ΠΠ΅ΠΊΡΠΎΡΠ½ΠΎΠ΅ ΠΏΡΠΎΠΈΠ·Π²Π΅Π΄Π΅Π½ΠΈΠ΅ Π²Π΅ΠΊΡΠΎΡΠΎΠ².
ΠΠ΅ΠΊΡΠΎΡΠ½ΠΎΠ΅ ΠΏΡΠΎΠΈΠ·Π²Π΅Π΄Π΅Π½ΠΈΠ΅ Π²Π΅ΠΊΡΠΎΡΠΎΠ².ΠΠ°Π²ΠΈΠ³Π°ΡΠΈΡ ΠΏΠΎ ΡΡΡΠ°Π½ΠΈΡΠ΅:
- ΠΠΏΡΠ΅Π΄Π΅Π»Π΅Π½ΠΈΠ΅ Π²Π΅ΠΊΡΠΎΡΠ½ΠΎΠ³ΠΎ ΠΏΡΠΎΠΈΠ·Π²Π΅Π΄Π΅Π½ΠΈΡ Π²Π΅ΠΊΡΠΎΡΠΎΠ²
- Π€ΠΎΡΠΌΡΠ»Ρ Π²ΡΡΠΈΡΠ»Π΅Π½ΠΈΡ Π²Π΅ΠΊΡΠΎΡΠ½ΠΎΠ³ΠΎ ΠΏΡΠΎΠΈΠ·Π²Π΅Π΄Π΅Π½ΠΈΡ Π²Π΅ΠΊΡΠΎΡΠΎΠ²
- Π‘Π²ΠΎΠΉΡΡΠ²Π° Π²Π΅ΠΊΡΠΎΡΠ½ΠΎΠ³ΠΎ ΠΏΡΠΎΠΈΠ·Π²Π΅Π΄Π΅Π½ΠΈΡ Π²Π΅ΠΊΡΠΎΡΠΎΠ²
- ΠΡΠΈΠΌΠ΅ΡΡ Π·Π°Π΄Π°Ρ Π½Π° Π²ΡΡΠΈΡΠ»Π΅Π½ΠΈΡ Π²Π΅ΠΊΡΠΎΡΠ½ΠΎΠ³ΠΎ ΠΏΡΠΎΠΈΠ·Π²Π΅Π΄Π΅Π½ΠΈΡ Π²Π΅ΠΊΡΠΎΡΠΎΠ²
ΠΠ½Π»Π°ΠΉΠ½ ΠΊΠ°Π»ΡΠΊΡΠ»ΡΡΠΎΡ. ΠΠ΅ΠΊΡΠΎΡΠ½ΠΎΠ΅ ΠΏΡΠΎΠΈΠ·Π²Π΅Π΄Π΅Π½ΠΈΠ΅ Π²Π΅ΠΊΡΠΎΡΠΎΠ².
Π£ΠΏΡΠ°ΠΆΠ½Π΅Π½ΠΈΡ Π½Π° ΡΠ΅ΠΌΡ Π²Π΅ΠΊΡΠΎΡΠ½ΠΎΠ΅ ΠΏΡΠΎΠΈΠ·Π²Π΅Π΄Π΅Π½ΠΈΠ΅ Π²Π΅ΠΊΡΠΎΡΠΎΠ².
ΠΠΏΡΠ΅Π΄Π΅Π»Π΅Π½ΠΈΠ΅. ΠΠ΅ΠΊΡΠΎΡΠ½ΡΠΌ ΠΏΡΠΎΠΈΠ·Π²Π΅Π΄Π΅Π½ΠΈΠ΅ΠΌ Π²Π΅ΠΊΡΠΎΡΠ° a Π½Π° Π²Π΅ΠΊΡΠΎΡ b Π½Π°Π·ΡΠ²Π°Π΅ΡΡΡ Π²Π΅ΠΊΡΠΎΡ c, Π΄Π»ΠΈΠ½Π° ΠΊΠΎΡΠΎΡΠΎΠ³ΠΎ ΡΠΈΡΠ»Π΅Π½Π½ΠΎ ΡΠ°Π²Π½Π° ΠΏΠ»ΠΎΡΠ°Π΄ΠΈ ΠΏΠ°ΡΠ°Π»Π»Π΅Π»ΠΎΠ³ΡΠ°ΠΌΠΌΠ° ΠΏΠΎΡΡΡΠΎΠ΅Π½Π½ΠΎΠ³ΠΎ Π½Π° Π²Π΅ΠΊΡΠΎΡΠ°Ρ a ΠΈ b, ΠΏΠ΅ΡΠΏΠ΅Π½Π΄ΠΈΠΊΡΠ»ΡΡΠ½ΡΠΉ ΠΊ ΠΏΠ»ΠΎΡΠΊΠΎΡΡΠΈ ΡΡΠΈΡ Π²Π΅ΠΊΡΠΎΡΠΎΠ² ΠΈ Π½Π°ΠΏΡΠ°Π²Π»Π΅Π½Π½ΡΠΉ ΡΠ°ΠΊ, ΡΡΠΎΠ± Π½Π°ΠΈΠΌΠ΅Π½ΡΡΠ΅Π΅ Π²ΡΠ°ΡΠ΅Π½ΠΈΠ΅ ΠΎΡ a ΠΊ b Π²ΠΎΠΊΡΡΠ³ Π²Π΅ΠΊΡΠΎΡΠ° c ΠΎΡΡΡΠ΅ΡΡΠ²Π»ΡΠ»ΠΎΡΡ ΠΏΡΠΎΡΠΈΠ² ΡΠ°ΡΠΎΠ²ΠΎΠΉ ΡΡΡΠ΅Π»ΠΊΠΈ, Π΅ΡΠ»ΠΈ ΡΠΌΠΎΡΡΠ΅ΡΡ Ρ ΠΊΠΎΠ½ΡΠ° Π²Π΅ΠΊΡΠΎΡΠ° c (ΡΠΈΡ. 1).
ΡΠΈΡ. 1 |
Π€ΠΎΡΠΌΡΠ»Ρ Π²ΡΡΠΈΡΠ»Π΅Π½ΠΈΡ Π²Π΅ΠΊΡΠΎΡΠ½ΠΎΠ³ΠΎ ΠΏΡΠΎΠΈΠ·Π²Π΅Π΄Π΅Π½ΠΈΡ Π²Π΅ΠΊΡΠΎΡΠΎΠ²
ΠΠ΅ΠΊΡΠΎΡΠ½ΠΎΠ΅ ΠΏΡΠΎΠΈΠ·Π²Π΅Π΄Π΅Π½ΠΈΠ΅
a Γ b = ijkaxayazbxbybz = i (aybz β azby) β j (axbz β azbx) + k (axby β aybx)
a Γ b = {aybz β azby; azbx β axbz; axby β aybx}
Π‘Π²ΠΎΠΉΡΡΠ²Π° Π²Π΅ΠΊΡΠΎΡΠ½ΠΎΠ³ΠΎ ΠΏΡΠΎΠΈΠ·Π²Π΅Π΄Π΅Π½ΠΈΡ Π²Π΅ΠΊΡΠΎΡΠΎΠ²
ΠΠ΅ΠΎΠΌΠ΅ΡΡΠΈΡΠ΅ΡΠΊΠΈΠΉ ΡΠΌΡΡΠ» Π²Π΅ΠΊΡΠΎΡΠ½ΠΎΠ³ΠΎ ΠΏΡΠΎΠΈΠ·Π²Π΅Π΄Π΅Π½ΠΈΡ.
ΠΠΎΠ΄ΡΠ»Ρ Π²Π΅ΠΊΡΠΎΡΠ½ΠΎΠ³ΠΎ ΠΏΡΠΎΠΈΠ·Π²Π΅Π΄Π΅Π½ΠΈΡ Π΄Π²ΡΡ Π²Π΅ΠΊΡΠΎΡΠΎΠ² a ΠΈ b ΡΠ°Π²Π΅Π½ ΠΏΠ»ΠΎΡΠ°Π΄ΠΈ ΠΏΠ°ΡΠ°Π»Π»Π΅Π»ΠΎΠ³ΡΠ°ΠΌΠΌΠ° ΠΏΠΎΡΡΡΠΎΠ΅Π½Π½ΠΎΠ³ΠΎ Π½Π° ΡΡΠΈΡ Π²Π΅ΠΊΡΠΎΡΠ°Ρ :
SΠΏΠ°ΡΠ°Π» = [a Γ b]
ΠΠ΅ΠΎΠΌΠ΅ΡΡΠΈΡΠ΅ΡΠΊΠΈΠΉ ΡΠΌΡΡΠ» Π²Π΅ΠΊΡΠΎΡΠ½ΠΎΠ³ΠΎ ΠΏΡΠΎΠΈΠ·Π²Π΅Π΄Π΅Π½ΠΈΡ.
ΠΠ»ΠΎΡΠ°Π΄Ρ ΡΡΠ΅ΡΠ³ΠΎΠ»ΡΠ½ΠΈΠΊΠ° ΠΏΠΎΡΡΡΠΎΠ΅Π½Π½ΠΎΠ³ΠΎ Π½Π° Π²Π΅ΠΊΡΠΎΡΠ°Ρ a ΠΈ b ΡΠ°Π²Π½Π° ΠΏΠΎΠ»ΠΎΠ²ΠΈΠ½Π΅ ΠΌΠΎΠ΄ΡΠ»Ρ Π²Π΅ΠΊΡΠΎΡΠ½ΠΎΠ³ΠΎ ΠΏΡΠΎΠΈΠ·Π²Π΅Π΄Π΅Π½ΠΈΡ ΡΡΠΈΡ Π²Π΅ΠΊΡΠΎΡΠΎΠ²:
SΞ = 1 |a Γ b| 2 ΠΠ΅ΠΊΡΠΎΡΠ½ΠΎΠ΅ ΠΏΡΠΎΠΈΠ·Π²Π΅Π΄Π΅Π½ΠΈΡ Π΄Π²ΡΡ Π½Π΅ Π½ΡΠ»Π΅Π²ΡΡ Π²Π΅ΠΊΡΠΎΡΠΎΠ² a ΠΈ b ΡΠ°Π²Π½ΠΎ Π½ΡΠ»Ρ ΡΠΎΠ³Π΄Π° ΠΈ ΡΠΎΠ»ΡΠΊΠΎ ΡΠΎΠ³Π΄Π°, ΠΊΠΎΠ³Π΄Π° Π²Π΅ΠΊΡΠΎΡΠ° ΠΊΠΎΠ»Π»ΠΈΠ½Π΅Π°ΡΠ½Ρ.
ΠΠ΅ΠΊΡΠΎΡ c, ΡΠ°Π²Π½ΡΠΉ Π²Π΅ΠΊΡΠΎΡΠ½ΠΎΠΌΡ ΠΏΡΠΎΠΈΠ·Π²Π΅Π΄Π΅Π½ΠΈΡ Π½Π΅ Π½ΡΠ»Π΅Π²ΡΡ Π²Π΅ΠΊΡΠΎΡΠΎΠ² a ΠΈ b, ΠΏΠ΅ΡΠΏΠ΅Π½Π΄ΠΈΠΊΡΠ»ΡΡΠ΅Π½ ΡΡΠΈΠΌ Π²Π΅ΠΊΡΠΎΡΠ°ΠΌ.
a Γ b = -b Γ a
(k a) Γ b = a Γ (k b) = k (a Γ b)
(a + b) Γ c = a Γ c + b Γ c
ΠΡΠΈΠΌΠ΅ΡΡ Π·Π°Π΄Π°Ρ Π½Π° Π²ΡΡΠΈΡΠ»Π΅Π½ΠΈΡ Π²Π΅ΠΊΡΠΎΡΠ½ΠΎΠ³ΠΎ ΠΏΡΠΎΠΈΠ·Π²Π΅Π΄Π΅Π½ΠΈΡ Π²Π΅ΠΊΡΠΎΡΠΎΠ²
ΠΡΠΈΠΌΠ΅Ρ 1. ΠΠ°ΠΉΡΠΈ Π²Π΅ΠΊΡΠΎΡΠ½ΠΎΠ΅ ΠΏΡΠΎΠΈΠ·Π²Π΅Π΄Π΅Π½ΠΈΠ΅ Π²Π΅ΠΊΡΠΎΡΠΎΠ² a = {1; 2; 3} ΠΈ b = {2; 1; -2}.
Π Π΅ΡΠ΅Π½ΠΈΠ΅:
a Γ b = | i | j | k | = |
1 | 2 | 3 | ||
2 | 1 | -2 |
= i(2 Β· (-2) β 3 Β· 1) β j(1 Β· (-2) β 2 Β· 3) + k(1 Β· 1 β 2 Β· 2) =
= i(-4 β 3) β j(-2 β 6) + k(1 β 4) = -7i + 8j β 3k = {-7; 8; -3}
ΠΡΠΈΠΌΠ΅Ρ 2. ΠΠ°ΠΉΡΠΈ ΠΏΠ»ΠΎΡΠ°Π΄Ρ ΡΡΠ΅ΡΠ³ΠΎΠ»ΡΠ½ΠΈΠΊΠ° ΠΎΠ±ΡΠ°Π·ΠΎΠ²Π°Π½Π½ΠΎΠ³ΠΎ Π²Π΅ΠΊΡΠΎΡΠ°ΠΌΠΈ a = {-1; 2; -2} ΠΈ b = {2; 1; -1}.
Π Π΅ΡΠ΅Π½ΠΈΠ΅: ΠΠ°ΠΉΠ΄Π΅ΠΌ Π²Π΅ΠΊΡΠΎΡΠ½ΠΎΠ΅ ΠΏΡΠΎΠΈΠ·Π²Π΅Π΄Π΅Π½ΠΈΠ΅ ΡΡΠΈΡ Π²Π΅ΠΊΡΠΎΡΠΎΠ²:
a Γ b = | i | j | k | = |
-1 | 2 | -2 | ||
2 | 1 | -1 |
= i(2 Β· (-1) β (-2) Β· 1) β j((-1) Β· (-1) β (-2) Β· 2) + k((-1) Β· 1 β 2 Β· 2) =
= i(-2 + 2) β j(1 + 4) + k(-1 β 4) = -5j β 5k = {0; -5; -5}
ΠΠ· ΡΠ²ΠΎΠΉΡΡΠ² Π²Π΅ΠΊΡΠΎΡΠ½ΠΎΠ³ΠΎ ΠΏΡΠΎΠΈΠ·Π²Π΅Π΄Π΅Π½ΠΈΡ:
SΞ =
12|a Γ b| =
12β02 + 52 + 52 =
12β25 + 25 =
12β50 =
5β22 = 2.
ΠΡΠ²Π΅Ρ: SΞ = 2.5β2.
ΠΠ΅ΠΊΡΠΎΡΠ° ΠΠ΅ΠΊΡΠΎΡ: ΠΎΠΏΡΠ΅Π΄Π΅Π»Π΅Π½ΠΈΠ΅ ΠΈ ΠΎΡΠ½ΠΎΠ²Π½ΡΠ΅ ΠΏΠΎΠ½ΡΡΠΈΡ ΠΠΏΡΠ΅Π΄Π΅Π»Π΅Π½ΠΈΠ΅ ΠΊΠΎΠΎΡΠ΄ΠΈΠ½Π°Ρ Π²Π΅ΠΊΡΠΎΡΠ° Π·Π°Π΄Π°Π½Π½ΠΎΠ³ΠΎ ΠΊΠΎΠΎΡΠ΄ΠΈΠ½Π°ΡΠ°ΠΌΠΈ Π΅Π³ΠΎ Π½Π°ΡΠ°Π»ΡΠ½ΠΎΠΉ ΠΈ ΠΊΠΎΠ½Π΅ΡΠ½ΠΎΠΉ ΡΠΎΡΠΊΠΈ ΠΠΎΠ΄ΡΠ»Ρ Π²Π΅ΠΊΡΠΎΡΠ°. ΠΠ»ΠΈΠ½Π° Π²Π΅ΠΊΡΠΎΡΠ° ΠΠ°ΠΏΡΠ°Π²Π»ΡΡΡΠΈΠ΅ ΠΊΠΎΡΠΈΠ½ΡΡΡ Π²Π΅ΠΊΡΠΎΡΠ° Π Π°Π²Π΅Π½ΡΡΠ²ΠΎ Π²Π΅ΠΊΡΠΎΡΠΎΠ² ΠΡΡΠΎΠ³ΠΎΠ½Π°Π»ΡΠ½ΠΎΡΡΡ Π²Π΅ΠΊΡΠΎΡΠΎΠ² ΠΠΎΠ»Π»ΠΈΠ½Π΅Π°ΡΠ½ΠΎΡΡΡ Π²Π΅ΠΊΡΠΎΡΠΎΠ² ΠΠΎΠΌΠΏΠ»Π°Π½Π°ΡΠ½ΠΎΡΡΡ Π²Π΅ΠΊΡΠΎΡΠΎΠ² Π£Π³ΠΎΠ» ΠΌΠ΅ΠΆΠ΄Ρ Π²Π΅ΠΊΡΠΎΡΠ°ΠΌΠΈ ΠΡΠΎΠ΅ΠΊΡΠΈΡ Π²Π΅ΠΊΡΠΎΡΠ° Π‘Π»ΠΎΠΆΠ΅Π½ΠΈΠ΅ ΠΈ Π²ΡΡΠΈΡΠ°Π½ΠΈΠ΅ Π²Π΅ΠΊΡΠΎΡΠΎΠ² Π£ΠΌΠ½ΠΎΠΆΠ΅Π½ΠΈΠ΅ Π²Π΅ΠΊΡΠΎΡΠ° Π½Π° ΡΠΈΡΠ»ΠΎ Π‘ΠΊΠ°Π»ΡΡΠ½ΠΎΠ΅ ΠΏΡΠΎΠΈΠ·Π²Π΅Π΄Π΅Π½ΠΈΠ΅ Π²Π΅ΠΊΡΠΎΡΠΎΠ² ΠΠ΅ΠΊΡΠΎΡΠ½ΠΎΠ΅ ΠΏΡΠΎΠΈΠ·Π²Π΅Π΄Π΅Π½ΠΈΠ΅ Π²Π΅ΠΊΡΠΎΡΠΎΠ² Π‘ΠΌΠ΅ΡΠ°Π½Π½ΠΎΠ΅ ΠΏΡΠΎΠΈΠ·Π²Π΅Π΄Π΅Π½ΠΈΠ΅ Π²Π΅ΠΊΡΠΎΡΠΎΠ² ΠΠΈΠ½Π΅ΠΉΠ½ΠΎ Π·Π°Π²ΠΈΡΠΈΠΌΡΠ΅ ΠΈ Π»ΠΈΠ½Π΅ΠΉΠ½ΠΎ Π½Π΅Π·Π°Π²ΠΈΡΠΈΠΌΡΠ΅ Π²Π΅ΠΊΡΠΎΡΠ° Π Π°Π·Π»ΠΎΠΆΠ΅Π½ΠΈΠ΅ Π²Π΅ΠΊΡΠΎΡΠ° ΠΏΠΎ Π±Π°Π·ΠΈΡΡ
ΠΠ½Π»Π°ΠΉΠ½ ΠΊΠ°Π»ΡΠΊΡΠ»ΡΡΠΎΡΡ Ρ Π²Π΅ΠΊΡΠΎΡΠ°ΠΌΠΈ
ΠΠ½Π»Π°ΠΉΠ½ ΡΠΏΡΠ°ΠΆΠ½Π΅Π½ΠΈΡ Ρ Π²Π΅ΠΊΡΠΎΡΠ°ΠΌΠΈ Π½Π° ΠΏΠ»ΠΎΡΠΊΠΎΡΡΠΈ
ΠΠ½Π»Π°ΠΉΠ½ ΡΠΏΡΠ°ΠΆΠ½Π΅Π½ΠΈΡ Ρ Π²Π΅ΠΊΡΠΎΡΠ°ΠΌΠΈ Π² ΠΏΡΠΎΡΡΡΠ°Π½ΡΡΠ²Π΅
ΠΠ°Ρ ΠΎΠΆΠ΄Π΅Π½ΠΈΠ΅ ΠΏΠ»ΠΎΡΠ°Π΄ΠΈ ΡΡΠ΅ΡΠ³ΠΎΠ»ΡΠ½ΠΈΠΊΠ° ΠΏΠΎ Π²Π΅ΠΊΡΠΎΡΠ°ΠΌ. ΠΠ΅ΠΊΡΠΎΡΠ½ΠΎΠ΅ ΠΏΡΠΎΠΈΠ·Π²Π΅Π΄Π΅Π½ΠΈΠ΅ Π²Π΅ΠΊΡΠΎΡΠΎΠ²
ΠΠΎΠ½ΡΡΠΎΠ»ΡΠ½Π°Ρ ΡΠ°Π±ΠΎΡΠ° β1
ΠΠ΅ΠΊΡΠΎΡΡ.
1-20. ΠΠ·Π²Π΅ΡΡΠ½Ρ Π΄Π»ΠΈΠ½Ρ Π²Π΅ΠΊΡΠΎΡΠΎΠ² ΠΈ ΠΈ; β ΡΠ³ΠΎΠ» ΠΌΠ΅ΠΆΠ΄Ρ ΡΡΠΈΠΌΠΈ Π²Π΅ΠΊΡΠΎΡΠ°ΠΌΠΈ.
ΠΡΡΠΈΡΠ»ΠΈΡΡ: 1) ΠΈ, 2) .3) ΠΠ°ΠΉΡΠΈ ΠΏΠ»ΠΎΡΠ°Π΄Ρ ΡΡΠ΅ΡΠ³ΠΎΠ»ΡΠ½ΠΈΠΊΠ°, ΠΏΠΎΡΡΡΠΎΠ΅Π½Π½ΠΎΠ³ΠΎ Π½Π° Π²Π΅ΠΊΡΠΎΡΠ°Ρ ΠΈ.
Π‘Π΄Π΅Π»Π°ΡΡ ΡΠ΅ΡΡΠ΅ΠΆ.
Π Π΅ΡΠ΅Π½ΠΈΠ΅. ΠΡΠΏΠΎΠ»ΡΠ·ΡΡ ΠΎΠΏΡΠ΅Π΄Π΅Π»Π΅Π½ΠΈΠ΅ ΡΠΊΠ°Π»ΡΡΠ½ΠΎΠ³ΠΎ ΠΏΡΠΎΠΈΠ·Π²Π΅Π΄Π΅Π½ΠΈΡ Π²Π΅ΠΊΡΠΎΡΠΎΠ²:
Π ΡΠ²ΠΎΠΉΡΡΠ²Π° ΡΠΊΠ°Π»ΡΡΠ½ΠΎΠ³ΠΎ ΠΏΡΠΎΠΈΠ·Π²Π΅Π΄Π΅Π½ΠΈΡ: ,
1) Π½Π°Ρ ΠΎΠ΄ΠΈΠΌ ΡΠΊΠ°Π»ΡΡΠ½ΡΠΉ ΠΊΠ²Π°Π΄ΡΠ°Ρ Π²Π΅ΠΊΡΠΎΡΠ°:
ΡΠΎ Π΅ΡΡΡ, Π’ΠΎΠ³Π΄Π° .
Π Π°ΡΡΡΠΆΠ΄Π°Ρ Π°Π½Π°Π»ΠΎΠ³ΠΈΡΠ½ΠΎ, ΠΏΠΎΠ»ΡΡΠ°Π΅ΠΌ
ΡΠΎ Π΅ΡΡΡ, Π’ΠΎΠ³Π΄Π° .
ΠΠΎ ΠΎΠΏΡΠ΅Π΄Π΅Π»Π΅Π½ΠΈΡ Π²Π΅ΠΊΡΠΎΡΠ½ΠΎΠ³ΠΎ ΠΏΡΠΎΠΈΠ·Π²Π΅Π΄Π΅Π½ΠΈΡ: ,
Ρ ΡΡΠ΅ΡΠΎΠΌ ΡΠΎΠ³ΠΎ, ΡΡΠΎ
ΠΠ»ΠΎΡΠ°Π΄Ρ ΡΡΠ΅ΡΠ³ΠΎΠ»ΡΠ½ΠΈΠΊΠ° ΠΏΠΎΡΡΡΠΎΠ΅Π½Π½ΠΎΠ³ΠΎ Π½Π° Π²Π΅ΠΊΡΠΎΡΠ°Ρ ΠΈ ΡΠ°Π²Π½Π°
21-40. ΠΠ·Π²Π΅ΡΡΠ½Ρ ΠΊΠΎΠΎΡΠ΄ΠΈΠ½Π°ΡΡ ΡΡΠ΅Ρ Π²Π΅ΡΡΠΈΠ½ A, B, D ΠΏΠ°ΡΠ°Π»Π»Π΅Π»ΠΎΠ³ΡΠ°ΠΌΠΌΠ° ABCD . Π‘ΡΠ΅Π΄ΡΡΠ²Π°ΠΌΠΈ Π²Π΅ΠΊΡΠΎΡΠ½ΠΎΠΉ Π°Π»Π³Π΅Π±ΡΡ ΡΡΠ΅Π±ΡΠ΅ΡΡΡ:
A (3;0;-7), B (2;4;6), D (-7;-5;1)
Π Π΅ΡΠ΅Π½ΠΈΠ΅.
ΠΠ·Π²Π΅ΡΡΠ½ΠΎ, ΡΡΠΎ Π΄ΠΈΠ°Π³ΠΎΠ½Π°Π»ΠΈ ΠΏΠ°ΡΠ°Π»Π»Π΅Π»ΠΎΠ³ΡΠ°ΠΌΠΌΠ° Π² ΡΠΎΡΠΊΠ΅ ΠΏΠ΅ΡΠ΅ΡΠ΅ΡΠ΅Π½ΠΈΡ Π΄Π΅Π»ΡΡΡΡ ΠΏΠΎΠΏΠΎΠ»Π°ΠΌ. ΠΠΎΡΡΠΎΠΌΡ ΠΊΠΎΠΎΡΠ΄ΠΈΠ½Π°ΡΡ ΡΠΎΡΠΊΠΈ

ΠΠΎΠ»ΡΡΠ°Π΅ΠΌ .
ΠΠ½Π°Ρ ΠΊΠΎΠΎΡΠ΄ΠΈΠ½Π°ΡΡ ΡΠΎΡΠΊΠΈ E β ΡΠ΅ΡΠ΅Π΄ΠΈΠ½Ρ Π΄ΠΈΠ°Π³ΠΎΠ½Π°Π»ΠΈ BD ΠΈ ΠΊΠΎΠΎΡΠ΄ΠΈΠ½Π°ΡΡ ΠΎΠ΄Π½ΠΎΠ³ΠΎ ΠΈΠ· Π΅Π³ΠΎ ΠΊΠΎΠ½ΡΠΎΠ² A (3;0;-7), ΠΏΠΎ ΡΠΎΡΠΌΡΠ»Π°ΠΌ ΠΎΠΏΡΠ΅Π΄Π΅Π»ΡΠ΅ΠΌ ΠΈΡΠΊΠΎΠΌΡΠ΅ ΠΊΠΎΠΎΡΠ΄ΠΈΠ½Π°ΡΡ Π²Π΅ΡΡΠΈΠ½Ρ Π‘ ΠΏΠ°ΡΠ°Π»Π»Π΅Π»ΠΎΠ³ΡΠ°ΠΌΠΌΠ°:
ΠΡΠ°ΠΊ, Π²Π΅ΡΡΠΈΠ½Π° .
2) Π§ΡΠΎΠ±Ρ Π½Π°ΠΉΡΠΈ ΠΏΡΠΎΠ΅ΠΊΡΠΈΡ Π²Π΅ΠΊΡΠΎΡΠ° Π½Π° Π²Π΅ΠΊΡΠΎΡ , Π½Π°ΠΉΠ΄Π΅ΠΌ ΠΊΠΎΠΎΡΠ΄ΠΈΠ½Π°ΡΡ ΡΡΠΈΡ Π²Π΅ΠΊΡΠΎΡΠΎΠ²: ,
Π°Π½Π°Π»ΠΎΠ³ΠΈΡΠ½ΠΎ . ΠΡΠΎΠ΅ΠΊΡΠΈΡ Π²Π΅ΠΊΡΠΎΡΠ° Π½Π° Π²Π΅ΠΊΡΠΎΡ , Π½Π°Ρ ΠΎΠ΄ΠΈΠΌ ΠΏΠΎ ΡΠΎΡΠΌΡΠ»Π΅:
3) Π£Π³ΠΎΠ» ΠΌΠ΅ΠΆΠ΄Ρ Π΄ΠΈΠ°Π³ΠΎΠ½Π°Π»ΡΠΌΠΈ ΠΏΠ°ΡΠ°Π»Π»Π΅Π»ΠΎΠ³ΡΠ°ΠΌΠΌΠ° Π½Π°Ρ ΠΎΠ΄ΠΈΠΌ ΠΊΠ°ΠΊ ΡΠ³ΠΎΠ» ΠΌΠ΅ΠΆΠ΄Ρ Π²Π΅ΠΊΡΠΎΡΠ°ΠΌΠΈ
Π ΠΏΠΎ ΡΠ²ΠΎΠΉΡΡΠ²Ρ ΡΠΊΠ°Π»ΡΡΠ½ΠΎΠ³ΠΎ ΠΏΡΠΎΠΈΠ·Π²Π΅Π΄Π΅Π½ΠΈΡ:
ΡΠΎΠ³Π΄Π°
4) ΠΠ»ΠΎΡΠ°Π΄Ρ ΠΏΠ°ΡΠ°Π»Π»Π΅Π»ΠΎΠ³ΡΠ°ΠΌΠΌΠ° Π½Π°Ρ ΠΎΠ΄ΠΈΠΌ ΠΊΠ°ΠΊ ΠΌΠΎΠ΄ΡΠ»Ρ Π²Π΅ΠΊΡΠΎΡΠ½ΠΎΠ³ΠΎ ΠΏΡΠΎΠΈΠ·Π²Π΅Π΄Π΅Π½ΠΈΡ:
5) ΠΠ±ΡΠ΅ΠΌ ΠΏΠΈΡΠ°ΠΌΠΈΠ΄Ρ Π½Π°Ρ ΠΎΠ΄ΠΈΠΌ ΠΊΠ°ΠΊ ΠΎΠ΄Π½Ρ ΡΠ΅ΡΡΡΡ ΠΌΠΎΠ΄ΡΠ»Ρ ΡΠΌΠ΅ΡΠ°Π½Π½ΠΎΠ³ΠΎ ΠΏΡΠΎΠΈΠ·Π²Π΅Π΄Π΅Π½ΠΈΡ Π²Π΅ΠΊΡΠΎΡΠΎΠ² , Π³Π΄Π΅ Π(0;0;0), ΡΠΎΠ³Π΄Π°
Π’ΠΎΠ³Π΄Π° ΠΈΡΠΊΠΎΠΌΡΠΉ ΠΎΠ±ΡΠ΅ΠΌ (ΠΊΡΠ±.Π΅Π΄.)
41-60. ΠΠ°Π½Ρ ΠΌΠ°ΡΡΠΈΡΡ:
Π Β·Π‘ -1 +3A T
ΠΠ±ΠΎΠ·Π½Π°ΡΠ΅Π½ΠΈΡ:
Π‘Π½Π°ΡΠ°Π»Π°
Π½Π°Ρ
ΠΎΠ΄ΠΈΠΌ ΠΎΠ±ΡΠ°ΡΠ½ΡΡ ΠΌΠ°ΡΡΠΈΡΡ ΠΊ ΠΌΠ°ΡΡΠΈΡΠ΅ Π‘.
ΠΠ»Ρ ΡΡΠΎΠ³ΠΎ Π½Π°Ρ ΠΎΠ΄ΠΈΠΌ Π΅Π΅ ΠΎΠΏΡΠ΅Π΄Π΅Π»ΠΈΡΠ΅Π»Ρ:
ΠΠΏΡΠ΅Π΄Π΅Π»ΠΈΡΠ΅Π»Ρ ΠΎΡΠ»ΠΈΡΠ΅Π½ ΠΎΡ Π½ΡΠ»Ρ, ΡΠ»Π΅Π΄ΠΎΠ²Π°ΡΠ΅Π»ΡΠ½ΠΎ, ΠΌΠ°ΡΡΠΈΡΠ° ΡΠ²Π»ΡΠ΅ΡΡΡ Π½Π΅Π²ΡΡΠΎΠΆΠ΄Π΅Π½Π½ΠΎΠΉ ΠΈ Π΄Π»Ρ Π½Π΅Π΅ ΠΌΠΎΠΆΠ½ΠΎ Π½Π°ΠΉΡΠΈ ΠΎΠ±ΡΠ°ΡΠ½ΡΡ ΠΌΠ°ΡΡΠΈΡΡ Π‘ -1
ΠΠ°ΠΉΠ΄Π΅ΠΌ Π°Π»Π³Π΅Π±ΡΠ°ΠΈΡΠ΅ΡΠΊΠΈΠ΅ Π΄ΠΎΠΏΠΎΠ»Π½Π΅Π½ΠΈΡ ΠΏΠΎ ΡΠΎΡΠΌΡΠ»Π΅ , Π³Π΄Π΅ β ΠΌΠΈΠ½ΠΎΡ ΡΠ»Π΅ΠΌΠ΅Π½ΡΠ° :
Π’ΠΎΠ³Π΄Π° , .
61β80. Π Π΅ΡΠΈΡΠ΅ ΡΠΈΡΡΠ΅ΠΌΡ Π»ΠΈΠ½Π΅ΠΉΠ½ΡΡ ΡΡΠ°Π²Π½Π΅Π½ΠΈΠΉ:
ΠΠ΅ΡΠΎΠ΄ΠΎΠΌ ΠΡΠ°ΠΌΠ΅ΡΠ°; 2. ΠΠ°ΡΡΠΈΡΠ½ΡΠΌ ΠΌΠ΅ΡΠΎΠ΄ΠΎΠΌ.
Π Π΅ΡΠ΅Π½ΠΈΠ΅.
Π°) ΠΌΠ΅ΡΠΎΠ΄ ΠΡΠ°ΠΌΠ΅ΡΠ°
ΠΠ°ΠΉΠ΄Π΅ΠΌ ΠΎΠΏΡΠ΅Π΄Π΅Π»ΠΈΡΠ΅Π»Ρ ΡΠΈΡΡΠ΅ΠΌΡ
Π’Π°ΠΊ ΠΊΠ°ΠΊ , ΡΠΎ ΡΠΈΡΡΠ΅ΠΌΠ° ΠΈΠΌΠ΅Π΅Ρ Π΅Π΄ΠΈΠ½ΡΡΠ²Π΅Π½Π½ΠΎΠ΅ ΡΠ΅ΡΠ΅Π½ΠΈΠ΅.
ΠΠ°ΠΉΠ΄Π΅ΠΌ ΠΎΠΏΡΠ΅Π΄Π΅Π»ΠΈΡΠ΅Π»ΠΈ ΠΈ , Π·Π°ΠΌΠ΅Π½ΠΈΠ² Π² ΠΌΠ°ΡΡΠΈΡΠ΅ ΠΊΠΎΡΡΡΠΈΡΠΈΠ΅Π½ΡΠΎΠ² ΡΠΎΠΎΡΠ²Π΅ΡΡΡΠ²Π΅Π½Π½ΠΎ ΠΏΠ΅ΡΠ²ΡΠΉ, Π²ΡΠΎΡΠΎΠΉ, ΡΡΠ΅ΡΠΈΠΉ ΡΡΠΎΠ»Π±ΡΡ ΡΡΠΎΠ»Π±ΡΠΎΠΌ ΡΠ²ΠΎΠ±ΠΎΠ΄Π½ΡΡ ΡΠ»Π΅Π½ΠΎΠ².
ΠΠΎ ΡΠΎΡΠΌΡΠ»Π°ΠΌ ΠΡΠ°ΠΌΠ΅ΡΠ°:
Π±) ΠΌΠ°ΡΡΠΈΡΠ½ΡΠΉ ΠΌΠ΅ΡΠΎΠ΄ (Ρ ΠΏΠΎΠΌΠΎΡΡΡ ΠΎΠ±ΡΠ°ΡΠ½ΠΎΠΉ ΠΌΠ°ΡΡΠΈΡΡ).
ΠΠ°Π½Π½ΡΡ ΡΠΈΡΡΠ΅ΠΌΡ Π·Π°ΠΏΠΈΡΠ΅ΠΌ Π² ΠΌΠ°ΡΡΠΈΡΠ½ΠΎΠΉ ΡΠΎΡΠΌΠ΅ ΠΈ ΡΠ΅ΡΠΈΠΌ Ρ ΠΏΠΎΠΌΠΎΡΡΡ ΠΎΠ±ΡΠ°ΡΠ½ΠΎΠΉ ΠΌΠ°ΡΡΠΈΡΡ.
ΠΡΡΡΡ Π β ΠΌΠ°ΡΡΠΈΡΠ° ΠΊΠΎΡΡΡΠΈΡΠΈΠ΅Π½ΡΠΎΠ² ΠΏΡΠΈ Π½Π΅ΠΈΠ·Π²Π΅ΡΡΠ½ΡΡ ; X β ΠΌΠ°ΡΡΠΈΡΠ°-ΡΡΠΎΠ»Π±Π΅Ρ Π½Π΅ΠΈΠ·Π²Π΅ΡΡΠ½ΡΡ x , y , z ΠΈ Π β ΠΌΠ°ΡΡΠΈΡΠ°-ΡΡΠΎΠ»Π±Π΅Ρ ΠΈΠ· ΡΠ²ΠΎΠ±ΠΎΠ΄Π½ΡΡ ΡΠ»Π΅Π½ΠΎΠ²:
ΠΠ΅Π²ΡΡ
ΡΠ°ΡΡΡ ΡΠΈΡΡΠ΅ΠΌΡ (1) ΠΌΠΎΠΆΠ½ΠΎ Π·Π°ΠΏΠΈΡΠ°ΡΡ Π² Π²ΠΈΠ΄Π΅
ΠΏΡΠΎΠΈΠ·Π²Π΅Π΄Π΅Π½ΠΈΡ ΠΌΠ°ΡΡΠΈΡ
, Π°
ΠΏΡΠ°Π²ΡΡ Π² Π²ΠΈΠ΄Π΅ ΠΌΠ°ΡΡΠΈΡΡ Π . Π‘Π»Π΅Π΄ΠΎΠ²Π°ΡΠ΅Π»ΡΠ½ΠΎ ΠΈΠΌΠ΅Π΅ΠΌ ΠΌΠ°ΡΡΠΈΡΠ½ΠΎΠ΅ ΡΡΠ°Π²Π½Π΅Π½ΠΈΠ΅
Π’Π°ΠΊ ΠΊΠ°ΠΊ ΠΎΠΏΡΠ΅Π΄Π΅Π»ΠΈΡΠ΅Π»Ρ ΠΌΠ°ΡΡΠΈΡΡ Π ΠΎΡΠ»ΠΈΡΠ΅Π½ ΠΎΡ Π½ΡΠ»Ρ (ΠΏΡΠ½ΠΊΡ Β«Π°Β»), ΡΠΎ ΠΌΠ°ΡΡΠΈΡΠ° Π ΠΈΠΌΠ΅Π΅Ρ ΠΎΠ±ΡΠ°ΡΠ½ΡΡ ΠΌΠ°ΡΡΠΈΡΡ . Π£ΠΌΠ½ΠΎΠΆΠΈΠΌ ΠΎΠ±Π΅ ΡΠ°ΡΡΠΈ ΡΠ°Π²Π΅Π½ΡΡΠ²Π° (2) ΡΠ»Π΅Π²Π° Π½Π° ΠΌΠ°ΡΡΠΈΡΡ , ΠΏΠΎΠ»ΡΡΠΈΠΌ
Π’Π°ΠΊ ΠΊΠ°ΠΊ , Π³Π΄Π΅ Π β Π΅Π΄ΠΈΠ½ΠΈΡΠ½Π°Ρ ΠΌΠ°ΡΡΠΈΡΠ°, Π° , ΡΠΎ
ΠΡΡΡΡ ΠΈΠΌΠ΅Π΅ΠΌ Π½Π΅Π²ΡΡΠΎΠΆΠ΄Π΅Π½Π½ΡΡ ΠΌΠ°ΡΡΠΈΡΡ Π:
Π’ΠΎΠ³Π΄Π° ΠΎΠ±ΡΠ°ΡΠ½ΡΡ ΠΌΠ°ΡΡΠΈΡΡ Π½Π°Ρ ΠΎΠ΄ΠΈΠΌ ΠΏΠΎ ΡΠΎΡΠΌΡΠ»Π΅:
Π³Π΄Π΅ A ij β Π°Π»Π³Π΅Π±ΡΠ°ΠΈΡΠ΅ΡΠΊΠΎΠ΅ Π΄ΠΎΠΏΠΎΠ»Π½Π΅Π½ΠΈΠ΅ ΡΠ»Π΅ΠΌΠ΅Π½ΡΠ° a ij Π² ΠΎΠΏΡΠ΅Π΄Π΅Π»ΠΈΡΠ΅Π»Π΅ ΠΌΠ°ΡΡΠΈΡΡ Π , ΠΊΠΎΡΠΎΡΠΎΠ΅ ΡΠ²Π»ΡΠ΅ΡΡΡ ΠΏΡΠΎΠΈΠ·Π²Π΅Π΄Π΅Π½ΠΈΠ΅ΠΌ (-1) i+j Π½Π° ΠΌΠΈΠ½ΠΎΡ (ΠΎΠΏΡΠ΅Π΄Π΅Π»ΠΈΡΠ΅Π»Ρ) n-1 ΠΏΠΎΡΡΠ΄ΠΊΠ°, ΠΏΠΎΠ»ΡΡΠ΅Π½Π½ΡΠΉ Π²ΡΡΠ΅ΡΠΊΠΈΠ²Π°Π½ΠΈΠ΅ΠΌ i-ΠΉ ΡΡΡΠΎΠΊΠΈ ΠΈ j-Π³ΠΎ ΡΡΠΎΠ»Π±ΡΠ° Π² ΠΎΠΏΡΠ΅Π΄Π΅Π»ΠΈΡΠ΅Π»Π΅ ΠΌΠ°ΡΡΠΈΡΡ Π:
ΠΡΡΡΠ΄Π° ΠΏΠΎΠ»ΡΡΠ°Π΅ΠΌ ΠΎΠ±ΡΠ°ΡΠ½ΡΡ ΠΌΠ°ΡΡΠΈΡΡ:
Π‘ΡΠΎΠ»Π±Π΅Ρ X: X=A -1 H
81β100. Π Π΅ΡΠΈΡΡ ΡΠΈΡΡΠ΅ΠΌΡ Π»ΠΈΠ½Π΅ΠΉΠ½ΡΡ ΡΡΠ°Π²Π½Π΅Π½ΠΈΠΉ ΠΌΠ΅ΡΠΎΠ΄ΠΎΠΌ ΠΠ°ΡΡΡΠ°
Π Π΅ΡΠ΅Π½ΠΈΠ΅. ΠΠ°ΠΏΠΈΡΠ΅ΠΌ ΡΠΈΡΡΠ΅ΠΌΡ Π² Π²ΠΈΠ΄Π΅ ΡΠ°ΡΡΠΈΡΠ΅Π½Π½ΠΎΠΉ ΠΌΠ°ΡΡΠΈΡΡ:
ΠΡΠΏΠΎΠ»Π½ΡΠ΅ΠΌ ΡΠ»Π΅ΠΌΠ΅Π½ΡΠ°ΡΠ½ΡΠ΅ ΠΏΡΠ΅ΠΎΠ±ΡΠ°Π·ΠΎΠ²Π°Π½ΠΈΡ ΡΠΎ ΡΡΡΠΎΠΊΠ°ΠΌΠΈ.
ΠΠ·
2-ΠΎΠΉ ΡΡΡΠΎΠΊΠΈ Π²ΡΡΠΈΡΠ°Π΅ΠΌ ΠΏΠ΅ΡΠ²ΡΡ ΡΡΡΠΎΠΊΡ,
ΡΠΌΠ½ΠΎΠΆΠ΅Π½Π½ΡΡ Π½Π° 2. ΠΠ· ΡΡΡΠΎΠΊΠΈ 3 Π²ΡΡΠΈΡΠ°Π΅ΠΌ
ΠΏΠ΅ΡΠ²ΡΡ ΡΡΡΠΎΠΊΡ, ΡΠΌΠ½ΠΎΠΆΠ΅Π½Π½ΡΡ Π½Π° 4. ΠΠ· ΡΡΡΠΎΠΊΠΈ
4 Π²ΡΡΠΈΡΠ°Π΅ΠΌ ΠΏΠ΅ΡΠ²ΡΡ ΡΡΡΠΎΠΊΡ, ΠΏΠΎΠ»ΡΡΠ°Π΅ΠΌ
ΠΌΠ°ΡΡΠΈΡΡ:
ΠΠ°Π»Π΅Π΅ ΠΏΠΎΠ»ΡΡΠ°Π΅ΠΌ Π½ΡΠ»Ρ Π² ΠΏΠ΅ΡΠ²ΠΎΠΌ ΡΡΠΎΠ»Π±ΡΠ΅ ΠΏΠΎΡΠ»Π΅Π΄ΡΡΡΠΈΡ ΡΡΡΠΎΠΊ, Π΄Π»Ρ ΡΡΠΎΠ³ΠΎ ΠΈΠ· Π²ΡΠΎΡΠΎΠΉ ΡΡΡΠΎΠΊΠΈ Π²ΡΡΠΈΡΠ°Π΅ΠΌ ΡΡΠ΅ΡΡΡ ΡΡΡΠΎΠΊΡ. ΠΠ· ΡΡΠ΅ΡΡΠ΅ΠΉ ΡΡΡΠΎΠΊΠΈ Π²ΡΡΠΈΡΠ°Π΅ΠΌ Π²ΡΠΎΡΡΡ ΡΡΡΠΎΠΊΡ, ΡΠΌΠ½ΠΎΠΆΠ΅Π½Π½ΡΡ Π½Π° 2. ΠΠ· ΡΠ΅ΡΠ²Π΅ΡΡΠΎΠΉ ΡΡΡΠΎΠΊΠΈ Π²ΡΡΠΈΡΠ°Π΅ΠΌ Π²ΡΠΎΡΡΡ ΡΡΡΠΎΠΊΡ, ΡΠΌΠ½ΠΎΠΆΠ΅Π½Π½ΡΡ Π½Π° 3. Π ΡΠ΅Π·ΡΠ»ΡΡΠ°ΡΠ΅ ΠΏΠΎΠ»ΡΡΠ°Π΅ΠΌ ΠΌΠ°ΡΡΠΈΡΡ Π²ΠΈΠ΄Π°:
ΠΠ· ΡΠ΅ΡΠ²Π΅ΡΡΠΎΠΉ ΡΡΡΠΎΠΊΠΈ Π²ΡΡΠΈΡΠ°Π΅ΠΌ ΡΡΠ΅ΡΡΡ.
ΠΠΎΠΌΠ΅Π½ΡΠ΅ΠΌ ΠΌΠ΅ΡΡΠ°ΠΌΠΈ ΠΏΡΠ΅Π΄ΠΏΠΎΡΠ»Π΅Π΄Π½ΡΡ ΠΈ ΠΏΠΎΡΠ»Π΅Π΄Π½ΡΡ ΡΡΡΠΎΠΊΠΈ:
ΠΠΎΡΠ»Π΅Π΄Π½ΡΡ ΠΌΠ°ΡΡΠΈΡΠ° ΡΠ°Π²Π½ΠΎΡΠΈΠ»ΡΠ½Π° ΡΠΈΡΡΠ΅ΠΌΠ΅ ΡΡΠ°Π²Π½Π΅Π½ΠΈΠΉ:
ΠΠ· ΠΏΠΎΡΠ»Π΅Π΄Π½Π΅Π³ΠΎ ΡΡΠ°Π²Π½Π΅Π½ΠΈΡ ΡΠΈΡΡΠ΅ΠΌΡ Π½Π°Ρ ΠΎΠ΄ΠΈΠΌ .
ΠΠΎΠ΄ΡΡΠ°Π²Π»ΡΡ Π² ΠΏΡΠ΅Π΄ΠΏΠΎΡΠ»Π΅Π΄Π½Π΅Π΅ ΡΡΠ°Π²Π½Π΅Π½ΠΈΠ΅, ΠΏΠΎΠ»ΡΡΠ°Π΅ΠΌ .
ΠΠ· Π²ΡΠΎΡΠΎΠ³ΠΎ ΡΡΠ°Π²Π½Π΅Π½ΠΈΡ ΡΠΈΡΡΠ΅ΠΌΡ ΡΠ»Π΅Π΄ΡΠ΅Ρ, ΡΡΠΎ
ΠΠ· ΠΏΠ΅ΡΠ²ΠΎΠ³ΠΎ ΡΡΠ°Π²Π½Π΅Π½ΠΈΡ Π½Π°Ρ ΠΎΠ΄ΠΈΠΌ Ρ :
ΠΡΠ²Π΅Ρ:
ΠΠΎΠ½ΡΡΠΎΠ»ΡΠ½Π°Ρ ΡΠ°Π±ΠΎΡΠ° β2
ΠΠ½Π°Π»ΠΈΡΠΈΡΠ΅ΡΠΊΠ°Ρ Π³Π΅ΠΎΠΌΠ΅ΡΡΠΈΡ
1-20. ΠΠ°Π½Ρ ΠΊΠΎΠΎΡΠ΄ΠΈΠ½Π°ΡΡ Π²Π΅ΡΡΠΈΠ½ ΡΡΠ΅ΡΠ³ΠΎΠ»ΡΠ½ΠΈΠΊΠ° ΠΠΠ‘. ΠΠ°ΠΉΡΠΈ:
1) Π΄Π»ΠΈΠ½Ρ ΡΡΠΎΡΠΎΠ½Ρ A Π ;
2) ΡΡΠ°Π²Π½Π΅Π½ΠΈΡ ΡΡΠΎΡΠΎΠ½ ΠΠ ΠΈ ΠΠ‘ ΠΈ ΠΈΡ ΡΠ³Π»ΠΎΠ²ΡΠ΅ ΠΊΠΎΡΡΡΠΈΡΠΈΠ΅Π½ΡΡ;
3) ΡΠ³ΠΎΠ» Π Π² ΡΠ°Π΄ΠΈΠ°Π½Π°Ρ Ρ ΡΠΎΡΠ½ΠΎΡΡΡΡ Π΄ΠΎ Π΄Π²ΡΡ Π·Π½Π°ΠΊΠΎΠ²;
4) ΡΡΠ°Π²Π½Π΅Π½ΠΈΠ΅ Π²ΡΡΠΎΡΡ CD ΠΈ Π΅Ρ Π΄Π»ΠΈΠ½Ρ;
5) ΡΡΠ°Π²Π½Π΅Π½ΠΈΠ΅ ΠΌΠ΅Π΄ΠΈΠ°Π½Ρ ΠΠ
Π²ΡΡΠΎΡΠΎΠΉ CD ;
Π ΠΏΠ°ΡΠ°Π»Π»Π΅Π»ΡΠ½ΠΎ ΡΡΠΎΡΠΎΠ½Π΅ ΠΠ,
7)
ΡΠ΄Π΅Π»Π°ΡΡ ΡΠ΅ΡΡΡΠΆ.
Π(3;6), Π(15;-3), Π‘(13;11)
Π Π΅ΡΠ΅Π½ΠΈΠ΅.
ΠΡΠΈΠΌΠ΅Π½ΡΡ (1), Π½Π°Ρ ΠΎΠ΄ΠΈΠΌ Π΄Π»ΠΈΠ½Ρ ΡΡΠΎΡΠΎΠ½Ρ ΠΠ :
2) ΡΡΠ°Π²Π½Π΅Π½ΠΈΡ ΡΡΠΎΡΠΎΠ½ ΠΠ ΠΈ ΠΠ‘ ΠΈ ΠΈΡ ΡΠ³Π»ΠΎΠ²ΡΠ΅ ΠΊΠΎΡΡΡΠΈΡΠΈΠ΅Π½ΡΡ:
Π£ΡΠ°Π²Π½Π΅Π½ΠΈΠ΅ ΠΏΡΡΠΌΠΎΠΉ, ΠΏΡΠΎΡ ΠΎΠ΄ΡΡΠ΅ΠΉ ΡΠ΅ΡΠ΅Π· ΡΠΎΡΠΊΠΈ ΠΈ , ΠΈΠΌΠ΅Π΅Ρ Π²ΠΈΠ΄
ΠΠΎΠ΄ΡΡΠ°Π²Π»ΡΡ Π² (2) ΠΊΠΎΠΎΡΠ΄ΠΈΠ½Π°ΡΡ ΡΠΎΡΠ΅ΠΊ Π ΠΈ Π , ΠΏΠΎΠ»ΡΡΠΈΠΌ ΡΡΠ°Π²Π½Π΅Π½ΠΈΠ΅ ΡΡΠΎΡΠΎΠ½Ρ ΠΠ :
(ΠΠ ).
(BC ).
3) ΡΠ³ΠΎΠ» Π Π² ΡΠ°Π΄ΠΈΠ°Π½Π°Ρ Ρ ΡΠΎΡΠ½ΠΎΡΡΡΡ Π΄ΠΎ Π΄Π²ΡΡ Π·Π½Π°ΠΊΠΎΠ².
ΠΠ·Π²Π΅ΡΡΠ½ΠΎ, ΡΡΠΎ ΡΠ°Π½Π³Π΅Π½Ρ ΡΠ³Π»Π° ΠΌΠ΅ΠΆΠ΄Ρ Π΄Π²ΡΠΌΡ ΠΏΡΡΠΌΡΠΌΠΈ, ΡΠ³Π»ΠΎΠ²ΡΠ΅ ΠΊΠΎΡΡΡΠΈΡΠΈΠ΅Π½ΡΡ, ΠΊΠΎΡΠΎΡΡΡ ΡΠΎΠΎΡΠ²Π΅ΡΡΡΠ²Π΅Π½Π½ΠΎ ΡΠ°Π²Π½Ρ ΠΈ Π²ΡΡΠΈΡΠ»ΡΠ΅ΡΡΡ ΠΏΠΎ ΡΠΎΡΠΌΡΠ»Π΅
ΠΡΠΊΠΎΠΌΡΠΉ ΡΠ³ΠΎΠ» Π ΠΎΠ±ΡΠ°Π·ΠΎΠ²Π°Π½ ΠΏΡΡΠΌΡΠΌΠΈ ΠΠ ΠΈ ΠΠ‘ , ΡΠ³Π»ΠΎΠ²ΡΠ΅ ΠΊΠΎΡΡΡΠΈΡΠΈΠ΅Π½ΡΡ ΠΊΠΎΡΠΎΡΡΡ Π½Π°ΠΉΠ΄Π΅Π½Ρ: ; . ΠΡΠΈΠΌΠ΅Π½ΡΡ (3), ΠΏΠΎΠ»ΡΡΠΈΠΌ
; , ΠΈΠ»ΠΈ
4) ΡΡΠ°Π²Π½Π΅Π½ΠΈΠ΅ Π²ΡΡΠΎΡΡ CD ΠΈ Π΅Ρ Π΄Π»ΠΈΠ½Π°.
Π Π°ΡΡΡΠΎΡΠ½ΠΈΠ΅ ΠΎΡ ΡΠΎΡΠΊΠΈ Π‘ Π΄ΠΎ ΠΏΡΡΠΌΠΎΠΉ ΠΠ:
5) ΡΡΠ°Π²Π½Π΅Π½ΠΈΠ΅ ΠΌΠ΅Π΄ΠΈΠ°Π½Ρ ΠΠ ΠΈ ΠΊΠΎΠΎΡΠ΄ΠΈΠ½Π°ΡΡ ΡΠΎΡΠΊΠΈ Π ΠΏΠ΅ΡΠ΅ΡΠ΅ΡΠ΅Π½ΠΈΡ ΡΡΠΎΠΉ ΠΌΠ΅Π΄ΠΈΠ°Π½Ρ Ρ
Π²ΡΡΠΎΡΠΎΠΉ CD .
ΡΠ΅ΡΠ΅Π΄ΠΈΠ½Π° ΡΡΠΎΡΠΎΠ½Ρ ΠΠ‘:
Π’ΠΎΠ³Π΄Π° ΡΡΠ°Π²Π½Π΅Π½ΠΈΠ΅ ΠΠ:
Π Π΅ΡΠ°Π΅ΠΌ ΡΠΈΡΡΠ΅ΠΌΡ ΡΡΠ°Π²Π½Π΅Π½ΠΈΠΉ:
6) ΡΡΠ°Π²Π½Π΅Π½ΠΈΠ΅ ΠΏΡΡΠΌΠΎΠΉ, ΠΏΡΠΎΡ ΠΎΠ΄ΡΡΠ΅ΠΉ ΡΠ΅ΡΠ΅Π· ΡΠΎΡΠΊΡ Π ΠΏΠ°ΡΠ°Π»Π»Π΅Π»ΡΠ½ΠΎ ΡΡΠΎΡΠΎΠ½Π΅ ΠΠ :
Π’Π°ΠΊ
ΠΊΠ°ΠΊ ΠΈΡΠΊΠΎΠΌΠ°Ρ ΠΏΡΡΠΌΠ°Ρ ΠΏΠ°ΡΠ°Π»Π»Π΅Π»ΡΠ½Π° ΡΡΠΎΡΠΎΠ½Π΅ ΠΠ , ΡΠΎ Π΅Π΅
ΡΠ³Π»ΠΎΠ²ΠΎΠΉ ΠΊΠΎΡΡΡΠΈΡΠΈΠ΅Π½Ρ Π±ΡΠ΄Π΅Ρ ΡΠ°Π²Π΅Π½ ΡΠ³Π»ΠΎΠ²ΠΎΠΌΡ
ΠΊΠΎΡΡΡΠΈΡΠΈΠ΅Π½ΡΡ ΠΏΡΡΠΌΠΎΠΉ ΠΠ . ΠΠΎΠ΄ΡΡΠ°Π²ΠΈΠ² Π² (4) ΠΊΠΎΠΎΡΠ΄ΠΈΠ½Π°ΡΡ Π½Π°ΠΉΠ΄Π΅Π½Π½ΠΎΠΉ
ΡΠΎΡΠΊΠΈ Π ΠΈ
ΡΠ³Π»ΠΎΠ²ΠΎΠΉ ΠΊΠΎΡΡΡΠΈΡΠΈΠ΅Π½Ρ
, ΠΏΠΎΠ»ΡΡΠΈΠΌ
; (KF ).
ΠΠ»ΠΎΡΠ°Π΄Ρ ΠΏΠ°ΡΠ°Π»Π»Π΅Π»ΠΎΠ³ΡΠ°ΠΌΠΌΠ° ΡΠ°Π²Π½Π° 12 ΠΊΠ². Π΅Π΄., Π΄Π²Π΅ Π΅Π³ΠΎ Π²Π΅ΡΡΠΈΠ½Ρ β ΡΠΎΡΠΊΠΈΠ(-1;3) ΠΈ Π(-2;4). ΠΠ°ΠΉΡΠΈ Π΄Π²Π΅ Π΄ΡΡΠ³ΠΈΠ΅ Π²Π΅ΡΡΠΈΠ½Ρ ΡΡΠΎΠ³ΠΎ ΠΏΠ°ΡΠ°Π»Π»Π΅Π»ΠΎΠ³ΡΠ°ΠΌΠΌΠ°, Π΅ΡΠ»ΠΈ ΠΈΠ·Π²Π΅ΡΡΠ½ΠΎ, ΡΡΠΎ ΡΠΎΡΠΊΠ° ΠΏΠ΅ΡΠ΅ΡΠ΅ΡΠ΅Π½ΠΈΡ Π΅Π³ΠΎ Π΄ΠΈΠ°Π³ΠΎΠ½Π°Π»Π΅ΠΉ Π»Π΅ΠΆΠΈΡ Π½Π° ΠΎΡΠΈ Π°Π±ΡΡΠΈΡΡ. Π‘Π΄Π΅Π»Π°ΡΡ ΡΠ΅ΡΡΡΠΆ.
Π Π΅ΡΠ΅Π½ΠΈΠ΅. ΠΡΡΡΡ ΡΠΎΡΠΊΠ° ΠΏΠ΅ΡΠ΅ΡΠ΅ΡΠ΅Π½ΠΈΡ Π΄ΠΈΠ°Π³ΠΎΠ½Π°Π»Π΅ΠΉ ΠΈΠΌΠ΅Π΅Ρ ΠΊΠΎΠΎΡΠ΄ΠΈΠ½Π°ΡΡ .
Π’ΠΎΠ³Π΄Π° ΠΎΡΠ΅Π²ΠΈΠ΄Π½ΠΎ, ΡΡΠΎ ΠΈ
ΡΠ»Π΅Π΄ΠΎΠ²Π°ΡΠ΅Π»ΡΠ½ΠΎ, ΠΊΠΎΠΎΡΠ΄ΠΈΠ½Π°ΡΡ Π²Π΅ΠΊΡΠΎΡΠΎΠ² .
ΠΠ»ΠΎΡΠ°Π΄Ρ ΠΏΠ°ΡΠ°Π»Π»Π΅Π»ΠΎΠ³ΡΠ°ΠΌΠΌΠ° Π½Π°Ρ ΠΎΠ΄ΠΈΠΌ ΠΏΠΎ ΡΠΎΡΠΌΡΠ»Π΅
Π’ΠΎΠ³Π΄Π° ΠΊΠΎΠΎΡΠ΄ΠΈΠ½Π°ΡΡ Π΄Π²ΡΡ Π΄ΡΡΠ³ΠΈΡ Π²Π΅ΡΡΠΈΠ½ .
Π Π·Π°Π΄Π°ΡΠ°Ρ 51-60 Π΄Π°Π½Ρ ΠΊΠΎΠΎΡΠ΄ΠΈΠ½Π°ΡΡ ΡΠΎΡΠ΅ΠΊ Π ΠΈ Π . Π’ΡΠ΅Π±ΡΠ΅ΡΡΡ:
Π‘ΠΎΡΡΠ°Π²ΠΈΡΡ ΠΊΠ°Π½ΠΎΠ½ΠΈΡΠ΅ΡΠΊΠΎΠ΅ ΡΡΠ°Π²Π½Π΅Π½ΠΈΠ΅ Π³ΠΈΠΏΠ΅ΡΠ±ΠΎΠ»Ρ, ΠΏΡΠΎΡ ΠΎΠ΄ΡΡΠ΅ΠΉ ΡΠ΅ΡΠ΅Π· Π΄Π°Π½Π½ΡΠ΅ ΡΠΎΡΠΊΠΈΠ ΠΈ Π, Π΅ΡΠ»ΠΈ ΡΠΎΠΊΡΡΡ Π³ΠΈΠΏΠ΅ΡΠ±ΠΎΠ»Ρ ΡΠ°ΡΠΏΠΎΠ»ΠΎΠΆΠ΅Π½Ρ Π½Π° ΠΎΡΠΈ Π°Π±ΡΡΠΈΡΡ;
ΠΠ°ΠΉΡΠΈ ΠΏΠΎΠ»ΡΠΎΡΠΈ, ΡΠΎΠΊΡΡΡ, ΡΠΊΡΡΠ΅Π½ΡΡΠΈΡΠΈΡΠ΅Ρ ΠΈ ΡΡΠ°Π²Π½Π΅Π½ΠΈΡ Π°ΡΠΈΠΌΠΏΡΠΎΡ ΡΡΠΎΠΉ Π³ΠΈΠΏΠ΅ΡΠ±ΠΎΠ»Ρ;
ΠΠ°ΠΉΡΠΈ Π²ΡΠ΅ ΡΠΎΡΠΊΠΈ ΠΏΠ΅ΡΠ΅ΡΠ΅ΡΠ΅Π½ΠΈΡ Π³ΠΈΠΏΠ΅ΡΠ±ΠΎΠ»Ρ Ρ ΠΎΠΊΡΡΠΆΠ½ΠΎΡΡΡΡ Ρ ΡΠ΅Π½ΡΡΠΎΠΌ Π² Π½Π°ΡΠ°Π»Π΅ ΠΊΠΎΠΎΡΠ΄ΠΈΠ½Π°Ρ, Π΅ΡΠ»ΠΈ ΡΡΠ° ΠΎΠΊΡΡΠΆΠ½ΠΎΡΡΡ ΠΏΡΠΎΡ ΠΎΠ΄ΠΈΡ ΡΠ΅ΡΠ΅Π· ΡΠΎΠΊΡΡΡ Π³ΠΈΠΏΠ΅ΡΠ±ΠΎΠ»Ρ;
ΠΠΎΡΡΡΠΎΠΈΡΡ
Π³ΠΈΠΏΠ΅ΡΠ±ΠΎΠ»Ρ, Π΅Ρ Π°ΡΠΈΠΌΠΏΡΠΎΡΡ ΠΈ ΠΎΠΊΡΡΠΆΠ½ΠΎΡΡΡ.
Π(6;-2), Π(-8;12).
Π Π΅ΡΠ΅Π½ΠΈΠ΅. Π£ΡΠ°Π²Π½Π΅Π½ΠΈΠ΅ ΠΈΡΠΊΠΎΠΌΠΎΠΉ Π³ΠΈΠΏΠ΅ΡΠ±ΠΎΠ»Ρ Π² ΠΊΠ°Π½ΠΎΠ½ΠΈΡΠ΅ΡΠΊΠΎΠΌ Π²ΠΈΠ΄Π΅ Π·Π°ΠΏΠΈΡΡΠ²Π°Π΅ΡΡΡ
Π³Π΄Π΅ a β Π΄Π΅ΠΉΡΡΠ²ΠΈΡΠ΅Π»ΡΠ½Π°Ρ ΠΏΠΎΠ»ΡΠΎΡΡ Π³ΠΈΠΏΠ΅ΡΠ±ΠΎΠ»Ρ, b β ΠΌΠ½ΠΈΠΌΠ°Ρ ΠΏΠΎΠ»ΡΠΎΡΡ. ΠΠΎΠ΄ΡΡΠ°Π²Π»ΡΡ ΠΊΠΎΠΎΡΠ΄ΠΈΠ½Π°ΡΡ ΡΠΎΡΠ΅ΠΊ Π ΠΈ Π Π² ΡΡΠΎ ΡΡΠ°Π²Π½Π΅Π½ΠΈΠ΅ Π½Π°ΠΉΠ΄Π΅ΠΌ ΡΡΠΈ ΠΏΠΎΠ»ΡΠΎΡΠΈ:
β ΡΡΠ°Π²Π½Π΅Π½ΠΈΠ΅ Π³ΠΈΠΏΠ΅ΡΠ±ΠΎΠ»Ρ: .
ΠΠΎΠ»ΡΠΎΡΠΈ Π°=4,
ΡΠΎΠΊΡΡΠ½ΠΎΠ΅ ΡΠ°ΡΡΡΠΎΡΠ½ΠΈΠ΅ Π€ΠΎΠΊΡΡΡ (-8,0) ΠΈ (8,0)
ΠΠΊΡΡΠ΅Π½ΡΡΠΈΡΠΈΡΠ΅Ρ
ΠΡΠΈΠΏΡΠΎΡΡ:
ΠΡΠ»ΠΈ ΠΎΠΊΡΡΠΆΠ½ΠΎΡΡΡ ΠΏΡΠΎΡ ΠΎΠ΄ΠΈΡ ΡΠ΅ΡΠ΅Π· Π½Π°ΡΠ°Π»ΠΎ ΠΊΠΎΠΎΡΠ΄ΠΈΠ½Π°Ρ, Π΅Π΅ ΡΡΠ°Π²Π½Π΅Π½ΠΈΠ΅
ΠΠΎΠ΄ΡΡΠ°Π²Π»ΡΡ ΠΎΠ΄ΠΈΠ½ ΠΈΠ· ΡΠΎΠΊΡΡΠΎΠ², Π½Π°Ρ ΠΎΠ΄ΠΈΠΌ ΠΈ ΡΡΠ°Π²Π½Π΅Π½ΠΈΠ΅ ΠΎΠΊΡΡΠΆΠ½ΠΎΡΡΠΈ
ΠΠ°Ρ ΠΎΠ΄ΠΈΠΌ ΡΠΎΡΠΊΠΈ ΠΏΠ΅ΡΠ΅ΡΠ΅ΡΠ΅Π½ΠΈΡ Π³ΠΈΠΏΠ΅ΡΠ±ΠΎΠ»Ρ ΠΈ ΠΎΠΊΡΡΠΆΠ½ΠΎΡΡΠΈ:
Π‘ΡΡΠΎΠΈΠΌ ΡΠ΅ΡΡΠ΅ΠΆ:
Π Π·Π°Π΄Π°ΡΠ°Ρ 61-80 ΠΏΠΎΡΡΡΠΎΠΈΡΡ Π³ΡΠ°ΡΠΈΠΊ ΡΡΠ½ΠΊΡΠΈΠΈ Π² ΠΏΠΎΠ»ΡΡΠ½ΠΎΠΉ ΡΠΈΡΡΠ΅ΠΌΠ΅ ΠΊΠΎΠΎΡΠ΄ΠΈΠ½Π°Ρ ΠΏΠΎ ΡΠΎΡΠΊΠ°ΠΌ, ΠΏΡΠΈΠ΄Π°Π²Π°Ρ οͺ Π·Π½Π°ΡΠ΅Π½ΠΈΡ ΡΠ΅ΡΠ΅Π· ΠΏΡΠΎΠΌΠ΅ΠΆΡΡΠΎΠΊ ο°/8 (0 ο£ οͺο£ 2ο°). ΠΠ°ΠΉΡΠΈ ΡΡΠ°Π²Π½Π΅Π½ΠΈΠ΅ Π»ΠΈΠ½ΠΈΠΈ Π² ΠΏΡΡΠΌΠΎΡΠ³ΠΎΠ»ΡΠ½ΠΎΠΉ Π΄Π΅ΠΊΠ°ΡΡΠΎΠ²ΠΎΠΉ ΡΠΈΡΡΠ΅ΠΌΠ΅ ΠΊΠΎΠΎΡΠ΄ΠΈΠ½Π°Ρ (ΠΏΠΎΠ»ΠΎΠΆΠΈΡΠ΅Π»ΡΠ½Π°Ρ ΠΏΠΎΠ»ΡΠΎΡΡ Π°Π±ΡΡΠΈΡΡ ΡΠΎΠ²ΠΏΠ°Π΄Π°Π΅Ρ Ρ ΠΏΠΎΠ»ΡΡΠ½ΠΎΠΉ ΠΎΡΡΡ, Π° ΠΏΠΎΠ»ΡΡ β Ρ Π½Π°ΡΠ°Π»ΠΎΠΌ ΠΊΠΎΠΎΡΠ΄ΠΈΠ½Π°Ρ).
Π Π΅ΡΠ΅Π½ΠΈΠ΅. ΠΠΎΡΡΡΠΎΠΈΠΌ Π»ΠΈΠ½ΠΈΡ ΠΏΠΎ ΡΠΎΡΠΊΠ°ΠΌ, ΠΏΡΠ΅Π΄Π²Π°ΡΠΈΡΠ΅Π»ΡΠ½ΠΎ
Π·Π°ΠΏΠΎΠ»Π½ΠΈΠ² ΡΠ°Π±Π»ΠΈΡΡ Π·Π½Π°ΡΠ΅Π½ΠΈΠΉ
ΠΈ Ο.
ΠΠΎΠΌΠ΅Ρ | Ο , | Ο, Π³ΡΠ°Π΄ΡΡΡ | ΠΠΎΠΌΠ΅Ρ | Ο , ΡΠ°Π΄ | Π³ΡΠ°Π΄ΡΡΡ | |||
3β(x 2 +2β1x + 1) -3β1 = 3(x+1) 2 β 3 Π΄Π΅Π»Π°Π΅ΠΌ Π²ΡΠ²ΠΎΠ΄, ΡΡΠΎ Π΄Π°Π½Π½ΠΎΠ΅ ΡΡΠ°Π²Π½Π΅Π½ΠΈΠ΅ ΠΎΠΏΡΠ΅Π΄Π΅Π»ΡΠ΅Ρ ΡΠ»Π»ΠΈΠΏΡ: ΠΠ°Π½Ρ
ΡΠΎΡΠΊΠΈ Π, Π, Π‘,
D. 1. Π£ΡΠ°Π²Π½Π΅Π½ΠΈΠ΅ ΠΏΠ»ΠΎΡΠΊΠΎΡΡΠΈ(Q ), ΠΏΡΠΎΡ ΠΎΠ΄ΡΡΠ΅ΠΉ ΡΠ΅ΡΠ΅Π· ΡΠΎΡΠΊΠΈΠ, Π, Π‘ D Π² ΠΏΠ»ΠΎΡΠΊΠΎΡΡΠΈ (Q) ; 2. Π£ΡΠ°Π²Π½Π΅Π½ΠΈΠ΅ ΠΏΡΡΠΌΠΎΠΉ (I), ΠΏΡΠΎΡ ΠΎΠ΄ΡΡΠ΅ΠΉ ΡΠ΅ΡΠ΅Π· ΡΠΎΡΠΊΠΈΠ ΠΈ D; 3. Π£Π³ΠΎΠ» ΠΌΠ΅ΠΆΠ΄Ρ ΠΏΠ»ΠΎΡΠΊΠΎΡΡΡΡ (Q) ΠΈ ΠΏΡΡΠΌΠΎΠΉ (I) ; 4. Π£ΡΠ°Π²Π½Π΅Π½ΠΈΠ΅ ΠΏΠ»ΠΎΡΠΊΠΎΡΡΠΈ (Π ), ΠΏΡΠΎΡ ΠΎΠ΄ΡΡΠ΅ΠΉ ΡΠ΅ΡΠ΅Π· ΡΠΎΡΠΊΡΠ ΠΏΠ΅ΡΠΏΠ΅Π½Π΄ΠΈΠΊΡΠ»ΡΡΠ½ΠΎ ΠΏΡΡΠΌΠΎΠΉ (I) ; 5. Π£Π³ΠΎΠ» ΠΌΠ΅ΠΆΠ΄Ρ ΠΏΠ»ΠΎΡΠΊΠΎΡΡΡΠΌΠΈ (Π ) ΠΈ (Q ) ; 6. Π£ΡΠ°Π²Π½Π΅Π½ΠΈΠ΅ ΠΏΡΡΠΌΠΎΠΉ (Ρ), ΠΏΡΠΎΡ ΠΎΠ΄ΡΡΠ΅ΠΉ ΡΠ΅ΡΠ΅Π· ΡΠΎΡΠΊΡ Π Π² Π½Π°ΠΏΡΠ°Π²Π»Π΅Π½ΠΈΠΈ Π΅Π΅ ΡΠ°Π΄ΠΈΡΡ-Π²Π΅ΠΊΡΠΎΡΠ°; 7. Π£Π³ΠΎΠ» ΠΌΠ΅ΠΆΠ΄Ρ ΠΏΡΡΠΌΡΠΌΠΈ (I) ΠΈ (Ρ). Π(9;-8;1), Π(-9;4;5), Π‘(9;-5;5), D (6;4;0) 1.
Π£ΡΠ°Π²Π½Π΅Π½ΠΈΠ΅ ΠΏΠ»ΠΎΡΠΊΠΎΡΡΠΈ(Q ), ΠΏΡΠΎΡ
ΠΎΠ΄ΡΡΠ΅ΠΉ ΡΠ΅ΡΠ΅Π· ΡΠΎΡΠΊΠΈ Π,
Π, Π‘ ΠΈ ΠΏΡΠΎΠ²Π΅ΡΠΈΡΡ, Π»Π΅ΠΆΠΈΡ Π»ΠΈ
ΡΠΎΡΠΊΠ° D Π²
ΠΏΠ»ΠΎΡΠΊΠΎΡΡΠΈ
ΠΎΠΏΡΠ΅Π΄Π΅Π»ΡΠ΅ΡΡΡ ΠΏΠΎ ΡΠΎΡΠΌΡΠ»Π΅
ΠΠ°ΠΉΡΠΈ
: 1) . 2) ΠΠ»ΠΎΡΠ°Π΄Ρ ΠΏΠ°ΡΠ°Π»Π»Π΅Π»ΠΎΠ³ΡΠ°ΠΌΠΌΠ°, ΠΏΠΎΡΡΡΠΎΠ΅Π½Π½ΠΎΠ³ΠΎ Π½Π° ΠΈ. ΠΠ΅ΡΠΎΠ΄ΠΈΡΠ΅ΡΠΊΠΈΠ΅ ΡΠ΅ΠΊΠΎΠΌΠ΅Π½Π΄Π°ΡΠΈΠΈ ΠΏΠΎ Π²ΡΠΏΠΎΠ»Π½Π΅Π½ΠΈΡ ΠΊΠΎΠ½ΡΡΠΎΠ»ΡΠ½ΡΡ ΡΠ°Π±ΠΎΡ Π΄Π»Ρ Π±Π°ΠΊΠ°Π»Π°Π²ΡΠΈΠ°ΡΠ° Π·Π°ΠΎΡΠ½ΠΎΠΉ ΡΠΎΡΠΌΡ ΠΎΠ±ΡΡΠ΅Π½ΠΈΡ ΠΏΠΎ ΠΊΠ²Π°Π»ΠΈΡΠΈΠΊΠ°ΡΠΈΠΈ 080100. 62 ΠΏΠΎ Π½Π°ΠΏΡΠ°Π²Π»Π΅Π½ΠΈΡΠΠ΅ΡΠΎΠ΄ΠΈΡΠ΅ΡΠΊΠΈΠ΅ ΡΠ΅ΠΊΠΎΠΌΠ΅Π½Π΄Π°ΡΠΈΠΈΠΠ°ΡΠ°Π»Π»Π΅Π»Π΅ΠΏΠΈΠΏΠ΅Π΄Π° ΠΈ ΠΎΠ±ΡΠ΅ΠΌ ΠΏΠΈΡΠ°ΠΌΠΈΠ΄Ρ, ΠΏΠΎΡΡΡΠΎΠ΅Π½Π½ΡΡ Π½Π° Π²Π΅ΠΊΡΠΎΡΠ°Ρ , ΠΈ. Π Π΅ΡΠ΅Π½ΠΈΠ΅: 2-=2(1;1;1)-(2;1;4)= (2;2;2)-(2;1;4)=(0;1;-2).. . . . . 4.ΠΠΠΠΠ§Π ΠΠΠ― ΠΠΠΠ’Π ΠΠΠ¬ΠΠ«Π₯ Π ΠΠΠΠ’ Π Π°Π·Π΄Π΅Π» I. ΠΠΈΠ½Π΅ΠΉΠ½Π°Ρ Π°Π»Π³Π΅Π±ΡΠ° . 1 β 10. ΠΠ°Π½Π°β¦ |
ΠΠ° Π΄Π°Π½Π½ΠΎΠΌ ΡΡΠΎΠΊΠ΅ ΠΌΡ ΡΠ°ΡΡΠΌΠΎΡΡΠΈΠΌ Π΅ΡΡ Π΄Π²Π΅ ΠΎΠΏΠ΅ΡΠ°ΡΠΈΠΈ Ρ Π²Π΅ΠΊΡΠΎΡΠ°ΠΌΠΈ: Π²Π΅ΠΊΡΠΎΡΠ½ΠΎΠ΅ ΠΏΡΠΎΠΈΠ·Π²Π΅Π΄Π΅Π½ΠΈΠ΅ Π²Π΅ΠΊΡΠΎΡΠΎΠ² ΠΈ ΡΠΌΠ΅ΡΠ°Π½Π½ΠΎΠ΅ ΠΏΡΠΎΠΈΠ·Π²Π΅Π΄Π΅Π½ΠΈΠ΅ Π²Π΅ΠΊΡΠΎΡΠΎΠ² (ΡΡΠ°Π·Ρ ΡΡΡΠ»ΠΊΠ°, ΠΊΠΎΠΌΡ Π½ΡΠΆΠ½ΠΎ ΠΈΠΌΠ΅Π½Π½ΠΎ ΠΎΠ½ΠΎ) . ΠΠΈΡΠ΅Π³ΠΎ ΡΡΡΠ°ΡΠ½ΠΎΠ³ΠΎ, ΡΠ°ΠΊ ΠΈΠ½ΠΎΠ³Π΄Π° Π±ΡΠ²Π°Π΅Ρ, ΡΡΠΎ Π΄Π»Ρ ΠΏΠΎΠ»Π½ΠΎΠ³ΠΎ ΡΡΠ°ΡΡΡΡ, ΠΏΠΎΠΌΠΈΠΌΠΎ ΡΠΊΠ°Π»ΡΡΠ½ΠΎΠ³ΠΎ ΠΏΡΠΎΠΈΠ·Π²Π΅Π΄Π΅Π½ΠΈΡ Π²Π΅ΠΊΡΠΎΡΠΎΠ² , ΡΡΠ΅Π±ΡΠ΅ΡΡΡ Π΅ΡΡ ΠΈ Π΅ΡΡ. Π’Π°ΠΊΠ°Ρ Π²ΠΎΡ Π²Π΅ΠΊΡΠΎΡΠ½Π°Ρ Π½Π°ΡΠΊΠΎΠΌΠ°Π½ΠΈΡ. ΠΠΎΠΆΠ΅Ρ ΡΠ»ΠΎΠΆΠΈΡΡΡΡ Π²ΠΏΠ΅ΡΠ°ΡΠ»Π΅Π½ΠΈΠ΅, ΡΡΠΎ ΠΌΡ Π·Π°Π»Π΅Π·Π°Π΅ΠΌ Π² Π΄Π΅Π±ΡΠΈ Π°Π½Π°Π»ΠΈΡΠΈΡΠ΅ΡΠΊΠΎΠΉ Π³Π΅ΠΎΠΌΠ΅ΡΡΠΈΠΈ. ΠΡΠΎ Π½Π΅ ΡΠ°ΠΊ. Π Π΄Π°Π½Π½ΠΎΠΌ ΡΠ°Π·Π΄Π΅Π»Π΅ Π²ΡΡΡΠ΅ΠΉ ΠΌΠ°ΡΠ΅ΠΌΠ°ΡΠΈΠΊΠΈ Π²ΠΎΠΎΠ±ΡΠ΅ ΠΌΠ°Π»ΠΎ Π΄ΡΠΎΠ², ΡΠ°Π·Π²Π΅ ΡΡΠΎ Π½Π° ΠΡΡΠ°ΡΠΈΠ½ΠΎ Ρ
Π²Π°ΡΠΈΡ. ΠΠ° ΡΠ°ΠΌΠΎΠΌ Π΄Π΅Π»Π΅ ΠΌΠ°ΡΠ΅ΡΠΈΠ°Π» ΠΎΡΠ΅Π½Ρ ΡΠ°ΡΠΏΡΠΎΡΡΡΠ°Π½Π΅Π½Π½ΡΠΉ ΠΈ ΠΏΡΠΎΡΡΠΎΠΉ β Π²ΡΡΠ΄ Π»ΠΈ ΡΠ»ΠΎΠΆΠ½Π΅Π΅, ΡΠ΅ΠΌ ΡΠΎ ΠΆΠ΅ ΡΠΊΠ°Π»ΡΡΠ½ΠΎΠ΅ ΠΏΡΠΎΠΈΠ·Π²Π΅Π΄Π΅Π½ΠΈΠ΅ , Π΄Π°ΠΆΠ΅ ΡΠΈΠΏΠΎΠ²ΡΡ
Π·Π°Π΄Π°Ρ ΠΏΠΎΠΌΠ΅Π½ΡΡΠ΅ Π±ΡΠ΄Π΅Ρ. ΠΠ»Π°Π²Π½ΠΎΠ΅ Π² Π°Π½Π°Π»ΠΈΡΠΈΡΠ΅ΡΠΊΠΎΠΉ Π³Π΅ΠΎΠΌΠ΅ΡΡΠΈΠΈ, ΠΊΠ°ΠΊ ΠΌΠ½ΠΎΠ³ΠΈΠ΅ ΡΠ±Π΅Π΄ΡΡΡΡ ΠΈΠ»ΠΈ ΡΠΆΠ΅ ΡΠ±Π΅Π΄ΠΈΠ»ΠΈΡΡ, ΠΠ ΠΠ¨ΠΠΠΠ’Π¬Π‘Π― Π ΠΠ«Π§ΠΠ‘ΠΠΠΠΠ―Π₯. ΠΠΎΠ²ΡΠΎΡΡΠΉΡΠ΅ ΠΊΠ°ΠΊ Π·Π°ΠΊΠ»ΠΈΠ½Π°Π½ΠΈΠ΅, ΠΈ Π±ΡΠ΄Π΅Ρ Π²Π°ΠΌ ΡΡΠ°ΡΡΡΠ΅ =)
ΠΡΠ»ΠΈ Π²Π΅ΠΊΡΠΎΡΡ ΡΠ²Π΅ΡΠΊΠ°ΡΡ Π³Π΄Π΅-ΡΠΎ Π΄Π°Π»Π΅ΠΊΠΎ, ΠΊΠ°ΠΊ ΠΌΠΎΠ»Π½ΠΈΠΈ Π½Π° Π³ΠΎΡΠΈΠ·ΠΎΠ½ΡΠ΅, Π½Π΅ Π±Π΅Π΄Π°, Π½Π°ΡΠ½ΠΈΡΠ΅ Ρ ΡΡΠΎΠΊΠ° ΠΠ΅ΠΊΡΠΎΡΡ Π΄Π»Ρ ΡΠ°ΠΉΠ½ΠΈΠΊΠΎΠ² , ΡΡΠΎΠ±Ρ Π²ΠΎΡΡΡΠ°Π½ΠΎΠ²ΠΈΡΡ ΠΈΠ»ΠΈ Π²Π½ΠΎΠ²Ρ ΠΏΡΠΈΠΎΠ±ΡΠ΅ΡΡΠΈ Π±Π°Π·ΠΎΠ²ΡΠ΅ Π·Π½Π°Π½ΠΈΡ ΠΎ Π²Π΅ΠΊΡΠΎΡΠ°Ρ . ΠΠΎΠ»Π΅Π΅ ΠΏΠΎΠ΄Π³ΠΎΡΠΎΠ²Π»Π΅Π½Π½ΡΠ΅ ΡΠΈΡΠ°ΡΠ΅Π»ΠΈ ΠΌΠΎΠ³ΡΡ Π·Π½Π°ΠΊΠΎΠΌΠΈΡΡΡΡ Ρ ΠΈΠ½ΡΠΎΡΠΌΠ°ΡΠΈΠ΅ΠΉ Π²ΡΠ±ΠΎΡΠΎΡΠ½ΠΎ, Ρ ΠΏΠΎΡΡΠ°ΡΠ°Π»ΡΡ ΡΠΎΠ±ΡΠ°ΡΡ ΠΌΠ°ΠΊΡΠΈΠΌΠ°Π»ΡΠ½ΠΎ ΠΏΠΎΠ»Π½ΡΡ ΠΊΠΎΠ»Π»Π΅ΠΊΡΠΈΡ ΠΏΡΠΈΠΌΠ΅ΡΠΎΠ², ΠΊΠΎΡΠΎΡΡΠ΅ ΡΠ°ΡΡΠΎ Π²ΡΡΡΠ΅ΡΠ°ΡΡΡΡ Π² ΠΏΡΠ°ΠΊΡΠΈΡΠ΅ΡΠΊΠΈΡ ΡΠ°Π±ΠΎΡΠ°Ρ
Π§Π΅ΠΌ Π²Π°Ρ ΡΡΠ°Π·Ρ ΠΏΠΎΡΠ°Π΄ΠΎΠ²Π°ΡΡ? ΠΠΎΠ³Π΄Π° Ρ Π±ΡΠ» ΠΌΠ°Π»Π΅Π½ΡΠΊΠΈΠΌ, ΡΠΎ ΡΠΌΠ΅Π» ΠΆΠΎΠ½Π³Π»ΠΈΡΠΎΠ²Π°ΡΡ Π΄Π²ΡΠΌΡ ΠΈ Π΄Π°ΠΆΠ΅ ΡΡΠ΅ΠΌΡ ΡΠ°ΡΠΈΠΊΠ°ΠΌΠΈ. ΠΠΎΠ²ΠΊΠΎ ΠΏΠΎΠ»ΡΡΠ°Π»ΠΎΡΡ. Π‘Π΅ΠΉΡΠ°Ρ ΠΆΠΎΠ½Π³Π»ΠΈΡΠΎΠ²Π°ΡΡ Π½Π΅ ΠΏΡΠΈΠ΄ΡΡΡΡ Π²ΠΎΠΎΠ±ΡΠ΅, ΠΏΠΎΡΠΊΠΎΠ»ΡΠΊΡ ΠΌΡ Π±ΡΠ΄Π΅ΠΌ ΡΠ°ΡΡΠΌΠ°ΡΡΠΈΠ²Π°ΡΡ ΡΠΎΠ»ΡΠΊΠΎ ΠΏΡΠΎΡΡΡΠ°Π½ΡΡΠ²Π΅Π½Π½ΡΠ΅ Π²Π΅ΠΊΡΠΎΡΡ , Π° ΠΏΠ»ΠΎΡΠΊΠΈΠ΅ Π²Π΅ΠΊΡΠΎΡΡ Ρ Π΄Π²ΡΠΌΡ ΠΊΠΎΠΎΡΠ΄ΠΈΠ½Π°ΡΠ°ΠΌΠΈ ΠΎΡΡΠ°Π½ΡΡΡΡ Π·Π° Π±ΠΎΡΡΠΎΠΌ. ΠΠΎΡΠ΅ΠΌΡ? Π’Π°ΠΊΠΈΠΌΠΈ ΡΠΆ ΡΠΎΠ΄ΠΈΠ»ΠΈΡΡ Π΄Π°Π½Π½ΡΠ΅ Π΄Π΅ΠΉΡΡΠ²ΠΈΡ β Π²Π΅ΠΊΡΠΎΡΠ½ΠΎΠ΅ ΠΈ ΡΠΌΠ΅ΡΠ°Π½Π½ΠΎΠ΅ ΠΏΡΠΎΠΈΠ·Π²Π΅Π΄Π΅Π½ΠΈΠ΅ Π²Π΅ΠΊΡΠΎΡΠΎΠ² ΠΎΠΏΡΠ΅Π΄Π΅Π»Π΅Π½Ρ ΠΈ ΡΠ°Π±ΠΎΡΠ°ΡΡ Π² ΡΡΡΡ
ΠΌΠ΅ΡΠ½ΠΎΠΌ ΠΏΡΠΎΡΡΡΠ°Π½ΡΡΠ²Π΅. Π£ΠΆΠ΅ ΠΏΡΠΎΡΠ΅!
Π Π΄Π°Π½Π½ΠΎΠΉ ΠΎΠΏΠ΅ΡΠ°ΡΠΈΠΈ, ΡΠΎΡΠ½ΠΎ ΡΠ°ΠΊ ΠΆΠ΅, ΠΊΠ°ΠΊ ΠΈ Π² ΡΠΊΠ°Π»ΡΡΠ½ΠΎΠΌ ΠΏΡΠΎΠΈΠ·Π²Π΅Π΄Π΅Π½ΠΈΠΈ, ΡΡΠ°ΡΡΠ²ΡΡΡ Π΄Π²Π° Π²Π΅ΠΊΡΠΎΡΠ° . ΠΡΡΡΡ ΡΡΠΎ Π±ΡΠ΄ΡΡ Π½Π΅ΡΠ»Π΅Π½Π½ΡΠ΅ Π±ΡΠΊΠ²Ρ .
Π‘Π°ΠΌΠΎ Π΄Π΅ΠΉΡΡΠ²ΠΈΠ΅ ΠΎΠ±ΠΎΠ·Π½Π°ΡΠ°Π΅ΡΡΡ ΡΠ»Π΅Π΄ΡΡΡΠΈΠΌ ΠΎΠ±ΡΠ°Π·ΠΎΠΌ: . Π‘ΡΡΠ΅ΡΡΠ²ΡΡΡ ΠΈ Π΄ΡΡΠ³ΠΈΠ΅ Π²Π°ΡΠΈΠ°Π½ΡΡ, Π½ΠΎ Ρ ΠΏΡΠΈΠ²ΡΠΊ ΠΎΠ±ΠΎΠ·Π½Π°ΡΠ°ΡΡ Π²Π΅ΠΊΡΠΎΡΠ½ΠΎΠ΅ ΠΏΡΠΎΠΈΠ·Π²Π΅Π΄Π΅Π½ΠΈΠ΅ Π²Π΅ΠΊΡΠΎΡΠΎΠ² ΠΈΠΌΠ΅Π½Π½ΠΎ ΡΠ°ΠΊ, Π² ΠΊΠ²Π°Π΄ΡΠ°ΡΠ½ΡΡ ΡΠΊΠΎΠ±ΠΊΠ°Ρ Ρ ΠΊΡΠ΅ΡΡΠΈΠΊΠΎΠΌ.
Π ΡΡΠ°Π·Ρ Π²ΠΎΠΏΡΠΎΡ : Π΅ΡΠ»ΠΈ Π² ΡΠΊΠ°Π»ΡΡΠ½ΠΎΠΌ ΠΏΡΠΎΠΈΠ·Π²Π΅Π΄Π΅Π½ΠΈΠΈ Π²Π΅ΠΊΡΠΎΡΠΎΠ² ΡΡΠ°ΡΡΠ²ΡΡΡ Π΄Π²Π° Π²Π΅ΠΊΡΠΎΡΠ°, ΠΈ Π·Π΄Π΅ΡΡ ΡΠΎΠΆΠ΅ ΡΠΌΠ½ΠΎΠΆΠ°ΡΡΡΡ Π΄Π²Π° Π²Π΅ΠΊΡΠΎΡΠ°, ΡΠΎΠ³Π΄Π° Π² ΡΡΠΌ ΡΠ°Π·Π½ΠΈΡΠ° ? Π―Π²Π½Π°Ρ ΡΠ°Π·Π½ΠΈΡΠ°, ΠΏΡΠ΅ΠΆΠ΄Π΅ Π²ΡΠ΅Π³ΠΎ, Π² Π ΠΠΠ£ΠΠ¬Π’ΠΠ’Π:
Π Π΅Π·ΡΠ»ΡΡΠ°ΡΠΎΠΌ ΡΠΊΠ°Π»ΡΡΠ½ΠΎΠ³ΠΎ ΠΏΡΠΎΠΈΠ·Π²Π΅Π΄Π΅Π½ΠΈΡ Π²Π΅ΠΊΡΠΎΡΠΎΠ² ΡΠ²Π»ΡΠ΅ΡΡΡ Π§ΠΠ‘ΠΠ:
Π Π΅Π·ΡΠ»ΡΡΠ°ΡΠΎΠΌ Π²Π΅ΠΊΡΠΎΡΠ½ΠΎΠ³ΠΎ ΠΏΡΠΎΠΈΠ·Π²Π΅Π΄Π΅Π½ΠΈΡ Π²Π΅ΠΊΡΠΎΡΠΎΠ² ΡΠ²Π»ΡΠ΅ΡΡΡ ΠΠΠΠ’ΠΠ : , ΡΠΎ Π΅ΡΡΡ ΡΠΌΠ½ΠΎΠΆΠ°Π΅ΠΌ Π²Π΅ΠΊΡΠΎΡΡ ΠΈ ΠΏΠΎΠ»ΡΡΠ°Π΅ΠΌ ΡΠ½ΠΎΠ²Π° Π²Π΅ΠΊΡΠΎΡ. ΠΠ°ΠΊΡΡΡΡΠΉ ΠΊΠ»ΡΠ±. Π‘ΠΎΠ±ΡΡΠ²Π΅Π½Π½ΠΎ, ΠΎΡΡΡΠ΄Π° ΠΈ Π½Π°Π·Π²Π°Π½ΠΈΠ΅ ΠΎΠΏΠ΅ΡΠ°ΡΠΈΠΈ. Π ΡΠ°Π·Π»ΠΈΡΠ½ΠΎΠΉ ΡΡΠ΅Π±Π½ΠΎΠΉ Π»ΠΈΡΠ΅ΡΠ°ΡΡΡΠ΅ ΠΎΠ±ΠΎΠ·Π½Π°ΡΠ΅Π½ΠΈΡ ΡΠΎΠΆΠ΅ ΠΌΠΎΠ³ΡΡ Π²Π°ΡΡΠΈΡΠΎΠ²Π°ΡΡΡΡ, Ρ Π±ΡΠ΄Ρ ΠΈΡΠΏΠΎΠ»ΡΠ·ΠΎΠ²Π°ΡΡ Π±ΡΠΊΠ²Ρ .
Π‘Π½Π°ΡΠ°Π»Π° Π±ΡΠ΄Π΅Ρ ΠΎΠΏΡΠ΅Π΄Π΅Π»Π΅Π½ΠΈΠ΅ Ρ ΠΊΠ°ΡΡΠΈΠ½ΠΊΠΎΠΉ, Π·Π°ΡΠ΅ΠΌ ΠΊΠΎΠΌΠΌΠ΅Π½ΡΠ°ΡΠΈΠΈ.
ΠΠΏΡΠ΅Π΄Π΅Π»Π΅Π½ΠΈΠ΅ : ΠΠ΅ΠΊΡΠΎΡΠ½ΡΠΌ ΠΏΡΠΎΠΈΠ·Π²Π΅Π΄Π΅Π½ΠΈΠ΅ΠΌ Π½Π΅ΠΊΠΎΠ»Π»ΠΈΠ½Π΅Π°ΡΠ½ΡΡ
Π²Π΅ΠΊΡΠΎΡΠΎΠ² , Π²Π·ΡΡΡΡ
Π² Π΄Π°Π½Π½ΠΎΠΌ ΠΏΠΎΡΡΠ΄ΠΊΠ΅ , Π½Π°Π·ΡΠ²Π°Π΅ΡΡΡ ΠΠΠΠ’ΠΠ , Π΄Π»ΠΈΠ½Π° ΠΊΠΎΡΠΎΡΠΎΠ³ΠΎ ΡΠΈΡΠ»Π΅Π½Π½ΠΎ ΡΠ°Π²Π½Π° ΠΏΠ»ΠΎΡΠ°Π΄ΠΈ ΠΏΠ°ΡΠ°Π»Π»Π΅Π»ΠΎΠ³ΡΠ°ΠΌΠΌΠ° , ΠΏΠΎΡΡΡΠΎΠ΅Π½Π½ΠΎΠ³ΠΎ Π½Π° Π΄Π°Π½Π½ΡΡ
Π²Π΅ΠΊΡΠΎΡΠ°Ρ
; Π²Π΅ΠΊΡΠΎΡ ΠΎΡΡΠΎΠ³ΠΎΠ½Π°Π»Π΅Π½ Π²Π΅ΠΊΡΠΎΡΠ°ΠΌ , ΠΈ Π½Π°ΠΏΡΠ°Π²Π»Π΅Π½ ΡΠ°ΠΊ, ΡΡΠΎ Π±Π°Π·ΠΈΡ ΠΈΠΌΠ΅Π΅Ρ ΠΏΡΠ°Π²ΡΡ ΠΎΡΠΈΠ΅Π½ΡΠ°ΡΠΈΡ:
Π Π°Π·Π±ΠΈΡΠ°Π΅ΠΌ ΠΎΠΏΡΠ΅Π΄Π΅Π»Π΅Π½ΠΈΠ΅ ΠΏΠΎ ΠΊΠΎΡΡΠΎΡΠΊΠ°ΠΌ, ΡΡΡ ΠΌΠ½ΠΎΠ³ΠΎ ΠΈΠ½ΡΠ΅ΡΠ΅ΡΠ½ΠΎΠ³ΠΎ!
ΠΡΠ°ΠΊ, ΠΌΠΎΠΆΠ½ΠΎ Π²ΡΠ΄Π΅Π»ΠΈΡΡ ΡΠ»Π΅Π΄ΡΡΡΠΈΠ΅ ΡΡΡΠ΅ΡΡΠ²Π΅Π½Π½ΡΠ΅ ΠΌΠΎΠΌΠ΅Π½ΡΡ:
1) ΠΡΡ ΠΎΠ΄Π½ΡΠ΅ Π²Π΅ΠΊΡΠΎΡΡ , ΠΎΠ±ΠΎΠ·Π½Π°ΡΠ΅Π½Π½ΡΠ΅ ΠΊΡΠ°ΡΠ½ΡΠΌΠΈ ΡΡΡΠ΅Π»ΠΊΠ°ΠΌΠΈ, ΠΏΠΎ ΠΎΠΏΡΠ΅Π΄Π΅Π»Π΅Π½ΠΈΡ Π½Π΅ ΠΊΠΎΠ»Π»ΠΈΠ½Π΅Π°ΡΠ½Ρ . Π‘Π»ΡΡΠ°ΠΉ ΠΊΠΎΠ»Π»ΠΈΠ½Π΅Π°ΡΠ½ΡΡ Π²Π΅ΠΊΡΠΎΡΠΎΠ² Π±ΡΠ΄Π΅Ρ ΡΠΌΠ΅ΡΡΠ½ΠΎ ΡΠ°ΡΡΠΌΠΎΡΡΠ΅ΡΡ ΡΡΡΡ ΠΏΠΎΠ·ΠΆΠ΅.
2) ΠΠ΅ΠΊΡΠΎΡΡ Π²Π·ΡΡΡ Π² ΡΡΡΠΎΠ³ΠΎ ΠΎΠΏΡΠ΅Π΄Π΅Π»ΡΠ½Π½ΠΎΠΌ ΠΏΠΎΡΡΠ΄ΠΊΠ΅ : β Β«Π°Β» ΡΠΌΠ½ΠΎΠΆΠ°Π΅ΡΡΡ Π½Π° Β«Π±ΡΒ» , Π° Π½Π΅ Β«Π±ΡΒ» Π½Π° Β«Π°Β». Π Π΅Π·ΡΠ»ΡΡΠ°ΡΠΎΠΌ ΡΠΌΠ½ΠΎΠΆΠ΅Π½ΠΈΡ Π²Π΅ΠΊΡΠΎΡΠΎΠ² ΡΠ²Π»ΡΠ΅ΡΡΡ ΠΠΠΠ’ΠΠ , ΠΊΠΎΡΠΎΡΡΠΉ ΠΎΠ±ΠΎΠ·Π½Π°ΡΠ΅Π½ ΡΠΈΠ½ΠΈΠΌ ΡΠ²Π΅ΡΠΎΠΌ. ΠΡΠ»ΠΈ Π²Π΅ΠΊΡΠΎΡΡ ΡΠΌΠ½ΠΎΠΆΠΈΡΡ Π² ΠΎΠ±ΡΠ°ΡΠ½ΠΎΠΌ ΠΏΠΎΡΡΠ΄ΠΊΠ΅, ΡΠΎ ΠΏΠΎΠ»ΡΡΠΈΠΌ ΡΠ°Π²Π½ΡΠΉ ΠΏΠΎ Π΄Π»ΠΈΠ½Π΅ ΠΈ ΠΏΡΠΎΡΠΈΠ²ΠΎΠΏΠΎΠ»ΠΎΠΆΠ½ΡΠΉ ΠΏΠΎ Π½Π°ΠΏΡΠ°Π²Π»Π΅Π½ΠΈΡ Π²Π΅ΠΊΡΠΎΡ (ΠΌΠ°Π»ΠΈΠ½ΠΎΠ²ΡΠΉ ΡΠ²Π΅Ρ). Π’ΠΎ Π΅ΡΡΡ, ΡΠΏΡΠ°Π²Π΅Π΄Π»ΠΈΠ²ΠΎ ΡΠ°Π²Π΅Π½ΡΡΠ²ΠΎ .
3) Π’Π΅ΠΏΠ΅ΡΡ ΠΏΠΎΠ·Π½Π°ΠΊΠΎΠΌΠΈΠΌΡΡ Ρ Π³Π΅ΠΎΠΌΠ΅ΡΡΠΈΡΠ΅ΡΠΊΠΈΠΌ ΡΠΌΡΡΠ»ΠΎΠΌ Π²Π΅ΠΊΡΠΎΡΠ½ΠΎΠ³ΠΎ ΠΏΡΠΎΠΈΠ·Π²Π΅Π΄Π΅Π½ΠΈΡ. ΠΡΠΎ ΠΎΡΠ΅Π½Ρ Π²Π°ΠΆΠ½ΡΠΉ ΠΏΡΠ½ΠΊΡ! ΠΠΠΠΠ ΡΠΈΠ½Π΅Π³ΠΎ Π²Π΅ΠΊΡΠΎΡΠ° (Π°, Π·Π½Π°ΡΠΈΡ, ΠΈ ΠΌΠ°Π»ΠΈΠ½ΠΎΠ²ΠΎΠ³ΠΎ Π²Π΅ΠΊΡΠΎΡΠ° ) ΡΠΈΡΠ»Π΅Π½Π½ΠΎ ΡΠ°Π²Π½Π° ΠΠΠΠ©ΠΠΠ ΠΏΠ°ΡΠ°Π»Π»Π΅Π»ΠΎΠ³ΡΠ°ΠΌΠΌΠ°, ΠΏΠΎΡΡΡΠΎΠ΅Π½Π½ΠΎΠ³ΠΎ Π½Π° Π²Π΅ΠΊΡΠΎΡΠ°Ρ . ΠΠ° ΡΠΈΡΡΠ½ΠΊΠ΅ Π΄Π°Π½Π½ΡΠΉ ΠΏΠ°ΡΠ°Π»Π»Π΅Π»ΠΎΠ³ΡΠ°ΠΌΠΌ Π·Π°ΡΡΡΠΈΡ ΠΎΠ²Π°Π½ ΡΡΡΠ½ΡΠΌ ΡΠ²Π΅ΡΠΎΠΌ.
ΠΡΠΈΠΌΠ΅ΡΠ°Π½ΠΈΠ΅ : ΡΠ΅ΡΡΡΠΆ ΡΠ²Π»ΡΠ΅ΡΡΡ ΡΡ Π΅ΠΌΠ°ΡΠΈΡΠ΅ΡΠΊΠΈΠΌ, ΠΈ, Π΅ΡΡΠ΅ΡΡΠ²Π΅Π½Π½ΠΎ, Π½ΠΎΠΌΠΈΠ½Π°Π»ΡΠ½Π°Ρ Π΄Π»ΠΈΠ½Π° Π²Π΅ΠΊΡΠΎΡΠ½ΠΎΠ³ΠΎ ΠΏΡΠΎΠΈΠ·Π²Π΅Π΄Π΅Π½ΠΈΡ Π½Π΅ ΡΠ°Π²Π½Π° ΠΏΠ»ΠΎΡΠ°Π΄ΠΈ ΠΏΠ°ΡΠ°Π»Π»Π΅Π»ΠΎΠ³ΡΠ°ΠΌΠΌΠ°.
ΠΡΠΏΠΎΠΌΠΈΠ½Π°Π΅ΠΌ ΠΎΠ΄Π½Ρ ΠΈΠ· Π³Π΅ΠΎΠΌΠ΅ΡΡΠΈΡΠ΅ΡΠΊΠΈΡ ΡΠΎΡΠΌΡΠ»: ΠΏΠ»ΠΎΡΠ°Π΄Ρ ΠΏΠ°ΡΠ°Π»Π»Π΅Π»ΠΎΠ³ΡΠ°ΠΌΠΌΠ° ΡΠ°Π²Π½Π° ΠΏΡΠΎΠΈΠ·Π²Π΅Π΄Π΅Π½ΠΈΡ ΡΠΌΠ΅ΠΆΠ½ΡΡ ΡΡΠΎΡΠΎΠ½ Π½Π° ΡΠΈΠ½ΡΡ ΡΠ³Π»Π° ΠΌΠ΅ΠΆΠ΄Ρ Π½ΠΈΠΌΠΈ . ΠΠΎΡΡΠΎΠΌΡ, ΠΈΡΡ ΠΎΠ΄Ρ ΠΈΠ· Π²ΡΡΠ΅ΡΠΊΠ°Π·Π°Π½Π½ΠΎΠ³ΠΎ, ΡΠΏΡΠ°Π²Π΅Π΄Π»ΠΈΠ²Π° ΡΠΎΡΠΌΡΠ»Π° Π²ΡΡΠΈΡΠ»Π΅Π½ΠΈΡ ΠΠΠΠΠ« Π²Π΅ΠΊΡΠΎΡΠ½ΠΎΠ³ΠΎ ΠΏΡΠΎΠΈΠ·Π²Π΅Π΄Π΅Π½ΠΈΡ:
ΠΠΎΠ΄ΡΡΡΠΊΠΈΠ²Π°Ρ, ΡΡΠΎ Π² ΡΠΎΡΠΌΡΠ»Π΅ ΡΠ΅ΡΡ ΠΈΠ΄ΡΡ ΠΎ ΠΠΠΠΠ Π²Π΅ΠΊΡΠΎΡΠ°, Π° Π½Π΅ ΠΎ ΡΠ°ΠΌΠΎΠΌ Π²Π΅ΠΊΡΠΎΡΠ΅ . ΠΠ°ΠΊΠΎΠ² ΠΏΡΠ°ΠΊΡΠΈΡΠ΅ΡΠΊΠΈΠΉ ΡΠΌΡΡΠ»? Π ΡΠΌΡΡΠ» ΡΠ°ΠΊΠΎΠ², ΡΡΠΎ Π² Π·Π°Π΄Π°ΡΠ°Ρ Π°Π½Π°Π»ΠΈΡΠΈΡΠ΅ΡΠΊΠΎΠΉ Π³Π΅ΠΎΠΌΠ΅ΡΡΠΈΠΈ ΠΏΠ»ΠΎΡΠ°Π΄Ρ ΠΏΠ°ΡΠ°Π»Π»Π΅Π»ΠΎΠ³ΡΠ°ΠΌΠΌΠ° ΡΠ°ΡΡΠΎ Π½Π°Ρ ΠΎΠ΄ΡΡ ΡΠ΅ΡΠ΅Π· ΠΏΠΎΠ½ΡΡΠΈΠ΅ Π²Π΅ΠΊΡΠΎΡΠ½ΠΎΠ³ΠΎ ΠΏΡΠΎΠΈΠ·Π²Π΅Π΄Π΅Π½ΠΈΡ:
ΠΠΎΠ»ΡΡΠΈΠΌ Π²ΡΠΎΡΡΡ Π²Π°ΠΆΠ½ΡΡ ΡΠΎΡΠΌΡΠ»Ρ. ΠΠΈΠ°Π³ΠΎΠ½Π°Π»Ρ ΠΏΠ°ΡΠ°Π»Π»Π΅Π»ΠΎΠ³ΡΠ°ΠΌΠΌΠ° (ΠΊΡΠ°ΡΠ½ΡΠΉ ΠΏΡΠ½ΠΊΡΠΈΡ) Π΄Π΅Π»ΠΈΡ Π΅Π³ΠΎ Π½Π° Π΄Π²Π° ΡΠ°Π²Π½ΡΡ
ΡΡΠ΅ΡΠ³ΠΎΠ»ΡΠ½ΠΈΠΊΠ°. Π‘Π»Π΅Π΄ΠΎΠ²Π°ΡΠ΅Π»ΡΠ½ΠΎ, ΠΏΠ»ΠΎΡΠ°Π΄Ρ ΡΡΠ΅ΡΠ³ΠΎΠ»ΡΠ½ΠΈΠΊΠ°, ΠΏΠΎΡΡΡΠΎΠ΅Π½Π½ΠΎΠ³ΠΎ Π½Π° Π²Π΅ΠΊΡΠΎΡΠ°Ρ
(ΠΊΡΠ°ΡΠ½Π°Ρ ΡΡΡΠΈΡ
ΠΎΠ²ΠΊΠ°), ΠΌΠΎΠΆΠ½ΠΎ Π½Π°ΠΉΡΠΈ ΠΏΠΎ ΡΠΎΡΠΌΡΠ»Π΅:
4) ΠΠ΅ ΠΌΠ΅Π½Π΅Π΅ Π²Π°ΠΆΠ½ΡΠΉ ΡΠ°ΠΊΡ ΡΠΎΡΡΠΎΠΈΡ Π² ΡΠΎΠΌ, ΡΡΠΎ Π²Π΅ΠΊΡΠΎΡ ΠΎΡΡΠΎΠ³ΠΎΠ½Π°Π»Π΅Π½ Π²Π΅ΠΊΡΠΎΡΠ°ΠΌ , ΡΠΎ Π΅ΡΡΡ . Π Π°Π·ΡΠΌΠ΅Π΅ΡΡΡ, ΠΏΡΠΎΡΠΈΠ²ΠΎΠΏΠΎΠ»ΠΎΠΆΠ½ΠΎ Π½Π°ΠΏΡΠ°Π²Π»Π΅Π½Π½ΡΠΉ Π²Π΅ΠΊΡΠΎΡ (ΠΌΠ°Π»ΠΈΠ½ΠΎΠ²Π°Ρ ΡΡΡΠ΅Π»ΠΊΠ°) ΡΠΎΠΆΠ΅ ΠΎΡΡΠΎΠ³ΠΎΠ½Π°Π»Π΅Π½ ΠΈΡΡ ΠΎΠ΄Π½ΡΠΌ Π²Π΅ΠΊΡΠΎΡΠ°ΠΌ .
5) ΠΠ΅ΠΊΡΠΎΡ Π½Π°ΠΏΡΠ°Π²Π»Π΅Π½ ΡΠ°ΠΊ, ΡΡΠΎ Π±Π°Π·ΠΈΡ ΠΈΠΌΠ΅Π΅Ρ ΠΏΡΠ°Π²ΡΡ ΠΎΡΠΈΠ΅Π½ΡΠ°ΡΠΈΡ. ΠΠ° ΡΡΠΎΠΊΠ΅ ΠΎ ΠΏΠ΅ΡΠ΅Ρ
ΠΎΠ΄Π΅ ΠΊ Π½ΠΎΠ²ΠΎΠΌΡ Π±Π°Π·ΠΈΡΡ Ρ Π΄ΠΎΡΡΠ°ΡΠΎΡΠ½ΠΎ ΠΏΠΎΠ΄ΡΠΎΠ±Π½ΠΎ ΡΠ°ΡΡΠΊΠ°Π·Π°Π» ΠΎΠ± ΠΎΡΠΈΠ΅Π½ΡΠ°ΡΠΈΠΈ ΠΏΠ»ΠΎΡΠΊΠΎΡΡΠΈ , ΠΈ ΡΠ΅ΠΉΡΠ°Ρ ΠΌΡ ΡΠ°Π·Π±Π΅ΡΡΠΌΡΡ, ΡΡΠΎ ΡΠ°ΠΊΠΎΠ΅ ΠΎΡΠΈΠ΅Π½ΡΠ°ΡΠΈΡ ΠΏΡΠΎΡΡΡΠ°Π½ΡΡΠ²Π°. ΠΠ±ΡΡΡΠ½ΡΡΡ Π±ΡΠ΄Ρ Π½Π° ΠΏΠ°Π»ΡΡΠ°Ρ
Π²Π°ΡΠ΅ΠΉ ΠΏΡΠ°Π²ΠΎΠΉ ΡΡΠΊΠΈ . ΠΡΡΠ»Π΅Π½Π½ΠΎ ΡΠΎΠ²ΠΌΠ΅ΡΡΠΈΡΠ΅ ΡΠΊΠ°Π·Π°ΡΠ΅Π»ΡΠ½ΡΠΉ ΠΏΠ°Π»Π΅Ρ Ρ Π²Π΅ΠΊΡΠΎΡΠΎΠΌ ΠΈ ΡΡΠ΅Π΄Π½ΠΈΠΉ ΠΏΠ°Π»Π΅Ρ Ρ Π²Π΅ΠΊΡΠΎΡΠΎΠΌ . ΠΠ΅Π·ΡΠΌΡΠ½Π½ΡΠΉ ΠΏΠ°Π»Π΅Ρ ΠΈ ΠΌΠΈΠ·ΠΈΠ½Π΅Ρ ΠΏΡΠΈΠΆΠΌΠΈΡΠ΅ ΠΊ Π»Π°Π΄ΠΎΠ½ΠΈ. Π ΡΠ΅Π·ΡΠ»ΡΡΠ°ΡΠ΅ Π±ΠΎΠ»ΡΡΠΎΠΉ ΠΏΠ°Π»Π΅Ρ β Π²Π΅ΠΊΡΠΎΡΠ½ΠΎΠ΅ ΠΏΡΠΎΠΈΠ·Π²Π΅Π΄Π΅Π½ΠΈΠ΅ Π±ΡΠ΄Π΅Ρ ΡΠΌΠΎΡΡΠ΅ΡΡ Π²Π²Π΅ΡΡ
. ΠΡΠΎ ΠΈ Π΅ΡΡΡ ΠΏΡΠ°Π²ΠΎΠΎΡΠΈΠ΅Π½ΡΠΈΡΠΎΠ²Π°Π½Π½ΡΠΉ Π±Π°Π·ΠΈΡ (Π½Π° ΡΠΈΡΡΠ½ΠΊΠ΅ ΠΈΠΌΠ΅Π½Π½ΠΎ ΠΎΠ½). Π’Π΅ΠΏΠ΅ΡΡ ΠΏΠΎΠΌΠ΅Π½ΡΠΉΡΠ΅ Π²Π΅ΠΊΡΠΎΡΡ (ΡΠΊΠ°Π·Π°ΡΠ΅Π»ΡΠ½ΡΠΉ ΠΈ ΡΡΠ΅Π΄Π½ΠΈΠΉ ΠΏΠ°Π»ΡΡΡ ) ΠΌΠ΅ΡΡΠ°ΠΌΠΈ, Π² ΡΠ΅Π·ΡΠ»ΡΡΠ°ΡΠ΅ Π±ΠΎΠ»ΡΡΠΎΠΉ ΠΏΠ°Π»Π΅Ρ ΡΠ°Π·Π²Π΅ΡΠ½ΡΡΡΡ, ΠΈ Π²Π΅ΠΊΡΠΎΡΠ½ΠΎΠ΅ ΠΏΡΠΎΠΈΠ·Π²Π΅Π΄Π΅Π½ΠΈΠ΅ ΡΠΆΠ΅ Π±ΡΠ΄Π΅Ρ ΡΠΌΠΎΡΡΠ΅ΡΡ Π²Π½ΠΈΠ·. ΠΡΠΎ ΡΠΎΠΆΠ΅ ΠΏΡΠ°Π²ΠΎΠΎΡΠΈΠ΅Π½ΡΠΈΡΠΎΠ²Π°Π½Π½ΡΠΉ Π±Π°Π·ΠΈΡ. ΠΠΎΠ·ΠΌΠΎΠΆΠ½ΠΎ, Ρ Π²Π°Ρ Π²ΠΎΠ·Π½ΠΈΠΊ Π²ΠΎΠΏΡΠΎΡ: Π° ΠΊΠ°ΠΊΠΎΠΉ Π±Π°Π·ΠΈΡ ΠΈΠΌΠ΅Π΅Ρ Π»Π΅Π²ΡΡ ΠΎΡΠΈΠ΅Π½ΡΠ°ΡΠΈΡ? Β«ΠΡΠΈΡΠ²ΠΎΠΉΡΠ΅Β» ΡΠ΅ΠΌ ΠΆΠ΅ ΠΏΠ°Π»ΡΡΠ°ΠΌ Π»Π΅Π²ΠΎΠΉ ΡΡΠΊΠΈ Π²Π΅ΠΊΡΠΎΡΡ , ΠΈ ΠΏΠΎΠ»Π£ΡΠΈΡΠ΅ Π»Π΅Π²ΡΠΉ Π±Π°Π·ΠΈΡ ΠΈ Π»Π΅Π²ΡΡ ΠΎΡΠΈΠ΅Π½ΡΠ°ΡΠΈΡ ΠΏΡΠΎΡΡΡΠ°Π½ΡΡΠ²Π° (Π² ΡΡΠΎΠΌ ΡΠ»ΡΡΠ°Π΅ Π±ΠΎΠ»ΡΡΠΎΠΉ ΠΏΠ°Π»Π΅Ρ ΡΠ°ΡΠΏΠΎΠ»ΠΎΠΆΠΈΡΡΡ ΠΏΠΎ Π½Π°ΠΏΡΠ°Π²Π»Π΅Π½ΠΈΡ Π½ΠΈΠΆΠ½Π΅Π³ΠΎ Π²Π΅ΠΊΡΠΎΡΠ°) . ΠΠ±ΡΠ°Π·Π½ΠΎ Π³ΠΎΠ²ΠΎΡΡ, Π΄Π°Π½Π½ΡΠ΅ Π±Π°Π·ΠΈΡΡ Β«Π·Π°ΠΊΡΡΡΠΈΠ²Π°ΡΡΒ» ΠΈΠ»ΠΈ ΠΎΡΠΈΠ΅Π½ΡΠΈΡΡΡΡ ΠΏΡΠΎΡΡΡΠ°Π½ΡΡΠ²ΠΎ Π² ΡΠ°Π·Π½ΡΠ΅ ΡΡΠΎΡΠΎΠ½Ρ. Π ΡΡΠΎ ΠΏΠΎΠ½ΡΡΠΈΠ΅ Π½Π΅ ΡΠ»Π΅Π΄ΡΠ΅Ρ ΡΡΠΈΡΠ°ΡΡ ΡΠ΅ΠΌ-ΡΠΎ Π½Π°Π΄ΡΠΌΠ°Π½Π½ΡΠΌ ΠΈΠ»ΠΈ Π°Π±ΡΡΡΠ°ΠΊΡΠ½ΡΠΌ β ΡΠ°ΠΊ, Π½Π°ΠΏΡΠΈΠΌΠ΅Ρ, ΠΎΡΠΈΠ΅Π½ΡΠ°ΡΠΈΡ ΠΏΡΠΎΡΡΡΠ°Π½ΡΡΠ²Π° ΠΌΠ΅Π½ΡΠ΅Ρ ΡΠ°ΠΌΠΎΠ΅ ΠΎΠ±ΡΡΠ½ΠΎΠ΅ Π·Π΅ΡΠΊΠ°Π»ΠΎ, ΠΈ Π΅ΡΠ»ΠΈ Β«Π²ΡΡΠ°ΡΠΈΡΡ ΠΎΡΡΠ°ΠΆΡΠ½Π½ΡΠΉ ΠΎΠ±ΡΠ΅ΠΊΡ ΠΈΠ· Π·Π°Π·Π΅ΡΠΊΠ°Π»ΡΡΒ», ΡΠΎ Π΅Π³ΠΎ Π² ΠΎΠ±ΡΠ΅ΠΌ ΡΠ»ΡΡΠ°Π΅ Π½Π΅ ΡΠ΄Π°ΡΡΡΡ ΡΠΎΠ²ΠΌΠ΅ΡΡΠΈΡΡ Ρ Β«ΠΎΡΠΈΠ³ΠΈΠ½Π°Π»ΠΎΠΌΒ». ΠΡΡΠ°ΡΠΈ, ΠΏΠΎΠ΄Π½Π΅ΡΠΈΡΠ΅ ΠΊ Π·Π΅ΡΠΊΠ°Π»Ρ ΡΡΠΈ ΠΏΠ°Π»ΡΡΠ° ΠΈ ΠΏΡΠΎΠ°Π½Π°Π»ΠΈΠ·ΠΈΡΡΠΉΡΠ΅ ΠΎΡΡΠ°ΠΆΠ΅Π½ΠΈΠ΅;-)
β¦ΠΊΠ°ΠΊ Π²ΡΡ-ΡΠ°ΠΊΠΈ Ρ ΠΎΡΠΎΡΠΎ, ΡΡΠΎ Π²Ρ ΡΠ΅ΠΏΠ΅ΡΡ Π·Π½Π°Π΅ΡΠ΅ ΠΎ ΠΏΡΠ°Π²ΠΎ- ΠΈ Π»Π΅Π²ΠΎΠΎΡΠΈΠ΅Π½ΡΠΈΡΠΎΠ²Π°Π½Π½ΡΡ Π±Π°Π·ΠΈΡΠ°Ρ , ΠΈΠ±ΠΎ ΡΡΡΠ°ΡΠ½Π« Π²ΡΡΠΊΠ°Π·ΡΠ²Π°Π½ΠΈΡ Π½Π΅ΠΊΠΎΡΠΎΡΡΡ Π»Π΅ΠΊΡΠΎΡΠΎΠ² ΠΎ ΡΠΌΠ΅Π½Π΅ ΠΎΡΠΈΠ΅Π½ΡΠ°ΡΠΈΠΈ =)
ΠΠ΅ΠΊΡΠΎΡΠ½ΠΎΠ΅ ΠΏΡΠΎΠΈΠ·Π²Π΅Π΄Π΅Π½ΠΈΠ΅ ΠΊΠΎΠ»Π»ΠΈΠ½Π΅Π°ΡΠ½ΡΡ Π²Π΅ΠΊΡΠΎΡΠΎΠ²ΠΠΏΡΠ΅Π΄Π΅Π»Π΅Π½ΠΈΠ΅ ΠΏΠΎΠ΄ΡΠΎΠ±Π½ΠΎ ΡΠ°Π·ΠΎΠ±ΡΠ°Π½ΠΎ, ΠΎΡΡΠ°Π»ΠΎΡΡ Π²ΡΡΡΠ½ΠΈΡΡ, ΡΡΠΎ ΠΏΡΠΎΠΈΡΡ
ΠΎΠ΄ΠΈΡ, ΠΊΠΎΠ³Π΄Π° Π²Π΅ΠΊΡΠΎΡΡ ΠΊΠΎΠ»Π»ΠΈΠ½Π΅Π°ΡΠ½Ρ. ΠΡΠ»ΠΈ Π²Π΅ΠΊΡΠΎΡΡ ΠΊΠΎΠ»Π»ΠΈΠ½Π΅Π°ΡΠ½Ρ, ΡΠΎ ΠΈΡ
ΠΌΠΎΠΆΠ½ΠΎ ΡΠ°ΡΠΏΠΎΠ»ΠΎΠΆΠΈΡΡ Π½Π° ΠΎΠ΄Π½ΠΎΠΉ ΠΏΡΡΠΌΠΎΠΉ ΠΈ Π½Π°Ρ ΠΏΠ°ΡΠ°Π»Π»Π΅Π»ΠΎΠ³ΡΠ°ΠΌΠΌ ΡΠΎΠΆΠ΅ Β«ΡΠΊΠ»Π°Π΄ΡΠ²Π°Π΅ΡΡΡΒ» Π² ΠΎΠ΄Π½Ρ ΠΏΡΡΠΌΡΡ. ΠΠ»ΠΎΡΠ°Π΄Ρ ΡΠ°ΠΊΠΎΠ³ΠΎ, ΠΊΠ°ΠΊ Π³ΠΎΠ²ΠΎΡΡΡ ΠΌΠ°ΡΠ΅ΠΌΠ°ΡΠΈΠΊΠΈ, Π²ΡΡΠΎΠΆΠ΄Π΅Π½Π½ΠΎΠ³ΠΎ ΠΏΠ°ΡΠ°Π»Π»Π΅Π»ΠΎΠ³ΡΠ°ΠΌΠΌΠ° ΡΠ°Π²Π½Π° Π½ΡΠ»Ρ. ΠΡΠΎ ΠΆΠ΅ ΡΠ»Π΅Π΄ΡΠ΅Ρ ΠΈ ΠΈΠ· ΡΠΎΡΠΌΡΠ»Ρ β ΡΠΈΠ½ΡΡ Π½ΡΠ»Ρ ΠΈΠ»ΠΈ 180-ΡΠΈ Π³ΡΠ°Π΄ΡΡΠΎΠ² ΡΠ°Π²Π΅Π½ Π½ΡΠ»Ρ, Π° Π·Π½Π°ΡΠΈΡ, ΠΈ ΠΏΠ»ΠΎΡΠ°Π΄Ρ Π½ΡΠ»Π΅Π²Π°Ρ
Π’Π°ΠΊΠΈΠΌ ΠΎΠ±ΡΠ°Π·ΠΎΠΌ, Π΅ΡΠ»ΠΈ , ΡΠΎ ΠΈ . ΠΠ±ΡΠ°ΡΠΈΡΠ΅ Π²Π½ΠΈΠΌΠ°Π½ΠΈΠ΅, ΡΡΠΎ ΡΠ°ΠΌΠΎ Π²Π΅ΠΊΡΠΎΡΠ½ΠΎΠ΅ ΠΏΡΠΎΠΈΠ·Π²Π΅Π΄Π΅Π½ΠΈΠ΅ ΡΠ°Π²Π½ΠΎ Π½ΡΠ»Π΅Π²ΠΎΠΌΡ Π²Π΅ΠΊΡΠΎΡΡ, Π½ΠΎ Π½Π° ΠΏΡΠ°ΠΊΡΠΈΠΊΠ΅ ΡΡΠΈΠΌ ΡΠ°ΡΡΠΎ ΠΏΡΠ΅Π½Π΅Π±ΡΠ΅Π³Π°ΡΡ ΠΈ ΠΏΠΈΡΡΡ, ΡΡΠΎ ΠΎΠ½ΠΎ ΡΠΎΠΆΠ΅ ΡΠ°Π²Π½ΠΎ Π½ΡΠ»Ρ.
Π§Π°ΡΡΠ½ΡΠΉ ΡΠ»ΡΡΠ°ΠΉ β Π²Π΅ΠΊΡΠΎΡΠ½ΠΎΠ΅ ΠΏΡΠΎΠΈΠ·Π²Π΅Π΄Π΅Π½ΠΈΠ΅ Π²Π΅ΠΊΡΠΎΡΠ° Π½Π° ΡΠ°ΠΌΠΎΠ³ΠΎ ΡΠ΅Π±Ρ:
Π‘ ΠΏΠΎΠΌΠΎΡΡΡ Π²Π΅ΠΊΡΠΎΡΠ½ΠΎΠ³ΠΎ ΠΏΡΠΎΠΈΠ·Π²Π΅Π΄Π΅Π½ΠΈΡ ΠΌΠΎΠΆΠ½ΠΎ ΠΏΡΠΎΠ²Π΅ΡΡΡΡ ΠΊΠΎΠ»Π»ΠΈΠ½Π΅Π°ΡΠ½ΠΎΡΡΡ ΡΡΡΡ ΠΌΠ΅ΡΠ½ΡΡ Π²Π΅ΠΊΡΠΎΡΠΎΠ², ΠΈ Π΄Π°Π½Π½ΡΡ Π·Π°Π΄Π°ΡΡ ΡΡΠ΅Π΄ΠΈ ΠΏΡΠΎΡΠΈΡ ΠΌΡ ΡΠΎΠΆΠ΅ ΡΠ°Π·Π±Π΅ΡΡΠΌ.
ΠΠ»Ρ ΡΠ΅ΡΠ΅Π½ΠΈΡ ΠΏΡΠ°ΠΊΡΠΈΡΠ΅ΡΠΊΠΈΡ ΠΏΡΠΈΠΌΠ΅ΡΠΎΠ² ΠΌΠΎΠΆΠ΅Ρ ΠΏΠΎΡΡΠ΅Π±ΠΎΠ²Π°ΡΡΡΡ ΡΡΠΈΠ³ΠΎΠ½ΠΎΠΌΠ΅ΡΡΠΈΡΠ΅ΡΠΊΠ°Ρ ΡΠ°Π±Π»ΠΈΡΠ° , ΡΡΠΎΠ±Ρ Π½Π°Ρ ΠΎΠ΄ΠΈΡΡ ΠΏΠΎ Π½Π΅ΠΉ Π·Π½Π°ΡΠ΅Π½ΠΈΡ ΡΠΈΠ½ΡΡΠΎΠ².
ΠΡ ΡΡΠΎ ΠΆΠ΅, ΡΠ°Π·ΠΆΠΈΠ³Π°Π΅ΠΌ ΠΎΠ³ΠΎΠ½Ρ:
ΠΡΠΈΠΌΠ΅Ρ 1
Π°) ΠΠ°ΠΉΡΠΈ Π΄Π»ΠΈΠ½Ρ Π²Π΅ΠΊΡΠΎΡΠ½ΠΎΠ³ΠΎ ΠΏΡΠΎΠΈΠ·Π²Π΅Π΄Π΅Π½ΠΈΡ Π²Π΅ΠΊΡΠΎΡΠΎΠ² , Π΅ΡΠ»ΠΈ
Π±) ΠΠ°ΠΉΡΠΈ ΠΏΠ»ΠΎΡΠ°Π΄Ρ ΠΏΠ°ΡΠ°Π»Π»Π΅Π»ΠΎΠ³ΡΠ°ΠΌΠΌΠ°, ΠΏΠΎΡΡΡΠΎΠ΅Π½Π½ΠΎΠ³ΠΎ Π½Π° Π²Π΅ΠΊΡΠΎΡΠ°Ρ , Π΅ΡΠ»ΠΈ
Π Π΅ΡΠ΅Π½ΠΈΠ΅ : ΠΠ΅Ρ, ΡΡΠΎ Π½Π΅ ΠΎΠΏΠ΅ΡΠ°ΡΠΊΠ°, ΠΈΡΡ
ΠΎΠ΄Π½ΡΠ΅ Π΄Π°Π½Π½ΡΠ΅ Π² ΠΏΡΠ½ΠΊΡΠ°Ρ
ΡΡΠ»ΠΎΠ²ΠΈΡ Ρ Π½Π°ΠΌΠ΅ΡΠ΅Π½Π½ΠΎ ΡΠ΄Π΅Π»Π°Π» ΠΎΠ΄ΠΈΠ½Π°ΠΊΠΎΠ²ΡΠΌΠΈ. ΠΠΎΡΠΎΠΌΡ ΡΡΠΎ ΠΎΡΠΎΡΠΌΠ»Π΅Π½ΠΈΠ΅ ΡΠ΅ΡΠ΅Π½ΠΈΠΉ Π±ΡΠ΄Π΅Ρ ΠΎΡΠ»ΠΈΡΠ°ΡΡΡΡ!
Π°) ΠΠΎ ΡΡΠ»ΠΎΠ²ΠΈΡ ΡΡΠ΅Π±ΡΠ΅ΡΡΡ Π½Π°ΠΉΡΠΈ Π΄Π»ΠΈΠ½Ρ Π²Π΅ΠΊΡΠΎΡΠ° (Π²Π΅ΠΊΡΠΎΡΠ½ΠΎΠ³ΠΎ ΠΏΡΠΎΠΈΠ·Π²Π΅Π΄Π΅Π½ΠΈΡ). ΠΠΎ ΡΠΎΠΎΡΠ²Π΅ΡΡΡΠ²ΡΡΡΠ΅ΠΉ ΡΠΎΡΠΌΡΠ»Π΅:
ΠΡΠ²Π΅Ρ :
ΠΠΎΠ»Ρ ΡΠΊΠΎΡΠΎ ΡΠΏΡΠ°ΡΠΈΠ²Π°Π»ΠΎΡΡ ΠΎ Π΄Π»ΠΈΠ½Π΅, ΡΠΎ Π² ΠΎΡΠ²Π΅ΡΠ΅ ΡΠΊΠ°Π·ΡΠ²Π°Π΅ΠΌ ΡΠ°Π·ΠΌΠ΅ΡΠ½ΠΎΡΡΡ β Π΅Π΄ΠΈΠ½ΠΈΡΡ.
Π±) ΠΠΎ ΡΡΠ»ΠΎΠ²ΠΈΡ ΡΡΠ΅Π±ΡΠ΅ΡΡΡ Π½Π°ΠΉΡΠΈ ΠΏΠ»ΠΎΡΠ°Π΄Ρ ΠΏΠ°ΡΠ°Π»Π»Π΅Π»ΠΎΠ³ΡΠ°ΠΌΠΌΠ°, ΠΏΠΎΡΡΡΠΎΠ΅Π½Π½ΠΎΠ³ΠΎ Π½Π° Π²Π΅ΠΊΡΠΎΡΠ°Ρ
. ΠΠ»ΠΎΡΠ°Π΄Ρ Π΄Π°Π½Π½ΠΎΠ³ΠΎ ΠΏΠ°ΡΠ°Π»Π»Π΅Π»ΠΎΠ³ΡΠ°ΠΌΠΌΠ° ΡΠΈΡΠ»Π΅Π½Π½ΠΎ ΡΠ°Π²Π½Π° Π΄Π»ΠΈΠ½Π΅ Π²Π΅ΠΊΡΠΎΡΠ½ΠΎΠ³ΠΎ ΠΏΡΠΎΠΈΠ·Π²Π΅Π΄Π΅Π½ΠΈΡ:
ΠΡΠ²Π΅Ρ :
ΠΠ±ΡΠ°ΡΠΈΡΠ΅ Π²Π½ΠΈΠΌΠ°Π½ΠΈΠ΅, ΡΡΠΎ Π² ΠΎΡΠ²Π΅ΡΠ΅ ΠΎ Π²Π΅ΠΊΡΠΎΡΠ½ΠΎΠΌ ΠΏΡΠΎΠΈΠ·Π²Π΅Π΄Π΅Π½ΠΈΠΈ ΡΠ΅ΡΠΈ Π½Π΅ ΠΈΠ΄ΡΡ Π²ΠΎΠΎΠ±ΡΠ΅, Π½Π°Ρ ΡΠΏΡΠ°ΡΠΈΠ²Π°Π»ΠΈ ΠΎ ΠΏΠ»ΠΎΡΠ°Π΄ΠΈ ΡΠΈΠ³ΡΡΡ , ΡΠΎΠΎΡΠ²Π΅ΡΡΡΠ²Π΅Π½Π½ΠΎ, ΡΠ°Π·ΠΌΠ΅ΡΠ½ΠΎΡΡΡ β ΠΊΠ²Π°Π΄ΡΠ°ΡΠ½ΡΠ΅ Π΅Π΄ΠΈΠ½ΠΈΡΡ.
ΠΡΠ΅Π³Π΄Π° ΡΠΌΠΎΡΡΠΈΠΌ, Π§Π’Π ΡΡΠ΅Π±ΡΠ΅ΡΡΡ Π½Π°ΠΉΡΠΈ ΠΏΠΎ ΡΡΠ»ΠΎΠ²ΠΈΡ, ΠΈ, ΠΈΡΡ
ΠΎΠ΄Ρ ΠΈΠ· ΡΡΠΎΠ³ΠΎ, ΡΠΎΡΠΌΡΠ»ΠΈΡΡΠ΅ΠΌ ΡΡΡΠΊΠΈΠΉ ΠΎΡΠ²Π΅Ρ. ΠΠΎΠΆΠ΅Ρ ΠΏΠΎΠΊΠ°Π·Π°ΡΡΡΡ Π±ΡΠΊΠ²ΠΎΠ΅Π΄ΡΡΠ²ΠΎΠΌ, Π½ΠΎ Π±ΡΠΊΠ²ΠΎΠ΅Π΄ΠΎΠ² ΡΡΠ΅Π΄ΠΈ ΠΏΡΠ΅ΠΏΠΎΠ΄Π°Π²Π°ΡΠ΅Π»Π΅ΠΉ Ρ
Π²Π°ΡΠ°Π΅Ρ, ΠΈ Π·Π°Π΄Π°Π½ΠΈΠ΅ Ρ Ρ
ΠΎΡΠΎΡΠΈΠΌΠΈ ΡΠ°Π½ΡΠ°ΠΌΠΈ Π²Π΅ΡΠ½ΡΡΡΡ Π½Π° Π΄ΠΎΡΠ°Π±ΠΎΡΠΊΡ. Π₯ΠΎΡΡ ΡΡΠΎ Π½Π΅ ΠΎΡΠΎΠ±ΠΎ Π½Π°ΡΡΠ½ΡΡΠ°Ρ ΠΏΡΠΈΠ΄ΠΈΡΠΊΠ° β Π΅ΡΠ»ΠΈ ΠΎΡΠ²Π΅Ρ Π½Π΅ΠΊΠΎΡΡΠ΅ΠΊΡΠ΅Π½, ΡΠΎ ΡΠΊΠ»Π°Π΄ΡΠ²Π°Π΅ΡΡΡ Π²ΠΏΠ΅ΡΠ°ΡΠ»Π΅Π½ΠΈΠ΅, ΡΡΠΎ ΡΠ΅Π»ΠΎΠ²Π΅ΠΊ Π½Π΅ ΡΠ°Π·Π±ΠΈΡΠ°Π΅ΡΡΡ Π² ΠΏΡΠΎΡΡΡΡ
Π²Π΅ΡΠ°Ρ
ΠΈ/ΠΈΠ»ΠΈ Π½Π΅ Π²Π½ΠΈΠΊ Π² ΡΡΡΡ Π·Π°Π΄Π°Π½ΠΈΡ. ΠΡΠΎΡ ΠΌΠΎΠΌΠ΅Π½Ρ Π²ΡΠ΅Π³Π΄Π° Π½ΡΠΆΠ½ΠΎ Π΄Π΅ΡΠΆΠ°ΡΡ Π½Π° ΠΊΠΎΠ½ΡΡΠΎΠ»Π΅, ΡΠ΅ΡΠ°Ρ Π»ΡΠ±ΡΡ Π·Π°Π΄Π°ΡΡ ΠΏΠΎ Π²ΡΡΡΠ΅ΠΉ ΠΌΠ°ΡΠ΅ΠΌΠ°ΡΠΈΠΊΠ΅, Π΄Π° ΠΈ ΠΏΠΎ Π΄ΡΡΠ³ΠΈΠΌ ΠΏΡΠ΅Π΄ΠΌΠ΅ΡΠ°ΠΌ ΡΠΎΠΆΠ΅.
ΠΡΠ΄Π° ΠΏΠΎΠ΄Π΅Π²Π°Π»Π°ΡΡ Π±ΠΎΠ»ΡΡΠ°Ρ Π±ΡΠΊΠΎΠ²ΠΊΠ° Β«ΡΠ½Β»? Π ΠΏΡΠΈΠ½ΡΠΈΠΏΠ΅, Π΅Ρ ΠΌΠΎΠΆΠ½ΠΎ Π±ΡΠ»ΠΎ Π΄ΠΎΠΏΠΎΠ»Π½ΠΈΡΠ΅Π»ΡΠ½ΠΎ ΠΏΡΠΈΠ»Π΅ΠΏΠΈΡΡ Π² ΡΠ΅ΡΠ΅Π½ΠΈΠ΅, Π½ΠΎ Π² ΡΠ΅Π»ΡΡ ΡΠΎΠΊΡΠ°ΡΠΈΡΡ Π·Π°ΠΏΠΈΡΡ, Ρ ΡΡΠΎΠ³ΠΎ Π½Π΅ ΡΠ΄Π΅Π»Π°Π». ΠΠ°Π΄Π΅ΡΡΡ, Π²ΡΠ΅ΠΌ ΠΏΠΎΠ½ΡΡΠ½ΠΎ, ΡΡΠΎ ΠΈ β ΡΡΠΎ ΠΎΠ±ΠΎΠ·Π½Π°ΡΠ΅Π½ΠΈΠ΅ ΠΎΠ΄Π½ΠΎΠ³ΠΎ ΠΈ ΡΠΎΠ³ΠΎ ΠΆΠ΅.
ΠΠΎΠΏΡΠ»ΡΡΠ½ΡΠΉ ΠΏΡΠΈΠΌΠ΅Ρ Π΄Π»Ρ ΡΠ°ΠΌΠΎΡΡΠΎΡΡΠ΅Π»ΡΠ½ΠΎΠ³ΠΎ ΡΠ΅ΡΠ΅Π½ΠΈΡ:
ΠΡΠΈΠΌΠ΅Ρ 2
ΠΠ°ΠΉΡΠΈ ΠΏΠ»ΠΎΡΠ°Π΄Ρ ΡΡΠ΅ΡΠ³ΠΎΠ»ΡΠ½ΠΈΠΊΠ°, ΠΏΠΎΡΡΡΠΎΠ΅Π½Π½ΠΎΠ³ΠΎ Π½Π° Π²Π΅ΠΊΡΠΎΡΠ°Ρ , Π΅ΡΠ»ΠΈ
Π€ΠΎΡΠΌΡΠ»Π° Π½Π°Ρ ΠΎΠΆΠ΄Π΅Π½ΠΈΡ ΠΏΠ»ΠΎΡΠ°Π΄ΠΈ ΡΡΠ΅ΡΠ³ΠΎΠ»ΡΠ½ΠΈΠΊΠ° ΡΠ΅ΡΠ΅Π· Π²Π΅ΠΊΡΠΎΡΠ½ΠΎΠ΅ ΠΏΡΠΎΠΈΠ·Π²Π΅Π΄Π΅Π½ΠΈΠ΅ Π΄Π°Π½Π° Π² ΠΊΠΎΠΌΠΌΠ΅Π½ΡΠ°ΡΠΈΡΡ ΠΊ ΠΎΠΏΡΠ΅Π΄Π΅Π»Π΅Π½ΠΈΡ. Π Π΅ΡΠ΅Π½ΠΈΠ΅ ΠΈ ΠΎΡΠ²Π΅Ρ Π² ΠΊΠΎΠ½ΡΠ΅ ΡΡΠΎΠΊΠ°.
ΠΠ° ΠΏΡΠ°ΠΊΡΠΈΠΊΠ΅ Π·Π°Π΄Π°ΡΠ° Π΄Π΅ΠΉΡΡΠ²ΠΈΡΠ΅Π»ΡΠ½ΠΎ ΠΎΡΠ΅Π½Ρ ΡΠ°ΡΠΏΡΠΎΡΡΡΠ°Π½Π΅Π½Π°, ΡΡΠ΅ΡΠ³ΠΎΠ»ΡΠ½ΠΈΠΊΠ°ΠΌΠΈ Π²ΠΎΠΎΠ±ΡΠ΅ ΠΌΠΎΠ³ΡΡ Π·Π°ΠΌΡΡΠΈΡΡ.
ΠΠ»Ρ ΡΠ΅ΡΠ΅Π½ΠΈΡ Π΄ΡΡΠ³ΠΈΡ Π·Π°Π΄Π°Ρ Π½Π°ΠΌ ΠΏΠΎΠ½Π°Π΄ΠΎΠ±ΡΡΡΡ:
Π‘Π²ΠΎΠΉΡΡΠ²Π° Π²Π΅ΠΊΡΠΎΡΠ½ΠΎΠ³ΠΎ ΠΏΡΠΎΠΈΠ·Π²Π΅Π΄Π΅Π½ΠΈΡ Π²Π΅ΠΊΡΠΎΡΠΎΠ²ΠΠ΅ΠΊΠΎΡΠΎΡΡΠ΅ ΡΠ²ΠΎΠΉΡΡΠ²Π° Π²Π΅ΠΊΡΠΎΡΠ½ΠΎΠ³ΠΎ ΠΏΡΠΎΠΈΠ·Π²Π΅Π΄Π΅Π½ΠΈΡ ΠΌΡ ΡΠΆΠ΅ ΡΠ°ΡΡΠΌΠΎΡΡΠ΅Π»ΠΈ, ΡΠ΅ΠΌ Π½Π΅ ΠΌΠ΅Π½Π΅Π΅, Ρ ΠΈΡ Π²ΠΊΠ»ΡΡΡ Π² Π΄Π°Π½Π½ΡΠΉ ΡΠΏΠΈΡΠΎΠΊ.
ΠΠ»Ρ ΠΏΡΠΎΠΈΠ·Π²ΠΎΠ»ΡΠ½ΡΡ Π²Π΅ΠΊΡΠΎΡΠΎΠ² ΠΈ ΠΏΡΠΎΠΈΠ·Π²ΠΎΠ»ΡΠ½ΠΎΠ³ΠΎ ΡΠΈΡΠ»Π° ΡΠΏΡΠ°Π²Π΅Π΄Π»ΠΈΠ²Ρ ΡΠ»Π΅Π΄ΡΡΡΠΈΠ΅ ΡΠ²ΠΎΠΉΡΡΠ²Π°:
1) Π Π΄ΡΡΠ³ΠΈΡ
ΠΈΡΡΠΎΡΠ½ΠΈΠΊΠ°Ρ
ΠΈΠ½ΡΠΎΡΠΌΠ°ΡΠΈΠΈ Π΄Π°Π½Π½ΡΠΉ ΠΏΡΠ½ΠΊΡ ΠΎΠ±ΡΡΠ½ΠΎ Π½Π΅ Π²ΡΠ΄Π΅Π»ΡΡΡ Π² ΡΠ²ΠΎΠΉΡΡΠ²Π°Ρ
, Π½ΠΎ ΠΎΠ½ ΠΎΡΠ΅Π½Ρ Π²Π°ΠΆΠ΅Π½ Π² ΠΏΡΠ°ΠΊΡΠΈΡΠ΅ΡΠΊΠΎΠΌ ΠΏΠ»Π°Π½Π΅. ΠΠΎΡΡΠΎΠΌΡ ΠΏΡΡΡΡ Π±ΡΠ΄Π΅Ρ.
2) β ΡΠ²ΠΎΠΉΡΡΠ²ΠΎ ΡΠΎΠΆΠ΅ ΡΠ°Π·ΠΎΠ±ΡΠ°Π½ΠΎ Π²ΡΡΠ΅, ΠΈΠ½ΠΎΠ³Π΄Π° Π΅Π³ΠΎ Π½Π°Π·ΡΠ²Π°ΡΡ Π°Π½ΡΠΈΠΊΠΎΠΌΠΌΡΡΠ°ΡΠΈΠ²Π½ΠΎΡΡΡΡ . ΠΠ½ΡΠΌΠΈ ΡΠ»ΠΎΠ²Π°ΠΌΠΈ, ΠΏΠΎΡΡΠ΄ΠΎΠΊ Π²Π΅ΠΊΡΠΎΡΠΎΠ² ΠΈΠΌΠ΅Π΅Ρ Π·Π½Π°ΡΠ΅Π½ΠΈΠ΅.
3) β ΡΠΎΡΠ΅ΡΠ°ΡΠ΅Π»ΡΠ½ΡΠ΅ ΠΈΠ»ΠΈ Π°ΡΡΠΎΡΠΈΠ°ΡΠΈΠ²Π½ΡΠ΅ Π·Π°ΠΊΠΎΠ½Ρ Π²Π΅ΠΊΡΠΎΡΠ½ΠΎΠ³ΠΎ ΠΏΡΠΎΠΈΠ·Π²Π΅Π΄Π΅Π½ΠΈΡ. ΠΠΎΠ½ΡΡΠ°Π½ΡΡ Π±Π΅Π·ΠΏΡΠΎΠ±Π»Π΅ΠΌΠ½ΠΎ Π²ΡΠ½ΠΎΡΡΡΡΡ Π·Π° ΠΏΡΠ΅Π΄Π΅Π»Ρ Π²Π΅ΠΊΡΠΎΡΠ½ΠΎΠ³ΠΎ ΠΏΡΠΎΠΈΠ·Π²Π΅Π΄Π΅Π½ΠΈΡ. ΠΠ΅ΠΉΡΡΠ²ΠΈΡΠ΅Π»ΡΠ½ΠΎ, ΡΠ΅Π³ΠΎ ΠΈΠΌ ΡΠ°ΠΌ Π΄Π΅Π»Π°ΡΡ?
4) β ΡΠ°ΡΠΏΡΠ΅Π΄Π΅Π»ΠΈΡΠ΅Π»ΡΠ½ΡΠ΅ ΠΈΠ»ΠΈ Π΄ΠΈΡΡΡΠΈΠ±ΡΡΠΈΠ²Π½ΡΠ΅ Π·Π°ΠΊΠΎΠ½Ρ Π²Π΅ΠΊΡΠΎΡΠ½ΠΎΠ³ΠΎ ΠΏΡΠΎΠΈΠ·Π²Π΅Π΄Π΅Π½ΠΈΡ. Π‘ ΡΠ°ΡΠΊΡΡΡΠΈΠ΅ΠΌ ΡΠΊΠΎΠ±ΠΎΠΊ ΡΠΎΠΆΠ΅ Π½Π΅Ρ ΠΏΡΠΎΠ±Π»Π΅ΠΌ.
Π ΠΊΠ°ΡΠ΅ΡΡΠ²Π΅ Π΄Π΅ΠΌΠΎΠ½ΡΡΡΠ°ΡΠΈΠΈ ΡΠ°ΡΡΠΌΠΎΡΡΠΈΠΌ ΠΊΠΎΡΠΎΡΠ΅Π½ΡΠΊΠΈΠΉ ΠΏΡΠΈΠΌΠ΅Ρ:
ΠΡΠΈΠΌΠ΅Ρ 3
ΠΠ°ΠΉΡΠΈ , Π΅ΡΠ»ΠΈ
Π Π΅ΡΠ΅Π½ΠΈΠ΅: ΠΠΎ ΡΡΠ»ΠΎΠ²ΠΈΡ ΡΠ½ΠΎΠ²Π° ΡΡΠ΅Π±ΡΠ΅ΡΡΡ Π½Π°ΠΉΡΠΈ Π΄Π»ΠΈΠ½Ρ Π²Π΅ΠΊΡΠΎΡΠ½ΠΎΠ³ΠΎ ΠΏΡΠΎΠΈΠ·Π²Π΅Π΄Π΅Π½ΠΈΡ. Π Π°ΡΠΏΠΈΡΠ΅ΠΌ Π½Π°ΡΡ ΠΌΠΈΠ½ΠΈΠ°ΡΡΡΡ:
(1) Π‘ΠΎΠ³Π»Π°ΡΠ½ΠΎ Π°ΡΡΠΎΡΠΈΠ°ΡΠΈΠ²Π½ΡΠΌ Π·Π°ΠΊΠΎΠ½Π°ΠΌ, Π²ΡΠ½ΠΎΡΠΈΠΌ ΠΊΠΎΠ½ΡΡΠ°Π½ΡΡ Π·Π° ΠΏΠ΅ΡΠ΅Π΄Π΅Π»Ρ Π²Π΅ΠΊΡΠΎΡΠ½ΠΎΠ³ΠΎ ΠΏΡΠΎΠΈΠ·Π²Π΅Π΄Π΅Π½ΠΈΡ.
(2) ΠΡΠ½ΠΎΡΠΈΠΌ ΠΊΠΎΠ½ΡΡΠ°Π½ΡΡ Π·Π° ΠΏΡΠ΅Π΄Π΅Π»Ρ ΠΌΠΎΠ΄ΡΠ»Ρ, ΠΏΡΠΈ ΡΡΠΎΠΌ ΠΌΠΎΠ΄ΡΠ»Ρ Β«ΡΡΠ΅Π΄Π°Π΅ΡΒ» Π·Π½Π°ΠΊ Β«ΠΌΠΈΠ½ΡΡΒ». ΠΠ»ΠΈΠ½Π° ΠΆΠ΅ Π½Π΅ ΠΌΠΎΠΆΠ΅Ρ Π±ΡΡΡ ΠΎΡΡΠΈΡΠ°ΡΠ΅Π»ΡΠ½ΠΎΠΉ.
(3) ΠΠ°Π»ΡΠ½Π΅ΠΉΡΠ΅Π΅ ΠΏΠΎΠ½ΡΡΠ½ΠΎ.
ΠΡΠ²Π΅Ρ :
ΠΠΎΡΠ° ΠΏΠΎΠ΄Π±ΡΠΎΡΠΈΡΡ Π΄ΡΠΎΠ² Π² ΠΎΠ³ΠΎΠ½Ρ:
ΠΡΠΈΠΌΠ΅Ρ 4
ΠΡΡΠΈΡΠ»ΠΈΡΡ ΠΏΠ»ΠΎΡΠ°Π΄Ρ ΡΡΠ΅ΡΠ³ΠΎΠ»ΡΠ½ΠΈΠΊΠ°, ΠΏΠΎΡΡΡΠΎΠ΅Π½Π½ΠΎΠ³ΠΎ Π½Π° Π²Π΅ΠΊΡΠΎΡΠ°Ρ , Π΅ΡΠ»ΠΈ
Π Π΅ΡΠ΅Π½ΠΈΠ΅ : ΠΠ»ΠΎΡΠ°Π΄Ρ ΡΡΠ΅ΡΠ³ΠΎΠ»ΡΠ½ΠΈΠΊΠ° Π½Π°ΠΉΠ΄ΡΠΌ ΠΏΠΎ ΡΠΎΡΠΌΡΠ»Π΅ . ΠΠ°Π³Π²ΠΎΠ·Π΄ΠΊΠ° ΡΠΎΡΡΠΎΠΈΡ Π² ΡΠΎΠΌ, ΡΡΠΎ Π²Π΅ΠΊΡΠΎΡΡ Β«ΡΡΒ» ΠΈ Β«Π΄ΡΒ» ΡΠ°ΠΌΠΈ ΠΏΡΠ΅Π΄ΡΡΠ°Π²Π»Π΅Π½Ρ Π² Π²ΠΈΠ΄Π΅ ΡΡΠΌΠΌ Π²Π΅ΠΊΡΠΎΡΠΎΠ². ΠΠ»Π³ΠΎΡΠΈΡΠΌ Π·Π΄Π΅ΡΡ ΡΡΠ°Π½Π΄Π°ΡΡΠ΅Π½ ΠΈ ΡΠ΅ΠΌ-ΡΠΎ Π½Π°ΠΏΠΎΠΌΠΈΠ½Π°Π΅Ρ ΠΏΡΠΈΠΌΠ΅ΡΡ β 3 ΠΈ 4 ΡΡΠΎΠΊΠ° Π‘ΠΊΠ°Π»ΡΡΠ½ΠΎΠ΅ ΠΏΡΠΎΠΈΠ·Π²Π΅Π΄Π΅Π½ΠΈΠ΅ Π²Π΅ΠΊΡΠΎΡΠΎΠ² . Π Π΅ΡΠ΅Π½ΠΈΠ΅ Π΄Π»Ρ ΡΡΠ½ΠΎΡΡΠΈ ΡΠ°Π·ΠΎΠ±ΡΡΠΌ Π½Π° ΡΡΠΈ ΡΡΠ°ΠΏΠ°:
1) ΠΠ° ΠΏΠ΅ΡΠ²ΠΎΠΌ ΡΠ°Π³Π΅ Π²ΡΡΠ°Π·ΠΈΠΌ Π²Π΅ΠΊΡΠΎΡΠ½ΠΎΠ΅ ΠΏΡΠΎΠΈΠ·Π²Π΅Π΄Π΅Π½ΠΈΠ΅ ΡΠ΅ΡΠ΅Π· Π²Π΅ΠΊΡΠΎΡΠ½ΠΎΠ΅ ΠΏΡΠΎΠΈΠ·Π²Π΅Π΄Π΅Π½ΠΈΠ΅ , ΠΏΠΎ ΡΡΡΠΈ, Π²ΡΡΠ°Π·ΠΈΠΌ Π²Π΅ΠΊΡΠΎΡ ΡΠ΅ΡΠ΅Π· Π²Π΅ΠΊΡΠΎΡ . Π Π΄Π»ΠΈΠ½Π°Ρ ΠΏΠΎΠΊΠ° Π½ΠΈ ΡΠ»ΠΎΠ²Π°!
(1) ΠΠΎΠ΄ΡΡΠ°Π²Π»ΡΠ΅ΠΌ Π²ΡΡΠ°ΠΆΠ΅Π½ΠΈΡ Π²Π΅ΠΊΡΠΎΡΠΎΠ² .
(2) ΠΡΠΏΠΎΠ»ΡΠ·ΡΡ Π΄ΠΈΡΡΡΠΈΠ±ΡΡΠΈΠ²Π½ΡΠ΅ Π·Π°ΠΊΠΎΠ½Ρ, ΡΠ°ΡΠΊΡΡΠ²Π°Π΅ΠΌ ΡΠΊΠΎΠ±ΠΊΠΈ ΠΏΠΎ ΠΏΡΠ°Π²ΠΈΠ»Ρ ΡΠΌΠ½ΠΎΠΆΠ΅Π½ΠΈΡ ΠΌΠ½ΠΎΠ³ΠΎΡΠ»Π΅Π½ΠΎΠ².
(3) ΠΡΠΏΠΎΠ»ΡΠ·ΡΡ Π°ΡΡΠΎΡΠΈΠ°ΡΠΈΠ²Π½ΡΠ΅ Π·Π°ΠΊΠΎΠ½Ρ, Π²ΡΠ½ΠΎΡΠΈΠΌ Π²ΡΠ΅ ΠΊΠΎΠ½ΡΡΠ°Π½ΡΡ Π·Π° ΠΏΡΠ΅Π΄Π΅Π»Ρ Π²Π΅ΠΊΡΠΎΡΠ½ΡΡ ΠΏΡΠΎΠΈΠ·Π²Π΅Π΄Π΅Π½ΠΈΠΉ. ΠΡΠΈ ΠΌΠ°Π»ΠΎΠΌΠ°Π»ΡΡΠΊΠΎΠΌ ΠΎΠΏΡΡΠ΅ Π΄Π΅ΠΉΡΡΠ²ΠΈΡ 2 ΠΈ 3 ΠΌΠΎΠΆΠ½ΠΎ Π²ΡΠΏΠΎΠ»Π½ΡΡΡ ΠΎΠ΄Π½ΠΎΠ²ΡΠ΅ΠΌΠ΅Π½Π½ΠΎ.
(4) ΠΠ΅ΡΠ²ΠΎΠ΅ ΠΈ ΠΏΠΎΡΠ»Π΅Π΄Π½Π΅Π΅ ΡΠ»Π°Π³Π°Π΅ΠΌΠΎΠ΅ ΡΠ°Π²Π½ΠΎ Π½ΡΠ»Ρ (Π½ΡΠ»Π΅Π²ΠΎΠΌΡ Π²Π΅ΠΊΡΠΎΡΡ) Π±Π»Π°Π³ΠΎΠ΄Π°ΡΡ ΠΏΡΠΈΡΡΠ½ΠΎΠΌΡ ΡΠ²ΠΎΠΉΡΡΠ²Ρ . ΠΠΎ Π²ΡΠΎΡΠΎΠΌ ΡΠ»Π°Π³Π°Π΅ΠΌΠΎΠΌ ΠΈΡΠΏΠΎΠ»ΡΠ·ΡΠ΅ΠΌ ΡΠ²ΠΎΠΉΡΡΠ²ΠΎ Π°Π½ΡΠΈΠΊΠΎΠΌΠΌΡΡΠ°ΡΠΈΠ²Π½ΠΎΡΡΠΈ Π²Π΅ΠΊΡΠΎΡΠ½ΠΎΠ³ΠΎ ΠΏΡΠΎΠΈΠ·Π²Π΅Π΄Π΅Π½ΠΈΡ:
(5) ΠΡΠΈΠ²ΠΎΠ΄ΠΈΠΌ ΠΏΠΎΠ΄ΠΎΠ±Π½ΡΠ΅ ΡΠ»Π°Π³Π°Π΅ΠΌΡΠ΅.
Π ΡΠ΅Π·ΡΠ»ΡΡΠ°ΡΠ΅ Π²Π΅ΠΊΡΠΎΡ ΠΎΠΊΠ°Π·Π°Π»ΡΡ Π²ΡΡΠ°ΠΆΠ΅Π½ ΡΠ΅ΡΠ΅Π· Π²Π΅ΠΊΡΠΎΡ, ΡΠ΅Π³ΠΎ ΠΈ ΡΡΠ΅Π±ΠΎΠ²Π°Π»ΠΎΡΡ Π΄ΠΎΡΡΠΈΡΡ:
2) ΠΠ° Π²ΡΠΎΡΠΎΠΌ ΡΠ°Π³Π΅ Π½Π°ΠΉΠ΄Π΅ΠΌ Π΄Π»ΠΈΠ½Ρ Π½ΡΠΆΠ½ΠΎΠ³ΠΎ Π½Π°ΠΌ Π²Π΅ΠΊΡΠΎΡΠ½ΠΎΠ³ΠΎ ΠΏΡΠΎΠΈΠ·Π²Π΅Π΄Π΅Π½ΠΈΡ. ΠΠ°Π½Π½ΠΎΠ΅ Π΄Π΅ΠΉΡΡΠ²ΠΈΠ΅ Π½Π°ΠΏΠΎΠΌΠΈΠ½Π°Π΅Ρ ΠΡΠΈΠΌΠ΅Ρ 3:
3) ΠΠ°ΠΉΠ΄ΡΠΌ ΠΏΠ»ΠΎΡΠ°Π΄Ρ ΠΈΡΠΊΠΎΠΌΠΎΠ³ΠΎ ΡΡΠ΅ΡΠ³ΠΎΠ»ΡΠ½ΠΈΠΊΠ°:
ΠΡΠ°ΠΏΡ 2-3 ΡΠ΅ΡΠ΅Π½ΠΈΡ ΠΌΠΎΠΆΠ½ΠΎ Π±ΡΠ»ΠΎ ΠΎΡΠΎΡΠΌΠΈΡΡ ΠΈ ΠΎΠ΄Π½ΠΎΠΉ ΡΡΡΠΎΠΊΠΎΠΉ.
ΠΡΠ²Π΅Ρ :
Π Π°ΡΡΠΌΠΎΡΡΠ΅Π½Π½Π°Ρ Π·Π°Π΄Π°ΡΠ° Π΄ΠΎΡΡΠ°ΡΠΎΡΠ½ΠΎ ΡΠ°ΡΠΏΡΠΎΡΡΡΠ°Π½Π΅Π½Π° Π² ΠΊΠΎΠ½ΡΡΠΎΠ»ΡΠ½ΡΡ ΡΠ°Π±ΠΎΡΠ°Ρ , Π²ΠΎΡ ΠΏΡΠΈΠΌΠ΅Ρ Π΄Π»Ρ ΡΠ°ΠΌΠΎΡΡΠΎΡΡΠ΅Π»ΡΠ½ΠΎΠ³ΠΎ ΡΠ΅ΡΠ΅Π½ΠΈΡ:
ΠΡΠΈΠΌΠ΅Ρ 5
ΠΠ°ΠΉΡΠΈ , Π΅ΡΠ»ΠΈ
ΠΡΠ°ΡΠΊΠΎΠ΅ ΡΠ΅ΡΠ΅Π½ΠΈΠ΅ ΠΈ ΠΎΡΠ²Π΅Ρ Π² ΠΊΠΎΠ½ΡΠ΅ ΡΡΠΎΠΊΠ°. ΠΠΎΡΠΌΠΎΡΡΠΈΠΌ, Π½Π°ΡΠΊΠΎΠ»ΡΠΊΠΎ Π²Ρ Π±ΡΠ»ΠΈ Π²Π½ΠΈΠΌΠ°ΡΠ΅Π»ΡΠ½Ρ ΠΏΡΠΈ ΠΈΠ·ΡΡΠ΅Π½ΠΈΠΈ ΠΏΡΠ΅Π΄ΡΠ΄ΡΡΠΈΡ ΠΏΡΠΈΠΌΠ΅ΡΠΎΠ²;-)
ΠΠ΅ΠΊΡΠΎΡΠ½ΠΎΠ΅ ΠΏΡΠΎΠΈΠ·Π²Π΅Π΄Π΅Π½ΠΈΠ΅ Π²Π΅ΠΊΡΠΎΡΠΎΠ² Π² ΠΊΠΎΠΎΡΠ΄ΠΈΠ½Π°ΡΠ°Ρ, Π·Π°Π΄Π°Π½Π½ΡΡ Π² ΠΎΡΡΠΎΠ½ΠΎΡΠΌΠΈΡΠΎΠ²Π°Π½Π½ΠΎΠΌ Π±Π°Π·ΠΈΡΠ΅ , Π²ΡΡΠ°ΠΆΠ°Π΅ΡΡΡ ΡΠΎΡΠΌΡΠ»ΠΎΠΉ :
Π€ΠΎΡΠΌΡΠ»Π° ΠΈ ΠΏΡΠ°Π²Π΄Π° ΠΏΡΠΎΡΡΠ΅ΡΠΊΠ°Ρ: Π² Π²Π΅ΡΡ
Π½ΡΡ ΡΡΡΠΎΠΊΡ ΠΎΠΏΡΠ΅Π΄Π΅Π»ΠΈΡΠ΅Π»Ρ Π·Π°ΠΏΠΈΡΡΠ²Π°Π΅ΠΌ ΠΊΠΎΠΎΡΠ΄ΠΈΠ½Π°ΡΠ½ΡΠ΅ Π²Π΅ΠΊΡΠΎΡΡ, Π²ΠΎ Π²ΡΠΎΡΡΡ ΠΈ ΡΡΠ΅ΡΡΡ ΡΡΡΠΎΠΊΠΈ Β«ΡΠΊΠ»Π°Π΄ΡΠ²Π°Π΅ΠΌΒ» ΠΊΠΎΠΎΡΠ΄ΠΈΠ½Π°ΡΡ Π²Π΅ΠΊΡΠΎΡΠΎΠ² , ΠΏΡΠΈΡΡΠΌ ΡΠΊΠ»Π°Π΄ΡΠ²Π°Π΅ΠΌ Π² ΡΡΡΠΎΠ³ΠΎΠΌ ΠΏΠΎΡΡΠ΄ΠΊΠ΅ β ΡΠ½Π°ΡΠ°Π»Π° ΠΊΠΎΠΎΡΠ΄ΠΈΠ½Π°ΡΡ Π²Π΅ΠΊΡΠΎΡΠ° Β«Π²ΡΒ», Π·Π°ΡΠ΅ΠΌ ΠΊΠΎΠΎΡΠ΄ΠΈΠ½Π°ΡΡ Π²Π΅ΠΊΡΠΎΡΠ° Β«Π΄ΡΠ±Π»Ρ-Π²ΡΒ». ΠΡΠ»ΠΈ Π²Π΅ΠΊΡΠΎΡΡ Π½ΡΠΆΠ½ΠΎ ΡΠΌΠ½ΠΎΠΆΠΈΡΡ Π² Π΄ΡΡΠ³ΠΎΠΌ ΠΏΠΎΡΡΠ΄ΠΊΠ΅, ΡΠΎ ΠΈ ΡΡΡΠΎΠΊΠΈ ΡΠ»Π΅Π΄ΡΠ΅Ρ ΠΏΠΎΠΌΠ΅Π½ΡΡΡ ΠΌΠ΅ΡΡΠ°ΠΌΠΈ:
ΠΡΠΈΠΌΠ΅Ρ 10
ΠΡΠΎΠ²Π΅ΡΠΈΡΡ, Π±ΡΠ΄ΡΡ Π»ΠΈ ΠΊΠΎΠ»Π»ΠΈΠ½Π΅Π°ΡΠ½Ρ ΡΠ»Π΅Π΄ΡΡΡΠΈΠ΅ Π²Π΅ΠΊΡΠΎΡΡ ΠΏΡΠΎΡΡΡΠ°Π½ΡΡΠ²Π°:
Π°)
Π±)
Π Π΅ΡΠ΅Π½ΠΈΠ΅ : ΠΡΠΎΠ²Π΅ΡΠΊΠ° ΠΎΡΠ½ΠΎΠ²Π°Π½Π° Π½Π° ΠΎΠ΄Π½ΠΎΠΌ ΠΈΠ· ΡΡΠ²Π΅ΡΠΆΠ΄Π΅Π½ΠΈΠΉ Π΄Π°Π½Π½ΠΎΠ³ΠΎ ΡΡΠΎΠΊΠ°: Π΅ΡΠ»ΠΈ Π²Π΅ΠΊΡΠΎΡΡ ΠΊΠΎΠ»Π»ΠΈΠ½Π΅Π°ΡΠ½Ρ, ΡΠΎ ΠΈΡ
Π²Π΅ΠΊΡΠΎΡΠ½ΠΎΠ΅ ΠΏΡΠΎΠΈΠ·Π²Π΅Π΄Π΅Π½ΠΈΠ΅ ΡΠ°Π²Π½ΠΎ Π½ΡΠ»Ρ (Π½ΡΠ»Π΅Π²ΠΎΠΌΡ Π²Π΅ΠΊΡΠΎΡΡ): .
Π°) ΠΠ°ΠΉΠ΄ΡΠΌ Π²Π΅ΠΊΡΠΎΡΠ½ΠΎΠ΅ ΠΏΡΠΎΠΈΠ·Π²Π΅Π΄Π΅Π½ΠΈΠ΅:
Π’Π°ΠΊΠΈΠΌ ΠΎΠ±ΡΠ°Π·ΠΎΠΌ, Π²Π΅ΠΊΡΠΎΡΡ Π½Π΅ ΠΊΠΎΠ»Π»ΠΈΠ½Π΅Π°ΡΠ½Ρ.
Π±) ΠΠ°ΠΉΠ΄ΡΠΌ Π²Π΅ΠΊΡΠΎΡΠ½ΠΎΠ΅ ΠΏΡΠΎΠΈΠ·Π²Π΅Π΄Π΅Π½ΠΈΠ΅:
ΠΡΠ²Π΅Ρ : Π°) Π½Π΅ ΠΊΠΎΠ»Π»ΠΈΠ½Π΅Π°ΡΠ½Ρ, Π±)
ΠΠΎΡ, ΠΏΠΎΠΆΠ°Π»ΡΠΉ, ΠΈ Π²ΡΠ΅ ΠΎΡΠ½ΠΎΠ²Π½ΡΠ΅ ΡΠ²Π΅Π΄Π΅Π½ΠΈΡ ΠΎ Π²Π΅ΠΊΡΠΎΡΠ½ΠΎΠΌ ΠΏΡΠΎΠΈΠ·Π²Π΅Π΄Π΅Π½ΠΈΠΈ Π²Π΅ΠΊΡΠΎΡΠΎΠ².
ΠΠ°Π½Π½ΡΠΉ ΡΠ°Π·Π΄Π΅Π» Π±ΡΠ΄Π΅Ρ Π½Π΅ ΠΎΡΠ΅Π½Ρ Π±ΠΎΠ»ΡΡΠΈΠΌ, ΡΠ°ΠΊ ΠΊΠ°ΠΊ Π·Π°Π΄Π°Ρ, Π³Π΄Π΅ ΠΈΡΠΏΠΎΠ»ΡΠ·ΡΠ΅ΡΡΡ ΡΠΌΠ΅ΡΠ°Π½Π½ΠΎΠ΅ ΠΏΡΠΎΠΈΠ·Π²Π΅Π΄Π΅Π½ΠΈΠ΅ Π²Π΅ΠΊΡΠΎΡΠΎΠ², Π½Π΅ΠΌΠ½ΠΎΠ³ΠΎ. Π€Π°ΠΊΡΠΈΡΠ΅ΡΠΊΠΈ Π²ΡΡ Π±ΡΠ΄Π΅Ρ ΡΠΏΠΈΡΠ°ΡΡΡΡ Π² ΠΎΠΏΡΠ΅Π΄Π΅Π»Π΅Π½ΠΈΠ΅, Π³Π΅ΠΎΠΌΠ΅ΡΡΠΈΡΠ΅ΡΠΊΠΈΠΉ ΡΠΌΡΡΠ» ΠΈ ΠΏΠ°ΡΡ ΡΠ°Π±ΠΎΡΠΈΡ ΡΠΎΡΠΌΡΠ».
Π‘ΠΌΠ΅ΡΠ°Π½Π½ΠΎΠ΅ ΠΏΡΠΎΠΈΠ·Π²Π΅Π΄Π΅Π½ΠΈΠ΅ Π²Π΅ΠΊΡΠΎΡΠΎΠ² β ΡΡΠΎ ΠΏΡΠΎΠΈΠ·Π²Π΅Π΄Π΅Π½ΠΈΠ΅ ΡΡΡΡ
Π²Π΅ΠΊΡΠΎΡΠΎΠ² :
ΠΠΎΡ ΡΠ°ΠΊ Π²ΠΎΡ ΠΎΠ½ΠΈ Π²ΡΡΡΡΠΎΠΈΠ»ΠΈΡΡ ΠΏΠ°ΡΠΎΠ²ΠΎΠ·ΠΈΠΊΠΎΠΌ ΠΈ ΠΆΠ΄ΡΡ, Π½Π΅ Π΄ΠΎΠΆΠ΄ΡΡΡΡ, ΠΊΠΎΠ³Π΄Π° ΠΈΡ Π²ΡΡΠΈΡΠ»ΡΡ.
Π‘Π½Π°ΡΠ°Π»Π° ΠΎΠΏΡΡΡ ΠΎΠΏΡΠ΅Π΄Π΅Π»Π΅Π½ΠΈΠ΅ ΠΈ ΠΊΠ°ΡΡΠΈΠ½ΠΊΠ°:
ΠΠΏΡΠ΅Π΄Π΅Π»Π΅Π½ΠΈΠ΅ : Π‘ΠΌΠ΅ΡΠ°Π½Π½ΡΠΌ ΠΏΡΠΎΠΈΠ·Π²Π΅Π΄Π΅Π½ΠΈΠ΅ΠΌ Π½Π΅ΠΊΠΎΠΌΠΏΠ»Π°Π½Π°ΡΠ½ΡΡ Π²Π΅ΠΊΡΠΎΡΠΎΠ² , Π²Π·ΡΡΡΡ Π² Π΄Π°Π½Π½ΠΎΠΌ ΠΏΠΎΡΡΠ΄ΠΊΠ΅ , Π½Π°Π·ΡΠ²Π°Π΅ΡΡΡ ΠΎΠ±ΡΡΠΌ ΠΏΠ°ΡΠ°Π»Π»Π΅Π»Π΅ΠΏΠΈΠΏΠ΅Π΄Π° , ΠΏΠΎΡΡΡΠΎΠ΅Π½Π½ΠΎΠ³ΠΎ Π½Π° Π΄Π°Π½Π½ΡΡ Π²Π΅ΠΊΡΠΎΡΠ°Ρ , ΡΠ½Π°Π±ΠΆΡΠ½Π½ΡΠΉ Π·Π½Π°ΠΊΠΎΠΌ Β«+Β», Π΅ΡΠ»ΠΈ Π±Π°Π·ΠΈΡ ΠΏΡΠ°Π²ΡΠΉ, ΠΈ Π·Π½Π°ΠΊΠΎΠΌ Β«βΒ», Π΅ΡΠ»ΠΈ Π±Π°Π·ΠΈΡ Π»Π΅Π²ΡΠΉ.
ΠΡΠΏΠΎΠ»Π½ΠΈΠΌ ΡΠΈΡΡΠ½ΠΎΠΊ. ΠΠ΅Π²ΠΈΠ΄ΠΈΠΌΡΠ΅ Π½Π°ΠΌ Π»ΠΈΠ½ΠΈΠΈ ΠΏΡΠΎΡΠ΅ΡΡΠ΅Π½Ρ ΠΏΡΠ½ΠΊΡΠΈΡΠΎΠΌ:
ΠΠΎΠ³ΡΡΠΆΠ°Π΅ΠΌΡΡ Π² ΠΎΠΏΡΠ΅Π΄Π΅Π»Π΅Π½ΠΈΠ΅:
2) ΠΠ΅ΠΊΡΠΎΡΡ Π²Π·ΡΡΡ Π² ΠΎΠΏΡΠ΅Π΄Π΅Π»ΡΠ½Π½ΠΎΠΌ ΠΏΠΎΡΡΠ΄ΠΊΠ΅ , ΡΠΎ Π΅ΡΡΡ ΠΏΠ΅ΡΠ΅ΡΡΠ°Π½ΠΎΠ²ΠΊΠ° Π²Π΅ΠΊΡΠΎΡΠΎΠ² Π² ΠΏΡΠΎΠΈΠ·Π²Π΅Π΄Π΅Π½ΠΈΠΈ , ΠΊΠ°ΠΊ Π²Ρ Π΄ΠΎΠ³Π°Π΄ΡΠ²Π°Π΅ΡΠ΅ΡΡ, Π½Π΅ ΠΏΡΠΎΡ
ΠΎΠ΄ΠΈΡ Π±Π΅Π· ΠΏΠΎΡΠ»Π΅Π΄ΡΡΠ²ΠΈΠΉ.
3) ΠΠ΅ΡΠ΅Π΄ ΡΠ΅ΠΌ, ΠΊΠ°ΠΊ ΠΏΡΠΎΠΊΠΎΠΌΠΌΠ΅Π½ΡΠΈΡΠΎΠ²Π°ΡΡ Π³Π΅ΠΎΠΌΠ΅ΡΡΠΈΡΠ΅ΡΠΊΠΈΠΉ ΡΠΌΡΡΠ», ΠΎΡΠΌΠ΅ΡΡ ΠΎΡΠ΅Π²ΠΈΠ΄Π½ΡΠΉ ΡΠ°ΠΊΡ: ΡΠΌΠ΅ΡΠ°Π½Π½ΠΎΠ΅ ΠΏΡΠΎΠΈΠ·Π²Π΅Π΄Π΅Π½ΠΈΠ΅ Π²Π΅ΠΊΡΠΎΡΠΎΠ² ΡΠ²Π»ΡΠ΅ΡΡΡ Π§ΠΠ‘ΠΠΠ : . Π ΡΡΠ΅Π±Π½ΠΎΠΉ Π»ΠΈΡΠ΅ΡΠ°ΡΡΡΠ΅ ΠΎΡΠΎΡΠΌΠ»Π΅Π½ΠΈΠ΅ ΠΌΠΎΠΆΠ΅Ρ Π±ΡΡΡ Π½Π΅ΡΠΊΠΎΠ»ΡΠΊΠΎ Π΄ΡΡΠ³ΠΈΠΌ, Ρ ΠΏΡΠΈΠ²ΡΠΊ ΠΎΠ±ΠΎΠ·Π½Π°ΡΠ°ΡΡ ΡΠΌΠ΅ΡΠ°Π½Π½ΠΎΠ΅ ΠΏΡΠΎΠΈΠ·Π²Π΅Π΄Π΅Π½ΠΈΠ΅ ΡΠ΅ΡΠ΅Π· , Π° ΡΠ΅Π·ΡΠ»ΡΡΠ°Ρ Π²ΡΡΠΈΡΠ»Π΅Π½ΠΈΠΉ Π±ΡΠΊΠ²ΠΎΠΉ Β«ΠΏΡΒ».
ΠΠΎ ΠΎΠΏΡΠ΅Π΄Π΅Π»Π΅Π½ΠΈΡ ΡΠΌΠ΅ΡΠ°Π½Π½ΠΎΠ΅ ΠΏΡΠΎΠΈΠ·Π²Π΅Π΄Π΅Π½ΠΈΠ΅ β ΡΡΠΎ ΠΎΠ±ΡΠ΅ΠΌ ΠΏΠ°ΡΠ°Π»Π»Π΅Π»Π΅ΠΏΠΈΠΏΠ΅Π΄Π° , ΠΏΠΎΡΡΡΠΎΠ΅Π½Π½ΠΎΠ³ΠΎ Π½Π° Π²Π΅ΠΊΡΠΎΡΠ°Ρ (ΡΠΈΠ³ΡΡΠ° ΠΏΡΠΎΡΠ΅ΡΡΠ΅Π½Π° ΠΊΡΠ°ΡΠ½ΡΠΌΠΈ Π²Π΅ΠΊΡΠΎΡΠ°ΠΌΠΈ ΠΈ Π»ΠΈΠ½ΠΈΡΠΌΠΈ ΡΡΡΠ½ΠΎΠ³ΠΎ ΡΠ²Π΅ΡΠ°). Π’ΠΎ Π΅ΡΡΡ, ΡΠΈΡΠ»ΠΎ ΡΠ°Π²Π½ΠΎ ΠΎΠ±ΡΠ΅ΠΌΡ Π΄Π°Π½Π½ΠΎΠ³ΠΎ ΠΏΠ°ΡΠ°Π»Π»Π΅Π»Π΅ΠΏΠΈΠΏΠ΅Π΄Π°.
ΠΡΠΈΠΌΠ΅ΡΠ°Π½ΠΈΠ΅ : ΡΠ΅ΡΡΡΠΆ ΡΠ²Π»ΡΠ΅ΡΡΡ ΡΡ Π΅ΠΌΠ°ΡΠΈΡΠ΅ΡΠΊΠΈΠΌ.
4) ΠΠ΅ Π±ΡΠ΄Π΅ΠΌ Π·Π°Π½ΠΎΠ²ΠΎ ΠΏΠ°ΡΠΈΡΡΡΡ Ρ ΠΏΠΎΠ½ΡΡΠΈΠ΅ΠΌ ΠΎΡΠΈΠ΅Π½ΡΠ°ΡΠΈΠΈ Π±Π°Π·ΠΈΡΠ° ΠΈ ΠΏΡΠΎΡΡΡΠ°Π½ΡΡΠ²Π°. Π‘ΠΌΡΡΠ» Π·Π°ΠΊΠ»ΡΡΠΈΡΠ΅Π»ΡΠ½ΠΎΠΉ ΡΠ°ΡΡΠΈ ΡΠΎΡΡΠΎΠΈΡ Π² ΡΠΎΠΌ, ΡΡΠΎ ΠΊ ΠΎΠ±ΡΡΠΌΡ ΠΌΠΎΠΆΠ΅Ρ Π΄ΠΎΠ±Π°Π²Π»ΡΡΡΡΡ Π·Π½Π°ΠΊ ΠΌΠΈΠ½ΡΡ. ΠΡΠΎΡΡΡΠΌΠΈ ΡΠ»ΠΎΠ²Π°ΠΌΠΈ, ΡΠΌΠ΅ΡΠ°Π½Π½ΠΎΠ΅ ΠΏΡΠΎΠΈΠ·Π²Π΅Π΄Π΅Π½ΠΈΠ΅ ΠΌΠΎΠΆΠ΅Ρ Π±ΡΡΡ ΠΎΡΡΠΈΡΠ°ΡΠ΅Π»ΡΠ½ΡΠΌ: .
ΠΠ΅ΠΏΠΎΡΡΠ΅Π΄ΡΡΠ²Π΅Π½Π½ΠΎ ΠΈΠ· ΠΎΠΏΡΠ΅Π΄Π΅Π»Π΅Π½ΠΈΡ ΡΠ»Π΅Π΄ΡΠ΅Ρ ΡΠΎΡΠΌΡΠ»Π° Π²ΡΡΠΈΡΠ»Π΅Π½ΠΈΡ ΠΎΠ±ΡΠ΅ΠΌΠ° ΠΏΠ°ΡΠ°Π»Π»Π΅Π»Π΅ΠΏΠΈΠΏΠ΅Π΄Π°, ΠΏΠΎΡΡΡΠΎΠ΅Π½Π½ΠΎΠ³ΠΎ Π½Π° Π²Π΅ΠΊΡΠΎΡΠ°Ρ .
Π ΡΡΠΎΠΉ ΡΡΠ°ΡΡΠ΅ ΠΌΡ ΠΏΠΎΠ΄ΡΠΎΠ±Π½ΠΎ ΠΎΡΡΠ°Π½ΠΎΠ²ΠΈΠΌΡΡ Π½Π° ΠΏΠΎΠ½ΡΡΠΈΠΈ Π²Π΅ΠΊΡΠΎΡΠ½ΠΎΠ³ΠΎ ΠΏΡΠΎΠΈΠ·Π²Π΅Π΄Π΅Π½ΠΈΡ Π΄Π²ΡΡ
Π²Π΅ΠΊΡΠΎΡΠΎΠ². ΠΡ Π΄Π°Π΄ΠΈΠΌ Π½Π΅ΠΎΠ±Ρ
ΠΎΠ΄ΠΈΠΌΡΠ΅ ΠΎΠΏΡΠ΅Π΄Π΅Π»Π΅Π½ΠΈΡ, Π·Π°ΠΏΠΈΡΠ΅ΠΌ ΡΠΎΡΠΌΡΠ»Ρ Π΄Π»Ρ Π½Π°Ρ
ΠΎΠΆΠ΄Π΅Π½ΠΈΡ ΠΊΠΎΠΎΡΠ΄ΠΈΠ½Π°Ρ Π²Π΅ΠΊΡΠΎΡΠ½ΠΎΠ³ΠΎ ΠΏΡΠΎΠΈΠ·Π²Π΅Π΄Π΅Π½ΠΈΡ, ΠΏΠ΅ΡΠ΅ΡΠΈΡΠ»ΠΈΠΌ ΠΈ ΠΎΠ±ΠΎΡΠ½ΡΠ΅ΠΌ Π΅Π³ΠΎ ΡΠ²ΠΎΠΉΡΡΠ²Π°. ΠΠΎΡΠ»Π΅ ΡΡΠΎΠ³ΠΎ ΠΎΡΡΠ°Π½ΠΎΠ²ΠΈΠΌΡΡ Π½Π° Π³Π΅ΠΎΠΌΠ΅ΡΡΠΈΡΠ΅ΡΠΊΠΎΠΌ ΡΠΌΡΡΠ»Π΅ Π²Π΅ΠΊΡΠΎΡΠ½ΠΎΠ³ΠΎ ΠΏΡΠΎΠΈΠ·Π²Π΅Π΄Π΅Π½ΠΈΡ Π΄Π²ΡΡ
Π²Π΅ΠΊΡΠΎΡΠΎΠ² ΠΈ ΡΠ°ΡΡΠΌΠΎΡΡΠΈΠΌ ΡΠ΅ΡΠ΅Π½ΠΈΡ ΡΠ°Π·Π»ΠΈΡΠ½ΡΡ
Ρ
Π°ΡΠ°ΠΊΡΠ΅ΡΠ½ΡΡ
ΠΏΡΠΈΠΌΠ΅ΡΠΎΠ².
ΠΠ°Π²ΠΈΠ³Π°ΡΠΈΡ ΠΏΠΎ ΡΡΡΠ°Π½ΠΈΡΠ΅.
ΠΠΏΡΠ΅Π΄Π΅Π»Π΅Π½ΠΈΠ΅ Π²Π΅ΠΊΡΠΎΡΠ½ΠΎΠ³ΠΎ ΠΏΡΠΎΠΈΠ·Π²Π΅Π΄Π΅Π½ΠΈΡ.
ΠΡΠ΅ΠΆΠ΄Π΅ ΡΠ΅ΠΌ Π΄Π°ΡΡ ΠΎΠΏΡΠ΅Π΄Π΅Π»Π΅Π½ΠΈΠ΅ Π²Π΅ΠΊΡΠΎΡΠ½ΠΎΠ³ΠΎ ΠΏΡΠΎΠΈΠ·Π²Π΅Π΄Π΅Π½ΠΈΡ, ΡΠ°Π·Π±Π΅ΡΠ΅ΠΌΡΡ Ρ ΠΎΡΠΈΠ΅Π½ΡΠ°ΡΠΈΠ΅ΠΉ ΡΠΏΠΎΡΡΠ΄ΠΎΡΠ΅Π½Π½ΠΎΠΉ ΡΡΠΎΠΉΠΊΠΈ Π²Π΅ΠΊΡΠΎΡΠΎΠ² Π² ΡΡΠ΅Ρ ΠΌΠ΅ΡΠ½ΠΎΠΌ ΠΏΡΠΎΡΡΡΠ°Π½ΡΡΠ²Π΅.
ΠΡΠ»ΠΎΠΆΠΈΠΌ Π²Π΅ΠΊΡΠΎΡΡ ΠΎΡ ΠΎΠ΄Π½ΠΎΠΉ ΡΠΎΡΠΊΠΈ. Π Π·Π°Π²ΠΈΡΠΈΠΌΠΎΡΡΠΈ ΠΎΡ Π½Π°ΠΏΡΠ°Π²Π»Π΅Π½ΠΈΡ Π²Π΅ΠΊΡΠΎΡΠ° ΡΡΠΎΠΉΠΊΠ° ΠΌΠΎΠΆΠ΅Ρ Π±ΡΡΡ ΠΏΡΠ°Π²ΠΎΠΉ ΠΈΠ»ΠΈ Π»Π΅Π²ΠΎΠΉ. ΠΠΎΡΠΌΠΎΡΡΠΈΠΌ Ρ ΠΊΠΎΠ½ΡΠ° Π²Π΅ΠΊΡΠΎΡΠ° Π½Π° ΡΠΎ, ΠΊΠ°ΠΊ ΠΏΡΠΎΠΈΡΡ ΠΎΠ΄ΠΈΡ ΠΊΡΠ°ΡΡΠ°ΠΉΡΠΈΠΉ ΠΏΠΎΠ²ΠΎΡΠΎΡ ΠΎΡ Π²Π΅ΠΊΡΠΎΡΠ° ΠΊ . ΠΡΠ»ΠΈ ΠΊΡΠ°ΡΡΠ°ΠΉΡΠΈΠΉ ΠΏΠΎΠ²ΠΎΡΠΎΡ ΠΏΡΠΎΠΈΡΡ ΠΎΠ΄ΠΈΡ ΠΏΡΠΎΡΠΈΠ² ΡΠ°ΡΠΎΠ²ΠΎΠΉ ΡΡΡΠ΅Π»ΠΊΠΈ, ΡΠΎ ΡΡΠΎΠΉΠΊΠ° Π²Π΅ΠΊΡΠΎΡΠΎΠ² Π½Π°Π·ΡΠ²Π°Π΅ΡΡΡ ΠΏΡΠ°Π²ΠΎΠΉ , Π² ΠΏΡΠΎΡΠΈΠ²Π½ΠΎΠΌ ΡΠ»ΡΡΠ°Π΅ β Π»Π΅Π²ΠΎΠΉ .
Π’Π΅ΠΏΠ΅ΡΡ Π²ΠΎΠ·ΡΠΌΠ΅ΠΌ Π΄Π²Π° Π½Π΅ ΠΊΠΎΠ»Π»ΠΈΠ½Π΅Π°ΡΠ½ΡΡ
Π²Π΅ΠΊΡΠΎΡΠ° ΠΈ . ΠΡΠ»ΠΎΠΆΠΈΠΌ ΠΎΡ ΡΠΎΡΠΊΠΈ Π
Π²Π΅ΠΊΡΠΎΡΡ ΠΈ . ΠΠΎΡΡΡΠΎΠΈΠΌ Π½Π΅ΠΊΠΎΡΠΎΡΡΠΉ Π²Π΅ΠΊΡΠΎΡ , ΠΏΠ΅ΡΠΏΠ΅Π½Π΄ΠΈΠΊΡΠ»ΡΡΠ½ΡΠΉ ΠΎΠ΄Π½ΠΎΠ²ΡΠ΅ΠΌΠ΅Π½Π½ΠΎ ΠΈ ΠΈ . ΠΡΠ΅Π²ΠΈΠ΄Π½ΠΎ, ΡΡΠΎ ΠΏΡΠΈ ΠΏΠΎΡΡΡΠΎΠ΅Π½ΠΈΠΈ Π²Π΅ΠΊΡΠΎΡΠ° ΠΌΡ ΠΌΠΎΠΆΠ΅ΠΌ ΠΏΠΎΡΡΡΠΏΠΈΡΡ Π΄Π²ΠΎΡΠΊΠΎ, Π·Π°Π΄Π°Π² Π΅ΠΌΡ Π»ΠΈΠ±ΠΎ ΠΎΠ΄Π½ΠΎ Π½Π°ΠΏΡΠ°Π²Π»Π΅Π½ΠΈΠ΅, Π»ΠΈΠ±ΠΎ ΠΏΡΠΎΡΠΈΠ²ΠΎΠΏΠΎΠ»ΠΎΠΆΠ½ΠΎΠ΅ (ΡΠΌΠΎΡΡΠΈΡΠ΅ ΠΈΠ»Π»ΡΡΡΡΠ°ΡΠΈΡ).
Π Π·Π°Π²ΠΈΡΠΈΠΌΠΎΡΡΠΈ ΠΎΡ Π½Π°ΠΏΡΠ°Π²Π»Π΅Π½ΠΈΡ Π²Π΅ΠΊΡΠΎΡΠ° ΡΠΏΠΎΡΡΠ΄ΠΎΡΠ΅Π½Π½Π°Ρ ΡΡΠΎΠΉΠΊΠ° Π²Π΅ΠΊΡΠΎΡΠΎΠ² ΠΌΠΎΠΆΠ΅Ρ Π±ΡΡΡ ΠΏΡΠ°Π²ΠΎΠΉ ΠΈΠ»ΠΈ Π»Π΅Π²ΠΎΠΉ.
Π’Π°ΠΊ ΠΌΡ Π²ΠΏΠ»ΠΎΡΠ½ΡΡ ΠΏΠΎΠ΄ΠΎΡΠ»ΠΈ ΠΊ ΠΎΠΏΡΠ΅Π΄Π΅Π»Π΅Π½ΠΈΡ Π²Π΅ΠΊΡΠΎΡΠ½ΠΎΠ³ΠΎ ΠΏΡΠΎΠΈΠ·Π²Π΅Π΄Π΅Π½ΠΈΡ. ΠΠ½ΠΎ Π΄Π°Π΅ΡΡΡ Π΄Π»Ρ Π΄Π²ΡΡ Π²Π΅ΠΊΡΠΎΡΠΎΠ², Π·Π°Π΄Π°Π½Π½ΡΡ Π² ΠΏΡΡΠΌΠΎΡΠ³ΠΎΠ»ΡΠ½ΠΎΠΉ ΡΠΈΡΡΠ΅ΠΌΠ΅ ΠΊΠΎΠΎΡΠ΄ΠΈΠ½Π°Ρ ΡΡΠ΅Ρ ΠΌΠ΅ΡΠ½ΠΎΠ³ΠΎ ΠΏΡΠΎΡΡΡΠ°Π½ΡΡΠ²Π°.
ΠΠΏΡΠ΅Π΄Π΅Π»Π΅Π½ΠΈΠ΅.
ΠΠ΅ΠΊΡΠΎΡΠ½ΡΠΌ ΠΏΡΠΎΠΈΠ·Π²Π΅Π΄Π΅Π½ΠΈΠ΅ΠΌ Π΄Π²ΡΡ Π²Π΅ΠΊΡΠΎΡΠΎΠ² ΠΈ , Π·Π°Π΄Π°Π½Π½ΡΡ Π² ΠΏΡΡΠΌΠΎΡΠ³ΠΎΠ»ΡΠ½ΠΎΠΉ ΡΠΈΡΡΠ΅ΠΌΠ΅ ΠΊΠΎΠΎΡΠ΄ΠΈΠ½Π°Ρ ΡΡΠ΅Ρ ΠΌΠ΅ΡΠ½ΠΎΠ³ΠΎ ΠΏΡΠΎΡΡΡΠ°Π½ΡΡΠ²Π°, Π½Π°Π·ΡΠ²Π°Π΅ΡΡΡ ΡΠ°ΠΊΠΎΠΉ Π²Π΅ΠΊΡΠΎΡ , ΡΡΠΎ
ΠΠ΅ΠΊΡΠΎΡΠ½ΠΎΠ΅ ΠΏΡΠΎΠΈΠ·Π²Π΅Π΄Π΅Π½ΠΈΠ΅ Π²Π΅ΠΊΡΠΎΡΠΎΠ² ΠΈ ΠΎΠ±ΠΎΠ·Π½Π°ΡΠ°Π΅ΡΡΡ ΠΊΠ°ΠΊ .
ΠΠΎΠΎΡΠ΄ΠΈΠ½Π°ΡΡ Π²Π΅ΠΊΡΠΎΡΠ½ΠΎΠ³ΠΎ ΠΏΡΠΎΠΈΠ·Π²Π΅Π΄Π΅Π½ΠΈΡ.
Π‘Π΅ΠΉΡΠ°Ρ Π΄Π°Π΄ΠΈΠΌ Π²ΡΠΎΡΠΎΠ΅ ΠΎΠΏΡΠ΅Π΄Π΅Π»Π΅Π½ΠΈΠ΅ Π²Π΅ΠΊΡΠΎΡΠ½ΠΎΠ³ΠΎ ΠΏΡΠΎΠΈΠ·Π²Π΅Π΄Π΅Π½ΠΈΡ, ΠΊΠΎΡΠΎΡΠΎΠ΅ ΠΏΠΎΠ·Π²ΠΎΠ»ΡΠ΅Ρ Π½Π°Ρ ΠΎΠ΄ΠΈΡΡ Π΅Π³ΠΎ ΠΊΠΎΠΎΡΠ΄ΠΈΠ½Π°ΡΡ ΠΏΠΎ ΠΊΠΎΠΎΡΠ΄ΠΈΠ½Π°ΡΠ°ΠΌ Π·Π°Π΄Π°Π½Π½ΡΡ Π²Π΅ΠΊΡΠΎΡΠΎΠ² ΠΈ.
ΠΠΏΡΠ΅Π΄Π΅Π»Π΅Π½ΠΈΠ΅.
Π ΠΏΡΡΠΌΠΎΡΠ³ΠΎΠ»ΡΠ½ΠΎΠΉ ΡΠΈΡΡΠ΅ΠΌΠ΅ ΠΊΠΎΠΎΡΠ΄ΠΈΠ½Π°Ρ ΡΡΠ΅Ρ ΠΌΠ΅ΡΠ½ΠΎΠ³ΠΎ ΠΏΡΠΎΡΡΡΠ°Π½ΡΡΠ²Π° Π²Π΅ΠΊΡΠΎΡΠ½ΠΎΠ΅ ΠΏΡΠΎΠΈΠ·Π²Π΅Π΄Π΅Π½ΠΈΠ΅ Π΄Π²ΡΡ Π²Π΅ΠΊΡΠΎΡΠΎΠ² ΠΈ Π΅ΡΡΡ Π²Π΅ΠΊΡΠΎΡ , Π³Π΄Π΅ β ΠΊΠΎΠΎΡΠ΄ΠΈΠ½Π°ΡΠ½ΡΠ΅ Π²Π΅ΠΊΡΠΎΡΡ.
ΠΡΠΎ ΠΎΠΏΡΠ΅Π΄Π΅Π»Π΅Π½ΠΈΠ΅ Π΄Π°Π΅Ρ Π½Π°ΠΌ Π²Π΅ΠΊΡΠΎΡΠ½ΠΎΠ΅ ΠΏΡΠΎΠΈΠ·Π²Π΅Π΄Π΅Π½ΠΈΠ΅ Π² ΠΊΠΎΠΎΡΠ΄ΠΈΠ½Π°ΡΠ½ΠΎΠΉ ΡΠΎΡΠΌΠ΅.
ΠΠ΅ΠΊΡΠΎΡΠ½ΠΎΠ΅ ΠΏΡΠΎΠΈΠ·Π²Π΅Π΄Π΅Π½ΠΈΠ΅ ΡΠ΄ΠΎΠ±Π½ΠΎ ΠΏΡΠ΅Π΄ΡΡΠ°Π²Π»ΡΡΡ Π² Π²ΠΈΠ΄Π΅ ΠΎΠΏΡΠ΅Π΄Π΅Π»ΠΈΡΠ΅Π»Ρ ΠΊΠ²Π°Π΄ΡΠ°ΡΠ½ΠΎΠΉ ΠΌΠ°ΡΡΠΈΡΡ ΡΡΠ΅ΡΡΠ΅Π³ΠΎ ΠΏΠΎΡΡΠ΄ΠΊΠ°, ΠΏΠ΅ΡΠ²Π°Ρ ΡΡΡΠΎΠΊΠ° ΠΊΠΎΡΠΎΡΠΎΠΉ Π΅ΡΡΡ ΠΎΡΡΡ , Π²ΠΎ Π²ΡΠΎΡΠΎΠΉ ΡΡΡΠΎΠΊΠ΅ Π½Π°Ρ
ΠΎΠ΄ΡΡΡΡ ΠΊΠΎΠΎΡΠ΄ΠΈΠ½Π°ΡΡ Π²Π΅ΠΊΡΠΎΡΠ° , Π° Π² ΡΡΠ΅ΡΡΠ΅ΠΉ β ΠΊΠΎΠΎΡΠ΄ΠΈΠ½Π°ΡΡ Π²Π΅ΠΊΡΠΎΡΠ° Π² Π·Π°Π΄Π°Π½Π½ΠΎΠΉ ΠΏΡΡΠΌΠΎΡΠ³ΠΎΠ»ΡΠ½ΠΎΠΉ ΡΠΈΡΡΠ΅ΠΌΠ΅ ΠΊΠΎΠΎΡΠ΄ΠΈΠ½Π°Ρ:
ΠΡΠ»ΠΈ ΡΠ°Π·Π»ΠΎΠΆΠΈΡΡ ΡΡΠΎΡ ΠΎΠΏΡΠ΅Π΄Π΅Π»ΠΈΡΠ΅Π»Ρ ΠΏΠΎ ΡΠ»Π΅ΠΌΠ΅Π½ΡΠ°ΠΌ ΠΏΠ΅ΡΠ²ΠΎΠΉ ΡΡΡΠΎΠΊΠΈ, ΡΠΎ ΠΏΠΎΠ»ΡΡΠΈΠΌ ΡΠ°Π²Π΅Π½ΡΡΠ²ΠΎ ΠΈΠ· ΠΎΠΏΡΠ΅Π΄Π΅Π»Π΅Π½ΠΈΡ Π²Π΅ΠΊΡΠΎΡΠ½ΠΎΠ³ΠΎ ΠΏΡΠΎΠΈΠ·Π²Π΅Π΄Π΅Π½ΠΈΡ Π² ΠΊΠΎΠΎΡΠ΄ΠΈΠ½Π°ΡΠ°Ρ (ΠΏΡΠΈ Π½Π΅ΠΎΠ±Ρ ΠΎΠ΄ΠΈΠΌΠΎΡΡΠΈ ΠΎΠ±ΡΠ°ΡΠ°ΠΉΡΠ΅ΡΡ ΠΊ ΡΡΠ°ΡΡΠ΅ ):
Π‘Π»Π΅Π΄ΡΠ΅Ρ ΠΎΡΠΌΠ΅ΡΠΈΡΡ, ΡΡΠΎ ΠΊΠΎΠΎΡΠ΄ΠΈΠ½Π°ΡΠ½Π°Ρ ΡΠΎΡΠΌΠ° Π²Π΅ΠΊΡΠΎΡΠ½ΠΎΠ³ΠΎ ΠΏΡΠΎΠΈΠ·Π²Π΅Π΄Π΅Π½ΠΈΡ ΠΏΠΎΠ»Π½ΠΎΡΡΡΡ ΡΠΎΠ³Π»Π°ΡΡΠ΅ΡΡΡ Ρ ΠΎΠΏΡΠ΅Π΄Π΅Π»Π΅Π½ΠΈΠ΅ΠΌ, Π΄Π°Π½Π½ΡΠΌ Π² ΠΏΠ΅ΡΠ²ΠΎΠΌ ΠΏΡΠ½ΠΊΡΠ΅ ΡΡΠΎΠΉ ΡΡΠ°ΡΡΠΈ. ΠΠΎΠ»Π΅Π΅ ΡΠΎΠ³ΠΎ, ΡΡΠΈ Π΄Π²Π° ΠΎΠΏΡΠ΅Π΄Π΅Π»Π΅Π½ΠΈΡ Π²Π΅ΠΊΡΠΎΡΠ½ΠΎΠ³ΠΎ ΠΏΡΠΎΠΈΠ·Π²Π΅Π΄Π΅Π½ΠΈΡ ΡΠΊΠ²ΠΈΠ²Π°Π»Π΅Π½ΡΠ½Ρ. ΠΠΎΠΊΠ°Π·Π°ΡΠ΅Π»ΡΡΡΠ²ΠΎ ΡΡΠΎΠ³ΠΎ ΡΠ°ΠΊΡΠ° ΠΌΠΎΠΆΠ΅ΡΠ΅ ΠΏΠΎΡΠΌΠΎΡΡΠ΅ΡΡ Π² ΠΊΠ½ΠΈΠ³Π΅, ΡΠΊΠ°Π·Π°Π½Π½ΠΎΠΉ Π² ΠΊΠΎΠ½ΡΠ΅ ΡΡΠ°ΡΡΠΈ.
Π‘Π²ΠΎΠΉΡΡΠ²Π° Π²Π΅ΠΊΡΠΎΡΠ½ΠΎΠ³ΠΎ ΠΏΡΠΎΠΈΠ·Π²Π΅Π΄Π΅Π½ΠΈΡ.
Π’Π°ΠΊ ΠΊΠ°ΠΊ Π²Π΅ΠΊΡΠΎΡΠ½ΠΎΠ΅ ΠΏΡΠΎΠΈΠ·Π²Π΅Π΄Π΅Π½ΠΈΠ΅ Π² ΠΊΠΎΠΎΡΠ΄ΠΈΠ½Π°ΡΠ°Ρ ΠΏΡΠ΅Π΄ΡΡΠ°Π²ΠΈΠΌΠΎ Π² Π²ΠΈΠ΄Π΅ ΠΎΠΏΡΠ΅Π΄Π΅Π»ΠΈΡΠ΅Π»Ρ ΠΌΠ°ΡΡΠΈΡΡ , ΡΠΎ Π½Π° ΠΎΡΠ½ΠΎΠ²Π°Π½ΠΈΠΈ Π»Π΅Π³ΠΊΠΎ ΠΎΠ±ΠΎΡΠ½ΠΎΠ²ΡΠ²Π°ΡΡΡΡ ΡΠ»Π΅Π΄ΡΡΡΠΈΠ΅ ΡΠ²ΠΎΠΉΡΡΠ²Π° Π²Π΅ΠΊΡΠΎΡΠ½ΠΎΠ³ΠΎ ΠΏΡΠΎΠΈΠ·Π²Π΅Π΄Π΅Π½ΠΈΡ :
ΠΠ»Ρ ΠΏΡΠΈΠΌΠ΅ΡΠ° Π΄ΠΎΠΊΠ°ΠΆΠ΅ΠΌ ΡΠ²ΠΎΠΉΡΡΠ²ΠΎ Π°Π½ΡΠΈΠΊΠΎΠΌΠΌΡΡΠ°ΡΠΈΠ²Π½ΠΎΡΡΠΈ Π²Π΅ΠΊΡΠΎΡΠ½ΠΎΠ³ΠΎ ΠΏΡΠΎΠΈΠ·Π²Π΅Π΄Π΅Π½ΠΈΡ.
ΠΠΎ ΠΎΠΏΡΠ΅Π΄Π΅Π»Π΅Π½ΠΈΡ ΠΈ . ΠΠ°ΠΌ ΠΈΠ·Π²Π΅ΡΡΠ½ΠΎ, ΡΡΠΎ Π·Π½Π°ΡΠ΅Π½ΠΈΠ΅ ΠΎΠΏΡΠ΅Π΄Π΅Π»ΠΈΡΠ΅Π»Ρ ΠΌΠ°ΡΡΠΈΡΡ ΠΈΠ·ΠΌΠ΅Π½ΡΠ΅ΡΡΡ Π½Π° ΠΏΡΠΎΡΠΈΠ²ΠΎΠΏΠΎΠ»ΠΎΠΆΠ½ΠΎΠ΅, Π΅ΡΠ»ΠΈ ΠΏΠ΅ΡΠ΅ΡΡΠ°Π²ΠΈΡΡ ΠΌΠ΅ΡΡΠ°ΠΌΠΈ Π΄Π²Π΅ ΡΡΡΠΎΠΊΠΈ, ΠΏΠΎΡΡΠΎΠΌΡ, , ΡΡΠΎ Π΄ΠΎΠΊΠ°Π·ΡΠ²Π°Π΅Ρ ΡΠ²ΠΎΠΉΡΡΠ²ΠΎ Π°Π½ΡΠΈΠΊΠΎΠΌΠΌΡΡΠ°ΡΠΈΠ²Π½ΠΎΡΡΠΈ Π²Π΅ΠΊΡΠΎΡΠ½ΠΎΠ³ΠΎ ΠΏΡΠΎΠΈΠ·Π²Π΅Π΄Π΅Π½ΠΈΡ.
ΠΠ΅ΠΊΡΠΎΡΠ½ΠΎΠ΅ ΠΏΡΠΎΠΈΠ·Π²Π΅Π΄Π΅Π½ΠΈΠ΅ β ΠΏΡΠΈΠΌΠ΅ΡΡ ΠΈ ΡΠ΅ΡΠ΅Π½ΠΈΡ.
Π ΠΎΡΠ½ΠΎΠ²Π½ΠΎΠΌ Π²ΡΡΡΠ΅ΡΠ°ΡΡΡΡ ΡΡΠΈ ΡΠΈΠΏΠ° Π·Π°Π΄Π°Ρ.
Π Π·Π°Π΄Π°ΡΠ°Ρ ΠΏΠ΅ΡΠ²ΠΎΠ³ΠΎ ΡΠΈΠΏΠ° Π·Π°Π΄Π°Π½Ρ Π΄Π»ΠΈΠ½Ρ Π΄Π²ΡΡ Π²Π΅ΠΊΡΠΎΡΠΎΠ² ΠΈ ΡΠ³ΠΎΠ» ΠΌΠ΅ΠΆΠ΄Ρ Π½ΠΈΠΌΠΈ, Π° ΡΡΠ΅Π±ΡΠ΅ΡΡΡ Π½Π°ΠΉΡΠΈ Π΄Π»ΠΈΠ½Ρ Π²Π΅ΠΊΡΠΎΡΠ½ΠΎΠ³ΠΎ ΠΏΡΠΎΠΈΠ·Π²Π΅Π΄Π΅Π½ΠΈΡ. Π ΡΡΠΎΠΌ ΡΠ»ΡΡΠ°Π΅ ΠΈΡΠΏΠΎΠ»ΡΠ·ΡΠ΅ΡΡΡ ΡΠΎΡΠΌΡΠ»Π° .
ΠΡΠΈΠΌΠ΅Ρ.
ΠΠ°ΠΉΠ΄ΠΈΡΠ΅ Π΄Π»ΠΈΠ½Ρ Π²Π΅ΠΊΡΠΎΡΠ½ΠΎΠ³ΠΎ ΠΏΡΠΎΠΈΠ·Π²Π΅Π΄Π΅Π½ΠΈΡ Π²Π΅ΠΊΡΠΎΡΠΎΠ² ΠΈ , Π΅ΡΠ»ΠΈ ΠΈΠ·Π²Π΅ΡΡΠ½ΠΎ .
Π Π΅ΡΠ΅Π½ΠΈΠ΅.
ΠΡ Π·Π½Π°Π΅ΠΌ ΠΈΠ· ΠΎΠΏΡΠ΅Π΄Π΅Π»Π΅Π½ΠΈΡ, ΡΡΠΎ Π΄Π»ΠΈΠ½Π° Π²Π΅ΠΊΡΠΎΡΠ½ΠΎΠ³ΠΎ ΠΏΡΠΎΠΈΠ·Π²Π΅Π΄Π΅Π½ΠΈΡ Π²Π΅ΠΊΡΠΎΡΠΎΠ² ΠΈ ΡΠ°Π²Π½Π° ΠΏΡΠΎΠΈΠ·Π²Π΅Π΄Π΅Π½ΠΈΡ Π΄Π»ΠΈΠ½ Π²Π΅ΠΊΡΠΎΡΠΎΠ² ΠΈ Π½Π° ΡΠΈΠ½ΡΡ ΡΠ³Π»Π° ΠΌΠ΅ΠΆΠ΄Ρ Π½ΠΈΠΌΠΈ, ΠΏΠΎΡΡΠΎΠΌΡ, .
ΠΡΠ²Π΅Ρ:
.
ΠΠ°Π΄Π°ΡΠΈ Π²ΡΠΎΡΠΎΠ³ΠΎ ΡΠΈΠΏΠ° ΡΠ²ΡΠ·Π°Π½Ρ Ρ ΠΊΠΎΠΎΡΠ΄ΠΈΠ½Π°ΡΠ°ΠΌΠΈ Π²Π΅ΠΊΡΠΎΡΠΎΠ², Π² Π½ΠΈΡ Π²Π΅ΠΊΡΠΎΡΠ½ΠΎΠ΅ ΠΏΡΠΎΠΈΠ·Π²Π΅Π΄Π΅Π½ΠΈΠ΅, Π΅Π³ΠΎ Π΄Π»ΠΈΠ½Π° ΠΈΠ»ΠΈ ΡΡΠΎ-Π»ΠΈΠ±ΠΎ Π΅ΡΠ΅ ΠΈΡΠ΅ΡΡΡ ΡΠ΅ΡΠ΅Π· ΠΊΠΎΠΎΡΠ΄ΠΈΠ½Π°ΡΡ Π·Π°Π΄Π°Π½Π½ΡΡ Π²Π΅ΠΊΡΠΎΡΠΎΠ² ΠΈ .
ΠΠ΄Π΅ΡΡ Π²ΠΎΠ·ΠΌΠΎΠΆΠ½Π° ΠΌΠ°ΡΡΠ° ΡΠ°Π·Π»ΠΈΡΠ½ΡΡ Π²Π°ΡΠΈΠ°Π½ΡΠΎΠ². Π ΠΏΡΠΈΠΌΠ΅ΡΡ, ΠΌΠΎΠ³ΡΡ Π±ΡΡΡ Π·Π°Π΄Π°Π½Ρ Π½Π΅ ΠΊΠΎΠΎΡΠ΄ΠΈΠ½Π°ΡΡ Π²Π΅ΠΊΡΠΎΡΠΎΠ² ΠΈ , Π° ΠΈΡ ΡΠ°Π·Π»ΠΎΠΆΠ΅Π½ΠΈΡ ΠΏΠΎ ΠΊΠΎΠΎΡΠ΄ΠΈΠ½Π°ΡΠ½ΡΠΌ Π²Π΅ΠΊΡΠΎΡΠ°ΠΌ Π²ΠΈΠ΄Π° ΠΈ , ΠΈΠ»ΠΈ Π²Π΅ΠΊΡΠΎΡΡ ΠΈ ΠΌΠΎΠ³ΡΡ Π±ΡΡΡ Π·Π°Π΄Π°Π½Ρ ΠΊΠΎΠΎΡΠ΄ΠΈΠ½Π°ΡΠ°ΠΌΠΈ ΡΠΎΡΠ΅ΠΊ ΠΈΡ Π½Π°ΡΠ°Π»Π° ΠΈ ΠΊΠΎΠ½ΡΠ°.
Π Π°ΡΡΠΌΠΎΡΡΠΈΠΌ Ρ Π°ΡΠ°ΠΊΡΠ΅ΡΠ½ΡΠ΅ ΠΏΡΠΈΠΌΠ΅ΡΡ.
ΠΡΠΈΠΌΠ΅Ρ.
Π ΠΏΡΡΠΌΠΎΡΠ³ΠΎΠ»ΡΠ½ΠΎΠΉ ΡΠΈΡΡΠ΅ΠΌΠ΅ ΠΊΠΎΠΎΡΠ΄ΠΈΠ½Π°Ρ Π·Π°Π΄Π°Π½Ρ Π΄Π²Π° Π²Π΅ΠΊΡΠΎΡΠ° . ΠΠ°ΠΉΠ΄ΠΈΡΠ΅ ΠΈΡ Π²Π΅ΠΊΡΠΎΡΠ½ΠΎΠ΅ ΠΏΡΠΎΠΈΠ·Π²Π΅Π΄Π΅Π½ΠΈΠ΅.
Π Π΅ΡΠ΅Π½ΠΈΠ΅.
ΠΠΎ Π²ΡΠΎΡΠΎΠΌΡ ΠΎΠΏΡΠ΅Π΄Π΅Π»Π΅Π½ΠΈΡ Π²Π΅ΠΊΡΠΎΡΠ½ΠΎΠ΅ ΠΏΡΠΎΠΈΠ·Π²Π΅Π΄Π΅Π½ΠΈΠ΅ Π΄Π²ΡΡ Π²Π΅ΠΊΡΠΎΡΠΎΠ² Π² ΠΊΠΎΠΎΡΠ΄ΠΈΠ½Π°ΡΠ°Ρ Π·Π°ΠΏΠΈΡΡΠ²Π°Π΅ΡΡΡ ΠΊΠ°ΠΊ:
Π ΡΠ°ΠΊΠΎΠΌΡ ΠΆΠ΅ ΡΠ΅Π·ΡΠ»ΡΡΠ°ΡΡ ΠΌΡ Π±Ρ ΠΏΡΠΈΡΠ»ΠΈ, Π΅ΡΠ»ΠΈ Π±Ρ Π²Π΅ΠΊΡΠΎΡΠ½ΠΎΠ΅ ΠΏΡΠΎΠΈΠ·Π²Π΅Π΄Π΅Π½ΠΈΠ΅ Π·Π°ΠΏΠΈΡΠ°Π»ΠΈ ΡΠ΅ΡΠ΅Π· ΠΎΠΏΡΠ΅Π΄Π΅Π»ΠΈΡΠ΅Π»Ρ
ΠΡΠ²Π΅Ρ:
.
ΠΡΠΈΠΌΠ΅Ρ.
ΠΠ°ΠΉΠ΄ΠΈΡΠ΅ Π΄Π»ΠΈΠ½Ρ Π²Π΅ΠΊΡΠΎΡΠ½ΠΎΠ³ΠΎ ΠΏΡΠΎΠΈΠ·Π²Π΅Π΄Π΅Π½ΠΈΡ Π²Π΅ΠΊΡΠΎΡΠΎΠ² ΠΈ , Π³Π΄Π΅ β ΠΎΡΡΡ ΠΏΡΡΠΌΠΎΡΠ³ΠΎΠ»ΡΠ½ΠΎΠΉ Π΄Π΅ΠΊΠ°ΡΡΠΎΠ²ΠΎΠΉ ΡΠΈΡΡΠ΅ΠΌΡ ΠΊΠΎΠΎΡΠ΄ΠΈΠ½Π°Ρ.
Π Π΅ΡΠ΅Π½ΠΈΠ΅.
Π‘Π½Π°ΡΠ°Π»Π° Π½Π°ΠΉΠ΄Π΅ΠΌ ΠΊΠΎΠΎΡΠ΄ΠΈΠ½Π°ΡΡ Π²Π΅ΠΊΡΠΎΡΠ½ΠΎΠ³ΠΎ ΠΏΡΠΎΠΈΠ·Π²Π΅Π΄Π΅Π½ΠΈΡ Π² Π·Π°Π΄Π°Π½Π½ΠΎΠΉ ΠΏΡΡΠΌΠΎΡΠ³ΠΎΠ»ΡΠ½ΠΎΠΉ ΡΠΈΡΡΠ΅ΠΌΠ΅ ΠΊΠΎΠΎΡΠ΄ΠΈΠ½Π°Ρ.
Π’Π°ΠΊ ΠΊΠ°ΠΊ Π²Π΅ΠΊΡΠΎΡΡ ΠΈ ΠΈΠΌΠ΅ΡΡ ΠΊΠΎΠΎΡΠ΄ΠΈΠ½Π°ΡΡ ΠΈ ΡΠΎΠΎΡΠ²Π΅ΡΡΡΠ²Π΅Π½Π½ΠΎ (ΠΏΡΠΈ Π½Π΅ΠΎΠ±Ρ ΠΎΠ΄ΠΈΠΌΠΎΡΡΠΈ ΡΠΌΠΎΡΡΠΈΡΠ΅ ΡΡΠ°ΡΡΡ ΠΊΠΎΠΎΡΠ΄ΠΈΠ½Π°ΡΡ Π²Π΅ΠΊΡΠΎΡΠ° Π² ΠΏΡΡΠΌΠΎΡΠ³ΠΎΠ»ΡΠ½ΠΎΠΉ ΡΠΈΡΡΠ΅ΠΌΠ΅ ΠΊΠΎΠΎΡΠ΄ΠΈΠ½Π°Ρ), ΡΠΎ ΠΏΠΎ Π²ΡΠΎΡΠΎΠΌΡ ΠΎΠΏΡΠ΅Π΄Π΅Π»Π΅Π½ΠΈΡ Π²Π΅ΠΊΡΠΎΡΠ½ΠΎΠ³ΠΎ ΠΏΡΠΎΠΈΠ·Π²Π΅Π΄Π΅Π½ΠΈΡ ΠΈΠΌΠ΅Π΅ΠΌ
Π’ΠΎ Π΅ΡΡΡ, Π²Π΅ΠΊΡΠΎΡΠ½ΠΎΠ΅ ΠΏΡΠΎΠΈΠ·Π²Π΅Π΄Π΅Π½ΠΈΠ΅ ΠΈΠΌΠ΅Π΅Ρ ΠΊΠΎΠΎΡΠ΄ΠΈΠ½Π°ΡΡ Π² Π·Π°Π΄Π°Π½Π½ΠΎΠΉ ΡΠΈΡΡΠ΅ΠΌΠ΅ ΠΊΠΎΠΎΡΠ΄ΠΈΠ½Π°Ρ.
ΠΠ»ΠΈΠ½Ρ Π²Π΅ΠΊΡΠΎΡΠ½ΠΎΠ³ΠΎ ΠΏΡΠΎΠΈΠ·Π²Π΅Π΄Π΅Π½ΠΈΡ Π½Π°Ρ
ΠΎΠ΄ΠΈΠΌ ΠΊΠ°ΠΊ ΠΊΠΎΡΠ΅Π½Ρ ΠΊΠ²Π°Π΄ΡΠ°ΡΠ½ΡΠΉ ΠΈΠ· ΡΡΠΌΠΌΡ ΠΊΠ²Π°Π΄ΡΠ°ΡΠΎΠ² Π΅Π³ΠΎ ΠΊΠΎΠΎΡΠ΄ΠΈΠ½Π°Ρ (ΡΡΡ ΡΠΎΡΠΌΡΠ»Ρ Π΄Π»ΠΈΠ½Ρ Π²Π΅ΠΊΡΠΎΡΠ° ΠΌΡ ΠΏΠΎΠ»ΡΡΠΈΠ»ΠΈ Π² ΡΠ°Π·Π΄Π΅Π»Π΅ Π½Π°Ρ
ΠΎΠΆΠ΄Π΅Π½ΠΈΠ΅ Π΄Π»ΠΈΠ½Ρ Π²Π΅ΠΊΡΠΎΡΠ°):
ΠΡΠ²Π΅Ρ:
.
ΠΡΠΈΠΌΠ΅Ρ.
Π ΠΏΡΡΠΌΠΎΡΠ³ΠΎΠ»ΡΠ½ΠΎΠΉ Π΄Π΅ΠΊΠ°ΡΡΠΎΠ²ΠΎΠΉ ΡΠΈΡΡΠ΅ΠΌΠ΅ ΠΊΠΎΠΎΡΠ΄ΠΈΠ½Π°Ρ Π·Π°Π΄Π°Π½Ρ ΠΊΠΎΠΎΡΠ΄ΠΈΠ½Π°ΡΡ ΡΡΠ΅Ρ ΡΠΎΡΠ΅ΠΊ . ΠΠ°ΠΉΠ΄ΠΈΡΠ΅ ΠΊΠ°ΠΊΠΎΠΉ-Π½ΠΈΠ±ΡΠ΄Ρ Π²Π΅ΠΊΡΠΎΡ, ΠΏΠ΅ΡΠΏΠ΅Π½Π΄ΠΈΠΊΡΠ»ΡΡΠ½ΡΠΉ ΠΈ ΠΎΠ΄Π½ΠΎΠ²ΡΠ΅ΠΌΠ΅Π½Π½ΠΎ.
Π Π΅ΡΠ΅Π½ΠΈΠ΅.
ΠΠ΅ΠΊΡΠΎΡΡ ΠΈ ΠΈΠΌΠ΅ΡΡ ΠΊΠΎΠΎΡΠ΄ΠΈΠ½Π°ΡΡ ΠΈ ΡΠΎΠΎΡΠ²Π΅ΡΡΡΠ²Π΅Π½Π½ΠΎ (ΡΠΌΠΎΡΡΠΈΡΠ΅ ΡΡΠ°ΡΡΡ Π½Π°Ρ
ΠΎΠΆΠ΄Π΅Π½ΠΈΠ΅ ΠΊΠΎΠΎΡΠ΄ΠΈΠ½Π°Ρ Π²Π΅ΠΊΡΠΎΡΠ° ΡΠ΅ΡΠ΅Π· ΠΊΠΎΠΎΡΠ΄ΠΈΠ½Π°ΡΡ ΡΠΎΡΠ΅ΠΊ). ΠΡΠ»ΠΈ Π½Π°ΠΉΡΠΈ Π²Π΅ΠΊΡΠΎΡΠ½ΠΎΠ΅ ΠΏΡΠΎΠΈΠ·Π²Π΅Π΄Π΅Π½ΠΈΠ΅ Π²Π΅ΠΊΡΠΎΡΠΎΠ² ΠΈ , ΡΠΎ ΠΎΠ½ΠΎ ΠΏΠΎ ΠΎΠΏΡΠ΅Π΄Π΅Π»Π΅Π½ΠΈΡ ΡΠ²Π»ΡΠ΅ΡΡΡ Π²Π΅ΠΊΡΠΎΡΠΎΠΌ, ΠΏΠ΅ΡΠΏΠ΅Π½Π΄ΠΈΠΊΡΠ»ΡΡΠ½ΡΠΌ ΠΈ ΠΊ ΠΈ ΠΊ , ΡΠΎ Π΅ΡΡΡ, ΡΠ²Π»ΡΠ΅ΡΡΡ ΡΠ΅ΡΠ΅Π½ΠΈΠ΅ΠΌ Π½Π°ΡΠ΅ΠΉ Π·Π°Π΄Π°ΡΠΈ. ΠΠ°ΠΉΠ΄Π΅ΠΌ Π΅Π³ΠΎ
ΠΡΠ²Π΅Ρ:
β ΠΎΠ΄ΠΈΠ½ ΠΈΠ· ΠΏΠ΅ΡΠΏΠ΅Π½Π΄ΠΈΠΊΡΠ»ΡΡΠ½ΡΡ Π²Π΅ΠΊΡΠΎΡΠΎΠ².
Π Π·Π°Π΄Π°ΡΠ°Ρ ΡΡΠ΅ΡΡΠ΅Π³ΠΎ ΡΠΈΠΏΠ° ΠΏΡΠΎΠ²Π΅ΡΡΠ΅ΡΡΡ Π½Π°Π²ΡΠΊ ΠΈΡΠΏΠΎΠ»ΡΠ·ΠΎΠ²Π°Π½ΠΈΡ ΡΠ²ΠΎΠΉΡΡΠ² Π²Π΅ΠΊΡΠΎΡΠ½ΠΎΠ³ΠΎ ΠΏΡΠΎΠΈΠ·Π²Π΅Π΄Π΅Π½ΠΈΡ Π²Π΅ΠΊΡΠΎΡΠΎΠ². ΠΠΎΡΠ»Π΅ ΠΏΡΠΈΠΌΠ΅Π½Π΅Π½ΠΈΡ ΡΠ²ΠΎΠΉΡΡΠ², ΠΏΡΠΈΠΌΠ΅Π½ΡΡΡΡΡ ΡΠΎΠΎΡΠ²Π΅ΡΡΡΠ²ΡΡΡΠΈΠ΅ ΡΠΎΡΠΌΡΠ»Ρ.
ΠΡΠΈΠΌΠ΅Ρ.
ΠΠ΅ΠΊΡΠΎΡΡ ΠΈ ΠΏΠ΅ΡΠΏΠ΅Π½Π΄ΠΈΠΊΡΠ»ΡΡΠ½Ρ ΠΈ ΠΈΡ Π΄Π»ΠΈΠ½Ρ ΡΠ°Π²Π½Ρ ΡΠΎΠΎΡΠ²Π΅ΡΡΡΠ²Π΅Π½Π½ΠΎ 3 ΠΈ 4 . ΠΠ°ΠΉΠ΄ΠΈΡΠ΅ Π΄Π»ΠΈΠ½Ρ Π²Π΅ΠΊΡΠΎΡΠ½ΠΎΠ³ΠΎ ΠΏΡΠΎΠΈΠ·Π²Π΅Π΄Π΅Π½ΠΈΡ .
Π Π΅ΡΠ΅Π½ΠΈΠ΅.
ΠΠΎ ΡΠ²ΠΎΠΉΡΡΠ²Ρ Π΄ΠΈΡΡΡΠΈΠ±ΡΡΠΈΠ²Π½ΠΎΡΡΠΈ Π²Π΅ΠΊΡΠΎΡΠ½ΠΎΠ³ΠΎ ΠΏΡΠΎΠΈΠ·Π²Π΅Π΄Π΅Π½ΠΈΡ ΠΌΡ ΠΌΠΎΠΆΠ΅ΠΌ Π·Π°ΠΏΠΈΡΠ°ΡΡ
Π ΡΠΈΠ»Ρ ΡΠΎΡΠ΅ΡΠ°ΡΠ΅Π»ΡΠ½ΠΎΠ³ΠΎ ΡΠ²ΠΎΠΉΡΡΠ²Π° Π²ΡΠ½Π΅ΡΠ΅ΠΌ ΡΠΈΡΠ»ΠΎΠ²ΡΠ΅ ΠΊΠΎΡΡΡΠΈΡΠΈΠ΅Π½ΡΡ Π·Π° Π·Π½Π°ΠΊ Π²Π΅ΠΊΡΠΎΡΠ½ΡΡ ΠΏΡΠΎΠΈΠ·Π²Π΅Π΄Π΅Π½ΠΈΠΉ Π² ΠΏΠΎΡΠ»Π΅Π΄Π½Π΅ΠΌ Π²ΡΡΠ°ΠΆΠ΅Π½ΠΈΠΈ:
ΠΠ΅ΠΊΡΠΎΡΠ½ΡΠ΅ ΠΏΡΠΎΠΈΠ·Π²Π΅Π΄Π΅Π½ΠΈΡ ΠΈ ΡΠ°Π²Π½Ρ Π½ΡΠ»Ρ, ΡΠ°ΠΊ ΠΊΠ°ΠΊ ΠΈ , ΡΠΎΠ³Π΄Π° .
Π’Π°ΠΊ ΠΊΠ°ΠΊ Π²Π΅ΠΊΡΠΎΡΠ½ΠΎΠ΅ ΠΏΡΠΎΠΈΠ·Π²Π΅Π΄Π΅Π½ΠΈΠ΅ Π°Π½ΡΠΈΠΊΠΎΠΌΠΌΡΡΠ°ΡΠΈΠ²Π½ΠΎ, ΡΠΎ .
ΠΡΠ°ΠΊ, Ρ ΠΏΠΎΠΌΠΎΡΡΡ ΡΠ²ΠΎΠΉΡΡΠ² Π²Π΅ΠΊΡΠΎΡΠ½ΠΎΠ³ΠΎ ΠΏΡΠΎΠΈΠ·Π²Π΅Π΄Π΅Π½ΠΈΡ ΠΌΡ ΠΏΡΠΈΡΠ»ΠΈ ΠΊ ΡΠ°Π²Π΅Π½ΡΡΠ²Ρ .
ΠΠΎ ΡΡΠ»ΠΎΠ²ΠΈΡ Π²Π΅ΠΊΡΠΎΡΡ ΠΈ ΠΏΠ΅ΡΠΏΠ΅Π½Π΄ΠΈΠΊΡΠ»ΡΡΠ½Ρ, ΡΠΎ Π΅ΡΡΡ ΡΠ³ΠΎΠ» ΠΌΠ΅ΠΆΠ΄Ρ Π½ΠΈΠΌΠΈ ΡΠ°Π²Π΅Π½ . Π’ΠΎ Π΅ΡΡΡ, Ρ Π½Π°Ρ Π΅ΡΡΡ Π²ΡΠ΅ Π΄Π°Π½Π½ΡΠ΅ Π΄Π»Ρ Π½Π°Ρ ΠΎΠΆΠ΄Π΅Π½ΠΈΡ ΡΡΠ΅Π±ΡΠ΅ΠΌΠΎΠΉ Π΄Π»ΠΈΠ½Ρ
ΠΡΠ²Π΅Ρ:
.
ΠΠ΅ΠΎΠΌΠ΅ΡΡΠΈΡΠ΅ΡΠΊΠΈΠΉ ΡΠΌΡΡΠ» Π²Π΅ΠΊΡΠΎΡΠ½ΠΎΠ³ΠΎ ΠΏΡΠΎΠΈΠ·Π²Π΅Π΄Π΅Π½ΠΈΡ.

ΠΠΎ ΠΎΠΏΡΠ΅Π΄Π΅Π»Π΅Π½ΠΈΡ Π΄Π»ΠΈΠ½Π° Π²Π΅ΠΊΡΠΎΡΠ½ΠΎΠ³ΠΎ ΠΏΡΠΎΠΈΠ·Π²Π΅Π΄Π΅Π½ΠΈΡ Π²Π΅ΠΊΡΠΎΡΠΎΠ² ΡΠ°Π²Π½Π° . Π ΠΈΠ· ΠΊΡΡΡΠ° Π³Π΅ΠΎΠΌΠ΅ΡΡΠΈΠΈ ΡΡΠ΅Π΄Π½Π΅ΠΉ ΡΠΊΠΎΠ»Ρ Π½Π°ΠΌ ΠΈΠ·Π²Π΅ΡΡΠ½ΠΎ, ΡΡΠΎ ΠΏΠ»ΠΎΡΠ°Π΄Ρ ΡΡΠ΅ΡΠ³ΠΎΠ»ΡΠ½ΠΈΠΊΠ° ΡΠ°Π²Π½Π° ΠΏΠΎΠ»ΠΎΠ²ΠΈΠ½Π΅ ΠΏΡΠΎΠΈΠ·Π²Π΅Π΄Π΅Π½ΠΈΡ Π΄Π»ΠΈΠ½ Π΄Π²ΡΡ ΡΡΠΎΡΠΎΠ½ ΡΡΠ΅ΡΠ³ΠΎΠ»ΡΠ½ΠΈΠΊΠ° Π½Π° ΡΠΈΠ½ΡΡ ΡΠ³Π»Π° ΠΌΠ΅ΠΆΠ΄Ρ Π½ΠΈΠΌΠΈ. Π‘Π»Π΅Π΄ΠΎΠ²Π°ΡΠ΅Π»ΡΠ½ΠΎ, Π΄Π»ΠΈΠ½Π° Π²Π΅ΠΊΡΠΎΡΠ½ΠΎΠ³ΠΎ ΠΏΡΠΎΠΈΠ·Π²Π΅Π΄Π΅Π½ΠΈΡ ΡΠ°Π²Π½Π° ΡΠ΄Π²ΠΎΠ΅Π½Π½ΠΎΠΉ ΠΏΠ»ΠΎΡΠ°Π΄ΠΈ ΡΡΠ΅ΡΠ³ΠΎΠ»ΡΠ½ΠΈΠΊΠ°, ΠΈΠΌΠ΅ΡΡΠ΅Π³ΠΎ ΡΡΠΎΡΠΎΠ½Π°ΠΌΠΈ Π²Π΅ΠΊΡΠΎΡΡ ΠΈ , Π΅ΡΠ»ΠΈ ΠΈΡ ΠΎΡΠ»ΠΎΠΆΠΈΡΡ ΠΎΡ ΠΎΠ΄Π½ΠΎΠΉ ΡΠΎΡΠΊΠΈ. ΠΡΡΠ³ΠΈΠΌΠΈ ΡΠ»ΠΎΠ²Π°ΠΌΠΈ, Π΄Π»ΠΈΠ½Π° Π²Π΅ΠΊΡΠΎΡΠ½ΠΎΠ³ΠΎ ΠΏΡΠΎΠΈΠ·Π²Π΅Π΄Π΅Π½ΠΈΡ Π²Π΅ΠΊΡΠΎΡΠΎΠ² ΠΈ ΡΠ°Π²Π½Π° ΠΏΠ»ΠΎΡΠ°Π΄ΠΈ ΠΏΠ°ΡΠ°Π»Π»Π΅Π»ΠΎΠ³ΡΠ°ΠΌΠΌΠ° ΡΠΎ ΡΡΠΎΡΠΎΠ½Π°ΠΌΠΈ ΠΈ ΠΈ ΡΠ³Π»ΠΎΠΌ ΠΌΠ΅ΠΆΠ΄Ρ Π½ΠΈΠΌΠΈ, ΡΠ°Π²Π½ΡΠΌ . Π ΡΡΠΎΠΌ ΡΠΎΡΡΠΎΠΈΡ Π³Π΅ΠΎΠΌΠ΅ΡΡΠΈΡΠ΅ΡΠΊΠΈΠΉ ΡΠΌΡΡΠ» Π²Π΅ΠΊΡΠΎΡΠ½ΠΎΠ³ΠΎ ΠΏΡΠΎΠΈΠ·Π²Π΅Π΄Π΅Π½ΠΈΡ.
ΠΡΠΈΠΌΠ΅ΡΡ ΠΊΠ°ΠΊ Π²ΡΡΠΈΡΠ»ΠΈΡΡ ΠΏΠ»ΠΎΡΠ°Π΄Ρ ΠΏΠ°ΡΠ°Π»Π»Π΅Π»ΠΎΠ³ΡΠ°ΠΌΠΌΠ°, ΠΏΠΎΡΡΡΠΎΠ΅Π½Π½ΠΎΠ³ΠΎ Π½Π° Π²Π΅ΠΊΡΠΎΡΠ°Ρ
ΠΡΠΏΠΎΠΌΠ½ΠΈΠΌ Π² Π½Π°ΡΠ°Π»Π΅, ΡΡΠΎ ΡΠ°ΠΊΠΎΠ΅ Π²Π΅ΠΊΡΠΎΡΠ½ΠΎΠ΅ ΠΏΡΠΎΠΈΠ·Π²Π΅Π΄Π΅Π½ΠΈΠ΅.
ΠΠ°ΠΌΠ΅ΡΠ°Π½ΠΈΠ΅ 1
ΠΠ΅ΠΊΡΠΎΡΠ½ΡΠΌ ΠΏΡΠΎΠΈΠ·Π²Π΅Π΄Π΅Π½ΠΈΠ΅ΠΌ Π΄Π»Ρ $\vec{a}$ ΠΈ $\vec{b}$ ΡΠ²Π»ΡΠ΅ΡΡΡ $\vec{c}$, ΠΏΡΠ΅Π΄ΡΡΠ°Π²Π»ΡΡΡΠΈΠΉ ΡΠΎΠ±ΠΎΠΉ Π½Π΅ΠΊΠΎΡΠΎΡΡΠΉ ΡΡΠ΅ΡΠΈΠΉ Π²Π΅ΠΊΡΠΎΡ $\vec{c}= |[ab]|$, ΠΏΡΠΈΡΡΠΌ ΡΡΠΎΡ Π²Π΅ΠΊΡΠΎΡ ΠΎΠ±Π»Π°Π΄Π°Π΅Ρ ΠΎΡΠΎΠ±Π΅Π½Π½ΡΠΌΠΈ ΡΠ²ΠΎΠΉΡΡΠ²Π°ΠΌΠΈ:
- CΠΊΠ°Π»ΡΡ ΠΏΠΎΠ»ΡΡΠ΅Π½Π½ΠΎΠ³ΠΎ Π²Π΅ΠΊΡΠΎΡΠ° β ΠΏΡΠΎΠΈΠ·Π²Π΅Π΄Π΅Π½ΠΈΠ΅ $|\vec{a}|$ ΠΈ $|\vec{b}|$ Π½Π° ΡΠΈΠ½ΡΡ ΡΠ³Π»Π° $\vec{c}= |[ab]|= |\vec{a}| \cdot |\vec{b}|\cdot \sin Ξ± \left(1\right)$;
- ΠΡΠ΅ $\vec{a}, \vec{b}$ ΠΈ $\vec{c}$ ΠΎΠ±ΡΠ°Π·ΡΡΡ ΠΏΡΠ°Π²ΡΡ ΡΡΠΎΠΉΠΊΡ;
- ΠΠΎΠ»ΡΡΠ΅Π½Π½ΡΠΉ Π²Π΅ΠΊΡΠΎΡ ΠΎΡΡΠΎΠ³ΠΎΠ½Π°Π»Π΅Π½ ΠΊ $\vec{a}$ ΠΈ $\vec{b}$.
ΠΡΠ»ΠΈ Π΄Π»Ρ Π²Π΅ΠΊΡΠΎΡΠΎΠ² ΠΏΡΠΈΡΡΡΡΡΠ²ΡΡΡ Π½Π΅ΠΊΠΎΡΠΎΡΡΠ΅ ΠΊΠΎΠΎΡΠ΄ΠΈΠ½Π°ΡΡ ($\vec{a}=\{x_1; y_1; z_1\}$ ΠΈ $\vec{b}= \{x_2; y_2; z_2\}$), ΡΠΎ ΠΈΡ Π²Π΅ΠΊΡΠΎΡΠ½ΠΎΠ΅ ΠΏΡΠΎΠΈΠ·Π²Π΅Π΄Π΅Π½ΠΈΠ΅ Π² Π΄Π΅ΠΊΠ°ΡΡΠΎΠ²ΠΎΠΉ ΡΠΈΡΡΠ΅ΠΌΠ΅ ΠΊΠΎΠΎΡΠ΄ΠΈΠ½Π°Ρ ΠΌΠΎΠΆΠ½ΠΎ ΠΎΠΏΡΠ΅Π΄Π΅Π»ΠΈΡΡ ΠΏΠΎ ΡΠΎΡΠΌΡΠ»Π΅:
$[a \times b] = \{y_1 \cdot z_2 β y_2 \cdot z_1; z_1 \cdot x_2 β z_2 \cdot x_1; x_2 \cdot y_2 β x_2 \cdot y_1\}$
ΠΠ΅Π³ΡΠ΅ Π²ΡΠ΅Π³ΠΎ Π·Π°ΠΏΠΎΠΌΠ½ΠΈΡΡ ΡΡΡ ΡΠΎΡΠΌΡΠ»Ρ Π·Π°ΠΏΠΈΡΠ°Π² Π² ΡΠΎΡΠΌΠ΅ ΠΎΠΏΡΠ΅Π΄Π΅Π»ΠΈΡΠ΅Π»Ρ:
$[ab] = \begin{array} {|ccc|} i & j & k \\ x_1 & y_1 & z_1 \\ x_2 & y_2 & z_2 \\ \end{array}$.
ΠΡΠ° ΡΠΎΡΠΌΡΠ»Π° Π²Π΅ΡΡΠΌΠ° ΡΠ΄ΠΎΠ±Π½Π° Π΄Π»Ρ ΠΈΡΠΏΠΎΠ»ΡΠ·ΠΎΠ²Π°Π½ΠΈΡ, Π½ΠΎ ΡΡΠΎΠ±Ρ ΠΏΠΎΠ½ΠΈΠΌΠ°ΡΡ, ΠΊΠ°ΠΊ Π΅Ρ ΠΈΡΠΏΠΎΠ»ΡΠ·ΠΎΠ²Π°ΡΡ, Π΄Π»Ρ Π½Π°ΡΠ°Π»Π° ΡΠ»Π΅Π΄ΡΠ΅Ρ ΠΎΠ·Π½Π°ΠΊΠΎΠΌΠΈΡΡΡΡ Ρ ΡΠ΅ΠΌΠΎΠΉ ΠΌΠ°ΡΡΠΈΡ ΠΈ ΠΈΡ ΠΎΠΏΡΠ΅Π΄Π΅Π»ΠΈΡΠ΅Π»Π΅ΠΉ.
ΠΠ»ΠΎΡΠ°Π΄Ρ ΠΏΠ°ΡΠ°Π»Π»Π΅Π»ΠΎΠ³ΡΠ°ΠΌΠΌΠ°, ΡΡΠΎΡΠΎΠ½Ρ ΠΊΠΎΡΠΎΡΠΎΠ³ΠΎ ΠΎΠΏΡΠ΅Π΄Π΅Π»ΡΡΡΡΡ Π΄Π²ΡΠΌΡ Π²Π΅ΠΊΡΠΎΡΠ°ΠΌΠΈ $\vec{a}$ ΠΈ $vec{b}$ ΡΠ°Π²Π½Π° ΡΠΊΠ°Π»ΡΡΡ Π²Π΅ΠΊΡΠΎΡΠ½ΠΎΠ³ΠΎ ΠΏΡΠΎΠΈΠ·Π²Π΅Π΄Π΅Π½ΠΈΡ Π΄Π°Π½Π½ΡΡ Π΄Π²ΡΡ Π²Π΅ΠΊΡΠΎΡΠΎΠ².
ΠΡΠΎ ΡΠΎΠΎΡΠ½ΠΎΡΠ΅Π½ΠΈΠ΅ ΡΠΎΠ²ΡΠ΅ΠΌ Π½Π΅ΡΠ»ΠΎΠΆΠ½ΠΎ Π²ΡΠ²Π΅ΡΡΠΈ.
ΠΡΠΏΠΎΠΌΠ½ΠΈΠΌ ΡΠΎΡΠΌΡΠ»Ρ Π΄Π»Ρ Π½Π°Ρ ΠΎΠΆΠ΄Π΅Π½ΠΈΡ ΠΏΠ»ΠΎΡΠ°Π΄ΠΈ ΠΎΠ±ΡΡΠ½ΠΎΠ³ΠΎ ΠΏΠ°ΡΠ°Π»Π»Π΅Π»ΠΎΠ³ΡΠ°ΠΌΠΌΠ°, ΠΊΠΎΡΠΎΡΡΠΉ ΠΌΠΎΠΆΠ½ΠΎ ΠΎΡ Π°ΡΠ°ΠΊΡΠ΅ΡΠΈΠ·ΠΎΠ²Π°ΡΡ ΠΎΠ±ΡΠ°Π·ΡΡΡΠΈΠΌΠΈ Π΅Π³ΠΎ ΠΎΡΡΠ΅Π·ΠΊΠ°ΠΌΠΈ $a$ ΠΈ $b$:
$S = a \cdot b \cdot \sin Ξ±$
ΠΡΠΈ ΡΡΠΎΠΌ Π΄Π»ΠΈΠ½Ρ ΡΡΠΎΡΠΎΠ½ ΡΠ°Π²Π½Ρ ΡΠΊΠ°Π»ΡΡΠ½ΡΠΌ Π·Π½Π°ΡΠ΅Π½ΠΈΡΠΌ Π²Π΅ΠΊΡΠΎΡΠΎΠ² $\vec{a}$ ΠΈ $\vec{b}$, ΡΡΠΎ Π²ΠΏΠΎΠ»Π½Π΅ ΡΠ΅Π±Π΅ ΠΏΠΎΠ΄Ρ
ΠΎΠ΄ΠΈΡ Π½Π°ΠΌ, ΡΠΎ Π΅ΡΡΡ, ΡΠΊΠ°Π»ΡΡ Π²Π΅ΠΊΡΠΎΡΠ½ΠΎΠ³ΠΎ ΠΏΡΠΎΠΈΠ·Π²Π΅Π΄Π΅Π½ΠΈΡ Π΄Π°Π½Π½ΡΡ
Π²Π΅ΠΊΡΠΎΡΠΎΠ² ΠΈ Π±ΡΠ΄Π΅Ρ ΠΏΠ»ΠΎΡΠ°Π΄ΡΡ ΡΠ°ΡΡΠΌΠ°ΡΡΠΈΠ²Π°Π΅ΠΌΠΎΠΉ ΡΠΈΠ³ΡΡΡ. 2} = \sqrt{1878} β 43, 34$.
ΠΠ°Π½Π½ΡΠΉ Ρ ΠΎΠ΄ ΡΠ°ΡΡΡΠΆΠ΄Π΅Π½ΠΈΠΉ ΡΠΏΡΠ°Π²Π΅Π΄Π»ΠΈΠ² Π½Π΅ ΡΠΎΠ»ΡΠΊΠΎ Π΄Π»Ρ Π½Π°Ρ ΠΎΠΆΠ΄Π΅Π½ΠΈΡ ΠΏΠ»ΠΎΡΠ°Π΄ΠΈ Π² 3-Ρ ΠΌΠ΅ΡΠ½ΠΎΠΌ ΠΏΡΠΎΡΡΡΠ°Π½ΡΡΠ²Π΅, Π½ΠΎ ΠΈ Π΄Π»Ρ Π΄Π²ΡΡ ΠΌΠ΅ΡΠ½ΠΎΠ³ΠΎ. ΠΠΎΠ·Π½Π°ΠΊΠΎΠΌΡΡΠ΅ΡΡ ΡΠΎ ΡΠ»Π΅Π΄ΡΡΡΠ΅ΠΉ Π·Π°Π΄Π°ΡΠΊΠΎΠΉ Π½Π° ΡΡΡ ΡΠ΅ΠΌΡ.
ΠΡΠΈΠΌΠ΅Ρ 2
ΠΡΡΠΈΡΠ»ΠΈΡΡ ΠΏΠ»ΠΎΡΠ°Π΄Ρ ΠΏΠ°ΡΠ°Π»Π»Π΅Π»ΠΎΠ³ΡΠ°ΠΌΠΌΠ°, Π΅ΡΠ»ΠΈ Π΅Π³ΠΎ ΠΎΠ±ΡΠ°Π·ΡΡΡΠΈΠ΅ ΠΎΡΡΠ΅Π·ΠΊΠΈ Π·Π°Π΄Π°ΡΡΡΡ Π²Π΅ΠΊΡΠΎΡΠ°ΠΌΠΈ $\vec{m}$ Ρ ΠΊΠΎΠΎΡΠ΄ΠΈΠ½Π°ΡΠ°ΠΌΠΈ $\{2; 3\}$ ΠΈ $\vec{d}$ Ρ ΠΊΠΎΠΎΡΠ΄ΠΈΠ½Π°ΡΠ°ΠΌΠΈ $\{-5; 6\}$.
Π Π΅ΡΠ΅Π½ΠΈΠ΅:
ΠΡΠ° Π·Π°Π΄Π°ΡΠ° ΠΏΡΠ΅Π΄ΡΡΠ°Π²Π»ΡΠ΅Ρ ΡΠΎΠ±ΠΎΠΉ ΡΠ°ΡΡΠ½ΡΠΉ ΠΏΡΠΈΠΌΠ΅Ρ Π·Π°Π΄Π°ΡΠΊΠΈ 1, ΡΠ΅ΡΡΠ½Π½ΠΎΠΉ Π²ΡΡΠ΅, Π½ΠΎ ΠΏΡΠΈ ΡΡΠΎΠΌ ΠΎΠ±Π° Π²Π΅ΠΊΡΠΎΡΠ° Π»Π΅ΠΆΠ°Ρ Π² ΠΎΠ΄Π½ΠΎΠΉ ΠΏΠ»ΠΎΡΠΊΠΎΡΡΠΈ, Π° ΡΡΠΎ Π·Π½Π°ΡΠΈΡ, ΡΡΠΎ ΡΡΠ΅ΡΡΡ ΠΊΠΎΠΎΡΠ΄ΠΈΠ½Π°ΡΡ, $z$, ΠΌΠΎΠΆΠ½ΠΎ ΠΏΡΠΈΠ½ΡΡΡ Π·Π° Π½ΡΠ»Ρ.
ΠΠΎΠ΄Π²Π΅Π΄ΡΠΌ ΠΈΡΠΎΠ³ΠΈ ΠΏΠΎ Π²ΡΠ΅ΠΌΡ Π²ΡΡΠ΅ΡΠΊΠ°Π·Π°Π½Π½ΠΎΠΌΡ, ΠΏΠ»ΠΎΡΠ°Π΄Ρ ΠΏΠ°ΡΠ°Π»Π»Π΅Π»ΠΎΠ³ΡΠ°ΠΌΠΌΠ° ΡΠΎΡΡΠ°Π²ΠΈΡ:
$S = \begin{array} {||cc||} 2 & 3\\ -5 & 6 \\ \end{array} = \sqrt{12 + 15} =3 \sqrt3$.
ΠΡΠΈΠΌΠ΅Ρ 3
ΠΠ°Π½Ρ Π²Π΅ΠΊΡΠΎΡΡ $\vec{a} = 3i β j + k; \vec{b}= 5i$. ΠΠΏΡΠ΅Π΄Π΅Π»ΠΈΡΠ΅ ΠΏΠ»ΠΎΡΠ°Π΄Ρ ΠΎΠ±ΡΠ°Π·ΡΠ΅ΠΌΠΎΠ³ΠΎ ΠΈΠΌΠΈ ΠΏΠ°ΡΠ°Π»Π»Π΅Π»ΠΎΠ³ΡΠ°ΠΌΠΌΠ°.
$[ \vec{a} \times \vec{b}] = (3i β j + k) \times 5i = 15 [i \times i] β 5 [j \times i] + [5k\times i]$
Π£ΠΏΡΠΎΡΡΠΈΠΌ ΡΠΎΠ³Π»Π°ΡΠ½ΠΎ ΠΏΡΠΈΠ²Π΅Π΄ΡΠ½Π½ΠΎΠΉ ΡΠ°Π±Π»ΠΈΡΠ΅ Π΄Π»Ρ Π΅Π΄ΠΈΠ½ΠΈΡΠ½ΡΡ Π²Π΅ΠΊΡΠΎΡΠΎΠ²:
Π ΠΈΡΡΠ½ΠΎΠΊ 1. 2} = 5\sqrt{2}$.
ΠΡΠ΅Π΄ΡΠ΄ΡΡΠΈΠ΅ Π·Π°Π΄Π°ΡΠΈ Π±ΡΠ»ΠΈ ΠΎ Π²Π΅ΠΊΡΠΎΡΠ°Ρ , ΠΊΠΎΠΎΡΠ΄ΠΈΠ½Π°ΡΡ ΠΊΠΎΡΠΎΡΡΡ Π·Π°Π΄Π°Π½Ρ Π² Π΄Π΅ΠΊΠ°ΡΡΠΎΠ²ΠΎΠΉ ΡΠΈΡΡΠ΅ΠΌΠ΅ ΠΊΠΎΠΎΡΠ΄ΠΈΠ½Π°Ρ, Π½ΠΎ ΡΠ°ΡΡΠΌΠΎΡΡΠΈΠΌ ΡΠ°ΠΊΠΆΠ΅ ΡΠ»ΡΡΠ°ΠΉ, Π΅ΡΠ»ΠΈ ΡΠ³ΠΎΠ» ΠΌΠ΅ΠΆΠ΄Ρ Π±Π°Π·ΠΈΡΠ½ΡΠΌΠΈ Π²Π΅ΠΊΡΠΎΡΠ°ΠΌΠΈ ΠΎΡΠ»ΠΈΡΠ°Π΅ΡΡΡ ΠΎΡ $90Β°$:
ΠΡΠΈΠΌΠ΅Ρ 4
ΠΠ΅ΠΊΡΠΎΡ $\vec{d} = 2a + 3b$, $\vec{f}= a β 4b$, Π΄Π»ΠΈΠ½Ρ $\vec{a}$ ΠΈ $\vec{b}$ ΡΠ°Π²Π½Ρ ΠΌΠ΅ΠΆΠ΄Ρ ΡΠΎΠ±ΠΎΠΉ ΠΈ ΡΠ°Π²Π½Ρ Π΅Π΄ΠΈΠ½ΠΈΡΠ΅, Π° ΡΠ³ΠΎΠ» ΠΌΠ΅ΠΆΠ΄Ρ $\vec{a}$ ΠΈ $\vec{b}$ ΡΠ°Π²Π΅Π½ 45Β°.
Π Π΅ΡΠ΅Π½ΠΈΠ΅:
ΠΡΡΠΈΡΠ»ΠΈΠΌ Π²Π΅ΠΊΡΠΎΡΠ½ΠΎΠ΅ ΠΏΡΠΎΠΈΠ·Π²Π΅Π΄Π΅Π½ΠΈΠ΅ $\vec{d} \times \vec{f}$:
$[\vec{d} \times \vec{f} ]= (2a + 3b) \times ( a β 4b) = 2 [a \times a] β 8 [a \times b] + 3 [b \times a] β 12 [b \times b]$.
ΠΠ»Ρ Π²Π΅ΠΊΡΠΎΡΠ½ΡΡ ΠΏΡΠΎΠΈΠ·Π²Π΅Π΄Π΅Π½ΠΈΠΉ ΡΠΎΠ³Π»Π°ΡΠ½ΠΎ ΠΈΡ ΡΠ²ΠΎΠΉΡΡΠ²Π°ΠΌ ΡΠΏΡΠ°Π²Π΅Π΄Π»ΠΈΠ²ΠΎ ΡΠ»Π΅Π΄ΡΡΡΠ΅Π΅: $[a \times a]$ ΠΈ $[b \times b]$ ΡΠ°Π²Π½Ρ Π½ΡΠ»Ρ, $[b \times a] = β [a \times b]$.
ΠΡΠΏΠΎΠ»ΡΠ·ΡΠ΅ΠΌ ΡΡΠΎ Π΄Π»Ρ ΡΠΏΡΠΎΡΠ΅Π½ΠΈΡ:
$[\vec{d} \times \vec{f} ]= -8[a \times b] + 3 [b \times a] = -8[a \times b] β 3[a \times b] =-11[a \times b]$.
Π’Π΅ΠΏΠ΅ΡΡ Π²ΠΎΡΠΏΠΎΠ»ΡΠ·ΡΠ΅ΠΌΡΡ ΡΠΎΡΠΌΡΠ»ΠΎΠΉ $(1)$ :
$[\vec{d} \times \vec{f} ] = |-11 [a \times b]| = 11 \cdot |a| \cdot |b| \cdot \sin Ξ± = 11 \cdot 1 \cdot 1 \cdot \frac12=5,5$.
Π‘ΠΎΠΎΠ±ΡΠ΅ΡΡΠ²ΠΎ ΡΠΊΡΠΏΠ΅ΡΡΠΎΠ² ΠΠ²ΡΠΎΡ24
ΠΠ²ΡΠΎΡ ΡΡΠΎΠΉ ΡΡΠ°ΡΡΠΈ ΠΠ°ΡΠ° ΠΏΠΎΡΠ»Π΅Π΄Π½Π΅Π³ΠΎ ΠΎΠ±Π½ΠΎΠ²Π»Π΅Π½ΠΈΡ ΡΡΠ°ΡΡΠΈ: 07.05.2022
ΠΠ΅ΠΊΡΠΎΡΠ½ΠΎΠ΅ ΠΏΡΠΎΠΈΠ·Π²Π΅Π΄Π΅Π½ΠΈΠ΅ Π²Π΅ΠΊΡΠΎΡΠΎΠ² ΠΏΠ»ΠΎΡΠ°Π΄Ρ ΡΡΠ΅ΡΠ³ΠΎΠ»ΡΠ½ΠΈΠΊΠ°. Π‘Π²ΠΎΠΉΡΡΠ²Π° Π²Π΅ΠΊΡΠΎΡΠ½ΠΎΠ³ΠΎ ΠΏΡΠΎΠΈΠ·Π²Π΅Π΄Π΅Π½ΠΈΡ
ΠΠΏΡΠ΅Π΄Π΅Π»Π΅Π½ΠΈΠ΅. ΠΠ΅ΠΊΡΠΎΡΠ½ΡΠΌ ΠΏΡΠΎΠΈΠ·Π²Π΅Π΄Π΅Π½ΠΈΠ΅ΠΌ Π²Π΅ΠΊΡΠΎΡΠ° Π° (ΠΌΠ½ΠΎΠΆΠΈΠΌΠΎΠ΅) Π½Π° Π½Π΅ ΠΊΠΎΠ»Π»ΠΈΠ½Π΅Π°ΡΠ½ΡΠΉ Π΅ΠΌΡ Π²Π΅ΠΊΡΠΎΡ (ΠΌΠ½ΠΎΠΆΠΈΡΠ΅Π»Ρ) Π½Π°Π·ΡΠ²Π°Π΅ΡΡΡ ΡΡΠ΅ΡΠΈΠΉ Π²Π΅ΠΊΡΠΎΡ Ρ (ΠΏΡΠΎΠΈΠ·Π²Π΅Π΄Π΅Π½ΠΈΠ΅), ΠΊΠΎΡΠΎΡΡΠΉ ΡΡΡΠΎΠΈΡΡΡ ΡΠ»Π΅Π΄ΡΡΡΠΈΠΌ ΠΎΠ±ΡΠ°Π·ΠΎΠΌ:
1) Π΅Π³ΠΎ ΠΌΠΎΠ΄ΡΠ»Ρ ΡΠΈΡΠ»Π΅Π½Π½ΠΎ ΡΠ°Π²Π΅Π½ ΠΏΠ»ΠΎΡΠ°Π΄ΠΈ ΠΏΠ°ΡΠ°Π»Π»Π΅Π»ΠΎΠ³ΡΠ°ΠΌΠΌΠ° Π½Π° ΡΠΈΡ. 155), ΠΏΠΎΡΡΡΠΎΠ΅Π½Π½ΠΎΠ³ΠΎ Π½Π° Π²Π΅ΠΊΡΠΎΡΠ°Ρ Ρ. Π΅. ΠΎΠ½ ΡΠ°Π²Π΅Π½ Π½Π°ΠΏΡΠ°Π²Π»Π΅Π½ΠΈΠ΅ ΠΏΠ΅ΡΠΏΠ΅Π½Π΄ΠΈΠΊΡΠ»ΡΡΠ½ΠΎ ΠΏΠ»ΠΎΡΠΊΠΎΡΡΠΈ ΡΠΏΠΎΠΌΡΠ½ΡΡΠΎΠ³ΠΎ ΠΏΠ°ΡΠ°Π»Π»Π΅Π»ΠΎΠ³ΡΠ°ΠΌΠΌΠ°;
3) ΠΏΡΠΈ ΡΡΠΎΠΌ Π½Π°ΠΏΡΠ°Π²Π»Π΅Π½ΠΈΠ΅ Π²Π΅ΠΊΡΠΎΡΠ° Ρ Π²ΡΠ±ΠΈΡΠ°Π΅ΡΡΡ (ΠΈΠ· Π΄Π²ΡΡ Π²ΠΎΠ·ΠΌΠΎΠΆΠ½ΡΡ ) ΡΠ°ΠΊ, ΡΡΠΎΠ±Ρ Π²Π΅ΠΊΡΠΎΡΡ Ρ ΡΠΎΡΡΠ°Π²Π»ΡΠ»ΠΈ ΠΏΡΠ°Π²ΡΡ ΡΠΈΡΡΠ΅ΠΌΡ (Β§ 110).
ΠΠ±ΠΎΠ·Π½Π°ΡΠ΅Π½ΠΈΠ΅: ΠΈΠ»ΠΈ
ΠΠΎΠΏΠΎΠ»Π½Π΅Π½ΠΈΠ΅ ΠΊ ΠΎΠΏΡΠ΅Π΄Π΅Π»Π΅Π½ΠΈΡ. ΠΡΠ»ΠΈ Π²Π΅ΠΊΡΠΎΡΡ ΠΊΠΎΠ»Π»ΠΈΠ½Π΅Π°ΡΠ½Ρ, ΡΠΎ ΡΠΈΠ³ΡΡΠ΅ ΡΡΠΈΡΠ°Ρ Π΅Π΅ (ΡΡΠ»ΠΎΠ²Π½ΠΎ) ΠΏΠ°ΡΠ°Π»Π»Π΅Π»ΠΎΠ³ΡΠ°ΠΌΠΌΠΎΠΌ, Π΅ΡΡΠ΅ΡΡΠ²Π΅Π½Π½ΠΎ ΠΏΡΠΈΠΏΠΈΡΠ°ΡΡ Π½ΡΠ»Π΅Π²ΡΡ ΠΏΠ»ΠΎΡΠ°Π΄Ρ. ΠΠΎΡΡΠΎΠΌΡ Π²Π΅ΠΊΡΠΎΡΠ½ΠΎΠ΅ ΠΏΡΠΎΠΈΠ·Π²Π΅Π΄Π΅Π½ΠΈΠ΅ ΠΊΠΎΠ»Π»ΠΈΠ½Π΅Π°ΡΠ½ΡΡ Π²Π΅ΠΊΡΠΎΡΠΎΠ² ΡΡΠΈΡΠ°Π΅ΡΡΡ ΡΠ°Π²Π½ΡΠΌ Π½ΡΠ»Ρ-Π²Π΅ΠΊΡΠΎΡΡ.
ΠΠΎΡΠΊΠΎΠ»ΡΠΊΡ Π½ΡΠ»Ρ-Π²Π΅ΠΊΡΠΎΡΡ ΠΌΠΎΠΆΠ½ΠΎ ΠΏΡΠΈΠΏΠΈΡΠ°ΡΡ Π»ΡΠ±ΠΎΠ΅ Π½Π°ΠΏΡΠ°Π²Π»Π΅Π½ΠΈΠ΅, ΡΡΠΎ ΡΠΎΠ³Π»Π°ΡΠ΅Π½ΠΈΠ΅ Π½Π΅ ΠΏΡΠΎΡΠΈΠ²ΠΎΡΠ΅ΡΠΈΡ ΠΏΡΠ½ΠΊΡΠ°ΠΌ 2 ΠΈ 3 ΠΎΠΏΡΠ΅Π΄Π΅Π»Π΅Π½ΠΈΡ.
ΠΠ°ΠΌΠ΅ΡΠ°Π½ΠΈΠ΅ 1. Π ΡΠ΅ΡΠΌΠΈΠ½Π΅ Β«Π²Π΅ΠΊΡΠΎΡΠ½ΠΎΠ΅ ΠΏΡΠΎΠΈΠ·Π²Π΅Π΄Π΅Π½ΠΈΠ΅Β» ΠΏΠ΅ΡΠ²ΠΎΠ΅ ΡΠ»ΠΎΠ²ΠΎ ΡΠΊΠ°Π·ΡΠ²Π°Π΅Ρ Π½Π° ΡΠΎ, ΡΡΠΎ ΡΠ΅Π·ΡΠ»ΡΡΠ°Ρ Π΄Π΅ΠΉΡΡΠ²ΠΈΡ Π΅ΡΡΡ Π²Π΅ΠΊΡΠΎΡ (Π² ΠΏΡΠΎΡΠΈΠ²ΠΎΠΏΠΎΠ»ΠΎΠΆΠ½ΠΎΡΡΡ ΡΠΊΠ°Π»ΡΡΠ½ΠΎΠΌΡ ΠΏΡΠΎΠΈΠ·Π²Π΅Π΄Π΅Π½ΠΈΡ; ΡΡ. Β§ 104, Π·Π°ΠΌΠ΅ΡΠ°Π½ΠΈΠ΅ 1).
ΠΡΠΈΠΌΠ΅Ρ 1. ΠΠ°ΠΉΡΠΈ Π²Π΅ΠΊΡΠΎΡΠ½ΠΎΠ΅ ΠΏΡΠΎΠΈΠ·Π²Π΅Π΄Π΅Π½ΠΈΠ΅ Π³Π΄Π΅ ΠΎΡΠ½ΠΎΠ²Π½ΡΠ΅ Π²Π΅ΠΊΡΠΎΡΡ ΠΏΡΠ°Π²ΠΎΠΉ ΡΠΈΡΡΠ΅ΠΌΡ ΠΊΠΎΠΎΡΠ΄ΠΈΠ½Π°Ρ (ΡΠΈΡ. 156).
1. Π’Π°ΠΊ ΠΊΠ°ΠΊ Π΄Π»ΠΈΠ½Ρ ΠΎΡΠ½ΠΎΠ²Π½ΡΡ Π²Π΅ΠΊΡΠΎΡΠΎΠ² ΡΠ°Π²Π½Ρ Π΅Π΄ΠΈΠ½ΠΈΡΠ΅ ΠΌΠ°ΡΡΡΠ°Π±Π°, ΡΠΎ ΠΏΠ»ΠΎΡΠ°Π΄Ρ ΠΏΠ°ΡΠ°Π»Π»Π΅Π»ΠΎΠ³ΡΠ°ΠΌΠΌΠ° (ΠΊΠ²Π°Π΄ΡΠ°ΡΠ°) ΡΠΈΡΠ»Π΅Π½Π½ΠΎ ΡΠ°Π²Π½Π° Π΅Π΄ΠΈΠ½ΠΈΡΠ΅. ΠΠ½Π°ΡΠΈΡ, ΠΌΠΎΠ΄ΡΠ»Ρ Π²Π΅ΠΊΡΠΎΡΠ½ΠΎΠ³ΠΎ ΠΏΡΠΎΠΈΠ·Π²Π΅Π΄Π΅Π½ΠΈΡ ΡΠ°Π²Π΅Π½ Π΅Π΄ΠΈΠ½ΠΈΡΠ΅.
2. Π’Π°ΠΊ ΠΊΠ°ΠΊ ΠΏΠ΅ΡΠΏΠ΅Π½Π΄ΠΈΠΊΡΠ»ΡΡ ΠΊ ΠΏΠ»ΠΎΡΠΊΠΎΡΡΠΈ Π΅ΡΡΡ ΠΎΡΡ ΡΠΎ ΠΈΡΠΊΠΎΠΌΠΎΠ΅ Π²Π΅ΠΊΡΠΎΡΠ½ΠΎΠ΅ ΠΏΡΠΎΠΈΠ·Π²Π΅Π΄Π΅Π½ΠΈΠ΅ Π΅ΡΡΡ Π²Π΅ΠΊΡΠΎΡ, ΠΊΠΎΠ»Π»ΠΈΠ½Π΅Π°ΡΠ½ΡΠΉ Π²Π΅ΠΊΡΠΎΡΡ ΠΊ; Π° ΡΠ°ΠΊ ΠΊΠ°ΠΊ ΠΎΠ±Π° ΠΎΠ½ΠΈ ΠΈΠΌΠ΅ΡΡ ΠΌΠΎΠ΄ΡΠ»Ρ 1, ΡΠΎ ΠΈΡΠΊΠΎΠΌΠΎΠ΅ Π²Π΅ΠΊΡΠΎΡΠ½ΠΎΠ΅ ΠΏΡΠΎΠΈΠ·Π²Π΅Π΄Π΅Π½ΠΈΠ΅ ΡΠ°Π²Π½ΠΎ Π»ΠΈΠ±ΠΎ k, Π»ΠΈΠ±ΠΎ -k.
3. ΠΠ· ΡΡΠΈΡ Π΄Π²ΡΡ Π²ΠΎΠ·ΠΌΠΎΠΆΠ½ΡΡ Π²Π΅ΠΊΡΠΎΡΠΎΠ² Π½Π°Π΄ΠΎ Π²ΡΠ±ΡΠ°ΡΡ ΠΏΠ΅ΡΠ²ΡΠΉ, ΡΠ°ΠΊ ΠΊΠ°ΠΊ Π²Π΅ΠΊΡΠΎΡΡ ΠΊ ΠΎΠ±ΡΠ°Π·ΡΡΡ ΠΏΡΠ°Π²ΡΡ ΡΠΈΡΡΠ΅ΠΌΡ (Π° Π²Π΅ΠΊΡΠΎΡΡ Π»Π΅Π²ΡΡ).
ΠΡΠΈΠΌΠ΅Ρ 2. ΠΠ°ΠΉΡΠΈ Π²Π΅ΠΊΡΠΎΡΠ½ΠΎΠ΅ ΠΏΡΠΎΠΈΠ·Π²Π΅Π΄Π΅Π½ΠΈΠ΅
Π Π΅ΡΠ΅Π½ΠΈΠ΅. ΠΠ°ΠΊ Π² ΠΏΡΠΈΠΌΠ΅ΡΠ΅ 1, Π·Π°ΠΊΠ»ΡΡΠ°Π΅ΠΌ, ΡΡΠΎ Π²Π΅ΠΊΡΠΎΡ ΡΠ°Π²Π΅Π½ Π»ΠΈΠ±ΠΎ k, Π»ΠΈΠ±ΠΎ -k. ΠΠΎ ΡΠ΅ΠΏΠ΅ΡΡ Π½Π°Π΄ΠΎ Π²ΡΠ±ΡΠ°ΡΡ -k, ΡΠ°ΠΊ ΠΊΠ°ΠΊ Π²Π΅ΠΊΡΠΎΡΡ ΠΎΠ±ΡΠ°Π·ΡΡΡ ΠΏΡΠ°Π²ΡΡ ΡΠΈΡΡΠ΅ΠΌΡ (Π° Π²Π΅ΠΊΡΠΎΡΡ Π»Π΅Π²ΡΡ). ΠΡΠ°ΠΊ,
ΠΡΠΈΠΌΠ΅Ρ 3. ΠΠ΅ΠΊΡΠΎΡΡ ΠΈΠΌΠ΅ΡΡ Π΄Π»ΠΈΠ½Ρ, ΡΠΎΠΎΡΠ²Π΅ΡΡΡΠ²Π΅Π½Π½ΠΎ ΡΠ°Π²Π½ΡΠ΅ 80 ΠΈ 50 ΡΠΌ, ΠΈ ΠΎΠ±ΡΠ°Π·ΡΡΡ ΡΠ³ΠΎΠ» 30Β°. ΠΡΠΈΠ½ΡΠ² Π·Π° Π΅Π΄ΠΈΠ½ΠΈΡΡ Π΄Π»ΠΈΠ½Ρ ΠΌΠ΅ΡΡ, Π½Π°ΠΉΡΠΈ Π΄Π»ΠΈΠ½Ρ Π²Π΅ΠΊΡΠΎΡΠ½ΠΎΠ³ΠΎ ΠΏΡΠΎΠΈΠ·Π²Π΅Π΄Π΅Π½ΠΈΡ Π°
Π Π΅ΡΠ΅Π½ΠΈΠ΅. ΠΠ»ΠΎΡΠ°Π΄Ρ ΠΏΠ°ΡΠ°Π»Π»Π΅Π»ΠΎΠ³ΡΠ°ΠΌΠΌΠ°, ΠΏΠΎΡΡΡΠΎΠ΅Π½Π½ΠΎΠ³ΠΎ Π½Π° Π²Π΅ΠΊΡΠΎΡΠ°Ρ ΡΠ°Π²Π½Π° ΠΠ»ΠΈΠ½Π° ΠΈΡΠΊΠΎΠΌΠΎΠ³ΠΎ Π²Π΅ΠΊΡΠΎΡΠ½ΠΎΠ³ΠΎ ΠΏΡΠΎΠΈΠ·Π²Π΅Π΄Π΅Π½ΠΈΡ ΡΠ°Π²Π½Π°
ΠΡΠΈΠΌΠ΅Ρ 4. ΠΠ°ΠΉΡΠΈ Π΄Π»ΠΈΠ½Ρ Π²Π΅ΠΊΡΠΎΡΠ½ΠΎΠ³ΠΎ ΠΏΡΠΎΠΈΠ·Π²Π΅Π΄Π΅Π½ΠΈΡ ΡΠ΅Ρ ΠΆΠ΅ Π²Π΅ΠΊΡΠΎΡΠΎΠ², ΠΏΡΠΈΠ½ΡΠ² Π·Π° Π΅Π΄ΠΈΠ½ΠΈΡΡ Π΄Π»ΠΈΠ½Ρ ΡΠ°Π½ΡΠΈΠΌΠ΅ΡΡ.
Π Π΅ΡΠ΅Π½ΠΈΠ΅. Π’Π°ΠΊ ΠΊΠ°ΠΊ ΠΏΠ»ΠΎΡΠ°Π΄Ρ ΠΏΠ°ΡΠ°Π»Π»Π΅Π»ΠΎΠ³ΡΠ°ΠΌΠΌΠ°, ΠΏΠΎΡΡΡΠΎΠ΅Π½Π½ΠΎΠ³ΠΎ Π½Π° Π²Π΅ΠΊΡΠΎΡΠ°Ρ ΡΠ°Π²Π½Π° ΡΠΎ Π΄Π»ΠΈΠ½Π° Π²Π΅ΠΊΡΠΎΡΠ½ΠΎΠ³ΠΎ ΠΏΡΠΎΠΈΠ·Π²Π΅Π΄Π΅Π½ΠΈΡ ΡΠ°Π²Π½Π° 2000 ΡΠΌ, Ρ. Π΅.
ΠΠ· ΡΡΠ°Π²Π½Π΅Π½ΠΈΡ ΠΏΡΠΈΠΌΠ΅ΡΠΎΠ² 3 ΠΈ 4 Π²ΠΈΠ΄Π½ΠΎ, ΡΡΠΎ Π΄Π»ΠΈΠ½Π° Π²Π΅ΠΊΡΠΎΡΠ° Π·Π°Π²ΠΈΡΠΈΡ Π½Π΅ ΡΠΎΠ»ΡΠΊΠΎ ΠΎΡ Π΄Π»ΠΈΠ½ ΡΠΎΠΌΠ½ΠΎΠΆΠΈΡΠ΅Π»Π΅ΠΉ Π½ΠΎ ΡΠ°ΠΊΠΆΠ΅ ΠΈ ΠΎΡ Π²ΡΠ±ΠΎΡΠ° Π΅Π΄ΠΈΠ½ΠΈΡΡ Π΄Π»ΠΈΠ½Ρ.
Π€ΠΈΠ·ΠΈΡΠ΅ΡΠΊΠΈΠΉ ΡΠΌΡΡΠ» Π²Π΅ΠΊΡΠΎΡΠ½ΠΎΠ³ΠΎ ΠΏΡΠΎΠΈΠ·Π²Π΅Π΄Π΅Π½ΠΈΡ. ΠΠ· ΠΌΠ½ΠΎΠ³ΠΎΡΠΈΡΠ»Π΅Π½Π½ΡΡ ΡΠΈΠ·ΠΈΡΠ΅ΡΠΊΠΈΡ Π²Π΅Π»ΠΈΡΠΈΠ½, ΠΈΠ·ΠΎΠ±ΡΠ°ΠΆΠ°Π΅ΠΌΡΡ Π²Π΅ΠΊΡΠΎΡΠ½ΡΠΌ ΠΏΡΠΎΠΈΠ·Π²Π΅Π΄Π΅Π½ΠΈΠ΅ΠΌ, ΡΠ°ΡΡΠΌΠΎΡΡΠΈΠΌ ΡΠΎΠ»ΡΠΊΠΎ ΠΌΠΎΠΌΠ΅Π½Ρ ΡΠΈΠ»Ρ.
ΠΡΡΡΡ Π Π΅ΡΡΡ ΡΠΎΡΠΊΠ° ΠΏΡΠΈΠ»ΠΎΠΆΠ΅Π½ΠΈΡ ΡΠΈΠ»Ρ ΠΠΎΠΌΠ΅Π½ΡΠΎΠΌ ΡΠΈΠ»Ρ ΠΎΡΠ½ΠΎΡΠΈΡΠ΅Π»ΡΠΊΠΎ ΡΠΎΡΠΊΠΈ Π Π½Π°Π·ΡΠ²Π°Π΅ΡΡΡ Π²Π΅ΠΊΡΠΎΡΠ½ΠΎΠ΅ ΠΏΡΠΎΠΈΠ·Π²Π΅Π΄Π΅Π½ΠΈΠ΅ Π’Π°ΠΊ ΠΊΠ°ΠΊ ΠΌΠΎΠ΄ΡΠ»Ρ ΡΡΠΎΠ³ΠΎ Π²Π΅ΠΊΡΠΎΡΠ½ΠΎΠ³ΠΎ ΠΏΡΠΎΠΈΠ·Π²Π΅Π΄Π΅Π½ΠΈΡ ΡΠΈΡΠ»Π΅Π½Π½ΠΎ ΡΠ°Π²Π΅Π½ ΠΏΠ»ΠΎΡΠ°Π΄ΠΈ ΠΏΠ°ΡΠ°Π»Π»Π΅Π»ΠΎΠ³ΡΠ°ΠΌΠΌΠ° (ΡΠΈΡ. 157), ΡΠΎ ΠΌΠΎΠ΄ΡΠ»Ρ ΠΌΠΎΠΌΠ΅Π½ΡΠ° ΡΠ°Π²Π½ΡΠ΅ΡΡΡ ΠΏΡΠΎΠΈΠ·Π²Π΅Π΄Π΅Π½ΠΈΡ ΠΎΡΠ½ΠΎΠ²Π°Π½ΠΈΡ Π½Π° Π²ΡΡΠΎΡΡ Ρ. Π΅. ΡΠΈΠ»Π΅, ΡΠΌΠ½ΠΎΠΆΠ΅Π½Π½ΠΎΠΉ Π½Π° ΡΠ°ΡΡΡΠΎΡΠ½ΠΈΠ΅ ΠΎΡ ΡΠΎΡΠΊΠΈ Π Π΄ΠΎ ΠΏΡΡΠΌΠΎΠΉ, Π²Π΄ΠΎΠ»Ρ ΠΊΠΎΡΠΎΡΠΎΠΉ Π΄Π΅ΠΉΡΡΠ²ΡΠ΅Ρ ΡΠΈΠ»Π°.
Π ΠΌΠ΅Ρ Π°Π½ΠΈΠΊΠ΅ Π΄ΠΎΠΊΠ°Π·ΡΠ²Π°Π΅ΡΡΡ, ΡΡΠΎ Π΄Π»Ρ ΡΠ°Π²Π½ΠΎΠ²Π΅ΡΠΈΡ ΡΠ²Π΅ΡΠ΄ΠΎΠ³ΠΎ ΡΠ΅Π»Π° Π½Π΅ΠΎΠ±Ρ ΠΎΠ΄ΠΈΠΌΠΎ, ΡΡΠΎΠ±Ρ ΡΠ°Π²Π½ΡΠ»Π°ΡΡ Π½ΡΠ»Ρ Π½Π΅ ΡΠΎΠ»ΡΠΊΠΎ ΡΡΠΌΠΌΠ° Π²Π΅ΠΊΡΠΎΡΠΎΠ² , ΠΏΡΠ΅Π΄ΡΡΠ°Π²Π»ΡΡΡΠΈΡ ΡΠΈΠ»Ρ, ΠΏΡΠΈΠ»ΠΎΠΆΠ΅Π½Π½ΡΠ΅ ΠΊ ΡΠ΅Π»Ρ, Π½ΠΎ ΡΠ°ΠΊΠΆΠ΅ ΠΈ ΡΡΠΌΠΌΠ° ΠΌΠΎΠΌΠ΅Π½ΡΠΎΠ² ΡΠΈΠ». Π ΡΠΎΠΌ ΡΠ»ΡΡΠ°Π΅, ΠΊΠΎΠ³Π΄Π° Π²ΡΠ΅ ΡΠΈΠ»Ρ ΠΏΠ°ΡΠ°Π»Π»Π΅Π»ΡΠ½Ρ ΠΎΠ΄Π½ΠΎΠΉ ΠΏΠ»ΠΎΡΠΊΠΎΡΡΠΈ, ΡΠ»ΠΎΠΆΠ΅Π½ΠΈΠ΅ Π²Π΅ΠΊΡΠΎΡΠΎΠ², ΠΏΡΠ΅Π΄ΡΡΠ°Π²Π»ΡΡΡΠΈΡ ΠΌΠΎΠΌΠ΅Π½ΡΡ, ΠΌΠΎΠΆΠ½ΠΎ Π·Π°ΠΌΠ΅Π½ΠΈΡΡ ΡΠ»ΠΎΠΆΠ΅Π½ΠΈΠ΅ΠΌ ΠΈ Π²ΡΡΠΈΡΠ°Π½ΠΈΠ΅ΠΌ ΠΈΡ ΠΌΠΎΠ΄ΡΠ»Π΅ΠΉ. ΠΠΎ ΠΏΡΠΈ ΠΏΡΠΎΠΈΠ·Π²ΠΎΠ»ΡΠ½ΡΡ Π½Π°ΠΏΡΠ°Π²Π»Π΅Π½ΠΈΡΡ ΡΠΈΠ» ΡΠ°ΠΊΠ°Ρ Π·Π°ΠΌΠ΅Π½Π° Π½Π΅Π²ΠΎΠ·ΠΌΠΎΠΆΠ½Π°. Π ΡΠΎΠΎΡΠ²Π΅ΡΡΡΠ²ΠΈΠΈ Ρ ΡΡΠΈΠΌ Π²Π΅ΠΊΡΠΎΡΠ½ΠΎΠ΅ ΠΏΡΠΎΠΈΠ·Π²Π΅Π΄Π΅Π½ΠΈΠ΅ ΠΎΠΏΡΠ΅Π΄Π΅Π»ΡΠ΅ΡΡΡ ΠΈΠΌΠ΅Π½Π½ΠΎ ΠΊΠ°ΠΊ Π²Π΅ΠΊΡΠΎΡ, Π° Π½Π΅ ΠΊΠ°ΠΊ ΡΠΈΡΠ»ΠΎ.
7.1. ΠΠΏΡΠ΅Π΄Π΅Π»Π΅Π½ΠΈΠ΅ Π²Π΅ΠΊΡΠΎΡΠ½ΠΎΠ³ΠΎ ΠΏΡΠΎΠΈΠ·Π²Π΅Π΄Π΅Π½ΠΈΡ
Π’ΡΠΈ Π½Π΅ΠΊΠΎΠΌΠΏΠ»Π°Π½Π°ΡΠ½ΡΡ Π²Π΅ΠΊΡΠΎΡΠ° a , b ΠΈ Ρ , Π²Π·ΡΡΡΠ΅ Π² ΡΠΊΠ°Π·Π°Π½Π½ΠΎΠΌ ΠΏΠΎΡΡΠ΄ΠΊΠ΅, ΠΎΠ±ΡΠ°Π·ΡΡΡ ΠΏΡΠ°Π²ΡΡ ΡΡΠΎΠΉΠΊΡ, Π΅ΡΠ»ΠΈ Ρ ΠΊΠΎΠ½ΡΠ° ΡΡΠ΅ΡΡΠ΅Π³ΠΎ Π²Π΅ΠΊΡΠΎΡΠ° Ρ ΠΊΡΠ°ΡΡΠ°ΠΉΡΠΈΠΉ ΠΏΠΎΠ²ΠΎΡΠΎΡ ΠΎΡ ΠΏΠ΅ΡΠ²ΠΎΠ³ΠΎ Π²Π΅ΠΊΡΠΎΡΠ° Π° ΠΊΠΎ Π²ΡΠΎΡΠΎΠΌΡ Π²Π΅ΠΊΡΠΎΡΡ b Π²ΠΈΠ΄Π΅Π½ ΡΠΎΠ²Π΅ΡΡΠ°ΡΡΠΈΠΌΡΡ ΠΏΡΠΎΡΠΈΠ² ΡΠ°ΡΠΎΠ²ΠΎΠΉ ΡΡΡΠ΅Π»ΠΊΠΈ, ΠΈ Π»Π΅Π²ΡΡ, Π΅ΡΠ»ΠΈ ΠΏΠΎ ΡΠ°ΡΠΎΠ²ΠΎΠΉ (ΡΠΌ. ΡΠΈΡ. 16).
ΠΠ΅ΠΊΡΠΎΡΠ½ΡΠΌ ΠΏΡΠΎΠΈΠ·Π²Π΅Π΄Π΅Π½ΠΈΠ΅ΠΌ Π²Π΅ΠΊΡΠΎΡΠ° Π° Π½Π° Π²Π΅ΠΊΡΠΎΡ b Π½Π°Π·ΡΠ²Π°Π΅ΡΡΡ Π²Π΅ΠΊΡΠΎΡ Ρ , ΠΊΠΎΡΠΎΡΡΠΉ:
1. j
;
2) |k |=1, Π½ΠΎ | i x j | = |i | |J | sin(90Β°)=1;
3) Π²Π΅ΠΊΡΠΎΡΡ i , j ΠΈ k ΠΎΠ±ΡΠ°Π·ΡΡΡ ΠΏΡΠ°Π²ΡΡ ΡΡΠΎΠΉΠΊΡ (ΡΠΌ. ΡΠΈΡ. 16).
7.2. Π‘Π²ΠΎΠΉΡΡΠ²Π° Π²Π΅ΠΊΡΠΎΡΠ½ΠΎΠ³ΠΎ ΠΏΡΠΎΠΈΠ·Π²Π΅Π΄Π΅Π½ΠΈΡ
1. ΠΡΠΈ ΠΏΠ΅ΡΠ΅ΡΡΠ°Π½ΠΎΠ²ΠΊΠ΅ ΡΠΎΠΌΠ½ΠΎΠΆΠΈΡΠ΅Π»Π΅ΠΉ Π²Π΅ΠΊΡΠΎΡΠ½ΠΎΠ΅ ΠΏΡΠΎΠΈΠ·Π²Π΅Π΄Π΅Π½ΠΈΠ΅ ΠΌΠ΅Π½ΡΠ΅Ρ Π·Π½Π°ΠΊ, Ρ.Π΅. Π° Ρ b =(b Ρ a ) (ΡΠΌ. ΡΠΈΡ. 19).
ΠΠ΅ΠΊΡΠΎΡΡ Π° Ρ b ΠΈ b Ρ Π° ΠΊΠΎΠ»Π»ΠΈΠ½Π΅Π°ΡΠ½Ρ, ΠΈΠΌΠ΅ΡΡ ΠΎΠ΄ΠΈΠ½Π°ΠΊΠΎΠ²ΡΠ΅ ΠΌΠΎΠ΄ΡΠ»ΠΈ (ΠΏΠ»ΠΎΡΠ°Π΄Ρ ΠΏΠ°ΡΠ°Π»Π»Π΅Π»ΠΎΠ³ΡΠ°ΠΌΠΌΠ° ΠΎΡΡΠ°Π΅ΡΡΡ Π½Π΅ΠΈΠ·ΠΌΠ΅Π½Π½ΠΎΠΉ), Π½ΠΎ ΠΏΡΠΎΡΠΈΠ²ΠΎΠΏΠΎΠ»ΠΎΠΆΠ½ΠΎ Π½Π°ΠΏΡΠ°Π²Π»Π΅Π½Ρ (ΡΡΠΎΠΉΠΊΠΈ Π° , b , Π° Ρ b ΠΈ a , b , b x a ΠΏΡΠΎΡΠΈΠ²ΠΎΠΏΠΎΠ»ΠΎΠΆΠ½ΠΎΠΉ ΠΎΡΠΈΠ΅Π½ΡΠ°ΡΠΈΠΈ). Π‘ΡΠ°Π»ΠΎ Π±ΡΡΡ a xb = -(b xa ).
2. ΠΠ΅ΠΊΡΠΎΡΠ½ΠΎΠ΅ ΠΏΡΠΎΠΈΠ·Π²Π΅Π΄Π΅Π½ΠΈΠ΅ ΠΎΠ±Π»Π°Π΄Π°Π΅Ρ ΡΠΎΡΠ΅ΡΠ°ΡΠ΅Π»ΡΠ½ΡΠΌ ΡΠ²ΠΎΠΉΡΡΠ²ΠΎΠΌ ΠΎΡΠ½ΠΎΡΠΈΡΠ΅Π»ΡΠ½ΠΎ ΡΠΊΠ°Π»ΡΡΠ½ΠΎΠ³ΠΎ ΠΌΠ½ΠΎΠΆΠΈΡΠ΅Π»Ρ, Ρ. Π΅. l (Π° Ρ b ) = (l Π° ) Ρ b = Π° Ρ (l b ).
ΠΡΡΡΡ l >0. ΠΠ΅ΠΊΡΠΎΡ l (Π° Ρ b ) ΠΏΠ΅ΡΠΏΠ΅Π½Π΄ΠΈΠΊΡΠ»ΡΡΠ΅Π½ Π²Π΅ΠΊΡΠΎΡΠ°ΠΌ Π° ΠΈ b . ΠΠ΅ΠΊΡΠΎΡ ( l Π° )Ρ b ΡΠ°ΠΊΠΆΠ΅ ΠΏΠ΅ΡΠΏΠ΅Π½Π΄ΠΈΠΊΡΠ»ΡΡΠ΅Π½ Π²Π΅ΠΊΡΠΎΡΠ°ΠΌ Π° ΠΈ b (Π²Π΅ΠΊΡΠΎΡΡ Π° , l Π° Π»Π΅ΠΆΠ°Ρ Π² ΠΎΠ΄Π½ΠΎΠΉ ΠΏΠ»ΠΎΡΠΊΠΎΡΡΠΈ). ΠΠ½Π°ΡΠΈΡ, Π²Π΅ΠΊΡΠΎΡΡ l (Π° Ρ b ) ΠΈ ( l Π° )Ρ b ΠΊΠΎΠ»Π»ΠΈΠ½Π΅Π°ΡΠ½Ρ. ΠΡΠ΅Π²ΠΈΠ΄Π½ΠΎ, ΡΡΠΎ ΠΈ Π½Π°ΠΏΡΠ°Π²Π»Π΅Π½ΠΈΡ ΠΈΡ ΡΠΎΠ²ΠΏΠ°Π΄Π°ΡΡ. ΠΠΌΠ΅ΡΡ ΠΎΠ΄ΠΈΠ½Π°ΠΊΠΎΠ²ΡΡ Π΄Π»ΠΈΠ½Ρ:
ΠΠΎΡΡΠΎΠΌΡ l
(a
Ρ
b
)=
l
Π°
Ρ
b
. ΠΠ½Π°Π»ΠΎΠ³ΠΈΡΠ½ΠΎ Π΄ΠΎΠΊΠ°Π·ΡΠ²Π°Π΅ΡΡΡ ΠΏΡΠΈ
l
3. ΠΠ²Π° Π½Π΅Π½ΡΠ»Π΅Π²ΡΡ Π²Π΅ΠΊΡΠΎΡΠ° Π° ΠΈ b ΠΊΠΎΠ»Π»ΠΈΠ½Π΅Π°ΡΠ½Ρ ΡΠΎΠ³Π΄Π° ΠΈ ΡΠΎΠ»ΡΠΊΠΎ ΡΠΎΠ³Π΄Π°, ΠΊΠΎΠ³Π΄Π° ΠΈΡ Π²Π΅ΠΊΡΠΎΡΠ½ΠΎΠ΅ ΠΏΡΠΎΠΈΠ·Π²Π΅Π΄Π΅Π½ΠΈΠ΅ ΡΠ°Π²Π½ΠΎ Π½ΡΠ»Π΅Π²ΠΎΠΌΡ Π²Π΅ΠΊΡΠΎΡΡ, Ρ. Π΅. Π° ||b Π° Ρ b =0 .
Π ΡΠ°ΡΡΠ½ΠΎΡΡΠΈ, i *i =j *j =k *k =0 .
4. ΠΠ΅ΠΊΡΠΎΡΠ½ΠΎΠ΅ ΠΏΡΠΎΠΈΠ·Π²Π΅Π΄Π΅Π½ΠΈΠ΅ ΠΎΠ±Π»Π°Π΄Π°Π΅Ρ ΡΠ°ΡΠΏΡΠ΅Π΄Π΅Π»ΠΈΡΠ΅Π»ΡΠ½ΡΠΌ ΡΠ²ΠΎΠΉΡΡΠ²ΠΎΠΌ:
(a +b ) Ρ Ρ = Π° Ρ Ρ +b Ρ Ρ .
ΠΡΠΈΠΌΠ΅ΠΌ Π±Π΅Π· Π΄ΠΎΠΊΠ°Π·Π°ΡΠ΅Π»ΡΡΡΠ²Π°.
7.3. ΠΡΡΠ°ΠΆΠ΅Π½ΠΈΠ΅ Π²Π΅ΠΊΡΠΎΡΠ½ΠΎΠ³ΠΎ ΠΏΡΠΎΠΈΠ·Π²Π΅Π΄Π΅Π½ΠΈΡ ΡΠ΅ΡΠ΅Π· ΠΊΠΎΠΎΡΠ΄ΠΈΠ½Π°ΡΡ
ΠΡ Π±ΡΠ΄Π΅ΠΌ ΠΈΡΠΏΠΎΠ»ΡΠ·ΠΎΠ²Π°ΡΡ ΡΠ°Π±Π»ΠΈΡΡ Π²Π΅ΠΊΡΠΎΡΠ½ΠΎΠ³ΠΎ ΠΏΡΠΎΠΈΠ·Π²Π΅Π΄Π΅Π½ΠΈΡ Π²Π΅ΠΊΡΠΎΡΠΎΠ² i , j ΠΈ k :
Π΅ΡΠ»ΠΈ Π½Π°ΠΏΡΠ°Π²Π»Π΅Π½ΠΈΠ΅ ΠΊΡΠ°ΡΡΠ°ΠΉΡΠ΅Π³ΠΎ ΠΏΡΡΠΈ ΠΎΡ ΠΏΠ΅ΡΠ²ΠΎΠ³ΠΎ Π²Π΅ΠΊΡΠΎΡΠ° ΠΊ Π²ΡΠΎΡΠΎΠΌΡ ΡΠΎΠ²ΠΏΠ°Π΄Π°Π΅Ρ Ρ Π½Π°ΠΏΡΠ°Π²Π»Π΅Π½ΠΈΠ΅ΠΌ ΡΡΡΠ΅Π»ΠΊΠΈ, ΡΠΎ ΠΏΡΠΎΠΈΠ·Π²Π΅Π΄Π΅Π½ΠΈΠ΅ ΡΠ°Π²Π½ΠΎ ΡΡΠ΅ΡΡΠ΅ΠΌΡ Π²Π΅ΠΊΡΠΎΡΡ, Π΅ΡΠ»ΠΈ Π½Π΅ ΡΠΎΠ²ΠΏΠ°Π΄Π°Π΅Ρ β ΡΡΠ΅ΡΠΈΠΉ Π²Π΅ΠΊΡΠΎΡ Π±Π΅ΡΠ΅ΡΡΡ ΡΠΎ Π·Π½Π°ΠΊΠΎΠΌ Β«ΠΌΠΈΠ½ΡΡΒ».
ΠΡΡΡΡ Π·Π°Π΄Π°Π½Ρ Π΄Π²Π° Π²Π΅ΠΊΡΠΎΡΠ° Π° =Π° Ρ i +a y j +a z k ΠΈ b =b x i +b y j +b z k . ΠΠ°ΠΉΠ΄Π΅ΠΌ Π²Π΅ΠΊΡΠΎΡΠ½ΠΎΠ΅ ΠΏΡΠΎΠΈΠ·Π²Π΅Π΄Π΅Π½ΠΈΠ΅ ΡΡΠΈΡ Π²Π΅ΠΊΡΠΎΡΠΎΠ², ΠΏΠ΅ΡΠ΅ΠΌΠ½ΠΎΠΆΠ°Ρ ΠΈΡ ΠΊΠ°ΠΊ ΠΌΠ½ΠΎΠ³ΠΎΡΠ»Π΅Π½Ρ (ΡΠΎΠ³Π»Π°ΡΠ½ΠΎ ΡΠ²ΠΎΠΉΡΡΠ² Π²Π΅ΠΊΡΠΎΡΠ½ΠΎΠ³ΠΎ ΠΏΡΠΎΠΈΠ·Π²Π΅Π΄Π΅Π½ΠΈΡ):
ΠΠΎΠ»ΡΡΠ΅Π½Π½ΡΡ ΡΠΎΡΠΌΡΠ»Ρ ΠΌΠΎΠΆΠ½ΠΎ Π·Π°ΠΏΠΈΡΠ°ΡΡ Π΅ΡΠ΅ ΠΊΠΎΡΠΎΡΠ΅:
ΡΠ°ΠΊ ΠΊΠ°ΠΊ ΠΏΡΠ°Π²Π°Ρ ΡΠ°ΡΡΡ ΡΠ°Π²Π΅Π½ΡΡΠ²Π° (7. 1) ΡΠΎΠΎΡΠ²Π΅ΡΡΡΠ²ΡΠ΅Ρ ΡΠ°Π·Π»ΠΎΠΆΠ΅Π½ΠΈΡ
ΠΎΠΏΡΠ΅Π΄Π΅Π»ΠΈΡΠ΅Π»Ρ ΡΡΠ΅ΡΡΠ΅Π³ΠΎ ΠΏΠΎΡΡΠ΄ΠΊΠ° ΠΏΠΎ ΡΠ»Π΅ΠΌΠ΅Π½ΡΠ°ΠΌ ΠΏΠ΅ΡΠ²ΠΎΠΉ ΡΡΡΠΎΠΊΠΈ.Π Π°Π²Π΅Π½ΡΡΠ²ΠΎ (7.2) Π»Π΅Π³ΠΊΠΎ
Π·Π°ΠΏΠΎΠΌΠΈΠ½Π°Π΅ΡΡΡ.
7.4. ΠΠ΅ΠΊΠΎΡΠΎΡΡΠ΅ ΠΏΡΠΈΠ»ΠΎΠΆΠ΅Π½ΠΈΡ Π²Π΅ΠΊΡΠΎΡΠ½ΠΎΠ³ΠΎ ΠΏΡΠΎΠΈΠ·Π²Π΅Π΄Π΅Π½ΠΈΡ
Π£ΡΡΠ°Π½ΠΎΠ²Π»Π΅Π½ΠΈΠ΅ ΠΊΠΎΠ»Π»ΠΈΠ½Π΅Π°ΡΠ½ΠΎΡΡΠΈ Π²Π΅ΠΊΡΠΎΡΠΎΠ²
ΠΠ°Ρ ΠΎΠΆΠ΄Π΅Π½ΠΈΠ΅ ΠΏΠ»ΠΎΡΠ°Π΄ΠΈ ΠΏΠ°ΡΠ°Π»Π»Π΅Π»ΠΎΠ³ΡΠ°ΠΌΠΌΠ° ΠΈ ΡΡΠ΅ΡΠ³ΠΎΠ»ΡΠ½ΠΈΠΊΠ°
Π‘ΠΎΠ³Π»Π°ΡΠ½ΠΎ ΠΎΠΏΡΠ΅Π΄Π΅Π»Π΅Π½ΠΈΡ Π²Π΅ΠΊΡΠΎΡΠ½ΠΎΠ³ΠΎ ΠΏΡΠΎΠΈΠ·Π²Π΅Π΄Π΅Π½ΠΈΡ Π²Π΅ΠΊΡΠΎΡΠΎΠ² Π° ΠΈ b |Π° Ρ b | = |Π° | * |b |sin g , Ρ. Π΅. S ΠΏΠ°Ρ = |Π° Ρ b |. Π, Π·Π½Π°ΡΠΈΡ, D S =1/2|Π° Ρ b |.
ΠΠΏΡΠ΅Π΄Π΅Π»Π΅Π½ΠΈΠ΅ ΠΌΠΎΠΌΠ΅Π½ΡΠ° ΡΠΈΠ»Ρ ΠΎΡΠ½ΠΎΡΠΈΡΠ΅Π»ΡΠ½ΠΎ ΡΠΎΡΠΊΠΈ
ΠΡΡΡΡ Π² ΡΠΎΡΠΊΠ΅ Π ΠΏΡΠΈΠ»ΠΎΠΆΠ΅Π½Π° ΡΠΈΠ»Π° F =ΠΠ ΠΈ ΠΏΡΡΡΡ Π β Π½Π΅ΠΊΠΎΡΠΎΡΠ°Ρ ΡΠΎΡΠΊΠ° ΠΏΡΠΎΡΡΡΠ°Π½ΡΡΠ²Π° (ΡΠΌ. ΡΠΈΡ. 20).
ΠΠ· ΡΠΈΠ·ΠΈΠΊΠΈ ΠΈΠ·Π²Π΅ΡΡΠ½ΠΎ, ΡΡΠΎ ΠΌΠΎΠΌΠ΅Π½ΡΠΎΠΌ ΡΠΈ Π»Ρ F ΠΎΡΠ½ΠΎΡΠΈΡΠ΅Π»ΡΠ½ΠΎ ΡΠΎΡΠΊΠΈ Π Π½Π°Π·ΡΠ²Π°Π΅ΡΡΡ Π²Π΅ΠΊΡΠΎΡ Π , ΠΊΠΎΡΠΎΡΡΠΉ ΠΏΡΠΎΡ ΠΎΠ΄ΠΈΡ ΡΠ΅ΡΠ΅Π· ΡΠΎΡΠΊΡ Π ΠΈ:
1) ΠΏΠ΅ΡΠΏΠ΅Π½Π΄ΠΈΠΊΡΠ»ΡΡΠ΅Π½ ΠΏΠ»ΠΎΡΠΊΠΎΡΡΠΈ, ΠΏΡΠΎΡ ΠΎΠ΄ΡΡΠ΅ΠΉ ΡΠ΅ΡΠ΅Π· ΡΠΎΡΠΊΠΈ Π, Π, Π;
2) ΡΠΈΡΠ»Π΅Π½Π½ΠΎ ΡΠ°Π²Π΅Π½ ΠΏΡΠΎΠΈΠ·Π²Π΅Π΄Π΅Π½ΠΈΡ ΡΠΈΠ»Ρ Π½Π° ΠΏΠ»Π΅ΡΠΎ
3) ΠΎΠ±ΡΠ°Π·ΡΠ΅Ρ ΠΏΡΠ°Π²ΡΡ ΡΡΠΎΠΉΠΊΡ Ρ Π²Π΅ΠΊΡΠΎΡΠ°ΠΌΠΈ ΠΠ
ΠΈ
A
Π
.
Π‘ΡΠ°Π»ΠΎ Π±ΡΡΡ, Π =ΠΠ Ρ F .
ΠΠ°Ρ ΠΎΠΆΠ΄Π΅Π½ΠΈΠ΅ Π»ΠΈΠ½Π΅ΠΉΠ½ΠΎΠΉ ΡΠΊΠΎΡΠΎΡΡΠΈ Π²ΡΠ°ΡΠ΅Π½ΠΈΡ
Π‘ΠΊΠΎΡΠΎΡΡΡ v ΡΠΎΡΠΊΠΈ Π ΡΠ²Π΅ΡΠ΄ΠΎΠ³ΠΎ ΡΠ΅Π»Π°, Π²ΡΠ°ΡΠ°ΡΡΠ΅Π³ΠΎΡΡ Ρ ΡΠ³Π»ΠΎΠ²ΠΎΠΉ ΡΠΊΠΎΡΠΎΡΡΡΡ w Π²ΠΎΠΊΡΡΠ³ Π½Π΅ΠΏΠΎΠ΄Π²ΠΈΠΆΠ½ΠΎΠΉ ΠΎΡΠΈ, ΠΎΠΏΡΠ΅Π΄Π΅Π»ΡΠ΅ΡΡΡ ΡΠΎΡΠΌΡΠ»ΠΎΠΉ ΠΠΉΠ»Π΅ΡΠ° v =w Ρ r , Π³Π΄Π΅ r =ΠΠ , Π³Π΄Π΅ Π-Π½Π΅ΠΊΠΎΡΠΎΡΠ°Ρ Π½Π΅ΠΏΠΎΠ΄Π²ΠΈΠΆΠ½Π°Ρ ΡΠΎΡΠΊΠ° ΠΎΡΠΈ (ΡΠΌ. ΡΠΈΡ. 21).
ΠΠ° Π΄Π°Π½Π½ΠΎΠΌ ΡΡΠΎΠΊΠ΅ ΠΌΡ ΡΠ°ΡΡΠΌΠΎΡΡΠΈΠΌ Π΅ΡΡ Π΄Π²Π΅ ΠΎΠΏΠ΅ΡΠ°ΡΠΈΠΈ Ρ Π²Π΅ΠΊΡΠΎΡΠ°ΠΌΠΈ: Π²Π΅ΠΊΡΠΎΡΠ½ΠΎΠ΅ ΠΏΡΠΎΠΈΠ·Π²Π΅Π΄Π΅Π½ΠΈΠ΅ Π²Π΅ΠΊΡΠΎΡΠΎΠ² ΠΈ ΡΠΌΠ΅ΡΠ°Π½Π½ΠΎΠ΅ ΠΏΡΠΎΠΈΠ·Π²Π΅Π΄Π΅Π½ΠΈΠ΅ Π²Π΅ΠΊΡΠΎΡΠΎΠ² (ΡΡΠ°Π·Ρ ΡΡΡΠ»ΠΊΠ°, ΠΊΠΎΠΌΡ Π½ΡΠΆΠ½ΠΎ ΠΈΠΌΠ΅Π½Π½ΠΎ ΠΎΠ½ΠΎ) . ΠΠΈΡΠ΅Π³ΠΎ ΡΡΡΠ°ΡΠ½ΠΎΠ³ΠΎ, ΡΠ°ΠΊ ΠΈΠ½ΠΎΠ³Π΄Π° Π±ΡΠ²Π°Π΅Ρ, ΡΡΠΎ Π΄Π»Ρ ΠΏΠΎΠ»Π½ΠΎΠ³ΠΎ ΡΡΠ°ΡΡΡΡ, ΠΏΠΎΠΌΠΈΠΌΠΎ ΡΠΊΠ°Π»ΡΡΠ½ΠΎΠ³ΠΎ ΠΏΡΠΎΠΈΠ·Π²Π΅Π΄Π΅Π½ΠΈΡ Π²Π΅ΠΊΡΠΎΡΠΎΠ² , ΡΡΠ΅Π±ΡΠ΅ΡΡΡ Π΅ΡΡ ΠΈ Π΅ΡΡ. Π’Π°ΠΊΠ°Ρ Π²ΠΎΡ Π²Π΅ΠΊΡΠΎΡΠ½Π°Ρ Π½Π°ΡΠΊΠΎΠΌΠ°Π½ΠΈΡ. ΠΠΎΠΆΠ΅Ρ ΡΠ»ΠΎΠΆΠΈΡΡΡΡ Π²ΠΏΠ΅ΡΠ°ΡΠ»Π΅Π½ΠΈΠ΅, ΡΡΠΎ ΠΌΡ Π·Π°Π»Π΅Π·Π°Π΅ΠΌ Π² Π΄Π΅Π±ΡΠΈ Π°Π½Π°Π»ΠΈΡΠΈΡΠ΅ΡΠΊΠΎΠΉ Π³Π΅ΠΎΠΌΠ΅ΡΡΠΈΠΈ. ΠΡΠΎ Π½Π΅ ΡΠ°ΠΊ. Π Π΄Π°Π½Π½ΠΎΠΌ ΡΠ°Π·Π΄Π΅Π»Π΅ Π²ΡΡΡΠ΅ΠΉ ΠΌΠ°ΡΠ΅ΠΌΠ°ΡΠΈΠΊΠΈ Π²ΠΎΠΎΠ±ΡΠ΅ ΠΌΠ°Π»ΠΎ Π΄ΡΠΎΠ², ΡΠ°Π·Π²Π΅ ΡΡΠΎ Π½Π° ΠΡΡΠ°ΡΠΈΠ½ΠΎ Ρ
Π²Π°ΡΠΈΡ. ΠΠ° ΡΠ°ΠΌΠΎΠΌ Π΄Π΅Π»Π΅ ΠΌΠ°ΡΠ΅ΡΠΈΠ°Π» ΠΎΡΠ΅Π½Ρ ΡΠ°ΡΠΏΡΠΎΡΡΡΠ°Π½Π΅Π½Π½ΡΠΉ ΠΈ ΠΏΡΠΎΡΡΠΎΠΉ β Π²ΡΡΠ΄ Π»ΠΈ ΡΠ»ΠΎΠΆΠ½Π΅Π΅, ΡΠ΅ΠΌ ΡΠΎ ΠΆΠ΅ ΡΠΊΠ°Π»ΡΡΠ½ΠΎΠ΅ ΠΏΡΠΎΠΈΠ·Π²Π΅Π΄Π΅Π½ΠΈΠ΅ , Π΄Π°ΠΆΠ΅ ΡΠΈΠΏΠΎΠ²ΡΡ
Π·Π°Π΄Π°Ρ ΠΏΠΎΠΌΠ΅Π½ΡΡΠ΅ Π±ΡΠ΄Π΅Ρ. ΠΠ»Π°Π²Π½ΠΎΠ΅ Π² Π°Π½Π°Π»ΠΈΡΠΈΡΠ΅ΡΠΊΠΎΠΉ Π³Π΅ΠΎΠΌΠ΅ΡΡΠΈΠΈ, ΠΊΠ°ΠΊ ΠΌΠ½ΠΎΠ³ΠΈΠ΅ ΡΠ±Π΅Π΄ΡΡΡΡ ΠΈΠ»ΠΈ ΡΠΆΠ΅ ΡΠ±Π΅Π΄ΠΈΠ»ΠΈΡΡ, ΠΠ ΠΠ¨ΠΠΠΠ’Π¬Π‘Π― Π ΠΠ«Π§ΠΠ‘ΠΠΠΠΠ―Π₯. ΠΠΎΠ²ΡΠΎΡΡΠΉΡΠ΅ ΠΊΠ°ΠΊ Π·Π°ΠΊΠ»ΠΈΠ½Π°Π½ΠΈΠ΅, ΠΈ Π±ΡΠ΄Π΅Ρ Π²Π°ΠΌ ΡΡΠ°ΡΡΡΠ΅ =)
ΠΡΠ»ΠΈ Π²Π΅ΠΊΡΠΎΡΡ ΡΠ²Π΅ΡΠΊΠ°ΡΡ Π³Π΄Π΅-ΡΠΎ Π΄Π°Π»Π΅ΠΊΠΎ, ΠΊΠ°ΠΊ ΠΌΠΎΠ»Π½ΠΈΠΈ Π½Π° Π³ΠΎΡΠΈΠ·ΠΎΠ½ΡΠ΅, Π½Π΅ Π±Π΅Π΄Π°, Π½Π°ΡΠ½ΠΈΡΠ΅ Ρ ΡΡΠΎΠΊΠ° ΠΠ΅ΠΊΡΠΎΡΡ Π΄Π»Ρ ΡΠ°ΠΉΠ½ΠΈΠΊΠΎΠ² , ΡΡΠΎΠ±Ρ Π²ΠΎΡΡΡΠ°Π½ΠΎΠ²ΠΈΡΡ ΠΈΠ»ΠΈ Π²Π½ΠΎΠ²Ρ ΠΏΡΠΈΠΎΠ±ΡΠ΅ΡΡΠΈ Π±Π°Π·ΠΎΠ²ΡΠ΅ Π·Π½Π°Π½ΠΈΡ ΠΎ Π²Π΅ΠΊΡΠΎΡΠ°Ρ . ΠΠΎΠ»Π΅Π΅ ΠΏΠΎΠ΄Π³ΠΎΡΠΎΠ²Π»Π΅Π½Π½ΡΠ΅ ΡΠΈΡΠ°ΡΠ΅Π»ΠΈ ΠΌΠΎΠ³ΡΡ Π·Π½Π°ΠΊΠΎΠΌΠΈΡΡΡΡ Ρ ΠΈΠ½ΡΠΎΡΠΌΠ°ΡΠΈΠ΅ΠΉ Π²ΡΠ±ΠΎΡΠΎΡΠ½ΠΎ, Ρ ΠΏΠΎΡΡΠ°ΡΠ°Π»ΡΡ ΡΠΎΠ±ΡΠ°ΡΡ ΠΌΠ°ΠΊΡΠΈΠΌΠ°Π»ΡΠ½ΠΎ ΠΏΠΎΠ»Π½ΡΡ ΠΊΠΎΠ»Π»Π΅ΠΊΡΠΈΡ ΠΏΡΠΈΠΌΠ΅ΡΠΎΠ², ΠΊΠΎΡΠΎΡΡΠ΅ ΡΠ°ΡΡΠΎ Π²ΡΡΡΠ΅ΡΠ°ΡΡΡΡ Π² ΠΏΡΠ°ΠΊΡΠΈΡΠ΅ΡΠΊΠΈΡ ΡΠ°Π±ΠΎΡΠ°Ρ
Π§Π΅ΠΌ Π²Π°Ρ ΡΡΠ°Π·Ρ ΠΏΠΎΡΠ°Π΄ΠΎΠ²Π°ΡΡ? ΠΠΎΠ³Π΄Π° Ρ Π±ΡΠ» ΠΌΠ°Π»Π΅Π½ΡΠΊΠΈΠΌ, ΡΠΎ ΡΠΌΠ΅Π» ΠΆΠΎΠ½Π³Π»ΠΈΡΠΎΠ²Π°ΡΡ Π΄Π²ΡΠΌΡ ΠΈ Π΄Π°ΠΆΠ΅ ΡΡΠ΅ΠΌΡ ΡΠ°ΡΠΈΠΊΠ°ΠΌΠΈ. ΠΠΎΠ²ΠΊΠΎ ΠΏΠΎΠ»ΡΡΠ°Π»ΠΎΡΡ. Π‘Π΅ΠΉΡΠ°Ρ ΠΆΠΎΠ½Π³Π»ΠΈΡΠΎΠ²Π°ΡΡ Π½Π΅ ΠΏΡΠΈΠ΄ΡΡΡΡ Π²ΠΎΠΎΠ±ΡΠ΅, ΠΏΠΎΡΠΊΠΎΠ»ΡΠΊΡ ΠΌΡ Π±ΡΠ΄Π΅ΠΌ ΡΠ°ΡΡΠΌΠ°ΡΡΠΈΠ²Π°ΡΡ ΡΠΎΠ»ΡΠΊΠΎ ΠΏΡΠΎΡΡΡΠ°Π½ΡΡΠ²Π΅Π½Π½ΡΠ΅ Π²Π΅ΠΊΡΠΎΡΡ , Π° ΠΏΠ»ΠΎΡΠΊΠΈΠ΅ Π²Π΅ΠΊΡΠΎΡΡ Ρ Π΄Π²ΡΠΌΡ ΠΊΠΎΠΎΡΠ΄ΠΈΠ½Π°ΡΠ°ΠΌΠΈ ΠΎΡΡΠ°Π½ΡΡΡΡ Π·Π° Π±ΠΎΡΡΠΎΠΌ. ΠΠΎΡΠ΅ΠΌΡ? Π’Π°ΠΊΠΈΠΌΠΈ ΡΠΆ ΡΠΎΠ΄ΠΈΠ»ΠΈΡΡ Π΄Π°Π½Π½ΡΠ΅ Π΄Π΅ΠΉΡΡΠ²ΠΈΡ β Π²Π΅ΠΊΡΠΎΡΠ½ΠΎΠ΅ ΠΈ ΡΠΌΠ΅ΡΠ°Π½Π½ΠΎΠ΅ ΠΏΡΠΎΠΈΠ·Π²Π΅Π΄Π΅Π½ΠΈΠ΅ Π²Π΅ΠΊΡΠΎΡΠΎΠ² ΠΎΠΏΡΠ΅Π΄Π΅Π»Π΅Π½Ρ ΠΈ ΡΠ°Π±ΠΎΡΠ°ΡΡ Π² ΡΡΡΡ ΠΌΠ΅ΡΠ½ΠΎΠΌ ΠΏΡΠΎΡΡΡΠ°Π½ΡΡΠ²Π΅. Π£ΠΆΠ΅ ΠΏΡΠΎΡΠ΅!
Π Π΄Π°Π½Π½ΠΎΠΉ ΠΎΠΏΠ΅ΡΠ°ΡΠΈΠΈ, ΡΠΎΡΠ½ΠΎ ΡΠ°ΠΊ ΠΆΠ΅, ΠΊΠ°ΠΊ ΠΈ Π² ΡΠΊΠ°Π»ΡΡΠ½ΠΎΠΌ ΠΏΡΠΎΠΈΠ·Π²Π΅Π΄Π΅Π½ΠΈΠΈ, ΡΡΠ°ΡΡΠ²ΡΡΡ Π΄Π²Π° Π²Π΅ΠΊΡΠΎΡΠ° . ΠΡΡΡΡ ΡΡΠΎ Π±ΡΠ΄ΡΡ Π½Π΅ΡΠ»Π΅Π½Π½ΡΠ΅ Π±ΡΠΊΠ²Ρ .
Π‘Π°ΠΌΠΎ Π΄Π΅ΠΉΡΡΠ²ΠΈΠ΅ ΠΎΠ±ΠΎΠ·Π½Π°ΡΠ°Π΅ΡΡΡ ΡΠ»Π΅Π΄ΡΡΡΠΈΠΌ ΠΎΠ±ΡΠ°Π·ΠΎΠΌ: . Π‘ΡΡΠ΅ΡΡΠ²ΡΡΡ ΠΈ Π΄ΡΡΠ³ΠΈΠ΅ Π²Π°ΡΠΈΠ°Π½ΡΡ, Π½ΠΎ Ρ ΠΏΡΠΈΠ²ΡΠΊ ΠΎΠ±ΠΎΠ·Π½Π°ΡΠ°ΡΡ Π²Π΅ΠΊΡΠΎΡΠ½ΠΎΠ΅ ΠΏΡΠΎΠΈΠ·Π²Π΅Π΄Π΅Π½ΠΈΠ΅ Π²Π΅ΠΊΡΠΎΡΠΎΠ² ΠΈΠΌΠ΅Π½Π½ΠΎ ΡΠ°ΠΊ, Π² ΠΊΠ²Π°Π΄ΡΠ°ΡΠ½ΡΡ ΡΠΊΠΎΠ±ΠΊΠ°Ρ Ρ ΠΊΡΠ΅ΡΡΠΈΠΊΠΎΠΌ.
Π ΡΡΠ°Π·Ρ Π²ΠΎΠΏΡΠΎΡ : Π΅ΡΠ»ΠΈ Π² ΡΠΊΠ°Π»ΡΡΠ½ΠΎΠΌ ΠΏΡΠΎΠΈΠ·Π²Π΅Π΄Π΅Π½ΠΈΠΈ Π²Π΅ΠΊΡΠΎΡΠΎΠ² ΡΡΠ°ΡΡΠ²ΡΡΡ Π΄Π²Π° Π²Π΅ΠΊΡΠΎΡΠ°, ΠΈ Π·Π΄Π΅ΡΡ ΡΠΎΠΆΠ΅ ΡΠΌΠ½ΠΎΠΆΠ°ΡΡΡΡ Π΄Π²Π° Π²Π΅ΠΊΡΠΎΡΠ°, ΡΠΎΠ³Π΄Π° Π² ΡΡΠΌ ΡΠ°Π·Π½ΠΈΡΠ° ? Π―Π²Π½Π°Ρ ΡΠ°Π·Π½ΠΈΡΠ°, ΠΏΡΠ΅ΠΆΠ΄Π΅ Π²ΡΠ΅Π³ΠΎ, Π² Π ΠΠΠ£ΠΠ¬Π’ΠΠ’Π:
Π Π΅Π·ΡΠ»ΡΡΠ°ΡΠΎΠΌ ΡΠΊΠ°Π»ΡΡΠ½ΠΎΠ³ΠΎ ΠΏΡΠΎΠΈΠ·Π²Π΅Π΄Π΅Π½ΠΈΡ Π²Π΅ΠΊΡΠΎΡΠΎΠ² ΡΠ²Π»ΡΠ΅ΡΡΡ Π§ΠΠ‘ΠΠ:
Π Π΅Π·ΡΠ»ΡΡΠ°ΡΠΎΠΌ Π²Π΅ΠΊΡΠΎΡΠ½ΠΎΠ³ΠΎ ΠΏΡΠΎΠΈΠ·Π²Π΅Π΄Π΅Π½ΠΈΡ Π²Π΅ΠΊΡΠΎΡΠΎΠ² ΡΠ²Π»ΡΠ΅ΡΡΡ ΠΠΠΠ’ΠΠ : , ΡΠΎ Π΅ΡΡΡ ΡΠΌΠ½ΠΎΠΆΠ°Π΅ΠΌ Π²Π΅ΠΊΡΠΎΡΡ ΠΈ ΠΏΠΎΠ»ΡΡΠ°Π΅ΠΌ ΡΠ½ΠΎΠ²Π° Π²Π΅ΠΊΡΠΎΡ. ΠΠ°ΠΊΡΡΡΡΠΉ ΠΊΠ»ΡΠ±. Π‘ΠΎΠ±ΡΡΠ²Π΅Π½Π½ΠΎ, ΠΎΡΡΡΠ΄Π° ΠΈ Π½Π°Π·Π²Π°Π½ΠΈΠ΅ ΠΎΠΏΠ΅ΡΠ°ΡΠΈΠΈ. Π ΡΠ°Π·Π»ΠΈΡΠ½ΠΎΠΉ ΡΡΠ΅Π±Π½ΠΎΠΉ Π»ΠΈΡΠ΅ΡΠ°ΡΡΡΠ΅ ΠΎΠ±ΠΎΠ·Π½Π°ΡΠ΅Π½ΠΈΡ ΡΠΎΠΆΠ΅ ΠΌΠΎΠ³ΡΡ Π²Π°ΡΡΠΈΡΠΎΠ²Π°ΡΡΡΡ, Ρ Π±ΡΠ΄Ρ ΠΈΡΠΏΠΎΠ»ΡΠ·ΠΎΠ²Π°ΡΡ Π±ΡΠΊΠ²Ρ .
ΠΠΏΡΠ΅Π΄Π΅Π»Π΅Π½ΠΈΠ΅ Π²Π΅ΠΊΡΠΎΡΠ½ΠΎΠ³ΠΎ ΠΏΡΠΎΠΈΠ·Π²Π΅Π΄Π΅Π½ΠΈΡΠ‘Π½Π°ΡΠ°Π»Π° Π±ΡΠ΄Π΅Ρ ΠΎΠΏΡΠ΅Π΄Π΅Π»Π΅Π½ΠΈΠ΅ Ρ ΠΊΠ°ΡΡΠΈΠ½ΠΊΠΎΠΉ, Π·Π°ΡΠ΅ΠΌ ΠΊΠΎΠΌΠΌΠ΅Π½ΡΠ°ΡΠΈΠΈ.
ΠΠΏΡΠ΅Π΄Π΅Π»Π΅Π½ΠΈΠ΅ : ΠΠ΅ΠΊΡΠΎΡΠ½ΡΠΌ ΠΏΡΠΎΠΈΠ·Π²Π΅Π΄Π΅Π½ΠΈΠ΅ΠΌ Π½Π΅ΠΊΠΎΠ»Π»ΠΈΠ½Π΅Π°ΡΠ½ΡΡ
Π²Π΅ΠΊΡΠΎΡΠΎΠ² , Π²Π·ΡΡΡΡ
Π² Π΄Π°Π½Π½ΠΎΠΌ ΠΏΠΎΡΡΠ΄ΠΊΠ΅ , Π½Π°Π·ΡΠ²Π°Π΅ΡΡΡ ΠΠΠΠ’ΠΠ , Π΄Π»ΠΈΠ½Π° ΠΊΠΎΡΠΎΡΠΎΠ³ΠΎ ΡΠΈΡΠ»Π΅Π½Π½ΠΎ ΡΠ°Π²Π½Π° ΠΏΠ»ΠΎΡΠ°Π΄ΠΈ ΠΏΠ°ΡΠ°Π»Π»Π΅Π»ΠΎΠ³ΡΠ°ΠΌΠΌΠ° , ΠΏΠΎΡΡΡΠΎΠ΅Π½Π½ΠΎΠ³ΠΎ Π½Π° Π΄Π°Π½Π½ΡΡ
Π²Π΅ΠΊΡΠΎΡΠ°Ρ
; Π²Π΅ΠΊΡΠΎΡ ΠΎΡΡΠΎΠ³ΠΎΠ½Π°Π»Π΅Π½ Π²Π΅ΠΊΡΠΎΡΠ°ΠΌ , ΠΈ Π½Π°ΠΏΡΠ°Π²Π»Π΅Π½ ΡΠ°ΠΊ, ΡΡΠΎ Π±Π°Π·ΠΈΡ ΠΈΠΌΠ΅Π΅Ρ ΠΏΡΠ°Π²ΡΡ ΠΎΡΠΈΠ΅Π½ΡΠ°ΡΠΈΡ:
Π Π°Π·Π±ΠΈΡΠ°Π΅ΠΌ ΠΎΠΏΡΠ΅Π΄Π΅Π»Π΅Π½ΠΈΠ΅ ΠΏΠΎ ΠΊΠΎΡΡΠΎΡΠΊΠ°ΠΌ, ΡΡΡ ΠΌΠ½ΠΎΠ³ΠΎ ΠΈΠ½ΡΠ΅ΡΠ΅ΡΠ½ΠΎΠ³ΠΎ!
ΠΡΠ°ΠΊ, ΠΌΠΎΠΆΠ½ΠΎ Π²ΡΠ΄Π΅Π»ΠΈΡΡ ΡΠ»Π΅Π΄ΡΡΡΠΈΠ΅ ΡΡΡΠ΅ΡΡΠ²Π΅Π½Π½ΡΠ΅ ΠΌΠΎΠΌΠ΅Π½ΡΡ:
1) ΠΡΡ
ΠΎΠ΄Π½ΡΠ΅ Π²Π΅ΠΊΡΠΎΡΡ , ΠΎΠ±ΠΎΠ·Π½Π°ΡΠ΅Π½Π½ΡΠ΅ ΠΊΡΠ°ΡΠ½ΡΠΌΠΈ ΡΡΡΠ΅Π»ΠΊΠ°ΠΌΠΈ, ΠΏΠΎ ΠΎΠΏΡΠ΅Π΄Π΅Π»Π΅Π½ΠΈΡ Π½Π΅ ΠΊΠΎΠ»Π»ΠΈΠ½Π΅Π°ΡΠ½Ρ . Π‘Π»ΡΡΠ°ΠΉ ΠΊΠΎΠ»Π»ΠΈΠ½Π΅Π°ΡΠ½ΡΡ
Π²Π΅ΠΊΡΠΎΡΠΎΠ² Π±ΡΠ΄Π΅Ρ ΡΠΌΠ΅ΡΡΠ½ΠΎ ΡΠ°ΡΡΠΌΠΎΡΡΠ΅ΡΡ ΡΡΡΡ ΠΏΠΎΠ·ΠΆΠ΅.
2) ΠΠ΅ΠΊΡΠΎΡΡ Π²Π·ΡΡΡ Π² ΡΡΡΠΎΠ³ΠΎ ΠΎΠΏΡΠ΅Π΄Π΅Π»ΡΠ½Π½ΠΎΠΌ ΠΏΠΎΡΡΠ΄ΠΊΠ΅ : β Β«Π°Β» ΡΠΌΠ½ΠΎΠΆΠ°Π΅ΡΡΡ Π½Π° Β«Π±ΡΒ» , Π° Π½Π΅ Β«Π±ΡΒ» Π½Π° Β«Π°Β». Π Π΅Π·ΡΠ»ΡΡΠ°ΡΠΎΠΌ ΡΠΌΠ½ΠΎΠΆΠ΅Π½ΠΈΡ Π²Π΅ΠΊΡΠΎΡΠΎΠ² ΡΠ²Π»ΡΠ΅ΡΡΡ ΠΠΠΠ’ΠΠ , ΠΊΠΎΡΠΎΡΡΠΉ ΠΎΠ±ΠΎΠ·Π½Π°ΡΠ΅Π½ ΡΠΈΠ½ΠΈΠΌ ΡΠ²Π΅ΡΠΎΠΌ. ΠΡΠ»ΠΈ Π²Π΅ΠΊΡΠΎΡΡ ΡΠΌΠ½ΠΎΠΆΠΈΡΡ Π² ΠΎΠ±ΡΠ°ΡΠ½ΠΎΠΌ ΠΏΠΎΡΡΠ΄ΠΊΠ΅, ΡΠΎ ΠΏΠΎΠ»ΡΡΠΈΠΌ ΡΠ°Π²Π½ΡΠΉ ΠΏΠΎ Π΄Π»ΠΈΠ½Π΅ ΠΈ ΠΏΡΠΎΡΠΈΠ²ΠΎΠΏΠΎΠ»ΠΎΠΆΠ½ΡΠΉ ΠΏΠΎ Π½Π°ΠΏΡΠ°Π²Π»Π΅Π½ΠΈΡ Π²Π΅ΠΊΡΠΎΡ (ΠΌΠ°Π»ΠΈΠ½ΠΎΠ²ΡΠΉ ΡΠ²Π΅Ρ). Π’ΠΎ Π΅ΡΡΡ, ΡΠΏΡΠ°Π²Π΅Π΄Π»ΠΈΠ²ΠΎ ΡΠ°Π²Π΅Π½ΡΡΠ²ΠΎ .
3) Π’Π΅ΠΏΠ΅ΡΡ ΠΏΠΎΠ·Π½Π°ΠΊΠΎΠΌΠΈΠΌΡΡ Ρ Π³Π΅ΠΎΠΌΠ΅ΡΡΠΈΡΠ΅ΡΠΊΠΈΠΌ ΡΠΌΡΡΠ»ΠΎΠΌ Π²Π΅ΠΊΡΠΎΡΠ½ΠΎΠ³ΠΎ ΠΏΡΠΎΠΈΠ·Π²Π΅Π΄Π΅Π½ΠΈΡ. ΠΡΠΎ ΠΎΡΠ΅Π½Ρ Π²Π°ΠΆΠ½ΡΠΉ ΠΏΡΠ½ΠΊΡ! ΠΠΠΠΠ ΡΠΈΠ½Π΅Π³ΠΎ Π²Π΅ΠΊΡΠΎΡΠ° (Π°, Π·Π½Π°ΡΠΈΡ, ΠΈ ΠΌΠ°Π»ΠΈΠ½ΠΎΠ²ΠΎΠ³ΠΎ Π²Π΅ΠΊΡΠΎΡΠ° ) ΡΠΈΡΠ»Π΅Π½Π½ΠΎ ΡΠ°Π²Π½Π° ΠΠΠΠ©ΠΠΠ ΠΏΠ°ΡΠ°Π»Π»Π΅Π»ΠΎΠ³ΡΠ°ΠΌΠΌΠ°, ΠΏΠΎΡΡΡΠΎΠ΅Π½Π½ΠΎΠ³ΠΎ Π½Π° Π²Π΅ΠΊΡΠΎΡΠ°Ρ . ΠΠ° ΡΠΈΡΡΠ½ΠΊΠ΅ Π΄Π°Π½Π½ΡΠΉ ΠΏΠ°ΡΠ°Π»Π»Π΅Π»ΠΎΠ³ΡΠ°ΠΌΠΌ Π·Π°ΡΡΡΠΈΡ ΠΎΠ²Π°Π½ ΡΡΡΠ½ΡΠΌ ΡΠ²Π΅ΡΠΎΠΌ.
ΠΡΠΈΠΌΠ΅ΡΠ°Π½ΠΈΠ΅ : ΡΠ΅ΡΡΡΠΆ ΡΠ²Π»ΡΠ΅ΡΡΡ ΡΡ Π΅ΠΌΠ°ΡΠΈΡΠ΅ΡΠΊΠΈΠΌ, ΠΈ, Π΅ΡΡΠ΅ΡΡΠ²Π΅Π½Π½ΠΎ, Π½ΠΎΠΌΠΈΠ½Π°Π»ΡΠ½Π°Ρ Π΄Π»ΠΈΠ½Π° Π²Π΅ΠΊΡΠΎΡΠ½ΠΎΠ³ΠΎ ΠΏΡΠΎΠΈΠ·Π²Π΅Π΄Π΅Π½ΠΈΡ Π½Π΅ ΡΠ°Π²Π½Π° ΠΏΠ»ΠΎΡΠ°Π΄ΠΈ ΠΏΠ°ΡΠ°Π»Π»Π΅Π»ΠΎΠ³ΡΠ°ΠΌΠΌΠ°.
ΠΡΠΏΠΎΠΌΠΈΠ½Π°Π΅ΠΌ ΠΎΠ΄Π½Ρ ΠΈΠ· Π³Π΅ΠΎΠΌΠ΅ΡΡΠΈΡΠ΅ΡΠΊΠΈΡ
ΡΠΎΡΠΌΡΠ»: ΠΏΠ»ΠΎΡΠ°Π΄Ρ ΠΏΠ°ΡΠ°Π»Π»Π΅Π»ΠΎΠ³ΡΠ°ΠΌΠΌΠ° ΡΠ°Π²Π½Π° ΠΏΡΠΎΠΈΠ·Π²Π΅Π΄Π΅Π½ΠΈΡ ΡΠΌΠ΅ΠΆΠ½ΡΡ
ΡΡΠΎΡΠΎΠ½ Π½Π° ΡΠΈΠ½ΡΡ ΡΠ³Π»Π° ΠΌΠ΅ΠΆΠ΄Ρ Π½ΠΈΠΌΠΈ . ΠΠΎΡΡΠΎΠΌΡ, ΠΈΡΡ
ΠΎΠ΄Ρ ΠΈΠ· Π²ΡΡΠ΅ΡΠΊΠ°Π·Π°Π½Π½ΠΎΠ³ΠΎ, ΡΠΏΡΠ°Π²Π΅Π΄Π»ΠΈΠ²Π° ΡΠΎΡΠΌΡΠ»Π° Π²ΡΡΠΈΡΠ»Π΅Π½ΠΈΡ ΠΠΠΠΠ« Π²Π΅ΠΊΡΠΎΡΠ½ΠΎΠ³ΠΎ ΠΏΡΠΎΠΈΠ·Π²Π΅Π΄Π΅Π½ΠΈΡ:
ΠΠΎΠ΄ΡΡΡΠΊΠΈΠ²Π°Ρ, ΡΡΠΎ Π² ΡΠΎΡΠΌΡΠ»Π΅ ΡΠ΅ΡΡ ΠΈΠ΄ΡΡ ΠΎ ΠΠΠΠΠ Π²Π΅ΠΊΡΠΎΡΠ°, Π° Π½Π΅ ΠΎ ΡΠ°ΠΌΠΎΠΌ Π²Π΅ΠΊΡΠΎΡΠ΅ . ΠΠ°ΠΊΠΎΠ² ΠΏΡΠ°ΠΊΡΠΈΡΠ΅ΡΠΊΠΈΠΉ ΡΠΌΡΡΠ»? Π ΡΠΌΡΡΠ» ΡΠ°ΠΊΠΎΠ², ΡΡΠΎ Π² Π·Π°Π΄Π°ΡΠ°Ρ Π°Π½Π°Π»ΠΈΡΠΈΡΠ΅ΡΠΊΠΎΠΉ Π³Π΅ΠΎΠΌΠ΅ΡΡΠΈΠΈ ΠΏΠ»ΠΎΡΠ°Π΄Ρ ΠΏΠ°ΡΠ°Π»Π»Π΅Π»ΠΎΠ³ΡΠ°ΠΌΠΌΠ° ΡΠ°ΡΡΠΎ Π½Π°Ρ ΠΎΠ΄ΡΡ ΡΠ΅ΡΠ΅Π· ΠΏΠΎΠ½ΡΡΠΈΠ΅ Π²Π΅ΠΊΡΠΎΡΠ½ΠΎΠ³ΠΎ ΠΏΡΠΎΠΈΠ·Π²Π΅Π΄Π΅Π½ΠΈΡ:
ΠΠΎΠ»ΡΡΠΈΠΌ Π²ΡΠΎΡΡΡ Π²Π°ΠΆΠ½ΡΡ ΡΠΎΡΠΌΡΠ»Ρ. ΠΠΈΠ°Π³ΠΎΠ½Π°Π»Ρ ΠΏΠ°ΡΠ°Π»Π»Π΅Π»ΠΎΠ³ΡΠ°ΠΌΠΌΠ° (ΠΊΡΠ°ΡΠ½ΡΠΉ ΠΏΡΠ½ΠΊΡΠΈΡ) Π΄Π΅Π»ΠΈΡ Π΅Π³ΠΎ Π½Π° Π΄Π²Π° ΡΠ°Π²Π½ΡΡ
ΡΡΠ΅ΡΠ³ΠΎΠ»ΡΠ½ΠΈΠΊΠ°. Π‘Π»Π΅Π΄ΠΎΠ²Π°ΡΠ΅Π»ΡΠ½ΠΎ, ΠΏΠ»ΠΎΡΠ°Π΄Ρ ΡΡΠ΅ΡΠ³ΠΎΠ»ΡΠ½ΠΈΠΊΠ°, ΠΏΠΎΡΡΡΠΎΠ΅Π½Π½ΠΎΠ³ΠΎ Π½Π° Π²Π΅ΠΊΡΠΎΡΠ°Ρ
(ΠΊΡΠ°ΡΠ½Π°Ρ ΡΡΡΠΈΡ
ΠΎΠ²ΠΊΠ°), ΠΌΠΎΠΆΠ½ΠΎ Π½Π°ΠΉΡΠΈ ΠΏΠΎ ΡΠΎΡΠΌΡΠ»Π΅:
4) ΠΠ΅ ΠΌΠ΅Π½Π΅Π΅ Π²Π°ΠΆΠ½ΡΠΉ ΡΠ°ΠΊΡ ΡΠΎΡΡΠΎΠΈΡ Π² ΡΠΎΠΌ, ΡΡΠΎ Π²Π΅ΠΊΡΠΎΡ ΠΎΡΡΠΎΠ³ΠΎΠ½Π°Π»Π΅Π½ Π²Π΅ΠΊΡΠΎΡΠ°ΠΌ , ΡΠΎ Π΅ΡΡΡ . Π Π°Π·ΡΠΌΠ΅Π΅ΡΡΡ, ΠΏΡΠΎΡΠΈΠ²ΠΎΠΏΠΎΠ»ΠΎΠΆΠ½ΠΎ Π½Π°ΠΏΡΠ°Π²Π»Π΅Π½Π½ΡΠΉ Π²Π΅ΠΊΡΠΎΡ (ΠΌΠ°Π»ΠΈΠ½ΠΎΠ²Π°Ρ ΡΡΡΠ΅Π»ΠΊΠ°) ΡΠΎΠΆΠ΅ ΠΎΡΡΠΎΠ³ΠΎΠ½Π°Π»Π΅Π½ ΠΈΡΡ ΠΎΠ΄Π½ΡΠΌ Π²Π΅ΠΊΡΠΎΡΠ°ΠΌ .
5) ΠΠ΅ΠΊΡΠΎΡ Π½Π°ΠΏΡΠ°Π²Π»Π΅Π½ ΡΠ°ΠΊ, ΡΡΠΎ Π±Π°Π·ΠΈΡ ΠΈΠΌΠ΅Π΅Ρ ΠΏΡΠ°Π²ΡΡ ΠΎΡΠΈΠ΅Π½ΡΠ°ΡΠΈΡ. ΠΠ° ΡΡΠΎΠΊΠ΅ ΠΎ ΠΏΠ΅ΡΠ΅Ρ
ΠΎΠ΄Π΅ ΠΊ Π½ΠΎΠ²ΠΎΠΌΡ Π±Π°Π·ΠΈΡΡ Ρ Π΄ΠΎΡΡΠ°ΡΠΎΡΠ½ΠΎ ΠΏΠΎΠ΄ΡΠΎΠ±Π½ΠΎ ΡΠ°ΡΡΠΊΠ°Π·Π°Π» ΠΎΠ± ΠΎΡΠΈΠ΅Π½ΡΠ°ΡΠΈΠΈ ΠΏΠ»ΠΎΡΠΊΠΎΡΡΠΈ , ΠΈ ΡΠ΅ΠΉΡΠ°Ρ ΠΌΡ ΡΠ°Π·Π±Π΅ΡΡΠΌΡΡ, ΡΡΠΎ ΡΠ°ΠΊΠΎΠ΅ ΠΎΡΠΈΠ΅Π½ΡΠ°ΡΠΈΡ ΠΏΡΠΎΡΡΡΠ°Π½ΡΡΠ²Π°. ΠΠ±ΡΡΡΠ½ΡΡΡ Π±ΡΠ΄Ρ Π½Π° ΠΏΠ°Π»ΡΡΠ°Ρ
Π²Π°ΡΠ΅ΠΉ ΠΏΡΠ°Π²ΠΎΠΉ ΡΡΠΊΠΈ . ΠΡΡΠ»Π΅Π½Π½ΠΎ ΡΠΎΠ²ΠΌΠ΅ΡΡΠΈΡΠ΅ ΡΠΊΠ°Π·Π°ΡΠ΅Π»ΡΠ½ΡΠΉ ΠΏΠ°Π»Π΅Ρ Ρ Π²Π΅ΠΊΡΠΎΡΠΎΠΌ ΠΈ ΡΡΠ΅Π΄Π½ΠΈΠΉ ΠΏΠ°Π»Π΅Ρ Ρ Π²Π΅ΠΊΡΠΎΡΠΎΠΌ . ΠΠ΅Π·ΡΠΌΡΠ½Π½ΡΠΉ ΠΏΠ°Π»Π΅Ρ ΠΈ ΠΌΠΈΠ·ΠΈΠ½Π΅Ρ ΠΏΡΠΈΠΆΠΌΠΈΡΠ΅ ΠΊ Π»Π°Π΄ΠΎΠ½ΠΈ. Π ΡΠ΅Π·ΡΠ»ΡΡΠ°ΡΠ΅ Π±ΠΎΠ»ΡΡΠΎΠΉ ΠΏΠ°Π»Π΅Ρ β Π²Π΅ΠΊΡΠΎΡΠ½ΠΎΠ΅ ΠΏΡΠΎΠΈΠ·Π²Π΅Π΄Π΅Π½ΠΈΠ΅ Π±ΡΠ΄Π΅Ρ ΡΠΌΠΎΡΡΠ΅ΡΡ Π²Π²Π΅ΡΡ
. ΠΡΠΎ ΠΈ Π΅ΡΡΡ ΠΏΡΠ°Π²ΠΎΠΎΡΠΈΠ΅Π½ΡΠΈΡΠΎΠ²Π°Π½Π½ΡΠΉ Π±Π°Π·ΠΈΡ (Π½Π° ΡΠΈΡΡΠ½ΠΊΠ΅ ΠΈΠΌΠ΅Π½Π½ΠΎ ΠΎΠ½). Π’Π΅ΠΏΠ΅ΡΡ ΠΏΠΎΠΌΠ΅Π½ΡΠΉΡΠ΅ Π²Π΅ΠΊΡΠΎΡΡ (ΡΠΊΠ°Π·Π°ΡΠ΅Π»ΡΠ½ΡΠΉ ΠΈ ΡΡΠ΅Π΄Π½ΠΈΠΉ ΠΏΠ°Π»ΡΡΡ ) ΠΌΠ΅ΡΡΠ°ΠΌΠΈ, Π² ΡΠ΅Π·ΡΠ»ΡΡΠ°ΡΠ΅ Π±ΠΎΠ»ΡΡΠΎΠΉ ΠΏΠ°Π»Π΅Ρ ΡΠ°Π·Π²Π΅ΡΠ½ΡΡΡΡ, ΠΈ Π²Π΅ΠΊΡΠΎΡΠ½ΠΎΠ΅ ΠΏΡΠΎΠΈΠ·Π²Π΅Π΄Π΅Π½ΠΈΠ΅ ΡΠΆΠ΅ Π±ΡΠ΄Π΅Ρ ΡΠΌΠΎΡΡΠ΅ΡΡ Π²Π½ΠΈΠ·. ΠΡΠΎ ΡΠΎΠΆΠ΅ ΠΏΡΠ°Π²ΠΎΠΎΡΠΈΠ΅Π½ΡΠΈΡΠΎΠ²Π°Π½Π½ΡΠΉ Π±Π°Π·ΠΈΡ. ΠΠΎΠ·ΠΌΠΎΠΆΠ½ΠΎ, Ρ Π²Π°Ρ Π²ΠΎΠ·Π½ΠΈΠΊ Π²ΠΎΠΏΡΠΎΡ: Π° ΠΊΠ°ΠΊΠΎΠΉ Π±Π°Π·ΠΈΡ ΠΈΠΌΠ΅Π΅Ρ Π»Π΅Π²ΡΡ ΠΎΡΠΈΠ΅Π½ΡΠ°ΡΠΈΡ? Β«ΠΡΠΈΡΠ²ΠΎΠΉΡΠ΅Β» ΡΠ΅ΠΌ ΠΆΠ΅ ΠΏΠ°Π»ΡΡΠ°ΠΌ Π»Π΅Π²ΠΎΠΉ ΡΡΠΊΠΈ Π²Π΅ΠΊΡΠΎΡΡ , ΠΈ ΠΏΠΎΠ»Π£ΡΠΈΡΠ΅ Π»Π΅Π²ΡΠΉ Π±Π°Π·ΠΈΡ ΠΈ Π»Π΅Π²ΡΡ ΠΎΡΠΈΠ΅Π½ΡΠ°ΡΠΈΡ ΠΏΡΠΎΡΡΡΠ°Π½ΡΡΠ²Π° (Π² ΡΡΠΎΠΌ ΡΠ»ΡΡΠ°Π΅ Π±ΠΎΠ»ΡΡΠΎΠΉ ΠΏΠ°Π»Π΅Ρ ΡΠ°ΡΠΏΠΎΠ»ΠΎΠΆΠΈΡΡΡ ΠΏΠΎ Π½Π°ΠΏΡΠ°Π²Π»Π΅Π½ΠΈΡ Π½ΠΈΠΆΠ½Π΅Π³ΠΎ Π²Π΅ΠΊΡΠΎΡΠ°) . ΠΠ±ΡΠ°Π·Π½ΠΎ Π³ΠΎΠ²ΠΎΡΡ, Π΄Π°Π½Π½ΡΠ΅ Π±Π°Π·ΠΈΡΡ Β«Π·Π°ΠΊΡΡΡΠΈΠ²Π°ΡΡΒ» ΠΈΠ»ΠΈ ΠΎΡΠΈΠ΅Π½ΡΠΈΡΡΡΡ ΠΏΡΠΎΡΡΡΠ°Π½ΡΡΠ²ΠΎ Π² ΡΠ°Π·Π½ΡΠ΅ ΡΡΠΎΡΠΎΠ½Ρ. Π ΡΡΠΎ ΠΏΠΎΠ½ΡΡΠΈΠ΅ Π½Π΅ ΡΠ»Π΅Π΄ΡΠ΅Ρ ΡΡΠΈΡΠ°ΡΡ ΡΠ΅ΠΌ-ΡΠΎ Π½Π°Π΄ΡΠΌΠ°Π½Π½ΡΠΌ ΠΈΠ»ΠΈ Π°Π±ΡΡΡΠ°ΠΊΡΠ½ΡΠΌ β ΡΠ°ΠΊ, Π½Π°ΠΏΡΠΈΠΌΠ΅Ρ, ΠΎΡΠΈΠ΅Π½ΡΠ°ΡΠΈΡ ΠΏΡΠΎΡΡΡΠ°Π½ΡΡΠ²Π° ΠΌΠ΅Π½ΡΠ΅Ρ ΡΠ°ΠΌΠΎΠ΅ ΠΎΠ±ΡΡΠ½ΠΎΠ΅ Π·Π΅ΡΠΊΠ°Π»ΠΎ, ΠΈ Π΅ΡΠ»ΠΈ Β«Π²ΡΡΠ°ΡΠΈΡΡ ΠΎΡΡΠ°ΠΆΡΠ½Π½ΡΠΉ ΠΎΠ±ΡΠ΅ΠΊΡ ΠΈΠ· Π·Π°Π·Π΅ΡΠΊΠ°Π»ΡΡΒ», ΡΠΎ Π΅Π³ΠΎ Π² ΠΎΠ±ΡΠ΅ΠΌ ΡΠ»ΡΡΠ°Π΅ Π½Π΅ ΡΠ΄Π°ΡΡΡΡ ΡΠΎΠ²ΠΌΠ΅ΡΡΠΈΡΡ Ρ Β«ΠΎΡΠΈΠ³ΠΈΠ½Π°Π»ΠΎΠΌΒ».
ΠΡΡΠ°ΡΠΈ, ΠΏΠΎΠ΄Π½Π΅ΡΠΈΡΠ΅ ΠΊ Π·Π΅ΡΠΊΠ°Π»Ρ ΡΡΠΈ ΠΏΠ°Π»ΡΡΠ° ΠΈ ΠΏΡΠΎΠ°Π½Π°Π»ΠΈΠ·ΠΈΡΡΠΉΡΠ΅ ΠΎΡΡΠ°ΠΆΠ΅Π½ΠΈΠ΅;-)
β¦ΠΊΠ°ΠΊ Π²ΡΡ-ΡΠ°ΠΊΠΈ Ρ ΠΎΡΠΎΡΠΎ, ΡΡΠΎ Π²Ρ ΡΠ΅ΠΏΠ΅ΡΡ Π·Π½Π°Π΅ΡΠ΅ ΠΎ ΠΏΡΠ°Π²ΠΎ- ΠΈ Π»Π΅Π²ΠΎΠΎΡΠΈΠ΅Π½ΡΠΈΡΠΎΠ²Π°Π½Π½ΡΡ Π±Π°Π·ΠΈΡΠ°Ρ , ΠΈΠ±ΠΎ ΡΡΡΠ°ΡΠ½Π« Π²ΡΡΠΊΠ°Π·ΡΠ²Π°Π½ΠΈΡ Π½Π΅ΠΊΠΎΡΠΎΡΡΡ Π»Π΅ΠΊΡΠΎΡΠΎΠ² ΠΎ ΡΠΌΠ΅Π½Π΅ ΠΎΡΠΈΠ΅Π½ΡΠ°ΡΠΈΠΈ =)
ΠΠ΅ΠΊΡΠΎΡΠ½ΠΎΠ΅ ΠΏΡΠΎΠΈΠ·Π²Π΅Π΄Π΅Π½ΠΈΠ΅ ΠΊΠΎΠ»Π»ΠΈΠ½Π΅Π°ΡΠ½ΡΡ Π²Π΅ΠΊΡΠΎΡΠΎΠ²ΠΠΏΡΠ΅Π΄Π΅Π»Π΅Π½ΠΈΠ΅ ΠΏΠΎΠ΄ΡΠΎΠ±Π½ΠΎ ΡΠ°Π·ΠΎΠ±ΡΠ°Π½ΠΎ, ΠΎΡΡΠ°Π»ΠΎΡΡ Π²ΡΡΡΠ½ΠΈΡΡ, ΡΡΠΎ ΠΏΡΠΎΠΈΡΡ ΠΎΠ΄ΠΈΡ, ΠΊΠΎΠ³Π΄Π° Π²Π΅ΠΊΡΠΎΡΡ ΠΊΠΎΠ»Π»ΠΈΠ½Π΅Π°ΡΠ½Ρ. ΠΡΠ»ΠΈ Π²Π΅ΠΊΡΠΎΡΡ ΠΊΠΎΠ»Π»ΠΈΠ½Π΅Π°ΡΠ½Ρ, ΡΠΎ ΠΈΡ ΠΌΠΎΠΆΠ½ΠΎ ΡΠ°ΡΠΏΠΎΠ»ΠΎΠΆΠΈΡΡ Π½Π° ΠΎΠ΄Π½ΠΎΠΉ ΠΏΡΡΠΌΠΎΠΉ ΠΈ Π½Π°Ρ ΠΏΠ°ΡΠ°Π»Π»Π΅Π»ΠΎΠ³ΡΠ°ΠΌΠΌ ΡΠΎΠΆΠ΅ Β«ΡΠΊΠ»Π°Π΄ΡΠ²Π°Π΅ΡΡΡΒ» Π² ΠΎΠ΄Π½Ρ ΠΏΡΡΠΌΡΡ. ΠΠ»ΠΎΡΠ°Π΄Ρ ΡΠ°ΠΊΠΎΠ³ΠΎ, ΠΊΠ°ΠΊ Π³ΠΎΠ²ΠΎΡΡΡ ΠΌΠ°ΡΠ΅ΠΌΠ°ΡΠΈΠΊΠΈ, Π²ΡΡΠΎΠΆΠ΄Π΅Π½Π½ΠΎΠ³ΠΎ ΠΏΠ°ΡΠ°Π»Π»Π΅Π»ΠΎΠ³ΡΠ°ΠΌΠΌΠ° ΡΠ°Π²Π½Π° Π½ΡΠ»Ρ. ΠΡΠΎ ΠΆΠ΅ ΡΠ»Π΅Π΄ΡΠ΅Ρ ΠΈ ΠΈΠ· ΡΠΎΡΠΌΡΠ»Ρ β ΡΠΈΠ½ΡΡ Π½ΡΠ»Ρ ΠΈΠ»ΠΈ 180-ΡΠΈ Π³ΡΠ°Π΄ΡΡΠΎΠ² ΡΠ°Π²Π΅Π½ Π½ΡΠ»Ρ, Π° Π·Π½Π°ΡΠΈΡ, ΠΈ ΠΏΠ»ΠΎΡΠ°Π΄Ρ Π½ΡΠ»Π΅Π²Π°Ρ
Π’Π°ΠΊΠΈΠΌ ΠΎΠ±ΡΠ°Π·ΠΎΠΌ, Π΅ΡΠ»ΠΈ , ΡΠΎ . Π‘ΡΡΠΎΠ³ΠΎ Π³ΠΎΠ²ΠΎΡΡ, ΡΠ°ΠΌΠΎ Π²Π΅ΠΊΡΠΎΡΠ½ΠΎΠ΅ ΠΏΡΠΎΠΈΠ·Π²Π΅Π΄Π΅Π½ΠΈΠ΅ ΡΠ°Π²Π½ΠΎ Π½ΡΠ»Π΅Π²ΠΎΠΌΡ Π²Π΅ΠΊΡΠΎΡΡ, Π½ΠΎ Π½Π° ΠΏΡΠ°ΠΊΡΠΈΠΊΠ΅ ΡΡΠΈΠΌ ΡΠ°ΡΡΠΎ ΠΏΡΠ΅Π½Π΅Π±ΡΠ΅Π³Π°ΡΡ ΠΈ ΠΏΠΈΡΡΡ, ΡΡΠΎ ΠΎΠ½ΠΎ ΠΏΡΠΎΡΡΠΎ ΡΠ°Π²Π½ΠΎ Π½ΡΠ»Ρ.
Π§Π°ΡΡΠ½ΡΠΉ ΡΠ»ΡΡΠ°ΠΉ β Π²Π΅ΠΊΡΠΎΡΠ½ΠΎΠ΅ ΠΏΡΠΎΠΈΠ·Π²Π΅Π΄Π΅Π½ΠΈΠ΅ Π²Π΅ΠΊΡΠΎΡΠ° Π½Π° ΡΠ°ΠΌΠΎΠ³ΠΎ ΡΠ΅Π±Ρ:
Π‘ ΠΏΠΎΠΌΠΎΡΡΡ Π²Π΅ΠΊΡΠΎΡΠ½ΠΎΠ³ΠΎ ΠΏΡΠΎΠΈΠ·Π²Π΅Π΄Π΅Π½ΠΈΡ ΠΌΠΎΠΆΠ½ΠΎ ΠΏΡΠΎΠ²Π΅ΡΡΡΡ ΠΊΠΎΠ»Π»ΠΈΠ½Π΅Π°ΡΠ½ΠΎΡΡΡ ΡΡΡΡ
ΠΌΠ΅ΡΠ½ΡΡ
Π²Π΅ΠΊΡΠΎΡΠΎΠ², ΠΈ Π΄Π°Π½Π½ΡΡ Π·Π°Π΄Π°ΡΡ ΡΡΠ΅Π΄ΠΈ ΠΏΡΠΎΡΠΈΡ
ΠΌΡ ΡΠΎΠΆΠ΅ ΡΠ°Π·Π±Π΅ΡΡΠΌ.
ΠΠ»Ρ ΡΠ΅ΡΠ΅Π½ΠΈΡ ΠΏΡΠ°ΠΊΡΠΈΡΠ΅ΡΠΊΠΈΡ ΠΏΡΠΈΠΌΠ΅ΡΠΎΠ² ΠΌΠΎΠΆΠ΅Ρ ΠΏΠΎΡΡΠ΅Π±ΠΎΠ²Π°ΡΡΡΡ ΡΡΠΈΠ³ΠΎΠ½ΠΎΠΌΠ΅ΡΡΠΈΡΠ΅ΡΠΊΠ°Ρ ΡΠ°Π±Π»ΠΈΡΠ° , ΡΡΠΎΠ±Ρ Π½Π°Ρ ΠΎΠ΄ΠΈΡΡ ΠΏΠΎ Π½Π΅ΠΉ Π·Π½Π°ΡΠ΅Π½ΠΈΡ ΡΠΈΠ½ΡΡΠΎΠ².
ΠΡ ΡΡΠΎ ΠΆΠ΅, ΡΠ°Π·ΠΆΠΈΠ³Π°Π΅ΠΌ ΠΎΠ³ΠΎΠ½Ρ:
ΠΡΠΈΠΌΠ΅Ρ 1
Π°) ΠΠ°ΠΉΡΠΈ Π΄Π»ΠΈΠ½Ρ Π²Π΅ΠΊΡΠΎΡΠ½ΠΎΠ³ΠΎ ΠΏΡΠΎΠΈΠ·Π²Π΅Π΄Π΅Π½ΠΈΡ Π²Π΅ΠΊΡΠΎΡΠΎΠ² , Π΅ΡΠ»ΠΈ
Π±) ΠΠ°ΠΉΡΠΈ ΠΏΠ»ΠΎΡΠ°Π΄Ρ ΠΏΠ°ΡΠ°Π»Π»Π΅Π»ΠΎΠ³ΡΠ°ΠΌΠΌΠ°, ΠΏΠΎΡΡΡΠΎΠ΅Π½Π½ΠΎΠ³ΠΎ Π½Π° Π²Π΅ΠΊΡΠΎΡΠ°Ρ , Π΅ΡΠ»ΠΈ
Π Π΅ΡΠ΅Π½ΠΈΠ΅ : ΠΠ΅Ρ, ΡΡΠΎ Π½Π΅ ΠΎΠΏΠ΅ΡΠ°ΡΠΊΠ°, ΠΈΡΡ ΠΎΠ΄Π½ΡΠ΅ Π΄Π°Π½Π½ΡΠ΅ Π² ΠΏΡΠ½ΠΊΡΠ°Ρ ΡΡΠ»ΠΎΠ²ΠΈΡ Ρ Π½Π°ΠΌΠ΅ΡΠ΅Π½Π½ΠΎ ΡΠ΄Π΅Π»Π°Π» ΠΎΠ΄ΠΈΠ½Π°ΠΊΠΎΠ²ΡΠΌΠΈ. ΠΠΎΡΠΎΠΌΡ ΡΡΠΎ ΠΎΡΠΎΡΠΌΠ»Π΅Π½ΠΈΠ΅ ΡΠ΅ΡΠ΅Π½ΠΈΠΉ Π±ΡΠ΄Π΅Ρ ΠΎΡΠ»ΠΈΡΠ°ΡΡΡΡ!
Π°) ΠΠΎ ΡΡΠ»ΠΎΠ²ΠΈΡ ΡΡΠ΅Π±ΡΠ΅ΡΡΡ Π½Π°ΠΉΡΠΈ Π΄Π»ΠΈΠ½Ρ Π²Π΅ΠΊΡΠΎΡΠ° (Π²Π΅ΠΊΡΠΎΡΠ½ΠΎΠ³ΠΎ ΠΏΡΠΎΠΈΠ·Π²Π΅Π΄Π΅Π½ΠΈΡ). ΠΠΎ ΡΠΎΠΎΡΠ²Π΅ΡΡΡΠ²ΡΡΡΠ΅ΠΉ ΡΠΎΡΠΌΡΠ»Π΅:
ΠΡΠ²Π΅Ρ :
ΠΠΎΠ»Ρ ΡΠΊΠΎΡΠΎ ΡΠΏΡΠ°ΡΠΈΠ²Π°Π»ΠΎΡΡ ΠΎ Π΄Π»ΠΈΠ½Π΅, ΡΠΎ Π² ΠΎΡΠ²Π΅ΡΠ΅ ΡΠΊΠ°Π·ΡΠ²Π°Π΅ΠΌ ΡΠ°Π·ΠΌΠ΅ΡΠ½ΠΎΡΡΡ β Π΅Π΄ΠΈΠ½ΠΈΡΡ.
Π±) ΠΠΎ ΡΡΠ»ΠΎΠ²ΠΈΡ ΡΡΠ΅Π±ΡΠ΅ΡΡΡ Π½Π°ΠΉΡΠΈ ΠΏΠ»ΠΎΡΠ°Π΄Ρ ΠΏΠ°ΡΠ°Π»Π»Π΅Π»ΠΎΠ³ΡΠ°ΠΌΠΌΠ°, ΠΏΠΎΡΡΡΠΎΠ΅Π½Π½ΠΎΠ³ΠΎ Π½Π° Π²Π΅ΠΊΡΠΎΡΠ°Ρ
. ΠΠ»ΠΎΡΠ°Π΄Ρ Π΄Π°Π½Π½ΠΎΠ³ΠΎ ΠΏΠ°ΡΠ°Π»Π»Π΅Π»ΠΎΠ³ΡΠ°ΠΌΠΌΠ° ΡΠΈΡΠ»Π΅Π½Π½ΠΎ ΡΠ°Π²Π½Π° Π΄Π»ΠΈΠ½Π΅ Π²Π΅ΠΊΡΠΎΡΠ½ΠΎΠ³ΠΎ ΠΏΡΠΎΠΈΠ·Π²Π΅Π΄Π΅Π½ΠΈΡ:
ΠΡΠ²Π΅Ρ :
ΠΠ±ΡΠ°ΡΠΈΡΠ΅ Π²Π½ΠΈΠΌΠ°Π½ΠΈΠ΅, ΡΡΠΎ Π² ΠΎΡΠ²Π΅ΡΠ΅ ΠΎ Π²Π΅ΠΊΡΠΎΡΠ½ΠΎΠΌ ΠΏΡΠΎΠΈΠ·Π²Π΅Π΄Π΅Π½ΠΈΠΈ ΡΠ΅ΡΠΈ Π½Π΅ ΠΈΠ΄ΡΡ Π²ΠΎΠΎΠ±ΡΠ΅, Π½Π°Ρ ΡΠΏΡΠ°ΡΠΈΠ²Π°Π»ΠΈ ΠΎ ΠΏΠ»ΠΎΡΠ°Π΄ΠΈ ΡΠΈΠ³ΡΡΡ , ΡΠΎΠΎΡΠ²Π΅ΡΡΡΠ²Π΅Π½Π½ΠΎ, ΡΠ°Π·ΠΌΠ΅ΡΠ½ΠΎΡΡΡ β ΠΊΠ²Π°Π΄ΡΠ°ΡΠ½ΡΠ΅ Π΅Π΄ΠΈΠ½ΠΈΡΡ.
ΠΡΠ΅Π³Π΄Π° ΡΠΌΠΎΡΡΠΈΠΌ, Π§Π’Π ΡΡΠ΅Π±ΡΠ΅ΡΡΡ Π½Π°ΠΉΡΠΈ ΠΏΠΎ ΡΡΠ»ΠΎΠ²ΠΈΡ, ΠΈ, ΠΈΡΡ ΠΎΠ΄Ρ ΠΈΠ· ΡΡΠΎΠ³ΠΎ, ΡΠΎΡΠΌΡΠ»ΠΈΡΡΠ΅ΠΌ ΡΡΡΠΊΠΈΠΉ ΠΎΡΠ²Π΅Ρ. ΠΠΎΠΆΠ΅Ρ ΠΏΠΎΠΊΠ°Π·Π°ΡΡΡΡ Π±ΡΠΊΠ²ΠΎΠ΅Π΄ΡΡΠ²ΠΎΠΌ, Π½ΠΎ Π±ΡΠΊΠ²ΠΎΠ΅Π΄ΠΎΠ² ΡΡΠ΅Π΄ΠΈ ΠΏΡΠ΅ΠΏΠΎΠ΄Π°Π²Π°ΡΠ΅Π»Π΅ΠΉ Ρ Π²Π°ΡΠ°Π΅Ρ, ΠΈ Π·Π°Π΄Π°Π½ΠΈΠ΅ Ρ Ρ ΠΎΡΠΎΡΠΈΠΌΠΈ ΡΠ°Π½ΡΠ°ΠΌΠΈ Π²Π΅ΡΠ½ΡΡΡΡ Π½Π° Π΄ΠΎΡΠ°Π±ΠΎΡΠΊΡ. Π₯ΠΎΡΡ ΡΡΠΎ Π½Π΅ ΠΎΡΠΎΠ±ΠΎ Π½Π°ΡΡΠ½ΡΡΠ°Ρ ΠΏΡΠΈΠ΄ΠΈΡΠΊΠ° β Π΅ΡΠ»ΠΈ ΠΎΡΠ²Π΅Ρ Π½Π΅ΠΊΠΎΡΡΠ΅ΠΊΡΠ΅Π½, ΡΠΎ ΡΠΊΠ»Π°Π΄ΡΠ²Π°Π΅ΡΡΡ Π²ΠΏΠ΅ΡΠ°ΡΠ»Π΅Π½ΠΈΠ΅, ΡΡΠΎ ΡΠ΅Π»ΠΎΠ²Π΅ΠΊ Π½Π΅ ΡΠ°Π·Π±ΠΈΡΠ°Π΅ΡΡΡ Π² ΠΏΡΠΎΡΡΡΡ Π²Π΅ΡΠ°Ρ ΠΈ/ΠΈΠ»ΠΈ Π½Π΅ Π²Π½ΠΈΠΊ Π² ΡΡΡΡ Π·Π°Π΄Π°Π½ΠΈΡ. ΠΡΠΎΡ ΠΌΠΎΠΌΠ΅Π½Ρ Π²ΡΠ΅Π³Π΄Π° Π½ΡΠΆΠ½ΠΎ Π΄Π΅ΡΠΆΠ°ΡΡ Π½Π° ΠΊΠΎΠ½ΡΡΠΎΠ»Π΅, ΡΠ΅ΡΠ°Ρ Π»ΡΠ±ΡΡ Π·Π°Π΄Π°ΡΡ ΠΏΠΎ Π²ΡΡΡΠ΅ΠΉ ΠΌΠ°ΡΠ΅ΠΌΠ°ΡΠΈΠΊΠ΅, Π΄Π° ΠΈ ΠΏΠΎ Π΄ΡΡΠ³ΠΈΠΌ ΠΏΡΠ΅Π΄ΠΌΠ΅ΡΠ°ΠΌ ΡΠΎΠΆΠ΅.
ΠΡΠ΄Π° ΠΏΠΎΠ΄Π΅Π²Π°Π»Π°ΡΡ Π±ΠΎΠ»ΡΡΠ°Ρ Π±ΡΠΊΠΎΠ²ΠΊΠ° Β«ΡΠ½Β»? Π ΠΏΡΠΈΠ½ΡΠΈΠΏΠ΅, Π΅Ρ ΠΌΠΎΠΆΠ½ΠΎ Π±ΡΠ»ΠΎ Π΄ΠΎΠΏΠΎΠ»Π½ΠΈΡΠ΅Π»ΡΠ½ΠΎ ΠΏΡΠΈΠ»Π΅ΠΏΠΈΡΡ Π² ΡΠ΅ΡΠ΅Π½ΠΈΠ΅, Π½ΠΎ Π² ΡΠ΅Π»ΡΡ ΡΠΎΠΊΡΠ°ΡΠΈΡΡ Π·Π°ΠΏΠΈΡΡ, Ρ ΡΡΠΎΠ³ΠΎ Π½Π΅ ΡΠ΄Π΅Π»Π°Π». ΠΠ°Π΄Π΅ΡΡΡ, Π²ΡΠ΅ΠΌ ΠΏΠΎΠ½ΡΡΠ½ΠΎ, ΡΡΠΎ ΠΈ β ΡΡΠΎ ΠΎΠ±ΠΎΠ·Π½Π°ΡΠ΅Π½ΠΈΠ΅ ΠΎΠ΄Π½ΠΎΠ³ΠΎ ΠΈ ΡΠΎΠ³ΠΎ ΠΆΠ΅.
ΠΠΎΠΏΡΠ»ΡΡΠ½ΡΠΉ ΠΏΡΠΈΠΌΠ΅Ρ Π΄Π»Ρ ΡΠ°ΠΌΠΎΡΡΠΎΡΡΠ΅Π»ΡΠ½ΠΎΠ³ΠΎ ΡΠ΅ΡΠ΅Π½ΠΈΡ:
ΠΡΠΈΠΌΠ΅Ρ 2
ΠΠ°ΠΉΡΠΈ ΠΏΠ»ΠΎΡΠ°Π΄Ρ ΡΡΠ΅ΡΠ³ΠΎΠ»ΡΠ½ΠΈΠΊΠ°, ΠΏΠΎΡΡΡΠΎΠ΅Π½Π½ΠΎΠ³ΠΎ Π½Π° Π²Π΅ΠΊΡΠΎΡΠ°Ρ , Π΅ΡΠ»ΠΈ
Π€ΠΎΡΠΌΡΠ»Π° Π½Π°Ρ
ΠΎΠΆΠ΄Π΅Π½ΠΈΡ ΠΏΠ»ΠΎΡΠ°Π΄ΠΈ ΡΡΠ΅ΡΠ³ΠΎΠ»ΡΠ½ΠΈΠΊΠ° ΡΠ΅ΡΠ΅Π· Π²Π΅ΠΊΡΠΎΡΠ½ΠΎΠ΅ ΠΏΡΠΎΠΈΠ·Π²Π΅Π΄Π΅Π½ΠΈΠ΅ Π΄Π°Π½Π° Π² ΠΊΠΎΠΌΠΌΠ΅Π½ΡΠ°ΡΠΈΡΡ
ΠΊ ΠΎΠΏΡΠ΅Π΄Π΅Π»Π΅Π½ΠΈΡ. Π Π΅ΡΠ΅Π½ΠΈΠ΅ ΠΈ ΠΎΡΠ²Π΅Ρ Π² ΠΊΠΎΠ½ΡΠ΅ ΡΡΠΎΠΊΠ°.
ΠΠ° ΠΏΡΠ°ΠΊΡΠΈΠΊΠ΅ Π·Π°Π΄Π°ΡΠ° Π΄Π΅ΠΉΡΡΠ²ΠΈΡΠ΅Π»ΡΠ½ΠΎ ΠΎΡΠ΅Π½Ρ ΡΠ°ΡΠΏΡΠΎΡΡΡΠ°Π½Π΅Π½Π°, ΡΡΠ΅ΡΠ³ΠΎΠ»ΡΠ½ΠΈΠΊΠ°ΠΌΠΈ Π²ΠΎΠΎΠ±ΡΠ΅ ΠΌΠΎΠ³ΡΡ Π·Π°ΠΌΡΡΠΈΡΡ.
ΠΠ»Ρ ΡΠ΅ΡΠ΅Π½ΠΈΡ Π΄ΡΡΠ³ΠΈΡ Π·Π°Π΄Π°Ρ Π½Π°ΠΌ ΠΏΠΎΠ½Π°Π΄ΠΎΠ±ΡΡΡΡ:
Π‘Π²ΠΎΠΉΡΡΠ²Π° Π²Π΅ΠΊΡΠΎΡΠ½ΠΎΠ³ΠΎ ΠΏΡΠΎΠΈΠ·Π²Π΅Π΄Π΅Π½ΠΈΡ Π²Π΅ΠΊΡΠΎΡΠΎΠ²ΠΠ΅ΠΊΠΎΡΠΎΡΡΠ΅ ΡΠ²ΠΎΠΉΡΡΠ²Π° Π²Π΅ΠΊΡΠΎΡΠ½ΠΎΠ³ΠΎ ΠΏΡΠΎΠΈΠ·Π²Π΅Π΄Π΅Π½ΠΈΡ ΠΌΡ ΡΠΆΠ΅ ΡΠ°ΡΡΠΌΠΎΡΡΠ΅Π»ΠΈ, ΡΠ΅ΠΌ Π½Π΅ ΠΌΠ΅Π½Π΅Π΅, Ρ ΠΈΡ Π²ΠΊΠ»ΡΡΡ Π² Π΄Π°Π½Π½ΡΠΉ ΡΠΏΠΈΡΠΎΠΊ.
ΠΠ»Ρ ΠΏΡΠΎΠΈΠ·Π²ΠΎΠ»ΡΠ½ΡΡ Π²Π΅ΠΊΡΠΎΡΠΎΠ² ΠΈ ΠΏΡΠΎΠΈΠ·Π²ΠΎΠ»ΡΠ½ΠΎΠ³ΠΎ ΡΠΈΡΠ»Π° ΡΠΏΡΠ°Π²Π΅Π΄Π»ΠΈΠ²Ρ ΡΠ»Π΅Π΄ΡΡΡΠΈΠ΅ ΡΠ²ΠΎΠΉΡΡΠ²Π°:
1) Π Π΄ΡΡΠ³ΠΈΡ ΠΈΡΡΠΎΡΠ½ΠΈΠΊΠ°Ρ ΠΈΠ½ΡΠΎΡΠΌΠ°ΡΠΈΠΈ Π΄Π°Π½Π½ΡΠΉ ΠΏΡΠ½ΠΊΡ ΠΎΠ±ΡΡΠ½ΠΎ Π½Π΅ Π²ΡΠ΄Π΅Π»ΡΡΡ Π² ΡΠ²ΠΎΠΉΡΡΠ²Π°Ρ , Π½ΠΎ ΠΎΠ½ ΠΎΡΠ΅Π½Ρ Π²Π°ΠΆΠ΅Π½ Π² ΠΏΡΠ°ΠΊΡΠΈΡΠ΅ΡΠΊΠΎΠΌ ΠΏΠ»Π°Π½Π΅. ΠΠΎΡΡΠΎΠΌΡ ΠΏΡΡΡΡ Π±ΡΠ΄Π΅Ρ.
2) β ΡΠ²ΠΎΠΉΡΡΠ²ΠΎ ΡΠΎΠΆΠ΅ ΡΠ°Π·ΠΎΠ±ΡΠ°Π½ΠΎ Π²ΡΡΠ΅, ΠΈΠ½ΠΎΠ³Π΄Π° Π΅Π³ΠΎ Π½Π°Π·ΡΠ²Π°ΡΡ Π°Π½ΡΠΈΠΊΠΎΠΌΠΌΡΡΠ°ΡΠΈΠ²Π½ΠΎΡΡΡΡ . ΠΠ½ΡΠΌΠΈ ΡΠ»ΠΎΠ²Π°ΠΌΠΈ, ΠΏΠΎΡΡΠ΄ΠΎΠΊ Π²Π΅ΠΊΡΠΎΡΠΎΠ² ΠΈΠΌΠ΅Π΅Ρ Π·Π½Π°ΡΠ΅Π½ΠΈΠ΅.
3) β ΡΠΎΡΠ΅ΡΠ°ΡΠ΅Π»ΡΠ½ΡΠ΅ ΠΈΠ»ΠΈ Π°ΡΡΠΎΡΠΈΠ°ΡΠΈΠ²Π½ΡΠ΅ Π·Π°ΠΊΠΎΠ½Ρ Π²Π΅ΠΊΡΠΎΡΠ½ΠΎΠ³ΠΎ ΠΏΡΠΎΠΈΠ·Π²Π΅Π΄Π΅Π½ΠΈΡ. ΠΠΎΠ½ΡΡΠ°Π½ΡΡ Π±Π΅Π·ΠΏΡΠΎΠ±Π»Π΅ΠΌΠ½ΠΎ Π²ΡΠ½ΠΎΡΡΡΡΡ Π·Π° ΠΏΡΠ΅Π΄Π΅Π»Ρ Π²Π΅ΠΊΡΠΎΡΠ½ΠΎΠ³ΠΎ ΠΏΡΠΎΠΈΠ·Π²Π΅Π΄Π΅Π½ΠΈΡ. ΠΠ΅ΠΉΡΡΠ²ΠΈΡΠ΅Π»ΡΠ½ΠΎ, ΡΠ΅Π³ΠΎ ΠΈΠΌ ΡΠ°ΠΌ Π΄Π΅Π»Π°ΡΡ?
4) β ΡΠ°ΡΠΏΡΠ΅Π΄Π΅Π»ΠΈΡΠ΅Π»ΡΠ½ΡΠ΅ ΠΈΠ»ΠΈ Π΄ΠΈΡΡΡΠΈΠ±ΡΡΠΈΠ²Π½ΡΠ΅ Π·Π°ΠΊΠΎΠ½Ρ Π²Π΅ΠΊΡΠΎΡΠ½ΠΎΠ³ΠΎ ΠΏΡΠΎΠΈΠ·Π²Π΅Π΄Π΅Π½ΠΈΡ. Π‘ ΡΠ°ΡΠΊΡΡΡΠΈΠ΅ΠΌ ΡΠΊΠΎΠ±ΠΎΠΊ ΡΠΎΠΆΠ΅ Π½Π΅Ρ ΠΏΡΠΎΠ±Π»Π΅ΠΌ.
Π ΠΊΠ°ΡΠ΅ΡΡΠ²Π΅ Π΄Π΅ΠΌΠΎΠ½ΡΡΡΠ°ΡΠΈΠΈ ΡΠ°ΡΡΠΌΠΎΡΡΠΈΠΌ ΠΊΠΎΡΠΎΡΠ΅Π½ΡΠΊΠΈΠΉ ΠΏΡΠΈΠΌΠ΅Ρ:
ΠΡΠΈΠΌΠ΅Ρ 3
ΠΠ°ΠΉΡΠΈ , Π΅ΡΠ»ΠΈ
Π Π΅ΡΠ΅Π½ΠΈΠ΅: ΠΠΎ ΡΡΠ»ΠΎΠ²ΠΈΡ ΡΠ½ΠΎΠ²Π° ΡΡΠ΅Π±ΡΠ΅ΡΡΡ Π½Π°ΠΉΡΠΈ Π΄Π»ΠΈΠ½Ρ Π²Π΅ΠΊΡΠΎΡΠ½ΠΎΠ³ΠΎ ΠΏΡΠΎΠΈΠ·Π²Π΅Π΄Π΅Π½ΠΈΡ. Π Π°ΡΠΏΠΈΡΠ΅ΠΌ Π½Π°ΡΡ ΠΌΠΈΠ½ΠΈΠ°ΡΡΡΡ:
(1) Π‘ΠΎΠ³Π»Π°ΡΠ½ΠΎ Π°ΡΡΠΎΡΠΈΠ°ΡΠΈΠ²Π½ΡΠΌ Π·Π°ΠΊΠΎΠ½Π°ΠΌ, Π²ΡΠ½ΠΎΡΠΈΠΌ ΠΊΠΎΠ½ΡΡΠ°Π½ΡΡ Π·Π° ΠΏΠ΅ΡΠ΅Π΄Π΅Π»Ρ Π²Π΅ΠΊΡΠΎΡΠ½ΠΎΠ³ΠΎ ΠΏΡΠΎΠΈΠ·Π²Π΅Π΄Π΅Π½ΠΈΡ.
(2) ΠΡΠ½ΠΎΡΠΈΠΌ ΠΊΠΎΠ½ΡΡΠ°Π½ΡΡ Π·Π° ΠΏΡΠ΅Π΄Π΅Π»Ρ ΠΌΠΎΠ΄ΡΠ»Ρ, ΠΏΡΠΈ ΡΡΠΎΠΌ ΠΌΠΎΠ΄ΡΠ»Ρ Β«ΡΡΠ΅Π΄Π°Π΅ΡΒ» Π·Π½Π°ΠΊ Β«ΠΌΠΈΠ½ΡΡΒ». ΠΠ»ΠΈΠ½Π° ΠΆΠ΅ Π½Π΅ ΠΌΠΎΠΆΠ΅Ρ Π±ΡΡΡ ΠΎΡΡΠΈΡΠ°ΡΠ΅Π»ΡΠ½ΠΎΠΉ.
(3) ΠΠ°Π»ΡΠ½Π΅ΠΉΡΠ΅Π΅ ΠΏΠΎΠ½ΡΡΠ½ΠΎ.
ΠΡΠ²Π΅Ρ :
ΠΠΎΡΠ° ΠΏΠΎΠ΄Π±ΡΠΎΡΠΈΡΡ Π΄ΡΠΎΠ² Π² ΠΎΠ³ΠΎΠ½Ρ:
ΠΡΠΈΠΌΠ΅Ρ 4
ΠΡΡΠΈΡΠ»ΠΈΡΡ ΠΏΠ»ΠΎΡΠ°Π΄Ρ ΡΡΠ΅ΡΠ³ΠΎΠ»ΡΠ½ΠΈΠΊΠ°, ΠΏΠΎΡΡΡΠΎΠ΅Π½Π½ΠΎΠ³ΠΎ Π½Π° Π²Π΅ΠΊΡΠΎΡΠ°Ρ , Π΅ΡΠ»ΠΈ
Π Π΅ΡΠ΅Π½ΠΈΠ΅ : ΠΠ»ΠΎΡΠ°Π΄Ρ ΡΡΠ΅ΡΠ³ΠΎΠ»ΡΠ½ΠΈΠΊΠ° Π½Π°ΠΉΠ΄ΡΠΌ ΠΏΠΎ ΡΠΎΡΠΌΡΠ»Π΅ . ΠΠ°Π³Π²ΠΎΠ·Π΄ΠΊΠ° ΡΠΎΡΡΠΎΠΈΡ Π² ΡΠΎΠΌ, ΡΡΠΎ Π²Π΅ΠΊΡΠΎΡΡ Β«ΡΡΒ» ΠΈ Β«Π΄ΡΒ» ΡΠ°ΠΌΠΈ ΠΏΡΠ΅Π΄ΡΡΠ°Π²Π»Π΅Π½Ρ Π² Π²ΠΈΠ΄Π΅ ΡΡΠΌΠΌ Π²Π΅ΠΊΡΠΎΡΠΎΠ². ΠΠ»Π³ΠΎΡΠΈΡΠΌ Π·Π΄Π΅ΡΡ ΡΡΠ°Π½Π΄Π°ΡΡΠ΅Π½ ΠΈ ΡΠ΅ΠΌ-ΡΠΎ Π½Π°ΠΏΠΎΠΌΠΈΠ½Π°Π΅Ρ ΠΏΡΠΈΠΌΠ΅ΡΡ β 3 ΠΈ 4 ΡΡΠΎΠΊΠ° Π‘ΠΊΠ°Π»ΡΡΠ½ΠΎΠ΅ ΠΏΡΠΎΠΈΠ·Π²Π΅Π΄Π΅Π½ΠΈΠ΅ Π²Π΅ΠΊΡΠΎΡΠΎΠ² . Π Π΅ΡΠ΅Π½ΠΈΠ΅ Π΄Π»Ρ ΡΡΠ½ΠΎΡΡΠΈ ΡΠ°Π·ΠΎΠ±ΡΡΠΌ Π½Π° ΡΡΠΈ ΡΡΠ°ΠΏΠ°:
1) ΠΠ° ΠΏΠ΅ΡΠ²ΠΎΠΌ ΡΠ°Π³Π΅ Π²ΡΡΠ°Π·ΠΈΠΌ Π²Π΅ΠΊΡΠΎΡΠ½ΠΎΠ΅ ΠΏΡΠΎΠΈΠ·Π²Π΅Π΄Π΅Π½ΠΈΠ΅ ΡΠ΅ΡΠ΅Π· Π²Π΅ΠΊΡΠΎΡΠ½ΠΎΠ΅ ΠΏΡΠΎΠΈΠ·Π²Π΅Π΄Π΅Π½ΠΈΠ΅ , ΠΏΠΎ ΡΡΡΠΈ, Π²ΡΡΠ°Π·ΠΈΠΌ Π²Π΅ΠΊΡΠΎΡ ΡΠ΅ΡΠ΅Π· Π²Π΅ΠΊΡΠΎΡ . Π Π΄Π»ΠΈΠ½Π°Ρ
ΠΏΠΎΠΊΠ° Π½ΠΈ ΡΠ»ΠΎΠ²Π°!
(1) ΠΠΎΠ΄ΡΡΠ°Π²Π»ΡΠ΅ΠΌ Π²ΡΡΠ°ΠΆΠ΅Π½ΠΈΡ Π²Π΅ΠΊΡΠΎΡΠΎΠ² .
(2) ΠΡΠΏΠΎΠ»ΡΠ·ΡΡ Π΄ΠΈΡΡΡΠΈΠ±ΡΡΠΈΠ²Π½ΡΠ΅ Π·Π°ΠΊΠΎΠ½Ρ, ΡΠ°ΡΠΊΡΡΠ²Π°Π΅ΠΌ ΡΠΊΠΎΠ±ΠΊΠΈ ΠΏΠΎ ΠΏΡΠ°Π²ΠΈΠ»Ρ ΡΠΌΠ½ΠΎΠΆΠ΅Π½ΠΈΡ ΠΌΠ½ΠΎΠ³ΠΎΡΠ»Π΅Π½ΠΎΠ².
(3) ΠΡΠΏΠΎΠ»ΡΠ·ΡΡ Π°ΡΡΠΎΡΠΈΠ°ΡΠΈΠ²Π½ΡΠ΅ Π·Π°ΠΊΠΎΠ½Ρ, Π²ΡΠ½ΠΎΡΠΈΠΌ Π²ΡΠ΅ ΠΊΠΎΠ½ΡΡΠ°Π½ΡΡ Π·Π° ΠΏΡΠ΅Π΄Π΅Π»Ρ Π²Π΅ΠΊΡΠΎΡΠ½ΡΡ ΠΏΡΠΎΠΈΠ·Π²Π΅Π΄Π΅Π½ΠΈΠΉ. ΠΡΠΈ ΠΌΠ°Π»ΠΎΠΌΠ°Π»ΡΡΠΊΠΎΠΌ ΠΎΠΏΡΡΠ΅ Π΄Π΅ΠΉΡΡΠ²ΠΈΡ 2 ΠΈ 3 ΠΌΠΎΠΆΠ½ΠΎ Π²ΡΠΏΠΎΠ»Π½ΡΡΡ ΠΎΠ΄Π½ΠΎΠ²ΡΠ΅ΠΌΠ΅Π½Π½ΠΎ.
(4) ΠΠ΅ΡΠ²ΠΎΠ΅ ΠΈ ΠΏΠΎΡΠ»Π΅Π΄Π½Π΅Π΅ ΡΠ»Π°Π³Π°Π΅ΠΌΠΎΠ΅ ΡΠ°Π²Π½ΠΎ Π½ΡΠ»Ρ (Π½ΡΠ»Π΅Π²ΠΎΠΌΡ Π²Π΅ΠΊΡΠΎΡΡ) Π±Π»Π°Π³ΠΎΠ΄Π°ΡΡ ΠΏΡΠΈΡΡΠ½ΠΎΠΌΡ ΡΠ²ΠΎΠΉΡΡΠ²Ρ . ΠΠΎ Π²ΡΠΎΡΠΎΠΌ ΡΠ»Π°Π³Π°Π΅ΠΌΠΎΠΌ ΠΈΡΠΏΠΎΠ»ΡΠ·ΡΠ΅ΠΌ ΡΠ²ΠΎΠΉΡΡΠ²ΠΎ Π°Π½ΡΠΈΠΊΠΎΠΌΠΌΡΡΠ°ΡΠΈΠ²Π½ΠΎΡΡΠΈ Π²Π΅ΠΊΡΠΎΡΠ½ΠΎΠ³ΠΎ ΠΏΡΠΎΠΈΠ·Π²Π΅Π΄Π΅Π½ΠΈΡ:
(5) ΠΡΠΈΠ²ΠΎΠ΄ΠΈΠΌ ΠΏΠΎΠ΄ΠΎΠ±Π½ΡΠ΅ ΡΠ»Π°Π³Π°Π΅ΠΌΡΠ΅.
Π ΡΠ΅Π·ΡΠ»ΡΡΠ°ΡΠ΅ Π²Π΅ΠΊΡΠΎΡ ΠΎΠΊΠ°Π·Π°Π»ΡΡ Π²ΡΡΠ°ΠΆΠ΅Π½ ΡΠ΅ΡΠ΅Π· Π²Π΅ΠΊΡΠΎΡ, ΡΠ΅Π³ΠΎ ΠΈ ΡΡΠ΅Π±ΠΎΠ²Π°Π»ΠΎΡΡ Π΄ΠΎΡΡΠΈΡΡ:
2) ΠΠ° Π²ΡΠΎΡΠΎΠΌ ΡΠ°Π³Π΅ Π½Π°ΠΉΠ΄Π΅ΠΌ Π΄Π»ΠΈΠ½Ρ Π½ΡΠΆΠ½ΠΎΠ³ΠΎ Π½Π°ΠΌ Π²Π΅ΠΊΡΠΎΡΠ½ΠΎΠ³ΠΎ ΠΏΡΠΎΠΈΠ·Π²Π΅Π΄Π΅Π½ΠΈΡ. ΠΠ°Π½Π½ΠΎΠ΅ Π΄Π΅ΠΉΡΡΠ²ΠΈΠ΅ Π½Π°ΠΏΠΎΠΌΠΈΠ½Π°Π΅Ρ ΠΡΠΈΠΌΠ΅Ρ 3:
3) ΠΠ°ΠΉΠ΄ΡΠΌ ΠΏΠ»ΠΎΡΠ°Π΄Ρ ΠΈΡΠΊΠΎΠΌΠΎΠ³ΠΎ ΡΡΠ΅ΡΠ³ΠΎΠ»ΡΠ½ΠΈΠΊΠ°:
ΠΡΠ°ΠΏΡ 2-3 ΡΠ΅ΡΠ΅Π½ΠΈΡ ΠΌΠΎΠΆΠ½ΠΎ Π±ΡΠ»ΠΎ ΠΎΡΠΎΡΠΌΠΈΡΡ ΠΈ ΠΎΠ΄Π½ΠΎΠΉ ΡΡΡΠΎΠΊΠΎΠΉ.
ΠΡΠ²Π΅Ρ :
Π Π°ΡΡΠΌΠΎΡΡΠ΅Π½Π½Π°Ρ Π·Π°Π΄Π°ΡΠ° Π΄ΠΎΡΡΠ°ΡΠΎΡΠ½ΠΎ ΡΠ°ΡΠΏΡΠΎΡΡΡΠ°Π½Π΅Π½Π° Π² ΠΊΠΎΠ½ΡΡΠΎΠ»ΡΠ½ΡΡ ΡΠ°Π±ΠΎΡΠ°Ρ , Π²ΠΎΡ ΠΏΡΠΈΠΌΠ΅Ρ Π΄Π»Ρ ΡΠ°ΠΌΠΎΡΡΠΎΡΡΠ΅Π»ΡΠ½ΠΎΠ³ΠΎ ΡΠ΅ΡΠ΅Π½ΠΈΡ:
ΠΡΠΈΠΌΠ΅Ρ 5
ΠΠ°ΠΉΡΠΈ , Π΅ΡΠ»ΠΈ
ΠΡΠ°ΡΠΊΠΎΠ΅ ΡΠ΅ΡΠ΅Π½ΠΈΠ΅ ΠΈ ΠΎΡΠ²Π΅Ρ Π² ΠΊΠΎΠ½ΡΠ΅ ΡΡΠΎΠΊΠ°. ΠΠΎΡΠΌΠΎΡΡΠΈΠΌ, Π½Π°ΡΠΊΠΎΠ»ΡΠΊΠΎ Π²Ρ Π±ΡΠ»ΠΈ Π²Π½ΠΈΠΌΠ°ΡΠ΅Π»ΡΠ½Ρ ΠΏΡΠΈ ΠΈΠ·ΡΡΠ΅Π½ΠΈΠΈ ΠΏΡΠ΅Π΄ΡΠ΄ΡΡΠΈΡ
ΠΏΡΠΈΠΌΠ΅ΡΠΎΠ²;-)
, Π·Π°Π΄Π°Π½Π½ΡΡ Π² ΠΎΡΡΠΎΠ½ΠΎΡΠΌΠΈΡΠΎΠ²Π°Π½Π½ΠΎΠΌ Π±Π°Π·ΠΈΡΠ΅ , Π²ΡΡΠ°ΠΆΠ°Π΅ΡΡΡ ΡΠΎΡΠΌΡΠ»ΠΎΠΉ :
Π€ΠΎΡΠΌΡΠ»Π° ΠΈ ΠΏΡΠ°Π²Π΄Π° ΠΏΡΠΎΡΡΠ΅ΡΠΊΠ°Ρ: Π² Π²Π΅ΡΡ
Π½ΡΡ ΡΡΡΠΎΠΊΡ ΠΎΠΏΡΠ΅Π΄Π΅Π»ΠΈΡΠ΅Π»Ρ Π·Π°ΠΏΠΈΡΡΠ²Π°Π΅ΠΌ ΠΊΠΎΠΎΡΠ΄ΠΈΠ½Π°ΡΠ½ΡΠ΅ Π²Π΅ΠΊΡΠΎΡΡ, Π²ΠΎ Π²ΡΠΎΡΡΡ ΠΈ ΡΡΠ΅ΡΡΡ ΡΡΡΠΎΠΊΠΈ Β«ΡΠΊΠ»Π°Π΄ΡΠ²Π°Π΅ΠΌΒ» ΠΊΠΎΠΎΡΠ΄ΠΈΠ½Π°ΡΡ Π²Π΅ΠΊΡΠΎΡΠΎΠ² , ΠΏΡΠΈΡΡΠΌ ΡΠΊΠ»Π°Π΄ΡΠ²Π°Π΅ΠΌ Π² ΡΡΡΠΎΠ³ΠΎΠΌ ΠΏΠΎΡΡΠ΄ΠΊΠ΅ β ΡΠ½Π°ΡΠ°Π»Π° ΠΊΠΎΠΎΡΠ΄ΠΈΠ½Π°ΡΡ Π²Π΅ΠΊΡΠΎΡΠ° Β«Π²ΡΒ», Π·Π°ΡΠ΅ΠΌ ΠΊΠΎΠΎΡΠ΄ΠΈΠ½Π°ΡΡ Π²Π΅ΠΊΡΠΎΡΠ° Β«Π΄ΡΠ±Π»Ρ-Π²ΡΒ». ΠΡΠ»ΠΈ Π²Π΅ΠΊΡΠΎΡΡ Π½ΡΠΆΠ½ΠΎ ΡΠΌΠ½ΠΎΠΆΠΈΡΡ Π² Π΄ΡΡΠ³ΠΎΠΌ ΠΏΠΎΡΡΠ΄ΠΊΠ΅, ΡΠΎ ΠΈ ΡΡΡΠΎΠΊΠΈ ΡΠ»Π΅Π΄ΡΠ΅Ρ ΠΏΠΎΠΌΠ΅Π½ΡΡΡ ΠΌΠ΅ΡΡΠ°ΠΌΠΈ:
ΠΡΠΈΠΌΠ΅Ρ 10
ΠΡΠΎΠ²Π΅ΡΠΈΡΡ, Π±ΡΠ΄ΡΡ Π»ΠΈ ΠΊΠΎΠ»Π»ΠΈΠ½Π΅Π°ΡΠ½Ρ ΡΠ»Π΅Π΄ΡΡΡΠΈΠ΅ Π²Π΅ΠΊΡΠΎΡΡ ΠΏΡΠΎΡΡΡΠ°Π½ΡΡΠ²Π°:
Π°)
Π±)
Π Π΅ΡΠ΅Π½ΠΈΠ΅ : ΠΡΠΎΠ²Π΅ΡΠΊΠ° ΠΎΡΠ½ΠΎΠ²Π°Π½Π° Π½Π° ΠΎΠ΄Π½ΠΎΠΌ ΠΈΠ· ΡΡΠ²Π΅ΡΠΆΠ΄Π΅Π½ΠΈΠΉ Π΄Π°Π½Π½ΠΎΠ³ΠΎ ΡΡΠΎΠΊΠ°: Π΅ΡΠ»ΠΈ Π²Π΅ΠΊΡΠΎΡΡ ΠΊΠΎΠ»Π»ΠΈΠ½Π΅Π°ΡΠ½Ρ, ΡΠΎ ΠΈΡ Π²Π΅ΠΊΡΠΎΡΠ½ΠΎΠ΅ ΠΏΡΠΎΠΈΠ·Π²Π΅Π΄Π΅Π½ΠΈΠ΅ ΡΠ°Π²Π½ΠΎ Π½ΡΠ»Ρ (Π½ΡΠ»Π΅Π²ΠΎΠΌΡ Π²Π΅ΠΊΡΠΎΡΡ): .
Π°) ΠΠ°ΠΉΠ΄ΡΠΌ Π²Π΅ΠΊΡΠΎΡΠ½ΠΎΠ΅ ΠΏΡΠΎΠΈΠ·Π²Π΅Π΄Π΅Π½ΠΈΠ΅:
Π’Π°ΠΊΠΈΠΌ ΠΎΠ±ΡΠ°Π·ΠΎΠΌ, Π²Π΅ΠΊΡΠΎΡΡ Π½Π΅ ΠΊΠΎΠ»Π»ΠΈΠ½Π΅Π°ΡΠ½Ρ.
Π±) ΠΠ°ΠΉΠ΄ΡΠΌ Π²Π΅ΠΊΡΠΎΡΠ½ΠΎΠ΅ ΠΏΡΠΎΠΈΠ·Π²Π΅Π΄Π΅Π½ΠΈΠ΅:
ΠΡΠ²Π΅Ρ : Π°) Π½Π΅ ΠΊΠΎΠ»Π»ΠΈΠ½Π΅Π°ΡΠ½Ρ, Π±)
ΠΠΎΡ, ΠΏΠΎΠΆΠ°Π»ΡΠΉ, ΠΈ Π²ΡΠ΅ ΠΎΡΠ½ΠΎΠ²Π½ΡΠ΅ ΡΠ²Π΅Π΄Π΅Π½ΠΈΡ ΠΎ Π²Π΅ΠΊΡΠΎΡΠ½ΠΎΠΌ ΠΏΡΠΎΠΈΠ·Π²Π΅Π΄Π΅Π½ΠΈΠΈ Π²Π΅ΠΊΡΠΎΡΠΎΠ².
ΠΠ°Π½Π½ΡΠΉ ΡΠ°Π·Π΄Π΅Π» Π±ΡΠ΄Π΅Ρ Π½Π΅ ΠΎΡΠ΅Π½Ρ Π±ΠΎΠ»ΡΡΠΈΠΌ, ΡΠ°ΠΊ ΠΊΠ°ΠΊ Π·Π°Π΄Π°Ρ, Π³Π΄Π΅ ΠΈΡΠΏΠΎΠ»ΡΠ·ΡΠ΅ΡΡΡ ΡΠΌΠ΅ΡΠ°Π½Π½ΠΎΠ΅ ΠΏΡΠΎΠΈΠ·Π²Π΅Π΄Π΅Π½ΠΈΠ΅ Π²Π΅ΠΊΡΠΎΡΠΎΠ², Π½Π΅ΠΌΠ½ΠΎΠ³ΠΎ. Π€Π°ΠΊΡΠΈΡΠ΅ΡΠΊΠΈ Π²ΡΡ Π±ΡΠ΄Π΅Ρ ΡΠΏΠΈΡΠ°ΡΡΡΡ Π² ΠΎΠΏΡΠ΅Π΄Π΅Π»Π΅Π½ΠΈΠ΅, Π³Π΅ΠΎΠΌΠ΅ΡΡΠΈΡΠ΅ΡΠΊΠΈΠΉ ΡΠΌΡΡΠ» ΠΈ ΠΏΠ°ΡΡ ΡΠ°Π±ΠΎΡΠΈΡ ΡΠΎΡΠΌΡΠ».
Π‘ΠΌΠ΅ΡΠ°Π½Π½ΠΎΠ΅ ΠΏΡΠΎΠΈΠ·Π²Π΅Π΄Π΅Π½ΠΈΠ΅ Π²Π΅ΠΊΡΠΎΡΠΎΠ² β ΡΡΠΎ ΠΏΡΠΎΠΈΠ·Π²Π΅Π΄Π΅Π½ΠΈΠ΅ ΡΡΡΡ
Π²Π΅ΠΊΡΠΎΡΠΎΠ² :
ΠΠΎΡ ΡΠ°ΠΊ Π²ΠΎΡ ΠΎΠ½ΠΈ Π²ΡΡΡΡΠΎΠΈΠ»ΠΈΡΡ ΠΏΠ°ΡΠΎΠ²ΠΎΠ·ΠΈΠΊΠΎΠΌ ΠΈ ΠΆΠ΄ΡΡ, Π½Π΅ Π΄ΠΎΠΆΠ΄ΡΡΡΡ, ΠΊΠΎΠ³Π΄Π° ΠΈΡ Π²ΡΡΠΈΡΠ»ΡΡ.
Π‘Π½Π°ΡΠ°Π»Π° ΠΎΠΏΡΡΡ ΠΎΠΏΡΠ΅Π΄Π΅Π»Π΅Π½ΠΈΠ΅ ΠΈ ΠΊΠ°ΡΡΠΈΠ½ΠΊΠ°:
ΠΠΏΡΠ΅Π΄Π΅Π»Π΅Π½ΠΈΠ΅ : Π‘ΠΌΠ΅ΡΠ°Π½Π½ΡΠΌ ΠΏΡΠΎΠΈΠ·Π²Π΅Π΄Π΅Π½ΠΈΠ΅ΠΌ Π½Π΅ΠΊΠΎΠΌΠΏΠ»Π°Π½Π°ΡΠ½ΡΡ Π²Π΅ΠΊΡΠΎΡΠΎΠ² , Π²Π·ΡΡΡΡ Π² Π΄Π°Π½Π½ΠΎΠΌ ΠΏΠΎΡΡΠ΄ΠΊΠ΅ , Π½Π°Π·ΡΠ²Π°Π΅ΡΡΡ ΠΎΠ±ΡΡΠΌ ΠΏΠ°ΡΠ°Π»Π»Π΅Π»Π΅ΠΏΠΈΠΏΠ΅Π΄Π° , ΠΏΠΎΡΡΡΠΎΠ΅Π½Π½ΠΎΠ³ΠΎ Π½Π° Π΄Π°Π½Π½ΡΡ Π²Π΅ΠΊΡΠΎΡΠ°Ρ , ΡΠ½Π°Π±ΠΆΡΠ½Π½ΡΠΉ Π·Π½Π°ΠΊΠΎΠΌ Β«+Β», Π΅ΡΠ»ΠΈ Π±Π°Π·ΠΈΡ ΠΏΡΠ°Π²ΡΠΉ, ΠΈ Π·Π½Π°ΠΊΠΎΠΌ Β«βΒ», Π΅ΡΠ»ΠΈ Π±Π°Π·ΠΈΡ Π»Π΅Π²ΡΠΉ.
ΠΡΠΏΠΎΠ»Π½ΠΈΠΌ ΡΠΈΡΡΠ½ΠΎΠΊ. ΠΠ΅Π²ΠΈΠ΄ΠΈΠΌΡΠ΅ Π½Π°ΠΌ Π»ΠΈΠ½ΠΈΠΈ ΠΏΡΠΎΡΠ΅ΡΡΠ΅Π½Ρ ΠΏΡΠ½ΠΊΡΠΈΡΠΎΠΌ:
ΠΠΎΠ³ΡΡΠΆΠ°Π΅ΠΌΡΡ Π² ΠΎΠΏΡΠ΅Π΄Π΅Π»Π΅Π½ΠΈΠ΅:
2) ΠΠ΅ΠΊΡΠΎΡΡ Π²Π·ΡΡΡ Π² ΠΎΠΏΡΠ΅Π΄Π΅Π»ΡΠ½Π½ΠΎΠΌ ΠΏΠΎΡΡΠ΄ΠΊΠ΅ , ΡΠΎ Π΅ΡΡΡ ΠΏΠ΅ΡΠ΅ΡΡΠ°Π½ΠΎΠ²ΠΊΠ° Π²Π΅ΠΊΡΠΎΡΠΎΠ² Π² ΠΏΡΠΎΠΈΠ·Π²Π΅Π΄Π΅Π½ΠΈΠΈ , ΠΊΠ°ΠΊ Π²Ρ Π΄ΠΎΠ³Π°Π΄ΡΠ²Π°Π΅ΡΠ΅ΡΡ, Π½Π΅ ΠΏΡΠΎΡ ΠΎΠ΄ΠΈΡ Π±Π΅Π· ΠΏΠΎΡΠ»Π΅Π΄ΡΡΠ²ΠΈΠΉ.
3) ΠΠ΅ΡΠ΅Π΄ ΡΠ΅ΠΌ, ΠΊΠ°ΠΊ ΠΏΡΠΎΠΊΠΎΠΌΠΌΠ΅Π½ΡΠΈΡΠΎΠ²Π°ΡΡ Π³Π΅ΠΎΠΌΠ΅ΡΡΠΈΡΠ΅ΡΠΊΠΈΠΉ ΡΠΌΡΡΠ», ΠΎΡΠΌΠ΅ΡΡ ΠΎΡΠ΅Π²ΠΈΠ΄Π½ΡΠΉ ΡΠ°ΠΊΡ: ΡΠΌΠ΅ΡΠ°Π½Π½ΠΎΠ΅ ΠΏΡΠΎΠΈΠ·Π²Π΅Π΄Π΅Π½ΠΈΠ΅ Π²Π΅ΠΊΡΠΎΡΠΎΠ² ΡΠ²Π»ΡΠ΅ΡΡΡ Π§ΠΠ‘ΠΠΠ : . Π ΡΡΠ΅Π±Π½ΠΎΠΉ Π»ΠΈΡΠ΅ΡΠ°ΡΡΡΠ΅ ΠΎΡΠΎΡΠΌΠ»Π΅Π½ΠΈΠ΅ ΠΌΠΎΠΆΠ΅Ρ Π±ΡΡΡ Π½Π΅ΡΠΊΠΎΠ»ΡΠΊΠΎ Π΄ΡΡΠ³ΠΈΠΌ, Ρ ΠΏΡΠΈΠ²ΡΠΊ ΠΎΠ±ΠΎΠ·Π½Π°ΡΠ°ΡΡ ΡΠΌΠ΅ΡΠ°Π½Π½ΠΎΠ΅ ΠΏΡΠΎΠΈΠ·Π²Π΅Π΄Π΅Π½ΠΈΠ΅ ΡΠ΅ΡΠ΅Π· , Π° ΡΠ΅Π·ΡΠ»ΡΡΠ°Ρ Π²ΡΡΠΈΡΠ»Π΅Π½ΠΈΠΉ Π±ΡΠΊΠ²ΠΎΠΉ Β«ΠΏΡΒ».
ΠΠΎ ΠΎΠΏΡΠ΅Π΄Π΅Π»Π΅Π½ΠΈΡ ΡΠΌΠ΅ΡΠ°Π½Π½ΠΎΠ΅ ΠΏΡΠΎΠΈΠ·Π²Π΅Π΄Π΅Π½ΠΈΠ΅ β ΡΡΠΎ ΠΎΠ±ΡΠ΅ΠΌ ΠΏΠ°ΡΠ°Π»Π»Π΅Π»Π΅ΠΏΠΈΠΏΠ΅Π΄Π° , ΠΏΠΎΡΡΡΠΎΠ΅Π½Π½ΠΎΠ³ΠΎ Π½Π° Π²Π΅ΠΊΡΠΎΡΠ°Ρ (ΡΠΈΠ³ΡΡΠ° ΠΏΡΠΎΡΠ΅ΡΡΠ΅Π½Π° ΠΊΡΠ°ΡΠ½ΡΠΌΠΈ Π²Π΅ΠΊΡΠΎΡΠ°ΠΌΠΈ ΠΈ Π»ΠΈΠ½ΠΈΡΠΌΠΈ ΡΡΡΠ½ΠΎΠ³ΠΎ ΡΠ²Π΅ΡΠ°). Π’ΠΎ Π΅ΡΡΡ, ΡΠΈΡΠ»ΠΎ ΡΠ°Π²Π½ΠΎ ΠΎΠ±ΡΠ΅ΠΌΡ Π΄Π°Π½Π½ΠΎΠ³ΠΎ ΠΏΠ°ΡΠ°Π»Π»Π΅Π»Π΅ΠΏΠΈΠΏΠ΅Π΄Π°.
ΠΡΠΈΠΌΠ΅ΡΠ°Π½ΠΈΠ΅ : ΡΠ΅ΡΡΡΠΆ ΡΠ²Π»ΡΠ΅ΡΡΡ ΡΡ Π΅ΠΌΠ°ΡΠΈΡΠ΅ΡΠΊΠΈΠΌ.
4) ΠΠ΅ Π±ΡΠ΄Π΅ΠΌ Π·Π°Π½ΠΎΠ²ΠΎ ΠΏΠ°ΡΠΈΡΡΡΡ Ρ ΠΏΠΎΠ½ΡΡΠΈΠ΅ΠΌ ΠΎΡΠΈΠ΅Π½ΡΠ°ΡΠΈΠΈ Π±Π°Π·ΠΈΡΠ° ΠΈ ΠΏΡΠΎΡΡΡΠ°Π½ΡΡΠ²Π°. Π‘ΠΌΡΡΠ» Π·Π°ΠΊΠ»ΡΡΠΈΡΠ΅Π»ΡΠ½ΠΎΠΉ ΡΠ°ΡΡΠΈ ΡΠΎΡΡΠΎΠΈΡ Π² ΡΠΎΠΌ, ΡΡΠΎ ΠΊ ΠΎΠ±ΡΡΠΌΡ ΠΌΠΎΠΆΠ΅Ρ Π΄ΠΎΠ±Π°Π²Π»ΡΡΡΡΡ Π·Π½Π°ΠΊ ΠΌΠΈΠ½ΡΡ. ΠΡΠΎΡΡΡΠΌΠΈ ΡΠ»ΠΎΠ²Π°ΠΌΠΈ, ΡΠΌΠ΅ΡΠ°Π½Π½ΠΎΠ΅ ΠΏΡΠΎΠΈΠ·Π²Π΅Π΄Π΅Π½ΠΈΠ΅ ΠΌΠΎΠΆΠ΅Ρ Π±ΡΡΡ ΠΎΡΡΠΈΡΠ°ΡΠ΅Π»ΡΠ½ΡΠΌ: .
ΠΠ΅ΠΏΠΎΡΡΠ΅Π΄ΡΡΠ²Π΅Π½Π½ΠΎ ΠΈΠ· ΠΎΠΏΡΠ΅Π΄Π΅Π»Π΅Π½ΠΈΡ ΡΠ»Π΅Π΄ΡΠ΅Ρ ΡΠΎΡΠΌΡΠ»Π° Π²ΡΡΠΈΡΠ»Π΅Π½ΠΈΡ ΠΎΠ±ΡΠ΅ΠΌΠ° ΠΏΠ°ΡΠ°Π»Π»Π΅Π»Π΅ΠΏΠΈΠΏΠ΅Π΄Π°, ΠΏΠΎΡΡΡΠΎΠ΅Π½Π½ΠΎΠ³ΠΎ Π½Π° Π²Π΅ΠΊΡΠΎΡΠ°Ρ .
Π ΡΡΠΎΠΉ ΡΡΠ°ΡΡΠ΅ ΠΌΡ ΠΏΠΎΠ΄ΡΠΎΠ±Π½ΠΎ ΠΎΡΡΠ°Π½ΠΎΠ²ΠΈΠΌΡΡ Π½Π° ΠΏΠΎΠ½ΡΡΠΈΠΈ Π²Π΅ΠΊΡΠΎΡΠ½ΠΎΠ³ΠΎ ΠΏΡΠΎΠΈΠ·Π²Π΅Π΄Π΅Π½ΠΈΡ Π΄Π²ΡΡ
Π²Π΅ΠΊΡΠΎΡΠΎΠ². ΠΡ Π΄Π°Π΄ΠΈΠΌ Π½Π΅ΠΎΠ±Ρ
ΠΎΠ΄ΠΈΠΌΡΠ΅ ΠΎΠΏΡΠ΅Π΄Π΅Π»Π΅Π½ΠΈΡ, Π·Π°ΠΏΠΈΡΠ΅ΠΌ ΡΠΎΡΠΌΡΠ»Ρ Π΄Π»Ρ Π½Π°Ρ
ΠΎΠΆΠ΄Π΅Π½ΠΈΡ ΠΊΠΎΠΎΡΠ΄ΠΈΠ½Π°Ρ Π²Π΅ΠΊΡΠΎΡΠ½ΠΎΠ³ΠΎ ΠΏΡΠΎΠΈΠ·Π²Π΅Π΄Π΅Π½ΠΈΡ, ΠΏΠ΅ΡΠ΅ΡΠΈΡΠ»ΠΈΠΌ ΠΈ ΠΎΠ±ΠΎΡΠ½ΡΠ΅ΠΌ Π΅Π³ΠΎ ΡΠ²ΠΎΠΉΡΡΠ²Π°. ΠΠΎΡΠ»Π΅ ΡΡΠΎΠ³ΠΎ ΠΎΡΡΠ°Π½ΠΎΠ²ΠΈΠΌΡΡ Π½Π° Π³Π΅ΠΎΠΌΠ΅ΡΡΠΈΡΠ΅ΡΠΊΠΎΠΌ ΡΠΌΡΡΠ»Π΅ Π²Π΅ΠΊΡΠΎΡΠ½ΠΎΠ³ΠΎ ΠΏΡΠΎΠΈΠ·Π²Π΅Π΄Π΅Π½ΠΈΡ Π΄Π²ΡΡ
Π²Π΅ΠΊΡΠΎΡΠΎΠ² ΠΈ ΡΠ°ΡΡΠΌΠΎΡΡΠΈΠΌ ΡΠ΅ΡΠ΅Π½ΠΈΡ ΡΠ°Π·Π»ΠΈΡΠ½ΡΡ
Ρ
Π°ΡΠ°ΠΊΡΠ΅ΡΠ½ΡΡ
ΠΏΡΠΈΠΌΠ΅ΡΠΎΠ².
ΠΠ°Π²ΠΈΠ³Π°ΡΠΈΡ ΠΏΠΎ ΡΡΡΠ°Π½ΠΈΡΠ΅.
ΠΠΏΡΠ΅Π΄Π΅Π»Π΅Π½ΠΈΠ΅ Π²Π΅ΠΊΡΠΎΡΠ½ΠΎΠ³ΠΎ ΠΏΡΠΎΠΈΠ·Π²Π΅Π΄Π΅Π½ΠΈΡ.
ΠΡΠ΅ΠΆΠ΄Π΅ ΡΠ΅ΠΌ Π΄Π°ΡΡ ΠΎΠΏΡΠ΅Π΄Π΅Π»Π΅Π½ΠΈΠ΅ Π²Π΅ΠΊΡΠΎΡΠ½ΠΎΠ³ΠΎ ΠΏΡΠΎΠΈΠ·Π²Π΅Π΄Π΅Π½ΠΈΡ, ΡΠ°Π·Π±Π΅ΡΠ΅ΠΌΡΡ Ρ ΠΎΡΠΈΠ΅Π½ΡΠ°ΡΠΈΠ΅ΠΉ ΡΠΏΠΎΡΡΠ΄ΠΎΡΠ΅Π½Π½ΠΎΠΉ ΡΡΠΎΠΉΠΊΠΈ Π²Π΅ΠΊΡΠΎΡΠΎΠ² Π² ΡΡΠ΅Ρ ΠΌΠ΅ΡΠ½ΠΎΠΌ ΠΏΡΠΎΡΡΡΠ°Π½ΡΡΠ²Π΅.
ΠΡΠ»ΠΎΠΆΠΈΠΌ Π²Π΅ΠΊΡΠΎΡΡ ΠΎΡ ΠΎΠ΄Π½ΠΎΠΉ ΡΠΎΡΠΊΠΈ. Π Π·Π°Π²ΠΈΡΠΈΠΌΠΎΡΡΠΈ ΠΎΡ Π½Π°ΠΏΡΠ°Π²Π»Π΅Π½ΠΈΡ Π²Π΅ΠΊΡΠΎΡΠ° ΡΡΠΎΠΉΠΊΠ° ΠΌΠΎΠΆΠ΅Ρ Π±ΡΡΡ ΠΏΡΠ°Π²ΠΎΠΉ ΠΈΠ»ΠΈ Π»Π΅Π²ΠΎΠΉ. ΠΠΎΡΠΌΠΎΡΡΠΈΠΌ Ρ ΠΊΠΎΠ½ΡΠ° Π²Π΅ΠΊΡΠΎΡΠ° Π½Π° ΡΠΎ, ΠΊΠ°ΠΊ ΠΏΡΠΎΠΈΡΡ ΠΎΠ΄ΠΈΡ ΠΊΡΠ°ΡΡΠ°ΠΉΡΠΈΠΉ ΠΏΠΎΠ²ΠΎΡΠΎΡ ΠΎΡ Π²Π΅ΠΊΡΠΎΡΠ° ΠΊ . ΠΡΠ»ΠΈ ΠΊΡΠ°ΡΡΠ°ΠΉΡΠΈΠΉ ΠΏΠΎΠ²ΠΎΡΠΎΡ ΠΏΡΠΎΠΈΡΡ ΠΎΠ΄ΠΈΡ ΠΏΡΠΎΡΠΈΠ² ΡΠ°ΡΠΎΠ²ΠΎΠΉ ΡΡΡΠ΅Π»ΠΊΠΈ, ΡΠΎ ΡΡΠΎΠΉΠΊΠ° Π²Π΅ΠΊΡΠΎΡΠΎΠ² Π½Π°Π·ΡΠ²Π°Π΅ΡΡΡ ΠΏΡΠ°Π²ΠΎΠΉ , Π² ΠΏΡΠΎΡΠΈΠ²Π½ΠΎΠΌ ΡΠ»ΡΡΠ°Π΅ β Π»Π΅Π²ΠΎΠΉ .
Π’Π΅ΠΏΠ΅ΡΡ Π²ΠΎΠ·ΡΠΌΠ΅ΠΌ Π΄Π²Π° Π½Π΅ ΠΊΠΎΠ»Π»ΠΈΠ½Π΅Π°ΡΠ½ΡΡ Π²Π΅ΠΊΡΠΎΡΠ° ΠΈ . ΠΡΠ»ΠΎΠΆΠΈΠΌ ΠΎΡ ΡΠΎΡΠΊΠΈ Π Π²Π΅ΠΊΡΠΎΡΡ ΠΈ . ΠΠΎΡΡΡΠΎΠΈΠΌ Π½Π΅ΠΊΠΎΡΠΎΡΡΠΉ Π²Π΅ΠΊΡΠΎΡ , ΠΏΠ΅ΡΠΏΠ΅Π½Π΄ΠΈΠΊΡΠ»ΡΡΠ½ΡΠΉ ΠΎΠ΄Π½ΠΎΠ²ΡΠ΅ΠΌΠ΅Π½Π½ΠΎ ΠΈ ΠΈ . ΠΡΠ΅Π²ΠΈΠ΄Π½ΠΎ, ΡΡΠΎ ΠΏΡΠΈ ΠΏΠΎΡΡΡΠΎΠ΅Π½ΠΈΠΈ Π²Π΅ΠΊΡΠΎΡΠ° ΠΌΡ ΠΌΠΎΠΆΠ΅ΠΌ ΠΏΠΎΡΡΡΠΏΠΈΡΡ Π΄Π²ΠΎΡΠΊΠΎ, Π·Π°Π΄Π°Π² Π΅ΠΌΡ Π»ΠΈΠ±ΠΎ ΠΎΠ΄Π½ΠΎ Π½Π°ΠΏΡΠ°Π²Π»Π΅Π½ΠΈΠ΅, Π»ΠΈΠ±ΠΎ ΠΏΡΠΎΡΠΈΠ²ΠΎΠΏΠΎΠ»ΠΎΠΆΠ½ΠΎΠ΅ (ΡΠΌΠΎΡΡΠΈΡΠ΅ ΠΈΠ»Π»ΡΡΡΡΠ°ΡΠΈΡ).
Π Π·Π°Π²ΠΈΡΠΈΠΌΠΎΡΡΠΈ ΠΎΡ Π½Π°ΠΏΡΠ°Π²Π»Π΅Π½ΠΈΡ Π²Π΅ΠΊΡΠΎΡΠ° ΡΠΏΠΎΡΡΠ΄ΠΎΡΠ΅Π½Π½Π°Ρ ΡΡΠΎΠΉΠΊΠ° Π²Π΅ΠΊΡΠΎΡΠΎΠ² ΠΌΠΎΠΆΠ΅Ρ Π±ΡΡΡ ΠΏΡΠ°Π²ΠΎΠΉ ΠΈΠ»ΠΈ Π»Π΅Π²ΠΎΠΉ.
Π’Π°ΠΊ ΠΌΡ Π²ΠΏΠ»ΠΎΡΠ½ΡΡ ΠΏΠΎΠ΄ΠΎΡΠ»ΠΈ ΠΊ ΠΎΠΏΡΠ΅Π΄Π΅Π»Π΅Π½ΠΈΡ Π²Π΅ΠΊΡΠΎΡΠ½ΠΎΠ³ΠΎ ΠΏΡΠΎΠΈΠ·Π²Π΅Π΄Π΅Π½ΠΈΡ. ΠΠ½ΠΎ Π΄Π°Π΅ΡΡΡ Π΄Π»Ρ Π΄Π²ΡΡ Π²Π΅ΠΊΡΠΎΡΠΎΠ², Π·Π°Π΄Π°Π½Π½ΡΡ Π² ΠΏΡΡΠΌΠΎΡΠ³ΠΎΠ»ΡΠ½ΠΎΠΉ ΡΠΈΡΡΠ΅ΠΌΠ΅ ΠΊΠΎΠΎΡΠ΄ΠΈΠ½Π°Ρ ΡΡΠ΅Ρ ΠΌΠ΅ΡΠ½ΠΎΠ³ΠΎ ΠΏΡΠΎΡΡΡΠ°Π½ΡΡΠ²Π°.
ΠΠΏΡΠ΅Π΄Π΅Π»Π΅Π½ΠΈΠ΅.
ΠΠ΅ΠΊΡΠΎΡΠ½ΡΠΌ ΠΏΡΠΎΠΈΠ·Π²Π΅Π΄Π΅Π½ΠΈΠ΅ΠΌ Π΄Π²ΡΡ Π²Π΅ΠΊΡΠΎΡΠΎΠ² ΠΈ , Π·Π°Π΄Π°Π½Π½ΡΡ Π² ΠΏΡΡΠΌΠΎΡΠ³ΠΎΠ»ΡΠ½ΠΎΠΉ ΡΠΈΡΡΠ΅ΠΌΠ΅ ΠΊΠΎΠΎΡΠ΄ΠΈΠ½Π°Ρ ΡΡΠ΅Ρ ΠΌΠ΅ΡΠ½ΠΎΠ³ΠΎ ΠΏΡΠΎΡΡΡΠ°Π½ΡΡΠ²Π°, Π½Π°Π·ΡΠ²Π°Π΅ΡΡΡ ΡΠ°ΠΊΠΎΠΉ Π²Π΅ΠΊΡΠΎΡ , ΡΡΠΎ
ΠΠ΅ΠΊΡΠΎΡΠ½ΠΎΠ΅ ΠΏΡΠΎΠΈΠ·Π²Π΅Π΄Π΅Π½ΠΈΠ΅ Π²Π΅ΠΊΡΠΎΡΠΎΠ² ΠΈ ΠΎΠ±ΠΎΠ·Π½Π°ΡΠ°Π΅ΡΡΡ ΠΊΠ°ΠΊ .
ΠΠΎΠΎΡΠ΄ΠΈΠ½Π°ΡΡ Π²Π΅ΠΊΡΠΎΡΠ½ΠΎΠ³ΠΎ ΠΏΡΠΎΠΈΠ·Π²Π΅Π΄Π΅Π½ΠΈΡ.
Π‘Π΅ΠΉΡΠ°Ρ Π΄Π°Π΄ΠΈΠΌ Π²ΡΠΎΡΠΎΠ΅ ΠΎΠΏΡΠ΅Π΄Π΅Π»Π΅Π½ΠΈΠ΅ Π²Π΅ΠΊΡΠΎΡΠ½ΠΎΠ³ΠΎ ΠΏΡΠΎΠΈΠ·Π²Π΅Π΄Π΅Π½ΠΈΡ, ΠΊΠΎΡΠΎΡΠΎΠ΅ ΠΏΠΎΠ·Π²ΠΎΠ»ΡΠ΅Ρ Π½Π°Ρ ΠΎΠ΄ΠΈΡΡ Π΅Π³ΠΎ ΠΊΠΎΠΎΡΠ΄ΠΈΠ½Π°ΡΡ ΠΏΠΎ ΠΊΠΎΠΎΡΠ΄ΠΈΠ½Π°ΡΠ°ΠΌ Π·Π°Π΄Π°Π½Π½ΡΡ Π²Π΅ΠΊΡΠΎΡΠΎΠ² ΠΈ.
ΠΠΏΡΠ΅Π΄Π΅Π»Π΅Π½ΠΈΠ΅.
Π ΠΏΡΡΠΌΠΎΡΠ³ΠΎΠ»ΡΠ½ΠΎΠΉ ΡΠΈΡΡΠ΅ΠΌΠ΅ ΠΊΠΎΠΎΡΠ΄ΠΈΠ½Π°Ρ ΡΡΠ΅Ρ ΠΌΠ΅ΡΠ½ΠΎΠ³ΠΎ ΠΏΡΠΎΡΡΡΠ°Π½ΡΡΠ²Π° Π²Π΅ΠΊΡΠΎΡΠ½ΠΎΠ΅ ΠΏΡΠΎΠΈΠ·Π²Π΅Π΄Π΅Π½ΠΈΠ΅ Π΄Π²ΡΡ Π²Π΅ΠΊΡΠΎΡΠΎΠ² ΠΈ Π΅ΡΡΡ Π²Π΅ΠΊΡΠΎΡ , Π³Π΄Π΅ β ΠΊΠΎΠΎΡΠ΄ΠΈΠ½Π°ΡΠ½ΡΠ΅ Π²Π΅ΠΊΡΠΎΡΡ.
ΠΡΠΎ ΠΎΠΏΡΠ΅Π΄Π΅Π»Π΅Π½ΠΈΠ΅ Π΄Π°Π΅Ρ Π½Π°ΠΌ Π²Π΅ΠΊΡΠΎΡΠ½ΠΎΠ΅ ΠΏΡΠΎΠΈΠ·Π²Π΅Π΄Π΅Π½ΠΈΠ΅ Π² ΠΊΠΎΠΎΡΠ΄ΠΈΠ½Π°ΡΠ½ΠΎΠΉ ΡΠΎΡΠΌΠ΅.
ΠΠ΅ΠΊΡΠΎΡΠ½ΠΎΠ΅ ΠΏΡΠΎΠΈΠ·Π²Π΅Π΄Π΅Π½ΠΈΠ΅ ΡΠ΄ΠΎΠ±Π½ΠΎ ΠΏΡΠ΅Π΄ΡΡΠ°Π²Π»ΡΡΡ Π² Π²ΠΈΠ΄Π΅ ΠΎΠΏΡΠ΅Π΄Π΅Π»ΠΈΡΠ΅Π»Ρ ΠΊΠ²Π°Π΄ΡΠ°ΡΠ½ΠΎΠΉ ΠΌΠ°ΡΡΠΈΡΡ ΡΡΠ΅ΡΡΠ΅Π³ΠΎ ΠΏΠΎΡΡΠ΄ΠΊΠ°, ΠΏΠ΅ΡΠ²Π°Ρ ΡΡΡΠΎΠΊΠ° ΠΊΠΎΡΠΎΡΠΎΠΉ Π΅ΡΡΡ ΠΎΡΡΡ , Π²ΠΎ Π²ΡΠΎΡΠΎΠΉ ΡΡΡΠΎΠΊΠ΅ Π½Π°Ρ
ΠΎΠ΄ΡΡΡΡ ΠΊΠΎΠΎΡΠ΄ΠΈΠ½Π°ΡΡ Π²Π΅ΠΊΡΠΎΡΠ° , Π° Π² ΡΡΠ΅ΡΡΠ΅ΠΉ β ΠΊΠΎΠΎΡΠ΄ΠΈΠ½Π°ΡΡ Π²Π΅ΠΊΡΠΎΡΠ° Π² Π·Π°Π΄Π°Π½Π½ΠΎΠΉ ΠΏΡΡΠΌΠΎΡΠ³ΠΎΠ»ΡΠ½ΠΎΠΉ ΡΠΈΡΡΠ΅ΠΌΠ΅ ΠΊΠΎΠΎΡΠ΄ΠΈΠ½Π°Ρ:
ΠΡΠ»ΠΈ ΡΠ°Π·Π»ΠΎΠΆΠΈΡΡ ΡΡΠΎΡ ΠΎΠΏΡΠ΅Π΄Π΅Π»ΠΈΡΠ΅Π»Ρ ΠΏΠΎ ΡΠ»Π΅ΠΌΠ΅Π½ΡΠ°ΠΌ ΠΏΠ΅ΡΠ²ΠΎΠΉ ΡΡΡΠΎΠΊΠΈ, ΡΠΎ ΠΏΠΎΠ»ΡΡΠΈΠΌ ΡΠ°Π²Π΅Π½ΡΡΠ²ΠΎ ΠΈΠ· ΠΎΠΏΡΠ΅Π΄Π΅Π»Π΅Π½ΠΈΡ Π²Π΅ΠΊΡΠΎΡΠ½ΠΎΠ³ΠΎ ΠΏΡΠΎΠΈΠ·Π²Π΅Π΄Π΅Π½ΠΈΡ Π² ΠΊΠΎΠΎΡΠ΄ΠΈΠ½Π°ΡΠ°Ρ (ΠΏΡΠΈ Π½Π΅ΠΎΠ±Ρ ΠΎΠ΄ΠΈΠΌΠΎΡΡΠΈ ΠΎΠ±ΡΠ°ΡΠ°ΠΉΡΠ΅ΡΡ ΠΊ ΡΡΠ°ΡΡΠ΅ ):
Π‘Π»Π΅Π΄ΡΠ΅Ρ ΠΎΡΠΌΠ΅ΡΠΈΡΡ, ΡΡΠΎ ΠΊΠΎΠΎΡΠ΄ΠΈΠ½Π°ΡΠ½Π°Ρ ΡΠΎΡΠΌΠ° Π²Π΅ΠΊΡΠΎΡΠ½ΠΎΠ³ΠΎ ΠΏΡΠΎΠΈΠ·Π²Π΅Π΄Π΅Π½ΠΈΡ ΠΏΠΎΠ»Π½ΠΎΡΡΡΡ ΡΠΎΠ³Π»Π°ΡΡΠ΅ΡΡΡ Ρ ΠΎΠΏΡΠ΅Π΄Π΅Π»Π΅Π½ΠΈΠ΅ΠΌ, Π΄Π°Π½Π½ΡΠΌ Π² ΠΏΠ΅ΡΠ²ΠΎΠΌ ΠΏΡΠ½ΠΊΡΠ΅ ΡΡΠΎΠΉ ΡΡΠ°ΡΡΠΈ. ΠΠΎΠ»Π΅Π΅ ΡΠΎΠ³ΠΎ, ΡΡΠΈ Π΄Π²Π° ΠΎΠΏΡΠ΅Π΄Π΅Π»Π΅Π½ΠΈΡ Π²Π΅ΠΊΡΠΎΡΠ½ΠΎΠ³ΠΎ ΠΏΡΠΎΠΈΠ·Π²Π΅Π΄Π΅Π½ΠΈΡ ΡΠΊΠ²ΠΈΠ²Π°Π»Π΅Π½ΡΠ½Ρ. ΠΠΎΠΊΠ°Π·Π°ΡΠ΅Π»ΡΡΡΠ²ΠΎ ΡΡΠΎΠ³ΠΎ ΡΠ°ΠΊΡΠ° ΠΌΠΎΠΆΠ΅ΡΠ΅ ΠΏΠΎΡΠΌΠΎΡΡΠ΅ΡΡ Π² ΠΊΠ½ΠΈΠ³Π΅, ΡΠΊΠ°Π·Π°Π½Π½ΠΎΠΉ Π² ΠΊΠΎΠ½ΡΠ΅ ΡΡΠ°ΡΡΠΈ.
Π‘Π²ΠΎΠΉΡΡΠ²Π° Π²Π΅ΠΊΡΠΎΡΠ½ΠΎΠ³ΠΎ ΠΏΡΠΎΠΈΠ·Π²Π΅Π΄Π΅Π½ΠΈΡ.
Π’Π°ΠΊ ΠΊΠ°ΠΊ Π²Π΅ΠΊΡΠΎΡΠ½ΠΎΠ΅ ΠΏΡΠΎΠΈΠ·Π²Π΅Π΄Π΅Π½ΠΈΠ΅ Π² ΠΊΠΎΠΎΡΠ΄ΠΈΠ½Π°ΡΠ°Ρ ΠΏΡΠ΅Π΄ΡΡΠ°Π²ΠΈΠΌΠΎ Π² Π²ΠΈΠ΄Π΅ ΠΎΠΏΡΠ΅Π΄Π΅Π»ΠΈΡΠ΅Π»Ρ ΠΌΠ°ΡΡΠΈΡΡ , ΡΠΎ Π½Π° ΠΎΡΠ½ΠΎΠ²Π°Π½ΠΈΠΈ Π»Π΅Π³ΠΊΠΎ ΠΎΠ±ΠΎΡΠ½ΠΎΠ²ΡΠ²Π°ΡΡΡΡ ΡΠ»Π΅Π΄ΡΡΡΠΈΠ΅ ΡΠ²ΠΎΠΉΡΡΠ²Π° Π²Π΅ΠΊΡΠΎΡΠ½ΠΎΠ³ΠΎ ΠΏΡΠΎΠΈΠ·Π²Π΅Π΄Π΅Π½ΠΈΡ :
ΠΠ»Ρ ΠΏΡΠΈΠΌΠ΅ΡΠ° Π΄ΠΎΠΊΠ°ΠΆΠ΅ΠΌ ΡΠ²ΠΎΠΉΡΡΠ²ΠΎ Π°Π½ΡΠΈΠΊΠΎΠΌΠΌΡΡΠ°ΡΠΈΠ²Π½ΠΎΡΡΠΈ Π²Π΅ΠΊΡΠΎΡΠ½ΠΎΠ³ΠΎ ΠΏΡΠΎΠΈΠ·Π²Π΅Π΄Π΅Π½ΠΈΡ.
ΠΠΎ ΠΎΠΏΡΠ΅Π΄Π΅Π»Π΅Π½ΠΈΡ ΠΈ . ΠΠ°ΠΌ ΠΈΠ·Π²Π΅ΡΡΠ½ΠΎ, ΡΡΠΎ Π·Π½Π°ΡΠ΅Π½ΠΈΠ΅ ΠΎΠΏΡΠ΅Π΄Π΅Π»ΠΈΡΠ΅Π»Ρ ΠΌΠ°ΡΡΠΈΡΡ ΠΈΠ·ΠΌΠ΅Π½ΡΠ΅ΡΡΡ Π½Π° ΠΏΡΠΎΡΠΈΠ²ΠΎΠΏΠΎΠ»ΠΎΠΆΠ½ΠΎΠ΅, Π΅ΡΠ»ΠΈ ΠΏΠ΅ΡΠ΅ΡΡΠ°Π²ΠΈΡΡ ΠΌΠ΅ΡΡΠ°ΠΌΠΈ Π΄Π²Π΅ ΡΡΡΠΎΠΊΠΈ, ΠΏΠΎΡΡΠΎΠΌΡ, , ΡΡΠΎ Π΄ΠΎΠΊΠ°Π·ΡΠ²Π°Π΅Ρ ΡΠ²ΠΎΠΉΡΡΠ²ΠΎ Π°Π½ΡΠΈΠΊΠΎΠΌΠΌΡΡΠ°ΡΠΈΠ²Π½ΠΎΡΡΠΈ Π²Π΅ΠΊΡΠΎΡΠ½ΠΎΠ³ΠΎ ΠΏΡΠΎΠΈΠ·Π²Π΅Π΄Π΅Π½ΠΈΡ.
ΠΠ΅ΠΊΡΠΎΡΠ½ΠΎΠ΅ ΠΏΡΠΎΠΈΠ·Π²Π΅Π΄Π΅Π½ΠΈΠ΅ β ΠΏΡΠΈΠΌΠ΅ΡΡ ΠΈ ΡΠ΅ΡΠ΅Π½ΠΈΡ.
Π ΠΎΡΠ½ΠΎΠ²Π½ΠΎΠΌ Π²ΡΡΡΠ΅ΡΠ°ΡΡΡΡ ΡΡΠΈ ΡΠΈΠΏΠ° Π·Π°Π΄Π°Ρ.
Π Π·Π°Π΄Π°ΡΠ°Ρ ΠΏΠ΅ΡΠ²ΠΎΠ³ΠΎ ΡΠΈΠΏΠ° Π·Π°Π΄Π°Π½Ρ Π΄Π»ΠΈΠ½Ρ Π΄Π²ΡΡ Π²Π΅ΠΊΡΠΎΡΠΎΠ² ΠΈ ΡΠ³ΠΎΠ» ΠΌΠ΅ΠΆΠ΄Ρ Π½ΠΈΠΌΠΈ, Π° ΡΡΠ΅Π±ΡΠ΅ΡΡΡ Π½Π°ΠΉΡΠΈ Π΄Π»ΠΈΠ½Ρ Π²Π΅ΠΊΡΠΎΡΠ½ΠΎΠ³ΠΎ ΠΏΡΠΎΠΈΠ·Π²Π΅Π΄Π΅Π½ΠΈΡ. Π ΡΡΠΎΠΌ ΡΠ»ΡΡΠ°Π΅ ΠΈΡΠΏΠΎΠ»ΡΠ·ΡΠ΅ΡΡΡ ΡΠΎΡΠΌΡΠ»Π° .
ΠΡΠΈΠΌΠ΅Ρ.
ΠΠ°ΠΉΠ΄ΠΈΡΠ΅ Π΄Π»ΠΈΠ½Ρ Π²Π΅ΠΊΡΠΎΡΠ½ΠΎΠ³ΠΎ ΠΏΡΠΎΠΈΠ·Π²Π΅Π΄Π΅Π½ΠΈΡ Π²Π΅ΠΊΡΠΎΡΠΎΠ² ΠΈ , Π΅ΡΠ»ΠΈ ΠΈΠ·Π²Π΅ΡΡΠ½ΠΎ .
Π Π΅ΡΠ΅Π½ΠΈΠ΅.
ΠΡ Π·Π½Π°Π΅ΠΌ ΠΈΠ· ΠΎΠΏΡΠ΅Π΄Π΅Π»Π΅Π½ΠΈΡ, ΡΡΠΎ Π΄Π»ΠΈΠ½Π° Π²Π΅ΠΊΡΠΎΡΠ½ΠΎΠ³ΠΎ ΠΏΡΠΎΠΈΠ·Π²Π΅Π΄Π΅Π½ΠΈΡ Π²Π΅ΠΊΡΠΎΡΠΎΠ² ΠΈ ΡΠ°Π²Π½Π° ΠΏΡΠΎΠΈΠ·Π²Π΅Π΄Π΅Π½ΠΈΡ Π΄Π»ΠΈΠ½ Π²Π΅ΠΊΡΠΎΡΠΎΠ² ΠΈ Π½Π° ΡΠΈΠ½ΡΡ ΡΠ³Π»Π° ΠΌΠ΅ΠΆΠ΄Ρ Π½ΠΈΠΌΠΈ, ΠΏΠΎΡΡΠΎΠΌΡ, .
ΠΡΠ²Π΅Ρ:
.
ΠΠ°Π΄Π°ΡΠΈ Π²ΡΠΎΡΠΎΠ³ΠΎ ΡΠΈΠΏΠ° ΡΠ²ΡΠ·Π°Π½Ρ Ρ ΠΊΠΎΠΎΡΠ΄ΠΈΠ½Π°ΡΠ°ΠΌΠΈ Π²Π΅ΠΊΡΠΎΡΠΎΠ², Π² Π½ΠΈΡ Π²Π΅ΠΊΡΠΎΡΠ½ΠΎΠ΅ ΠΏΡΠΎΠΈΠ·Π²Π΅Π΄Π΅Π½ΠΈΠ΅, Π΅Π³ΠΎ Π΄Π»ΠΈΠ½Π° ΠΈΠ»ΠΈ ΡΡΠΎ-Π»ΠΈΠ±ΠΎ Π΅ΡΠ΅ ΠΈΡΠ΅ΡΡΡ ΡΠ΅ΡΠ΅Π· ΠΊΠΎΠΎΡΠ΄ΠΈΠ½Π°ΡΡ Π·Π°Π΄Π°Π½Π½ΡΡ Π²Π΅ΠΊΡΠΎΡΠΎΠ² ΠΈ .
ΠΠ΄Π΅ΡΡ Π²ΠΎΠ·ΠΌΠΎΠΆΠ½Π° ΠΌΠ°ΡΡΠ° ΡΠ°Π·Π»ΠΈΡΠ½ΡΡ Π²Π°ΡΠΈΠ°Π½ΡΠΎΠ². Π ΠΏΡΠΈΠΌΠ΅ΡΡ, ΠΌΠΎΠ³ΡΡ Π±ΡΡΡ Π·Π°Π΄Π°Π½Ρ Π½Π΅ ΠΊΠΎΠΎΡΠ΄ΠΈΠ½Π°ΡΡ Π²Π΅ΠΊΡΠΎΡΠΎΠ² ΠΈ , Π° ΠΈΡ ΡΠ°Π·Π»ΠΎΠΆΠ΅Π½ΠΈΡ ΠΏΠΎ ΠΊΠΎΠΎΡΠ΄ΠΈΠ½Π°ΡΠ½ΡΠΌ Π²Π΅ΠΊΡΠΎΡΠ°ΠΌ Π²ΠΈΠ΄Π° ΠΈ , ΠΈΠ»ΠΈ Π²Π΅ΠΊΡΠΎΡΡ ΠΈ ΠΌΠΎΠ³ΡΡ Π±ΡΡΡ Π·Π°Π΄Π°Π½Ρ ΠΊΠΎΠΎΡΠ΄ΠΈΠ½Π°ΡΠ°ΠΌΠΈ ΡΠΎΡΠ΅ΠΊ ΠΈΡ Π½Π°ΡΠ°Π»Π° ΠΈ ΠΊΠΎΠ½ΡΠ°.
Π Π°ΡΡΠΌΠΎΡΡΠΈΠΌ Ρ Π°ΡΠ°ΠΊΡΠ΅ΡΠ½ΡΠ΅ ΠΏΡΠΈΠΌΠ΅ΡΡ.
ΠΡΠΈΠΌΠ΅Ρ.
Π ΠΏΡΡΠΌΠΎΡΠ³ΠΎΠ»ΡΠ½ΠΎΠΉ ΡΠΈΡΡΠ΅ΠΌΠ΅ ΠΊΠΎΠΎΡΠ΄ΠΈΠ½Π°Ρ Π·Π°Π΄Π°Π½Ρ Π΄Π²Π° Π²Π΅ΠΊΡΠΎΡΠ° . ΠΠ°ΠΉΠ΄ΠΈΡΠ΅ ΠΈΡ Π²Π΅ΠΊΡΠΎΡΠ½ΠΎΠ΅ ΠΏΡΠΎΠΈΠ·Π²Π΅Π΄Π΅Π½ΠΈΠ΅.
Π Π΅ΡΠ΅Π½ΠΈΠ΅.
ΠΠΎ Π²ΡΠΎΡΠΎΠΌΡ ΠΎΠΏΡΠ΅Π΄Π΅Π»Π΅Π½ΠΈΡ Π²Π΅ΠΊΡΠΎΡΠ½ΠΎΠ΅ ΠΏΡΠΎΠΈΠ·Π²Π΅Π΄Π΅Π½ΠΈΠ΅ Π΄Π²ΡΡ Π²Π΅ΠΊΡΠΎΡΠΎΠ² Π² ΠΊΠΎΠΎΡΠ΄ΠΈΠ½Π°ΡΠ°Ρ Π·Π°ΠΏΠΈΡΡΠ²Π°Π΅ΡΡΡ ΠΊΠ°ΠΊ:
Π ΡΠ°ΠΊΠΎΠΌΡ ΠΆΠ΅ ΡΠ΅Π·ΡΠ»ΡΡΠ°ΡΡ ΠΌΡ Π±Ρ ΠΏΡΠΈΡΠ»ΠΈ, Π΅ΡΠ»ΠΈ Π±Ρ Π²Π΅ΠΊΡΠΎΡΠ½ΠΎΠ΅ ΠΏΡΠΎΠΈΠ·Π²Π΅Π΄Π΅Π½ΠΈΠ΅ Π·Π°ΠΏΠΈΡΠ°Π»ΠΈ ΡΠ΅ΡΠ΅Π· ΠΎΠΏΡΠ΅Π΄Π΅Π»ΠΈΡΠ΅Π»Ρ
ΠΡΠ²Π΅Ρ:
.
ΠΡΠΈΠΌΠ΅Ρ.
ΠΠ°ΠΉΠ΄ΠΈΡΠ΅ Π΄Π»ΠΈΠ½Ρ Π²Π΅ΠΊΡΠΎΡΠ½ΠΎΠ³ΠΎ ΠΏΡΠΎΠΈΠ·Π²Π΅Π΄Π΅Π½ΠΈΡ Π²Π΅ΠΊΡΠΎΡΠΎΠ² ΠΈ , Π³Π΄Π΅ β ΠΎΡΡΡ ΠΏΡΡΠΌΠΎΡΠ³ΠΎΠ»ΡΠ½ΠΎΠΉ Π΄Π΅ΠΊΠ°ΡΡΠΎΠ²ΠΎΠΉ ΡΠΈΡΡΠ΅ΠΌΡ ΠΊΠΎΠΎΡΠ΄ΠΈΠ½Π°Ρ.
Π Π΅ΡΠ΅Π½ΠΈΠ΅.
Π‘Π½Π°ΡΠ°Π»Π° Π½Π°ΠΉΠ΄Π΅ΠΌ ΠΊΠΎΠΎΡΠ΄ΠΈΠ½Π°ΡΡ Π²Π΅ΠΊΡΠΎΡΠ½ΠΎΠ³ΠΎ ΠΏΡΠΎΠΈΠ·Π²Π΅Π΄Π΅Π½ΠΈΡ Π² Π·Π°Π΄Π°Π½Π½ΠΎΠΉ ΠΏΡΡΠΌΠΎΡΠ³ΠΎΠ»ΡΠ½ΠΎΠΉ ΡΠΈΡΡΠ΅ΠΌΠ΅ ΠΊΠΎΠΎΡΠ΄ΠΈΠ½Π°Ρ.
Π’Π°ΠΊ ΠΊΠ°ΠΊ Π²Π΅ΠΊΡΠΎΡΡ ΠΈ ΠΈΠΌΠ΅ΡΡ ΠΊΠΎΠΎΡΠ΄ΠΈΠ½Π°ΡΡ ΠΈ ΡΠΎΠΎΡΠ²Π΅ΡΡΡΠ²Π΅Π½Π½ΠΎ (ΠΏΡΠΈ Π½Π΅ΠΎΠ±Ρ ΠΎΠ΄ΠΈΠΌΠΎΡΡΠΈ ΡΠΌΠΎΡΡΠΈΡΠ΅ ΡΡΠ°ΡΡΡ ΠΊΠΎΠΎΡΠ΄ΠΈΠ½Π°ΡΡ Π²Π΅ΠΊΡΠΎΡΠ° Π² ΠΏΡΡΠΌΠΎΡΠ³ΠΎΠ»ΡΠ½ΠΎΠΉ ΡΠΈΡΡΠ΅ΠΌΠ΅ ΠΊΠΎΠΎΡΠ΄ΠΈΠ½Π°Ρ), ΡΠΎ ΠΏΠΎ Π²ΡΠΎΡΠΎΠΌΡ ΠΎΠΏΡΠ΅Π΄Π΅Π»Π΅Π½ΠΈΡ Π²Π΅ΠΊΡΠΎΡΠ½ΠΎΠ³ΠΎ ΠΏΡΠΎΠΈΠ·Π²Π΅Π΄Π΅Π½ΠΈΡ ΠΈΠΌΠ΅Π΅ΠΌ
Π’ΠΎ Π΅ΡΡΡ, Π²Π΅ΠΊΡΠΎΡΠ½ΠΎΠ΅ ΠΏΡΠΎΠΈΠ·Π²Π΅Π΄Π΅Π½ΠΈΠ΅ ΠΈΠΌΠ΅Π΅Ρ ΠΊΠΎΠΎΡΠ΄ΠΈΠ½Π°ΡΡ Π² Π·Π°Π΄Π°Π½Π½ΠΎΠΉ ΡΠΈΡΡΠ΅ΠΌΠ΅ ΠΊΠΎΠΎΡΠ΄ΠΈΠ½Π°Ρ.
ΠΠ»ΠΈΠ½Ρ Π²Π΅ΠΊΡΠΎΡΠ½ΠΎΠ³ΠΎ ΠΏΡΠΎΠΈΠ·Π²Π΅Π΄Π΅Π½ΠΈΡ Π½Π°Ρ
ΠΎΠ΄ΠΈΠΌ ΠΊΠ°ΠΊ ΠΊΠΎΡΠ΅Π½Ρ ΠΊΠ²Π°Π΄ΡΠ°ΡΠ½ΡΠΉ ΠΈΠ· ΡΡΠΌΠΌΡ ΠΊΠ²Π°Π΄ΡΠ°ΡΠΎΠ² Π΅Π³ΠΎ ΠΊΠΎΠΎΡΠ΄ΠΈΠ½Π°Ρ (ΡΡΡ ΡΠΎΡΠΌΡΠ»Ρ Π΄Π»ΠΈΠ½Ρ Π²Π΅ΠΊΡΠΎΡΠ° ΠΌΡ ΠΏΠΎΠ»ΡΡΠΈΠ»ΠΈ Π² ΡΠ°Π·Π΄Π΅Π»Π΅ Π½Π°Ρ
ΠΎΠΆΠ΄Π΅Π½ΠΈΠ΅ Π΄Π»ΠΈΠ½Ρ Π²Π΅ΠΊΡΠΎΡΠ°):
ΠΡΠ²Π΅Ρ:
.
ΠΡΠΈΠΌΠ΅Ρ.
Π ΠΏΡΡΠΌΠΎΡΠ³ΠΎΠ»ΡΠ½ΠΎΠΉ Π΄Π΅ΠΊΠ°ΡΡΠΎΠ²ΠΎΠΉ ΡΠΈΡΡΠ΅ΠΌΠ΅ ΠΊΠΎΠΎΡΠ΄ΠΈΠ½Π°Ρ Π·Π°Π΄Π°Π½Ρ ΠΊΠΎΠΎΡΠ΄ΠΈΠ½Π°ΡΡ ΡΡΠ΅Ρ ΡΠΎΡΠ΅ΠΊ . ΠΠ°ΠΉΠ΄ΠΈΡΠ΅ ΠΊΠ°ΠΊΠΎΠΉ-Π½ΠΈΠ±ΡΠ΄Ρ Π²Π΅ΠΊΡΠΎΡ, ΠΏΠ΅ΡΠΏΠ΅Π½Π΄ΠΈΠΊΡΠ»ΡΡΠ½ΡΠΉ ΠΈ ΠΎΠ΄Π½ΠΎΠ²ΡΠ΅ΠΌΠ΅Π½Π½ΠΎ.
Π Π΅ΡΠ΅Π½ΠΈΠ΅.
ΠΠ΅ΠΊΡΠΎΡΡ ΠΈ ΠΈΠΌΠ΅ΡΡ ΠΊΠΎΠΎΡΠ΄ΠΈΠ½Π°ΡΡ ΠΈ ΡΠΎΠΎΡΠ²Π΅ΡΡΡΠ²Π΅Π½Π½ΠΎ (ΡΠΌΠΎΡΡΠΈΡΠ΅ ΡΡΠ°ΡΡΡ Π½Π°Ρ
ΠΎΠΆΠ΄Π΅Π½ΠΈΠ΅ ΠΊΠΎΠΎΡΠ΄ΠΈΠ½Π°Ρ Π²Π΅ΠΊΡΠΎΡΠ° ΡΠ΅ΡΠ΅Π· ΠΊΠΎΠΎΡΠ΄ΠΈΠ½Π°ΡΡ ΡΠΎΡΠ΅ΠΊ). ΠΡΠ»ΠΈ Π½Π°ΠΉΡΠΈ Π²Π΅ΠΊΡΠΎΡΠ½ΠΎΠ΅ ΠΏΡΠΎΠΈΠ·Π²Π΅Π΄Π΅Π½ΠΈΠ΅ Π²Π΅ΠΊΡΠΎΡΠΎΠ² ΠΈ , ΡΠΎ ΠΎΠ½ΠΎ ΠΏΠΎ ΠΎΠΏΡΠ΅Π΄Π΅Π»Π΅Π½ΠΈΡ ΡΠ²Π»ΡΠ΅ΡΡΡ Π²Π΅ΠΊΡΠΎΡΠΎΠΌ, ΠΏΠ΅ΡΠΏΠ΅Π½Π΄ΠΈΠΊΡΠ»ΡΡΠ½ΡΠΌ ΠΈ ΠΊ ΠΈ ΠΊ , ΡΠΎ Π΅ΡΡΡ, ΡΠ²Π»ΡΠ΅ΡΡΡ ΡΠ΅ΡΠ΅Π½ΠΈΠ΅ΠΌ Π½Π°ΡΠ΅ΠΉ Π·Π°Π΄Π°ΡΠΈ. ΠΠ°ΠΉΠ΄Π΅ΠΌ Π΅Π³ΠΎ
ΠΡΠ²Π΅Ρ:
β ΠΎΠ΄ΠΈΠ½ ΠΈΠ· ΠΏΠ΅ΡΠΏΠ΅Π½Π΄ΠΈΠΊΡΠ»ΡΡΠ½ΡΡ Π²Π΅ΠΊΡΠΎΡΠΎΠ².
Π Π·Π°Π΄Π°ΡΠ°Ρ ΡΡΠ΅ΡΡΠ΅Π³ΠΎ ΡΠΈΠΏΠ° ΠΏΡΠΎΠ²Π΅ΡΡΠ΅ΡΡΡ Π½Π°Π²ΡΠΊ ΠΈΡΠΏΠΎΠ»ΡΠ·ΠΎΠ²Π°Π½ΠΈΡ ΡΠ²ΠΎΠΉΡΡΠ² Π²Π΅ΠΊΡΠΎΡΠ½ΠΎΠ³ΠΎ ΠΏΡΠΎΠΈΠ·Π²Π΅Π΄Π΅Π½ΠΈΡ Π²Π΅ΠΊΡΠΎΡΠΎΠ². ΠΠΎΡΠ»Π΅ ΠΏΡΠΈΠΌΠ΅Π½Π΅Π½ΠΈΡ ΡΠ²ΠΎΠΉΡΡΠ², ΠΏΡΠΈΠΌΠ΅Π½ΡΡΡΡΡ ΡΠΎΠΎΡΠ²Π΅ΡΡΡΠ²ΡΡΡΠΈΠ΅ ΡΠΎΡΠΌΡΠ»Ρ.
ΠΡΠΈΠΌΠ΅Ρ.
ΠΠ΅ΠΊΡΠΎΡΡ ΠΈ ΠΏΠ΅ΡΠΏΠ΅Π½Π΄ΠΈΠΊΡΠ»ΡΡΠ½Ρ ΠΈ ΠΈΡ Π΄Π»ΠΈΠ½Ρ ΡΠ°Π²Π½Ρ ΡΠΎΠΎΡΠ²Π΅ΡΡΡΠ²Π΅Π½Π½ΠΎ 3 ΠΈ 4 . ΠΠ°ΠΉΠ΄ΠΈΡΠ΅ Π΄Π»ΠΈΠ½Ρ Π²Π΅ΠΊΡΠΎΡΠ½ΠΎΠ³ΠΎ ΠΏΡΠΎΠΈΠ·Π²Π΅Π΄Π΅Π½ΠΈΡ .
Π Π΅ΡΠ΅Π½ΠΈΠ΅.
ΠΠΎ ΡΠ²ΠΎΠΉΡΡΠ²Ρ Π΄ΠΈΡΡΡΠΈΠ±ΡΡΠΈΠ²Π½ΠΎΡΡΠΈ Π²Π΅ΠΊΡΠΎΡΠ½ΠΎΠ³ΠΎ ΠΏΡΠΎΠΈΠ·Π²Π΅Π΄Π΅Π½ΠΈΡ ΠΌΡ ΠΌΠΎΠΆΠ΅ΠΌ Π·Π°ΠΏΠΈΡΠ°ΡΡ
Π ΡΠΈΠ»Ρ ΡΠΎΡΠ΅ΡΠ°ΡΠ΅Π»ΡΠ½ΠΎΠ³ΠΎ ΡΠ²ΠΎΠΉΡΡΠ²Π° Π²ΡΠ½Π΅ΡΠ΅ΠΌ ΡΠΈΡΠ»ΠΎΠ²ΡΠ΅ ΠΊΠΎΡΡΡΠΈΡΠΈΠ΅Π½ΡΡ Π·Π° Π·Π½Π°ΠΊ Π²Π΅ΠΊΡΠΎΡΠ½ΡΡ ΠΏΡΠΎΠΈΠ·Π²Π΅Π΄Π΅Π½ΠΈΠΉ Π² ΠΏΠΎΡΠ»Π΅Π΄Π½Π΅ΠΌ Π²ΡΡΠ°ΠΆΠ΅Π½ΠΈΠΈ:
ΠΠ΅ΠΊΡΠΎΡΠ½ΡΠ΅ ΠΏΡΠΎΠΈΠ·Π²Π΅Π΄Π΅Π½ΠΈΡ ΠΈ ΡΠ°Π²Π½Ρ Π½ΡΠ»Ρ, ΡΠ°ΠΊ ΠΊΠ°ΠΊ ΠΈ , ΡΠΎΠ³Π΄Π° .
Π’Π°ΠΊ ΠΊΠ°ΠΊ Π²Π΅ΠΊΡΠΎΡΠ½ΠΎΠ΅ ΠΏΡΠΎΠΈΠ·Π²Π΅Π΄Π΅Π½ΠΈΠ΅ Π°Π½ΡΠΈΠΊΠΎΠΌΠΌΡΡΠ°ΡΠΈΠ²Π½ΠΎ, ΡΠΎ .
ΠΡΠ°ΠΊ, Ρ ΠΏΠΎΠΌΠΎΡΡΡ ΡΠ²ΠΎΠΉΡΡΠ² Π²Π΅ΠΊΡΠΎΡΠ½ΠΎΠ³ΠΎ ΠΏΡΠΎΠΈΠ·Π²Π΅Π΄Π΅Π½ΠΈΡ ΠΌΡ ΠΏΡΠΈΡΠ»ΠΈ ΠΊ ΡΠ°Π²Π΅Π½ΡΡΠ²Ρ .
ΠΠΎ ΡΡΠ»ΠΎΠ²ΠΈΡ Π²Π΅ΠΊΡΠΎΡΡ ΠΈ ΠΏΠ΅ΡΠΏΠ΅Π½Π΄ΠΈΠΊΡΠ»ΡΡΠ½Ρ, ΡΠΎ Π΅ΡΡΡ ΡΠ³ΠΎΠ» ΠΌΠ΅ΠΆΠ΄Ρ Π½ΠΈΠΌΠΈ ΡΠ°Π²Π΅Π½ . Π’ΠΎ Π΅ΡΡΡ, Ρ Π½Π°Ρ Π΅ΡΡΡ Π²ΡΠ΅ Π΄Π°Π½Π½ΡΠ΅ Π΄Π»Ρ Π½Π°Ρ ΠΎΠΆΠ΄Π΅Π½ΠΈΡ ΡΡΠ΅Π±ΡΠ΅ΠΌΠΎΠΉ Π΄Π»ΠΈΠ½Ρ
ΠΡΠ²Π΅Ρ:
.
ΠΠ΅ΠΎΠΌΠ΅ΡΡΠΈΡΠ΅ΡΠΊΠΈΠΉ ΡΠΌΡΡΠ» Π²Π΅ΠΊΡΠΎΡΠ½ΠΎΠ³ΠΎ ΠΏΡΠΎΠΈΠ·Π²Π΅Π΄Π΅Π½ΠΈΡ.

ΠΠΎ ΠΎΠΏΡΠ΅Π΄Π΅Π»Π΅Π½ΠΈΡ Π΄Π»ΠΈΠ½Π° Π²Π΅ΠΊΡΠΎΡΠ½ΠΎΠ³ΠΎ ΠΏΡΠΎΠΈΠ·Π²Π΅Π΄Π΅Π½ΠΈΡ Π²Π΅ΠΊΡΠΎΡΠΎΠ² ΡΠ°Π²Π½Π° . Π ΠΈΠ· ΠΊΡΡΡΠ° Π³Π΅ΠΎΠΌΠ΅ΡΡΠΈΠΈ ΡΡΠ΅Π΄Π½Π΅ΠΉ ΡΠΊΠΎΠ»Ρ Π½Π°ΠΌ ΠΈΠ·Π²Π΅ΡΡΠ½ΠΎ, ΡΡΠΎ ΠΏΠ»ΠΎΡΠ°Π΄Ρ ΡΡΠ΅ΡΠ³ΠΎΠ»ΡΠ½ΠΈΠΊΠ° ΡΠ°Π²Π½Π° ΠΏΠΎΠ»ΠΎΠ²ΠΈΠ½Π΅ ΠΏΡΠΎΠΈΠ·Π²Π΅Π΄Π΅Π½ΠΈΡ Π΄Π»ΠΈΠ½ Π΄Π²ΡΡ ΡΡΠΎΡΠΎΠ½ ΡΡΠ΅ΡΠ³ΠΎΠ»ΡΠ½ΠΈΠΊΠ° Π½Π° ΡΠΈΠ½ΡΡ ΡΠ³Π»Π° ΠΌΠ΅ΠΆΠ΄Ρ Π½ΠΈΠΌΠΈ. Π‘Π»Π΅Π΄ΠΎΠ²Π°ΡΠ΅Π»ΡΠ½ΠΎ, Π΄Π»ΠΈΠ½Π° Π²Π΅ΠΊΡΠΎΡΠ½ΠΎΠ³ΠΎ ΠΏΡΠΎΠΈΠ·Π²Π΅Π΄Π΅Π½ΠΈΡ ΡΠ°Π²Π½Π° ΡΠ΄Π²ΠΎΠ΅Π½Π½ΠΎΠΉ ΠΏΠ»ΠΎΡΠ°Π΄ΠΈ ΡΡΠ΅ΡΠ³ΠΎΠ»ΡΠ½ΠΈΠΊΠ°, ΠΈΠΌΠ΅ΡΡΠ΅Π³ΠΎ ΡΡΠΎΡΠΎΠ½Π°ΠΌΠΈ Π²Π΅ΠΊΡΠΎΡΡ ΠΈ , Π΅ΡΠ»ΠΈ ΠΈΡ ΠΎΡΠ»ΠΎΠΆΠΈΡΡ ΠΎΡ ΠΎΠ΄Π½ΠΎΠΉ ΡΠΎΡΠΊΠΈ. ΠΡΡΠ³ΠΈΠΌΠΈ ΡΠ»ΠΎΠ²Π°ΠΌΠΈ, Π΄Π»ΠΈΠ½Π° Π²Π΅ΠΊΡΠΎΡΠ½ΠΎΠ³ΠΎ ΠΏΡΠΎΠΈΠ·Π²Π΅Π΄Π΅Π½ΠΈΡ Π²Π΅ΠΊΡΠΎΡΠΎΠ² ΠΈ ΡΠ°Π²Π½Π° ΠΏΠ»ΠΎΡΠ°Π΄ΠΈ ΠΏΠ°ΡΠ°Π»Π»Π΅Π»ΠΎΠ³ΡΠ°ΠΌΠΌΠ° ΡΠΎ ΡΡΠΎΡΠΎΠ½Π°ΠΌΠΈ ΠΈ ΠΈ ΡΠ³Π»ΠΎΠΌ ΠΌΠ΅ΠΆΠ΄Ρ Π½ΠΈΠΌΠΈ, ΡΠ°Π²Π½ΡΠΌ . Π ΡΡΠΎΠΌ ΡΠΎΡΡΠΎΠΈΡ Π³Π΅ΠΎΠΌΠ΅ΡΡΠΈΡΠ΅ΡΠΊΠΈΠΉ ΡΠΌΡΡΠ» Π²Π΅ΠΊΡΠΎΡΠ½ΠΎΠ³ΠΎ ΠΏΡΠΎΠΈΠ·Π²Π΅Π΄Π΅Π½ΠΈΡ.
ΠΠ΄ΠΈΠ½ΠΈΡΠ½ΡΠΉ Π²Π΅ΠΊΡΠΎΡ β ΡΡΠΎ Π²Π΅ΠΊΡΠΎΡ , Π°Π±ΡΠΎΠ»ΡΡΠ½Π°Ρ Π²Π΅Π»ΠΈΡΠΈΠ½Π° (ΠΌΠΎΠ΄ΡΠ»Ρ) ΠΊΠΎΡΠΎΡΠΎΠ³ΠΎ ΡΠ°Π²Π΅Π½ Π΅Π΄ΠΈΠ½ΠΈΡΠ΅. ΠΠ»Ρ ΠΎΠ±ΠΎΠ·Π½Π°ΡΠ΅Π½ΠΈΡ Π΅Π΄ΠΈΠ½ΠΈΡΠ½ΠΎΠ³ΠΎ Π²Π΅ΠΊΡΠΎΡΠ° ΠΌΡ Π±ΡΠ΄Π΅ΠΌ ΠΈΡΠΏΠΎΠ»ΡΠ·ΠΎΠ²Π°ΡΡ Π½ΠΈΠΆΠ½ΠΈΠΉ ΠΈΠ½Π΄Π΅ΠΊΡ Π΅. Π’Π°ΠΊ, Π΅ΡΠ»ΠΈ Π·Π°Π΄Π°Π½ Π²Π΅ΠΊΡΠΎΡ Π° , ΡΠΎ Π΅Π³ΠΎ Π΅Π΄ΠΈΠ½ΠΈΡΠ½ΡΠΌ Π²Π΅ΠΊΡΠΎΡΠΎΠΌ Π±ΡΠ΄Π΅Ρ Π²Π΅ΠΊΡΠΎΡ Π° Π΅. ΠΡΠΎΡ Π΅Π΄ΠΈΠ½ΠΈΡΠ½ΡΠΉ Π²Π΅ΠΊΡΠΎΡ Π½Π°ΠΏΡΠ°Π²Π»Π΅Π½ ΡΡΠ΄Π° ΠΆΠ΅, ΠΊΡΠ΄Π° Π½Π°ΠΏΡΠ°Π²Π»Π΅Π½ ΠΈ ΡΠ°ΠΌ Π²Π΅ΠΊΡΠΎΡ Π° , ΠΈ Π΅Π³ΠΎ ΠΌΠΎΠ΄ΡΠ»Ρ ΡΠ°Π²Π΅Π½ Π΅Π΄ΠΈΠ½ΠΈΡΠ΅, ΡΠΎ Π΅ΡΡΡ Π° Π΅ = 1.
ΠΡΠ΅Π²ΠΈΠ΄Π½ΠΎ, Π° = Π°Β·Π° Π΅ (Π° β ΠΌΠΎΠ΄ΡΠ»Ρ Π²Π΅ΠΊΡΠΎΡΠ° Π°) . ΠΡΠΎ ΡΠ»Π΅Π΄ΡΠ΅Ρ ΠΈΠ· ΠΏΡΠ°Π²ΠΈΠ»Π°, ΠΏΠΎ ΠΊΠΎΡΠΎΡΠΎΠΌΡ Π²ΡΠΏΠΎΠ»Π½ΡΠ΅ΡΡΡ ΠΎΠΏΠ΅ΡΠ°ΡΠΈΡ ΡΠΌΠ½ΠΎΠΆΠ΅Π½ΠΈΡ ΡΠΊΠ°Π»ΡΡΠ° Π½Π° Π²Π΅ΠΊΡΠΎΡ .
ΠΠ΄ΠΈΠ½ΠΈΡΠ½ΡΠ΅ Π²Π΅ΠΊΡΠΎΡΡ ΡΠ°ΡΡΠΎ ΡΠ²ΡΠ·ΡΠ²Π°ΡΡ Ρ ΠΊΠΎΠΎΡΠ΄ΠΈΠ½Π°ΡΠ½ΡΠΌΠΈ ΠΎΡΡΠΌΠΈ ΡΠΈΡΡΠ΅ΠΌΡ ΠΊΠΎΠΎΡΠ΄ΠΈΠ½Π°Ρ (Π² ΡΠ°ΡΡΠ½ΠΎΡΡΠΈ, Ρ ΠΎΡΡΠΌΠΈ Π΄Π΅ΠΊΠ°ΡΡΠΎΠ²ΠΎΠΉ ΡΠΈΡΡΠ΅ΠΌΡ ΠΊΠΎΠΎΡΠ΄ΠΈΠ½Π°Ρ). ΠΠ°ΠΏΡΠ°Π²Π»Π΅Π½ΠΈΡ ΡΡΠΈΡ Π²Π΅ΠΊΡΠΎΡΠΎΠ² ΡΠΎΠ²ΠΏΠ°Π΄Π°ΡΡ Ρ Π½Π°ΠΏΡΠ°Π²Π»Π΅Π½ΠΈΡΠΌΠΈ ΡΠΎΠΎΡΠ²Π΅ΡΡΡΠ²ΡΡΡΠΈΡ ΠΎΡΠ΅ΠΉ, Π° ΠΈΡ Π½Π°ΡΠ°Π»Π° ΡΠ°ΡΡΠΎ ΡΠΎΠ²ΠΌΠ΅ΡΠ°ΡΡ Ρ Π½Π°ΡΠ°Π»ΠΎΠΌ ΡΠΈΡΡΠ΅ΠΌΡ ΠΊΠΎΠΎΡΠ΄ΠΈΠ½Π°Ρ.
ΠΠ°ΠΏΠΎΠΌΠ½Ρ, ΡΡΠΎ Π΄Π΅ΠΊΠ°ΡΡΠΎΠ²ΠΎΠΉ ΡΠΈΡΡΠ΅ΠΌΠΎΠΉ ΠΊΠΎΠΎΡΠ΄ΠΈΠ½Π°Ρ Π² ΠΏΡΠΎΡΡΡΠ°Π½ΡΡΠ²Π΅ ΡΡΠ°Π΄ΠΈΡΠΈΠΎΠ½Π½ΠΎ Π½Π°Π·ΡΠ²Π°Π΅ΡΡΡ ΡΡΠΎΠΉΠΊΠ° Π²Π·Π°ΠΈΠΌΠ½ΠΎ ΠΏΠ΅ΡΠΏΠ΅Π½Π΄ΠΈΠΊΡΠ»ΡΡΠ½ΡΡ
ΠΎΡΠ΅ΠΉ, ΠΏΠ΅ΡΠ΅ΡΠ΅ΠΊΠ°ΡΡΠΈΡ
ΡΡ Π² ΡΠΎΡΠΊΠ΅, ΠΊΠΎΡΠΎΡΠ°Ρ Π½Π°Π·ΡΠ²Π°Π΅ΡΡΡ Π½Π°ΡΠ°Π»ΠΎΠΌ ΠΊΠΎΠΎΡΠ΄ΠΈΠ½Π°Ρ. ΠΠΎΠΎΡΠ΄ΠΈΠ½Π°ΡΠ½ΡΠ΅ ΠΎΡΠΈ ΠΎΠ±ΡΡΠ½ΠΎ ΠΎΠ±ΠΎΠ·Π½Π°ΡΠ°ΡΡ Π±ΡΠΊΠ²Π°ΠΌΠΈ X , Y , Z ΠΈ Π½Π°Π·ΡΠ²Π°ΡΡ ΡΠΎΠΎΡΠ²Π΅ΡΡΡΠ²Π΅Π½Π½ΠΎ ΠΎΡΡΡ Π°Π±ΡΡΠΈΡΡ, ΠΎΡΡΡ ΠΎΡΠ΄ΠΈΠ½Π°Ρ ΠΈ ΠΎΡΡΡ Π°ΠΏΠΏΠ»ΠΈΠΊΠ°Ρ. Π‘Π°ΠΌ ΠΠ΅ΠΊΠ°ΡΡ ΠΏΠΎΠ»ΡΠ·ΠΎΠ²Π°Π»ΡΡ ΡΠΎΠ»ΡΠΊΠΎ ΠΎΠ΄Π½ΠΎΠΉ ΠΎΡΡΡ, Π½Π° ΠΊΠΎΡΠΎΡΠΎΠΉ ΠΎΡΠΊΠ»Π°Π΄ΡΠ²Π°Π»ΠΈΡΡ Π°Π±ΡΡΠΈΡΡΡ. ΠΠ°ΡΠ»ΡΠ³Π° ΠΈΡΠΏΠΎΠ»ΡΠ·ΠΎΠ²Π°Π½ΠΈΡ ΡΠΈΡΡΠ΅ΠΌΡ ΠΎΡΠ΅ΠΉ ΠΏΡΠΈΠ½Π°Π΄Π»Π΅ΠΆΠΈΡ Π΅Π³ΠΎ ΡΡΠ΅Π½ΠΈΠΊΠ°ΠΌ. ΠΠΎΡΡΠΎΠΌΡ ΡΡΠ°Π·Π° Π΄Π΅ΠΊΠ°ΡΡΠΎΠ²Π° ΡΠΈΡΡΠ΅ΠΌΠ° ΠΊΠΎΠΎΡΠ΄ΠΈΠ½Π°Ρ ΠΈΡΡΠΎΡΠΈΡΠ΅ΡΠΊΠΈ ΠΎΡΠΈΠ±ΠΎΡΠ½Π°. ΠΡΡΡΠ΅ Π³ΠΎΠ²ΠΎΡΠΈΡΡ ΠΏΡΡΠΌΠΎΡΠ³ΠΎΠ»ΡΠ½Π°Ρ ΡΠΈΡΡΠ΅ΠΌΠ° ΠΊΠΎΠΎΡΠ΄ΠΈΠ½Π°Ρ ΠΈΠ»ΠΈ ΠΎΡΡΠΎΠ³ΠΎΠ½Π°Π»ΡΠ½Π°Ρ ΡΠΈΡΡΠ΅ΠΌΠ° ΠΊΠΎΠΎΡΠ΄ΠΈΠ½Π°Ρ . Π’Π΅ΠΌ Π½Π΅ ΠΌΠ΅Π½Π΅Π΅, ΠΈΠ·ΠΌΠ΅Π½ΡΡΡ ΡΡΠ°Π΄ΠΈΡΠΈΠΈ ΠΌΡ Π½Π΅ ΡΡΠ°Π½Π΅ΠΌ ΠΈ Π² Π΄Π°Π»ΡΠ½Π΅ΠΉΡΠ΅ΠΌ Π±ΡΠ΄Π΅ΠΌ ΡΡΠΈΡΠ°ΡΡ, ΡΡΠΎ Π΄Π΅ΠΊΠ°ΡΡΠΎΠ²Π° ΠΈ ΠΏΡΡΠΌΠΎΡΠ³ΠΎΠ»ΡΠ½Π°Ρ (ΠΎΡΡΠΎΠ³ΠΎΠ½Π°Π»ΡΠ½Π°Ρ) ΡΠΈΡΡΠ΅ΠΌΡ ΠΊΠΎΠΎΡΠ΄ΠΈΠ½Π°Ρ β ΡΡΠΎ ΠΎΠ΄Π½ΠΎ ΠΈ ΡΠΎ ΠΆΠ΅.
ΠΠ΄ΠΈΠ½ΠΈΡΠ½ΡΠΉ Π²Π΅ΠΊΡΠΎΡ , Π½Π°ΠΏΡΠ°Π²Π»Π΅Π½Π½ΡΠΉ Π²Π΄ΠΎΠ»Ρ ΠΎΡΠΈ Π₯, ΠΎΠ±ΠΎΠ·Π½Π°ΡΠ°Π΅ΡΡΡ i , Π΅Π΄ΠΈΠ½ΠΈΡΠ½ΡΠΉ Π²Π΅ΠΊΡΠΎΡ , Π½Π°ΠΏΡΠ°Π²Π»Π΅Π½Π½ΡΠΉ Π²Π΄ΠΎΠ»Ρ ΠΎΡΠΈ Y , ΠΎΠ±ΠΎΠ·Π½Π°ΡΠ°Π΅ΡΡΡ j , Π° Π΅Π΄ΠΈΠ½ΠΈΡΠ½ΡΠΉ Π²Π΅ΠΊΡΠΎΡ , Π½Π°ΠΏΡΠ°Π²Π»Π΅Π½Π½ΡΠΉ Π²Π΄ΠΎΠ»Ρ ΠΎΡΠΈ Z, ΠΎΠ±ΠΎΠ·Π½Π°ΡΠ°Π΅ΡΡΡ k . ΠΠ΅ΠΊΡΠΎΡΡ i , j , k Π½Π°Π·ΡΠ²Π°ΡΡΡΡ ΠΎΡΡΠ°ΠΌΠΈ (ΡΠΈΡ. 12, ΡΠ»Π΅Π²Π°), ΠΎΠ½ΠΈ ΠΈΠΌΠ΅ΡΡ Π΅Π΄ΠΈΠ½ΠΈΡΠ½ΡΠ΅ ΠΌΠΎΠ΄ΡΠ»ΠΈ, ΡΠΎ Π΅ΡΡΡ
i = 1, j = 1, k = 1.
ΠΡΠΈ ΠΈ ΠΎΡΡΡ ΠΏΡΡΠΌΠΎΡΠ³ΠΎΠ»ΡΠ½ΠΎΠΉ ΡΠΈΡΡΠ΅ΠΌΡ ΠΊΠΎΠΎΡΠ΄ΠΈΠ½Π°Ρ Π² Π½Π΅ΠΊΠΎΡΠΎΡΡΡ ΡΠ»ΡΡΠ°ΡΡ ΠΈΠΌΠ΅ΡΡ Π΄ΡΡΠ³ΠΈΠ΅ Π½Π°Π·Π²Π°Π½ΠΈΡ ΠΈ ΠΎΠ±ΠΎΠ·Π½Π°ΡΠ΅Π½ΠΈΡ. Π’Π°ΠΊ, ΠΎΡΡ Π°Π±ΡΡΠΈΡΡ X ΠΌΠΎΠΆΠ΅Ρ Π½Π°Π·ΡΠ²Π°ΡΡΡΡ ΠΊΠ°ΡΠ°ΡΠ΅Π»ΡΠ½ΠΎΠΉ ΠΎΡΡΡ, Π° Π΅Π΅ ΠΎΡΡ ΠΎΠ±ΠΎΠ·Π½Π°ΡΠ°Π΅ΡΡΡ Ο (Π³ΡΠ΅ΡΠ΅ΡΠΊΠ°Ρ ΡΡΡΠΎΡΠ½Π°Ρ Π±ΡΠΊΠ²Π° ΡΠ°Ρ), ΠΎΡΡ ΠΎΡΠ΄ΠΈΠ½Π°Ρ β ΠΎΡΡΡ Π½ΠΎΡΠΌΠ°Π»ΠΈ, Π΅Π΅ ΠΎΡΡ ΠΎΠ±ΠΎΠ·Π½Π°ΡΠ°Π΅ΡΡΡ n , ΠΎΡΡ Π°ΠΏΠΏΠ»ΠΈΠΊΠ°Ρ β ΠΎΡΡΡ Π±ΠΈΠ½ΠΎΡΠΌΠ°Π»ΠΈ, Π΅Π΅ ΠΎΡΡ ΠΎΠ±ΠΎΠ·Π½Π°ΡΠ°Π΅ΡΡΡ b . ΠΠ°ΡΠ΅ΠΌ ΠΌΠ΅Π½ΡΡΡ Π½Π°Π·Π²Π°Π½ΠΈΡ, Π΅ΡΠ»ΠΈ ΡΡΡΡ ΠΎΡΡΠ°Π΅ΡΡΡ ΡΠΎΠΉ ΠΆΠ΅?
ΠΠ΅Π»ΠΎ Π² ΡΠΎΠΌ, ΡΡΠΎ, Π½Π°ΠΏΡΠΈΠΌΠ΅Ρ, Π² ΠΌΠ΅Ρ
Π°Π½ΠΈΠΊΠ΅ ΠΏΡΠΈ ΠΈΠ·ΡΡΠ΅Π½ΠΈΠΈ Π΄Π²ΠΈΠΆΠ΅Π½ΠΈΡ ΡΠ΅Π» ΠΏΡΡΠΌΠΎΡΠ³ΠΎΠ»ΡΠ½Π°Ρ ΡΠΈΡΡΠ΅ΠΌΠ° ΠΊΠΎΠΎΡΠ΄ΠΈΠ½Π°Ρ ΠΈΡΠΏΠΎΠ»ΡΠ·ΡΠ΅ΡΡΡ ΠΎΡΠ΅Π½Ρ ΡΠ°ΡΡΠΎ. Π’Π°ΠΊ Π²ΠΎΡ, Π΅ΡΠ»ΠΈ ΡΠ°ΠΌΠ° ΡΠΈΡΡΠ΅ΠΌΠ° ΠΊΠΎΠΎΡΠ΄ΠΈΠ½Π°Ρ Π½Π΅ΠΏΠΎΠ΄Π²ΠΈΠΆΠ½Π°, Π° ΠΈΠ·ΠΌΠ΅Π½Π΅Π½ΠΈΠ΅ ΠΊΠΎΠΎΡΠ΄ΠΈΠ½Π°Ρ Π΄Π²ΠΈΠΆΡΡΠ΅Π³ΠΎΡΡ ΠΎΠ±ΡΠ΅ΠΊΡΠ° ΠΎΡΡΠ»Π΅ΠΆΠΈΠ²Π°Π΅ΡΡΡ Π² ΡΡΠΎΠΉ Π½Π΅ΠΏΠΎΠ΄Π²ΠΈΠΆΠ½ΠΎΠΉ ΡΠΈΡΡΠ΅ΠΌΠ΅, ΡΠΎ ΠΎΠ±ΡΡΠ½ΠΎ ΠΎΡΠΈ ΠΎΠ±ΠΎΠ·Π½Π°ΡΠ°ΡΡ X, Y, Z, Π° ΠΈΡ
ΠΎΡΡΡ ΡΠΎΠΎΡΠ²Π΅ΡΡΡΠ²Π΅Π½Π½ΠΎ i , j , k .
ΠΠΎ Π½Π΅ΡΠ΅Π΄ΠΊΠΎ, ΠΊΠΎΠ³Π΄Π° ΠΎΠ±ΡΠ΅ΠΊΡ Π΄Π²ΠΈΠΆΠ΅ΡΡΡ ΠΏΠΎ ΠΊΠ°ΠΊΠΎΠΉ-ΡΠΎ ΠΊΡΠΈΠ²ΠΎΠ»ΠΈΠ½Π΅ΠΉΠ½ΠΎΠΉ ΡΡΠ°Π΅ΠΊΡΠΎΡΠΈΠΈ (Π½Π°ΠΏΡΠΈΠΌΠ΅Ρ, ΠΏΠΎ ΠΎΠΊΡΡΠΆΠ½ΠΎΡΡΠΈ) Π±ΡΠ²Π°Π΅Ρ ΡΠ΄ΠΎΠ±Π½Π΅Π΅ ΡΠ°ΡΡΠΌΠ°ΡΡΠΈΠ²Π°ΡΡ ΠΌΠ΅Ρ
Π°Π½ΠΈΡΠ΅ΡΠΊΠΈΠ΅ ΠΏΡΠΎΡΠ΅ΡΡΡ Π² ΡΠΈΡΡΠ΅ΠΌΠ΅ ΠΊΠΎΠΎΡΠ΄ΠΈΠ½Π°Ρ, Π΄Π²ΠΈΠΆΡΡΠ΅ΠΉΡΡ Ρ ΡΡΠΈΠΌ ΠΎΠ±ΡΠ΅ΠΊΡΠΎΠΌ. ΠΠΌΠ΅Π½Π½ΠΎ Π΄Π»Ρ ΡΠ°ΠΊΠΎΠΉ Π΄Π²ΠΈΠΆΡΡΠ΅ΠΉΡΡ ΡΠΈΡΡΠ΅ΠΌΡ ΠΊΠΎΠΎΡΠ΄ΠΈΠ½Π°Ρ ΠΈ ΠΈΡΠΏΠΎΠ»ΡΠ·ΡΡΡΡΡ Π΄ΡΡΠ³ΠΈΠ΅ Π½Π°Π·Π²Π°Π½ΠΈΡ ΠΎΡΠ΅ΠΉ ΠΈ ΠΈΡ
ΠΎΡΡΠΎΠ². ΠΡΠΎΡΡΠΎ ΡΠ°ΠΊ ΠΏΡΠΈΠ½ΡΡΠΎ. Π ΡΡΠΎΠΌ ΡΠ»ΡΡΠ°Π΅ ΠΎΡΡ X Π½Π°ΠΏΡΠ°Π²Π»ΡΡΡ ΠΏΠΎ ΠΊΠ°ΡΠ°ΡΠ΅Π»ΡΠ½ΠΎΠΉ ΠΊ ΡΡΠ°Π΅ΠΊΡΠΎΡΠΈΠΈ Π² ΡΠΎΠΉ Π΅Π΅ ΡΠΎΡΠΊΠ΅, Π² ΠΊΠΎΡΠΎΡΠΎΠΉ Π² Π΄Π°Π½Π½ΡΠΉ ΠΌΠΎΠΌΠ΅Π½Ρ ΡΡΠΎΡ ΠΎΠ±ΡΠ΅ΠΊΡ Π½Π°Ρ
ΠΎΠ΄ΠΈΡΡΡ. Π ΡΠΎΠ³Π΄Π° ΡΡΡ ΠΎΡΡ Π½Π°Π·ΡΠ²Π°ΡΡ ΡΠΆΠ΅ Π½Π΅ ΠΎΡΡΡ X, Π° ΠΊΠ°ΡΠ°ΡΠ΅Π»ΡΠ½ΠΎΠΉ ΠΎΡΡΡ, Π° Π΅Π΅ ΠΎΡΡ ΠΎΠ±ΠΎΠ·Π½Π°ΡΠ°ΡΡ ΡΠΆΠ΅ Π½Π΅ i , Π° Ο . ΠΡΡ Y Π½Π°ΠΏΡΠ°Π²Π»ΡΡΡ ΠΏΠΎ ΡΠ°Π΄ΠΈΡΡΡ ΠΊΡΠΈΠ²ΠΈΠ·Π½Ρ ΡΡΠ°Π΅ΠΊΡΠΎΡΠΈΠΈ (Π² ΡΠ»ΡΡΠ°Π΅ Π΄Π²ΠΈΠΆΠ΅Π½ΠΈΡ ΠΏΠΎ ΠΎΠΊΡΡΠΆΠ½ΠΎΡΡΠΈ β ΠΊ ΡΠ΅Π½ΡΡΡ ΠΎΠΊΡΡΠΆΠ½ΠΎΡΡΠΈ). Π ΠΏΠΎΡΠΊΠΎΠ»ΡΠΊΡ ΡΠ°Π΄ΠΈΡΡ ΠΏΠ΅ΡΠΏΠ΅Π½Π΄ΠΈΠΊΡΠ»ΡΡΠ΅Π½ ΠΊΠ°ΡΠ°ΡΠ΅Π»ΡΠ½ΠΎΠΉ, ΡΠΎ ΠΎΡΡ Π½Π°Π·ΡΠ²Π°ΡΡ ΠΎΡΡΡ Π½ΠΎΡΠΌΠ°Π»ΠΈ (ΠΏΠ΅ΡΠΏΠ΅Π½Π΄ΠΈΠΊΡΠ»ΡΡ ΠΈ Π½ΠΎΡΠΌΠ°Π»Ρ β ΡΡΠΎ ΠΎΠ΄Π½ΠΎ ΠΈ ΡΠΎ ΠΆΠ΅). ΠΡΡ ΡΡΠΎΠΉ ΠΎΡΠΈ ΠΎΠ±ΠΎΠ·Π½Π°ΡΠ°ΡΡ ΡΠΆΠ΅ Π½Π΅ j , Π° n . Π’ΡΠ΅ΡΡΡ ΠΎΡΡ (Π±ΡΠ²ΡΠ°Ρ Z) ΠΏΠ΅ΡΠΏΠ΅Π½Π΄ΠΈΠΊΡΠ»ΡΡΠ½Π° Π΄Π²ΡΠΌ ΠΏΡΠ΅Π΄ΡΠ΄ΡΡΠΈΠΌ. ΠΡΠΎ β Π±ΠΈΠ½ΠΎΡΠΌΠ°Π»Ρ Ρ ΠΎΡΡΠΎΠΌ b (ΡΠΈΡ. 12, ΡΠΏΡΠ°Π²Π°). ΠΡΡΠ°ΡΠΈ, Π² ΡΡΠΎΠΌ ΡΠ»ΡΡΠ°Π΅ ΡΠ°ΠΊΡΡ ΠΏΡΡΠΌΠΎΡΠ³ΠΎΠ»ΡΠ½ΡΡ ΡΠΈΡΡΠ΅ΠΌΡ ΠΊΠΎΠΎΡΠ΄ΠΈΠ½Π°Ρ ΡΠ°ΡΡΠΎ Π½Π°Π·ΡΠ²Π°ΡΡ Β«Π΅ΡΡΠ΅ΡΡΠ²Π΅Π½Π½ΠΎΠΉΒ» ΠΈΠ»ΠΈ Π½Π°ΡΡΡΠ°Π»ΡΠ½ΠΎΠΉ.
ΠΠ΅ΠΊΡΠΎΡΠ½ΠΎΠ΅ ΠΏΡΠΎΠΈΠ·Π²Π΅Π΄Π΅Π½ΠΈΠ΅ Π²Π΅ΠΊΡΠΎΡΠΎΠ².
ΠΠ°Π²ΠΈΠ³Π°ΡΠΈΡ ΠΏΠΎ ΡΡΡΠ°Π½ΠΈΡΠ΅:
- ΠΠΏΡΠ΅Π΄Π΅Π»Π΅Π½ΠΈΠ΅ Π²Π΅ΠΊΡΠΎΡΠ½ΠΎΠ³ΠΎ ΠΏΡΠΎΠΈΠ·Π²Π΅Π΄Π΅Π½ΠΈΡ Π²Π΅ΠΊΡΠΎΡΠΎΠ²
- Π€ΠΎΡΠΌΡΠ»Ρ Π²ΡΡΠΈΡΠ»Π΅Π½ΠΈΡ Π²Π΅ΠΊΡΠΎΡΠ½ΠΎΠ³ΠΎ ΠΏΡΠΎΠΈΠ·Π²Π΅Π΄Π΅Π½ΠΈΡ Π²Π΅ΠΊΡΠΎΡΠΎΠ²
- Π‘Π²ΠΎΠΉΡΡΠ²Π° Π²Π΅ΠΊΡΠΎΡΠ½ΠΎΠ³ΠΎ ΠΏΡΠΎΠΈΠ·Π²Π΅Π΄Π΅Π½ΠΈΡ Π²Π΅ΠΊΡΠΎΡΠΎΠ²
- ΠΡΠΈΠΌΠ΅ΡΡ Π·Π°Π΄Π°Ρ Π½Π° Π²ΡΡΠΈΡΠ»Π΅Π½ΠΈΡ Π²Π΅ΠΊΡΠΎΡΠ½ΠΎΠ³ΠΎ ΠΏΡΠΎΠΈΠ·Π²Π΅Π΄Π΅Π½ΠΈΡ Π²Π΅ΠΊΡΠΎΡΠΎΠ²
ΠΠ½Π»Π°ΠΉΠ½ ΠΊΠ°Π»ΡΠΊΡΠ»ΡΡΠΎΡ. ΠΠ΅ΠΊΡΠΎΡΠ½ΠΎΠ΅ ΠΏΡΠΎΠΈΠ·Π²Π΅Π΄Π΅Π½ΠΈΠ΅ Π²Π΅ΠΊΡΠΎΡΠΎΠ².
Π£ΠΏΡΠ°ΠΆΠ½Π΅Π½ΠΈΡ Π½Π° ΡΠ΅ΠΌΡ Π²Π΅ΠΊΡΠΎΡΠ½ΠΎΠ΅ ΠΏΡΠΎΠΈΠ·Π²Π΅Π΄Π΅Π½ΠΈΠ΅ Π²Π΅ΠΊΡΠΎΡΠΎΠ².
ΠΠΏΡΠ΅Π΄Π΅Π»Π΅Π½ΠΈΠ΅. ΠΠ΅ΠΊΡΠΎΡΠ½ΡΠΌ ΠΏΡΠΎΠΈΠ·Π²Π΅Π΄Π΅Π½ΠΈΠ΅ΠΌ Π²Π΅ΠΊΡΠΎΡΠ° a Π½Π° Π²Π΅ΠΊΡΠΎΡ b Π½Π°Π·ΡΠ²Π°Π΅ΡΡΡ Π²Π΅ΠΊΡΠΎΡ c, Π΄Π»ΠΈΠ½Π° ΠΊΠΎΡΠΎΡΠΎΠ³ΠΎ ΡΠΈΡΠ»Π΅Π½Π½ΠΎ ΡΠ°Π²Π½Π° ΠΏΠ»ΠΎΡΠ°Π΄ΠΈ ΠΏΠ°ΡΠ°Π»Π»Π΅Π»ΠΎΠ³ΡΠ°ΠΌΠΌΠ° ΠΏΠΎΡΡΡΠΎΠ΅Π½Π½ΠΎΠ³ΠΎ Π½Π° Π²Π΅ΠΊΡΠΎΡΠ°Ρ
a ΠΈ b, ΠΏΠ΅ΡΠΏΠ΅Π½Π΄ΠΈΠΊΡΠ»ΡΡΠ½ΡΠΉ ΠΊ ΠΏΠ»ΠΎΡΠΊΠΎΡΡΠΈ ΡΡΠΈΡ
Π²Π΅ΠΊΡΠΎΡΠΎΠ² ΠΈ Π½Π°ΠΏΡΠ°Π²Π»Π΅Π½Π½ΡΠΉ ΡΠ°ΠΊ, ΡΡΠΎΠ± Π½Π°ΠΈΠΌΠ΅Π½ΡΡΠ΅Π΅ Π²ΡΠ°ΡΠ΅Π½ΠΈΠ΅ ΠΎΡ a ΠΊ b Π²ΠΎΠΊΡΡΠ³ Π²Π΅ΠΊΡΠΎΡΠ° c ΠΎΡΡΡΠ΅ΡΡΠ²Π»ΡΠ»ΠΎΡΡ ΠΏΡΠΎΡΠΈΠ² ΡΠ°ΡΠΎΠ²ΠΎΠΉ ΡΡΡΠ΅Π»ΠΊΠΈ, Π΅ΡΠ»ΠΈ ΡΠΌΠΎΡΡΠ΅ΡΡ Ρ ΠΊΠΎΠ½ΡΠ° Π²Π΅ΠΊΡΠΎΡΠ° c (ΡΠΈΡ. 1).
ΡΠΈΡ. 1 |
Π€ΠΎΡΠΌΡΠ»Ρ Π²ΡΡΠΈΡΠ»Π΅Π½ΠΈΡ Π²Π΅ΠΊΡΠΎΡΠ½ΠΎΠ³ΠΎ ΠΏΡΠΎΠΈΠ·Π²Π΅Π΄Π΅Π½ΠΈΡ Π²Π΅ΠΊΡΠΎΡΠΎΠ²
ΠΠ΅ΠΊΡΠΎΡΠ½ΠΎΠ΅ ΠΏΡΠΎΠΈΠ·Π²Π΅Π΄Π΅Π½ΠΈΠ΅ Π΄Π²ΡΡ Π²Π΅ΠΊΡΠΎΡΠΎΠ² a = {ax; ay; az} ΠΈ b = {bx; by; bz} Π² Π΄Π΅ΠΊΠ°ΡΡΠΎΠ²ΠΎΠΉ ΡΠΈΡΡΠ΅ΠΌΠ΅ ΠΊΠΎΠΎΡΠ΄ΠΈΠ½Π°Ρ β ΡΡΠΎ Π²Π΅ΠΊΡΠΎΡ, Π·Π½Π°ΡΠ΅Π½ΠΈΠ΅ ΠΊΠΎΡΠΎΡΠΎΠ³ΠΎ ΠΌΠΎΠΆΠ½ΠΎ Π²ΡΡΠΈΡΠ»ΠΈΡΡ, ΠΈΡΠΏΠΎΠ»ΡΠ·ΡΡ ΡΠ»Π΅Π΄ΡΡΡΠΈΠ΅ ΡΠΎΡΠΌΡΠ»Ρ:
a Γ b = | i | j | k | = i(aybz β azby) β j(axbz β azbx) + k(axby β aybx) |
ax | ay | az | ||
bx | by | bz |
a Γ b = {aybz β azby; azbx β axbz; axby β aybx}
Π‘Π²ΠΎΠΉΡΡΠ²Π° Π²Π΅ΠΊΡΠΎΡΠ½ΠΎΠ³ΠΎ ΠΏΡΠΎΠΈΠ·Π²Π΅Π΄Π΅Π½ΠΈΡ Π²Π΅ΠΊΡΠΎΡΠΎΠ²
ΠΠ΅ΠΎΠΌΠ΅ΡΡΠΈΡΠ΅ΡΠΊΠΈΠΉ ΡΠΌΡΡΠ» Π²Π΅ΠΊΡΠΎΡΠ½ΠΎΠ³ΠΎ ΠΏΡΠΎΠΈΠ·Π²Π΅Π΄Π΅Π½ΠΈΡ.
ΠΠΎΠ΄ΡΠ»Ρ Π²Π΅ΠΊΡΠΎΡΠ½ΠΎΠ³ΠΎ ΠΏΡΠΎΠΈΠ·Π²Π΅Π΄Π΅Π½ΠΈΡ Π΄Π²ΡΡ Π²Π΅ΠΊΡΠΎΡΠΎΠ² a ΠΈ b ΡΠ°Π²Π΅Π½ ΠΏΠ»ΠΎΡΠ°Π΄ΠΈ ΠΏΠ°ΡΠ°Π»Π»Π΅Π»ΠΎΠ³ΡΠ°ΠΌΠΌΠ° ΠΏΠΎΡΡΡΠΎΠ΅Π½Π½ΠΎΠ³ΠΎ Π½Π° ΡΡΠΈΡ Π²Π΅ΠΊΡΠΎΡΠ°Ρ :
SΠΏΠ°ΡΠ°Π» = [a Γ b]
ΠΠ΅ΠΎΠΌΠ΅ΡΡΠΈΡΠ΅ΡΠΊΠΈΠΉ ΡΠΌΡΡΠ» Π²Π΅ΠΊΡΠΎΡΠ½ΠΎΠ³ΠΎ ΠΏΡΠΎΠΈΠ·Π²Π΅Π΄Π΅Π½ΠΈΡ.
ΠΠ»ΠΎΡΠ°Π΄Ρ ΡΡΠ΅ΡΠ³ΠΎΠ»ΡΠ½ΠΈΠΊΠ° ΠΏΠΎΡΡΡΠΎΠ΅Π½Π½ΠΎΠ³ΠΎ Π½Π° Π²Π΅ΠΊΡΠΎΡΠ°Ρ a ΠΈ b ΡΠ°Π²Π½Π° ΠΏΠΎΠ»ΠΎΠ²ΠΈΠ½Π΅ ΠΌΠΎΠ΄ΡΠ»Ρ Π²Π΅ΠΊΡΠΎΡΠ½ΠΎΠ³ΠΎ ΠΏΡΠΎΠΈΠ·Π²Π΅Π΄Π΅Π½ΠΈΡ ΡΡΠΈΡ Π²Π΅ΠΊΡΠΎΡΠΎΠ²:
SΞ = 1 |a Γ b| 2 ΠΠ΅ΠΊΡΠΎΡΠ½ΠΎΠ΅ ΠΏΡΠΎΠΈΠ·Π²Π΅Π΄Π΅Π½ΠΈΡ Π΄Π²ΡΡ Π½Π΅ Π½ΡΠ»Π΅Π²ΡΡ Π²Π΅ΠΊΡΠΎΡΠΎΠ² a ΠΈ b ΡΠ°Π²Π½ΠΎ Π½ΡΠ»Ρ ΡΠΎΠ³Π΄Π° ΠΈ ΡΠΎΠ»ΡΠΊΠΎ ΡΠΎΠ³Π΄Π°, ΠΊΠΎΠ³Π΄Π° Π²Π΅ΠΊΡΠΎΡΠ° ΠΊΠΎΠ»Π»ΠΈΠ½Π΅Π°ΡΠ½Ρ.
ΠΠ΅ΠΊΡΠΎΡ c, ΡΠ°Π²Π½ΡΠΉ Π²Π΅ΠΊΡΠΎΡΠ½ΠΎΠΌΡ ΠΏΡΠΎΠΈΠ·Π²Π΅Π΄Π΅Π½ΠΈΡ Π½Π΅ Π½ΡΠ»Π΅Π²ΡΡ Π²Π΅ΠΊΡΠΎΡΠΎΠ² a ΠΈ b, ΠΏΠ΅ΡΠΏΠ΅Π½Π΄ΠΈΠΊΡΠ»ΡΡΠ΅Π½ ΡΡΠΈΠΌ Π²Π΅ΠΊΡΠΎΡΠ°ΠΌ.
a Γ b = -b Γ a
(k a) Γ b = a Γ (k b) = k (a Γ b)
(a + b) Γ c = a Γ c + b Γ c
ΠΡΠΈΠΌΠ΅ΡΡ Π·Π°Π΄Π°Ρ Π½Π° Π²ΡΡΠΈΡΠ»Π΅Π½ΠΈΡ Π²Π΅ΠΊΡΠΎΡΠ½ΠΎΠ³ΠΎ ΠΏΡΠΎΠΈΠ·Π²Π΅Π΄Π΅Π½ΠΈΡ Π²Π΅ΠΊΡΠΎΡΠΎΠ²
ΠΡΠΈΠΌΠ΅Ρ 1. ΠΠ°ΠΉΡΠΈ Π²Π΅ΠΊΡΠΎΡΠ½ΠΎΠ΅ ΠΏΡΠΎΠΈΠ·Π²Π΅Π΄Π΅Π½ΠΈΠ΅ Π²Π΅ΠΊΡΠΎΡΠΎΠ² a = {1; 2; 3} ΠΈ b = {2; 1; -2}.
Π Π΅ΡΠ΅Π½ΠΈΠ΅:
a Γ b = | i | j | k | = |
1 | 2 | 3 | ||
2 | 1 | -2 |
= i(2 Β· (-2) β 3 Β· 1) β j(1 Β· (-2) β 2 Β· 3) + k(1 Β· 1 β 2 Β· 2) =
= i(-4 β 3) β j(-2 β 6) + k(1 β 4) = -7i + 8j β 3k = {-7; 8; -3}
ΠΡΠΈΠΌΠ΅Ρ 2. ΠΠ°ΠΉΡΠΈ ΠΏΠ»ΠΎΡΠ°Π΄Ρ ΡΡΠ΅ΡΠ³ΠΎΠ»ΡΠ½ΠΈΠΊΠ° ΠΎΠ±ΡΠ°Π·ΠΎΠ²Π°Π½Π½ΠΎΠ³ΠΎ Π²Π΅ΠΊΡΠΎΡΠ°ΠΌΠΈ a = {-1; 2; -2} ΠΈ b = {2; 1; -1}.
Π Π΅ΡΠ΅Π½ΠΈΠ΅: ΠΠ°ΠΉΠ΄Π΅ΠΌ Π²Π΅ΠΊΡΠΎΡΠ½ΠΎΠ΅ ΠΏΡΠΎΠΈΠ·Π²Π΅Π΄Π΅Π½ΠΈΠ΅ ΡΡΠΈΡ Π²Π΅ΠΊΡΠΎΡΠΎΠ²:
a Γ b = | i | j | k | = |
-1 | 2 | -2 | ||
2 | 1 | -1 |
= i(2 Β· (-1) β (-2) Β· 1) β j((-1) Β· (-1) β (-2) Β· 2) + k((-1) Β· 1 β 2 Β· 2) =
= i(-2 + 2) β j(1 + 4) + k(-1 β 4) = -5j β 5k = {0; -5; -5}
ΠΠ· ΡΠ²ΠΎΠΉΡΡΠ² Π²Π΅ΠΊΡΠΎΡΠ½ΠΎΠ³ΠΎ ΠΏΡΠΎΠΈΠ·Π²Π΅Π΄Π΅Π½ΠΈΡ:
SΞ = | 1 | |a Γ b| = | 1 | β02 + 52 + 52 = | 1 | β25 + 25 = | 1 | β50 = | 5β2 |
2 | 2 | 2 | 2 | 2 |
ΠΡΠ²Π΅Ρ: SΞ = 2. 5β2.
ΠΠ΅ΠΊΡΠΎΡΠ° ΠΠ΅ΠΊΡΠΎΡ: ΠΎΠΏΡΠ΅Π΄Π΅Π»Π΅Π½ΠΈΠ΅ ΠΈ ΠΎΡΠ½ΠΎΠ²Π½ΡΠ΅ ΠΏΠΎΠ½ΡΡΠΈΡ ΠΠΏΡΠ΅Π΄Π΅Π»Π΅Π½ΠΈΠ΅ ΠΊΠΎΠΎΡΠ΄ΠΈΠ½Π°Ρ Π²Π΅ΠΊΡΠΎΡΠ° Π·Π°Π΄Π°Π½Π½ΠΎΠ³ΠΎ ΠΊΠΎΠΎΡΠ΄ΠΈΠ½Π°ΡΠ°ΠΌΠΈ Π΅Π³ΠΎ Π½Π°ΡΠ°Π»ΡΠ½ΠΎΠΉ ΠΈ ΠΊΠΎΠ½Π΅ΡΠ½ΠΎΠΉ ΡΠΎΡΠΊΠΈ ΠΠΎΠ΄ΡΠ»Ρ Π²Π΅ΠΊΡΠΎΡΠ°. ΠΠ»ΠΈΠ½Π° Π²Π΅ΠΊΡΠΎΡΠ° ΠΠ°ΠΏΡΠ°Π²Π»ΡΡΡΠΈΠ΅ ΠΊΠΎΡΠΈΠ½ΡΡΡ Π²Π΅ΠΊΡΠΎΡΠ° Π Π°Π²Π΅Π½ΡΡΠ²ΠΎ Π²Π΅ΠΊΡΠΎΡΠΎΠ² ΠΡΡΠΎΠ³ΠΎΠ½Π°Π»ΡΠ½ΠΎΡΡΡ Π²Π΅ΠΊΡΠΎΡΠΎΠ² ΠΠΎΠ»Π»ΠΈΠ½Π΅Π°ΡΠ½ΠΎΡΡΡ Π²Π΅ΠΊΡΠΎΡΠΎΠ² ΠΠΎΠΌΠΏΠ»Π°Π½Π°ΡΠ½ΠΎΡΡΡ Π²Π΅ΠΊΡΠΎΡΠΎΠ² Π£Π³ΠΎΠ» ΠΌΠ΅ΠΆΠ΄Ρ Π²Π΅ΠΊΡΠΎΡΠ°ΠΌΠΈ ΠΡΠΎΠ΅ΠΊΡΠΈΡ Π²Π΅ΠΊΡΠΎΡΠ° Π‘Π»ΠΎΠΆΠ΅Π½ΠΈΠ΅ ΠΈ Π²ΡΡΠΈΡΠ°Π½ΠΈΠ΅ Π²Π΅ΠΊΡΠΎΡΠΎΠ² Π£ΠΌΠ½ΠΎΠΆΠ΅Π½ΠΈΠ΅ Π²Π΅ΠΊΡΠΎΡΠ° Π½Π° ΡΠΈΡΠ»ΠΎ Π‘ΠΊΠ°Π»ΡΡΠ½ΠΎΠ΅ ΠΏΡΠΎΠΈΠ·Π²Π΅Π΄Π΅Π½ΠΈΠ΅ Π²Π΅ΠΊΡΠΎΡΠΎΠ² ΠΠ΅ΠΊΡΠΎΡΠ½ΠΎΠ΅ ΠΏΡΠΎΠΈΠ·Π²Π΅Π΄Π΅Π½ΠΈΠ΅ Π²Π΅ΠΊΡΠΎΡΠΎΠ² Π‘ΠΌΠ΅ΡΠ°Π½Π½ΠΎΠ΅ ΠΏΡΠΎΠΈΠ·Π²Π΅Π΄Π΅Π½ΠΈΠ΅ Π²Π΅ΠΊΡΠΎΡΠΎΠ² ΠΠΈΠ½Π΅ΠΉΠ½ΠΎ Π·Π°Π²ΠΈΡΠΈΠΌΡΠ΅ ΠΈ Π»ΠΈΠ½Π΅ΠΉΠ½ΠΎ Π½Π΅Π·Π°Π²ΠΈΡΠΈΠΌΡΠ΅ Π²Π΅ΠΊΡΠΎΡΠ° Π Π°Π·Π»ΠΎΠΆΠ΅Π½ΠΈΠ΅ Π²Π΅ΠΊΡΠΎΡΠ° ΠΏΠΎ Π±Π°Π·ΠΈΡΡ
ΠΠ½Π»Π°ΠΉΠ½ ΠΊΠ°Π»ΡΠΊΡΠ»ΡΡΠΎΡΡ Ρ Π²Π΅ΠΊΡΠΎΡΠ°ΠΌΠΈ
ΠΠ½Π»Π°ΠΉΠ½ ΡΠΏΡΠ°ΠΆΠ½Π΅Π½ΠΈΡ Ρ Π²Π΅ΠΊΡΠΎΡΠ°ΠΌΠΈ Π½Π° ΠΏΠ»ΠΎΡΠΊΠΎΡΡΠΈ
ΠΠ½Π»Π°ΠΉΠ½ ΡΠΏΡΠ°ΠΆΠ½Π΅Π½ΠΈΡ Ρ Π²Π΅ΠΊΡΠΎΡΠ°ΠΌΠΈ Π² ΠΏΡΠΎΡΡΡΠ°Π½ΡΡΠ²Π΅
ΠΡΠ±ΡΠ΅ Π½Π΅ΡΠ΅Π½Π·ΡΡΠ½ΡΠ΅ ΠΊΠΎΠΌΠΌΠ΅Π½ΡΠ°ΡΠΈΠΈ Π±ΡΠ΄ΡΡ ΡΠ΄Π°Π»Π΅Π½Ρ, Π° ΠΈΡ Π°Π²ΡΠΎΡΡ Π·Π°Π½Π΅ΡΠ΅Π½Ρ Π² ΡΠ΅ΡΠ½ΡΠΉ ΡΠΏΠΈΡΠΎΠΊ!
13. ΠΠ΅ΠΊΡΠΎΡΠ½ΠΎΠ΅ ΠΏΡΠΎΠΈΠ·Π²Π΅Π΄Π΅Π½ΠΈΠ΅ Π²Π΅ΠΊΡΠΎΡΠΎΠ², Π΅Π³ΠΎ ΡΠ²ΠΎΠΉΡΡΠ²Π°. ΠΠ»ΠΎΡΠ°Π΄Ρ ΠΏΠ°ΡΠ°Π»Π»Π΅Π»ΠΎΠ³ΡΠ°ΠΌΠΌΠ°.
ΠΠ΅ΠΊΡΠΎΡΠ½ΡΠΌ
ΠΏΡΠΎΠΈΠ·Π²Π΅Π΄Π΅Π½ΠΈΠ΅ΠΌΠ²Π΅ΠΊΡΠΎΡΠ° a Π½Π° Π²Π΅ΠΊΡΠΎΡ b
Π½Π°Π·ΡΠ²Π°Π΅ΡΡΡ Π²Π΅ΠΊΡΠΎΡ c, Π΄Π»ΠΈΠ½Π° ΠΊΠΎΡΠΎΡΠΎΠ³ΠΎ
ΡΠΈΡΠ»Π΅Π½Π½ΠΎ ΡΠ°Π²Π½Π° ΠΏΠ»ΠΎΡΠ°Π΄ΠΈ ΠΏΠ°ΡΠ°Π»Π»Π΅Π»ΠΎΠ³ΡΠ°ΠΌΠΌΠ°
ΠΏΠΎΡΡΡΠΎΠ΅Π½Π½ΠΎΠ³ΠΎ Π½Π° Π²Π΅ΠΊΡΠΎΡΠ°Ρ
a ΠΈ b,
ΠΏΠ΅ΡΠΏΠ΅Π½Π΄ΠΈΠΊΡΠ»ΡΡΠ½ΡΠΉ ΠΊ ΠΏΠ»ΠΎΡΠΊΠΎΡΡΠΈ ΡΡΠΈΡ
Π²Π΅ΠΊΡΠΎΡΠΎΠ² ΠΈ Π½Π°ΠΏΡΠ°Π²Π»Π΅Π½Π½ΡΠΉ ΡΠ°ΠΊ, ΡΡΠΎΠ±
Π½Π°ΠΈΠΌΠ΅Π½ΡΡΠ΅Π΅ Π²ΡΠ°ΡΠ΅Π½ΠΈΠ΅ ΠΎΡ a ΠΊ b Π²ΠΎΠΊΡΡΠ³
Π²Π΅ΠΊΡΠΎΡΠ° c ΠΎΡΡΡΠ΅ΡΡΠ²Π»ΡΠ»ΠΎΡΡ ΠΏΡΠΎΡΠΈΠ² ΡΠ°ΡΠΎΠ²ΠΎΠΉ
ΡΡΡΠ΅Π»ΠΊΠΈ, Π΅ΡΠ»ΠΈ ΡΠΌΠΎΡΡΠ΅ΡΡ Ρ ΠΊΠΎΠ½ΡΠ° Π²Π΅ΠΊΡΠΎΡΠ°
c (ΡΠΈΡ. 1).
ΡΠΈΡ. 1 |
Π€ΠΎΡΠΌΡΠ»Ρ Π²ΡΡΠΈΡΠ»Π΅Π½ΠΈΡ Π²Π΅ΠΊΡΠΎΡΠ½ΠΎΠ³ΠΎ ΠΏΡΠΎΠΈΠ·Π²Π΅Π΄Π΅Π½ΠΈΡ Π²Π΅ΠΊΡΠΎΡΠΎΠ²
ΠΠ΅ΠΊΡΠΎΡΠ½ΠΎΠ΅ ΠΏΡΠΎΠΈΠ·Π²Π΅Π΄Π΅Π½ΠΈΠ΅Π΄Π²ΡΡ Π²Π΅ΠΊΡΠΎΡΠΎΠ² a = {ax; ay; az} ΠΈ b = {bx; by; bz} Π² Π΄Π΅ΠΊΠ°ΡΡΠΎΠ²ΠΎΠΉ ΡΠΈΡΡΠ΅ΠΌΠ΅ ΠΊΠΎΠΎΡΠ΄ΠΈΠ½Π°Ρ β ΡΡΠΎ Π²Π΅ΠΊΡΠΎΡ, Π·Π½Π°ΡΠ΅Π½ΠΈΠ΅ ΠΊΠΎΡΠΎΡΠΎΠ³ΠΎ ΠΌΠΎΠΆΠ½ΠΎ Π²ΡΡΠΈΡΠ»ΠΈΡΡ, ΠΈΡΠΏΠΎΠ»ΡΠ·ΡΡ ΡΠ»Π΅Π΄ΡΡΡΠΈΠ΅ ΡΠΎΡΠΌΡΠ»Ρ:
a Γ b = | i | j | k | = i(aybz β azby) β j(axbz β azbx) + k(axby β aybx) |
ax | ay | az | ||
bx | by | bz |
a Γ b = {aybzβ azby; azbxβ axbz; axbyβ aybx}
Π‘Π²ΠΎΠΉΡΡΠ²Π° Π²Π΅ΠΊΡΠΎΡΠ½ΠΎΠ³ΠΎ ΠΏΡΠΎΠΈΠ·Π²Π΅Π΄Π΅Π½ΠΈΡ Π²Π΅ΠΊΡΠΎΡΠΎΠ²
ΠΠΎΠ΄ΡΠ»Ρ Π²Π΅ΠΊΡΠΎΡΠ½ΠΎΠ³ΠΎ ΠΏΡΠΎΠΈΠ·Π²Π΅Π΄Π΅Π½ΠΈΡ Π΄Π²ΡΡ Π²Π΅ΠΊΡΠΎΡΠΎΠ² a ΠΈ b ΡΠ°Π²Π΅Π½ ΠΏΠ»ΠΎΡΠ°Π΄ΠΈ ΠΏΠ°ΡΠ°Π»Π»Π΅Π»ΠΎΠ³ΡΠ°ΠΌΠΌΠ° ΠΏΠΎΡΡΡΠΎΠ΅Π½Π½ΠΎΠ³ΠΎ Π½Π° ΡΡΠΈΡ Π²Π΅ΠΊΡΠΎΡΠ°Ρ :
SΠΏΠ°ΡΠ°Π»= a Γ b]
ΠΠ»ΠΎΡΠ°Π΄Ρ ΡΡΠ΅ΡΠ³ΠΎΠ»ΡΠ½ΠΈΠΊΠ° ΠΏΠΎΡΡΡΠΎΠ΅Π½Π½ΠΎΠ³ΠΎ Π½Π° Π²Π΅ΠΊΡΠΎΡΠ°Ρ a ΠΈ b ΡΠ°Π²Π½Π° ΠΏΠΎΠ»ΠΎΠ²ΠΈΠ½Π΅ ΠΌΠΎΠ΄ΡΠ»Ρ Π²Π΅ΠΊΡΠΎΡΠ½ΠΎΠ³ΠΎ ΠΏΡΠΎΠΈΠ·Π²Π΅Π΄Π΅Π½ΠΈΡ ΡΡΠΈΡ Π²Π΅ΠΊΡΠΎΡΠΎΠ²:
SΞ = | 1 | |a Γ b| |
2 |
ΠΠ΅ΠΊΡΠΎΡΠ½ΠΎΠ΅ ΠΏΡΠΎΠΈΠ·Π²Π΅Π΄Π΅Π½ΠΈΡ Π΄Π²ΡΡ Π½Π΅ Π½ΡΠ»Π΅Π²ΡΡ Π²Π΅ΠΊΡΠΎΡΠΎΠ² a ΠΈ b ΡΠ°Π²Π½ΠΎ Π½ΡΠ»Ρ ΡΠΎΠ³Π΄Π° ΠΈ ΡΠΎΠ»ΡΠΊΠΎ ΡΠΎΠ³Π΄Π°, ΠΊΠΎΠ³Π΄Π° Π²Π΅ΠΊΡΠΎΡΠ° ΠΊΠΎΠ»Π»ΠΈΠ½Π΅Π°ΡΠ½Ρ.
ΠΠ΅ΠΊΡΠΎΡ c, ΡΠ°Π²Π½ΡΠΉ Π²Π΅ΠΊΡΠΎΡΠ½ΠΎΠΌΡ ΠΏΡΠΎΠΈΠ·Π²Π΅Π΄Π΅Π½ΠΈΡ Π½Π΅ Π½ΡΠ»Π΅Π²ΡΡ Π²Π΅ΠΊΡΠΎΡΠΎΠ² a ΠΈ b, ΠΏΠ΅ΡΠΏΠ΅Π½Π΄ΠΈΠΊΡΠ»ΡΡΠ΅Π½ ΡΡΠΈΠΌ Π²Π΅ΠΊΡΠΎΡΠ°ΠΌ.
a Γ b = -b Γ a
(k a) Γ b = a Γ (k b) = k (a Γ b)
(a + b) Γ c = a Γ c + b Γ c
14. Π‘ΠΌΠ΅ΡΠ°Π½Π½ΠΎΠ΅ ΠΏΡΠΎΠΈΠ·Π²Π΅Π΄Π΅Π½ΠΈΠ΅ Π²Π΅ΠΊΡΠΎΡΠΎΠ², Π΅Π³ΠΎ ΡΠ²ΠΎΠΉΡΡΠ²Π°. Π£ΡΠ»ΠΎΠ²ΠΈΠ΅ ΠΊΠΎΠΌΠΏΠ»Π°Π½Π°ΡΠ½ΠΎΡΡΠΈ Π²Π΅ΠΊΡΠΎΡΠ°. ΠΠ±ΡΠ΅ΠΌ ΠΏΠ°ΡΠ°Π»Π»Π΅Π»Π΅ΠΏΠΈΠΏΠ΅Π΄Π°. ΠΠ±ΡΡΠΌ ΠΏΠΈΡΠ°ΠΌΠΈΠ΄Ρ.
Π‘ΠΌΠ΅ΡΠ°Π½Π½ΡΠΌ ΠΏΡΠΎΠΈΠ·Π²Π΅Π΄Π΅Π½ΠΈΠ΅ΠΌ Π½Π΅ΠΊΠΎΠΌΠΏΠ»Π°Π½Π°ΡΠ½ΡΡ Π²Π΅ΠΊΡΠΎΡΠΎΠ²,Π²Π·ΡΡΡΡ Π² Π΄Π°Π½Π½ΠΎΠΌ ΠΏΠΎΡΡΠ΄ΠΊΠ΅, Π½Π°Π·ΡΠ²Π°Π΅ΡΡΡΠΎΠ±ΡΡΠΌ ΠΏΠ°ΡΠ°Π»Π»Π΅Π»Π΅ΠΏΠΈΠΏΠ΅Π΄Π°, ΠΏΠΎΡΡΡΠΎΠ΅Π½Π½ΠΎΠ³ΠΎ Π½Π° Π΄Π°Π½Π½ΡΡ Π²Π΅ΠΊΡΠΎΡΠ°Ρ , ΡΠ½Π°Π±ΠΆΡΠ½Π½ΡΠΉ Π·Π½Π°ΠΊΠΎΠΌ Β«+Β», Π΅ΡΠ»ΠΈ Π±Π°Π·ΠΈΡΠΏΡΠ°Π²ΡΠΉ, ΠΈ Π·Π½Π°ΠΊΠΎΠΌ Β«βΒ», Π΅ΡΠ»ΠΈ Π±Π°Π·ΠΈΡΠ»Π΅Π²ΡΠΉ.
1. Π‘ΠΌΠ΅ΡΠ°Π½Π½ΠΎΠ΅ ΠΏΡΠΎΠΈΠ·Π²Π΅Π΄Π΅Π½ΠΈΠ΅ Π½Π΅ ΠΌΠ΅Π½ΡΠ΅ΡΡΡ ΠΏΡΠΈ ΡΠΈΠΊΠ»ΠΈΡΠ΅ΡΠΊΠΎΠΉ ΠΏΠ΅ΡΠ΅ΡΡΠ°Π½ΠΎΠ²ΠΊΠ΅ Π΅Π³ΠΎ ΡΠΎΠΌΠ½ΠΎΠΆΠΈΡΠ΅Π»Π΅ΠΉ (Π½Π΅ ΠΌΠ΅Π½ΡΠ΅ΡΡΡ Π½ΠΈ ΠΎΠ±ΡΠ΅ΠΌ ΠΏΠ°ΡΠ°Π»Π»Π΅Π»Π΅ΠΏΠΈΠΏΠ΅Π΄Π°, Π½ΠΈ ΠΎΡΠΈΠ΅Π½ΡΠ°ΡΠΈΡ Π΅Π³ΠΎ ΡΠ΅Π±Π΅Ρ): .
2. Π‘ΠΌΠ΅ΡΠ°Π½Π½ΠΎΠ΅ ΠΏΡΠΎΠΈΠ·Π²Π΅Π΄Π΅Π½ΠΈΠ΅ Π½Π΅ ΠΌΠ΅Π½ΡΠ΅ΡΡΡΠ·Π½Π°ΠΊΠΎΠ² Π²Π΅ΠΊΡΠΎΡΠ½ΠΎΠ³ΠΎ ΠΈ ΡΠΊΠ°Π»ΡΡΠ½ΠΎΠ³ΠΎ ΡΠΌΠ½ΠΎΠΆΠ΅Π½ΠΈΡ:, ΠΏΠΎΡΡΠΎΠΌΡ ΡΠΌΠ΅ΡΠ°Π½Π½ΠΎΠ΅ ΠΏΡΠΎΠΈΠ·Π²Π΅Π΄Π΅Π½ΠΈΠ΅ Π·Π°ΠΏΠΈΡΡΠ²Π°ΡΡ.
3.
Π‘ΠΌΠ΅ΡΠ°Π½Π½ΠΎΠ΅
ΠΏΡΠΎΠΈΠ·Π²Π΅Π΄Π΅Π½ΠΈΠ΅ ΠΌΠ΅Π½ΡΠ΅Ρ ΡΠ²ΠΎΠΉ Π·Π½Π°ΠΊ ΠΏΡΠΈ
ΠΏΠ΅ΡΠ΅ΠΌΠ΅Π½Π΅ Π»ΡΠ±ΡΡ
Π΄Π²ΡΡ
Π²Π΅ΠΊΡΠΎΡ-ΡΠΎΠΌΠ½ΠΎΠΆΠΈΡΠ΅Π»Π΅ΠΉ:
,.
4. Π‘ΠΌΠ΅ΡΠ°Π½Π½ΠΎΠ΅ ΠΏΡΠΎΠΈΠ·Π²Π΅Π΄Π΅Π½ΠΈΠ΅ Π½Π΅Π½ΡΠ»Π΅Π²ΡΡ Π²Π΅ΠΊΡΠΎΡΠΎΠ² ,ΠΈΡΠ°Π²Π½ΠΎ Π½ΡΠ»Ρ ΡΠΎΠ³Π΄Π° ΠΈ ΡΠΎΠ»ΡΠΊΠΎ ΡΠΎΠ³Π΄Π°, ΠΊΠΎΠ³Π΄Π° ΠΎΠ½ΠΈ ΠΊΠΎΠΌΠΏΠ»Π°Π½Π°ΡΠ½Ρ:,, β ΠΊΠΎΠΌΠΏΠ»Π°Π½Π°ΡΠ½Ρ.
ΠΠΎΠΊΠ°Π·Π°ΡΠ΅Π»ΡΡΡΠ²ΠΎ. ΠΡΠ΅Π΄ΠΏΠΎΠ»ΠΎΠΆΠΈΠΌ, ΡΡΠΎ Π²Π΅ΠΊΡΠΎΡΡ ,ΠΈβ Π½Π΅ ΠΊΠΎΠΌΠΏΠ»Π°Π½Π°ΡΠ½Ρ. Π’ΠΎΠ³Π΄Π° ΠΌΠΎΠΆΠ½ΠΎ ΠΏΠΎΡΡΡΠΎΠΈΡΡ ΠΏΠ°ΡΠ°Π»Π»Π΅Π»Π΅ΠΏΠΈΠΏΠ΅Π΄ ΠΈΠΌΠ΅ΡΡΠΈΠΉ ΠΎΠ±ΡΠ΅ΠΌ, Ρ.Π΅., Π½ΠΎ ΡΡΠΎ ΠΏΡΠΎΡΠΈΠ²ΠΎΡΠ΅ΡΠΈΡ ΡΡΠ»ΠΎΠ²ΠΈΡ, ΡΠΎΠ³Π»Π°ΡΠ½ΠΎ ΠΊΠΎΡΠΎΡΠΎΠ³ΠΎ,. Π‘Π»Π΅Π΄ΠΎΠ²Π°ΡΠ΅Π»ΡΠ½ΠΎ, Π²Π΅ΠΊΡΠΎΡΡ,ΠΈβ ΠΊΠΎΠΌΠΏΠ»Π°Π½Π°ΡΠ½Ρ.
ΠΠ±ΡΠ°ΡΠ½ΠΎ, ΠΏΡΡΡΡ ,ΠΈβ ΠΊΠΎΠΌΠΏΠ»Π°Π½Π°ΡΠ½Ρ. Π’ΠΎΠ³Π΄Π° Π²Π΅ΠΊΡΠΎΡΠΈ ΠΏΠ΅ΡΠΏΠ΅Π½Π΄ΠΈΠΊΡΠ»ΡΡΠ΅Π½ ΠΏΠ»ΠΎΡΠΊΠΎΡΡΠΈ, Π² ΠΊΠΎΡΠΎΡΠΎΠΉ Π½Π°Ρ ΠΎΠ΄ΡΡΡΡ Π²Π΅ΠΊΡΠΎΡΡ,ΠΈ, Π·Π½Π°ΡΠΈΡ, ΠΎΠ½ ΠΏΠ΅ΡΠΏΠ΅Π½Π΄ΠΈΠΊΡΠ»ΡΡΠ΅Π½ Π»ΡΠ±ΠΎΠΌΡ Π²Π΅ΠΊΡΠΎΡΡ, Π»Π΅ΠΆΠ°ΡΠ΅ΠΌΡ Π² ΡΡΠΎΠΉ ΠΏΠ»ΠΎΡΠΊΠΎΡΡΠΈ, Π½Π°ΠΏΡΠΈΠΌΠ΅ΡΠΡΠΎ Π·Π½Π°ΡΠΈΡ, ΡΡΠΎ .
Π‘ΠΌΠ΅ΡΠ°Π½Π½ΠΎΠ΅ ΠΏΡΠΎΠΈΠ·Π²Π΅Π΄Π΅Π½ΠΈΠ΅ Π²Π΅ΠΊΡΠΎΡΠΎΠ², Π·Π°Π΄Π°Π½Π½ΡΡ ΡΠ²ΠΎΠΈΠΌΠΈ ΠΏΡΠΎΠ΅ΠΊΡΠΈΡΠΌΠΈ Π² Π΄Π΅ΠΊΠ°ΡΡΠΎΠ²ΠΎΠΉ ΡΠΈΡΡΠ΅ΠΌΠ΅ ΠΊΠΎΠΎΡΠ΄ΠΈΠ½Π°Ρ.
ΠΡΡΡΡ Π²Π΅ΠΊΡΠΎΡΡ Π·Π°Π΄Π°Π½Ρ ΡΠ²ΠΎΠΈΠΌΠΈ ΡΠ°Π·Π»ΠΎΠΆΠ΅Π½ΠΈΡΠΌΠΈ ΠΏΠΎ ΠΎΡΡΠ°ΠΌ Π² Π΄Π΅ΠΊΠ°ΡΡΠΎΠ²ΠΎΠΉ ΡΠΈΡΡΠ΅ΠΌΠ΅ ΠΊΠΎΠΎΡΠ΄ΠΈΠ½Π°Ρ:
, ΠΈ.
ΠΠ°ΠΉΠ΄Π΅ΠΌ ΠΈΡ ΡΠΌΠ΅ΡΠ°Π½Π½ΠΎΠ΅ ΠΏΡΠΎΠΈΠ·Π²Π΅Π΄Π΅Π½ΠΈΠ΅, ΠΈΡΠΏΠΎΠ»ΡΠ·ΡΡ Π²ΡΡΠ°ΠΆΠ΅Π½ΠΈΡ Π² ΠΊΠΎΠΎΡΠ΄ΠΈΠ½Π°ΡΠ°Ρ Π΄Π»Ρ Π²Π΅ΠΊΡΠΎΡΠ½ΠΎΠ³ΠΎ ΠΈ ΡΠΊΠ°Π»ΡΡΠ½ΠΎΠ³ΠΎ ΠΏΡΠΎΠΈΠ·Π²Π΅Π΄Π΅Π½ΠΈΠΉ:
.
ΠΡΠ°ΠΊ,
.
ΠΡΠΈΠ»ΠΎΠΆΠ΅Π½ΠΈΡ ΡΠΌΠ΅ΡΠ°Π½Π½ΠΎΠ³ΠΎ ΠΏΡΠΎΠΈΠ·Π²Π΅Π΄Π΅Π½ΠΈΡ:
1. ΠΠΏΡΠ΅Π΄Π΅Π»Π΅Π½ΠΈΠ΅ Π²Π·Π°ΠΈΠΌΠ½ΠΎΠΉ ΠΎΡΠΈΠ΅Π½ΡΠ°ΡΠΈΠΈ Π²Π΅ΠΊΡΠΎΡΠΎΠ² Π² ΠΏΡΠΎΡΡΡΠ°Π½ΡΡΠ²Π΅.
ΠΡΠ»ΠΈ ,ΠΈβ ΠΏΡΠ°Π²Π°Ρ ΡΡΠΎΠΉΠΊΠ°, Π΅ΡΠ»ΠΈΠ»Π΅Π²Π°Ρ.
2. Π£ΡΡΠ°Π½ΠΎΠ²Π»Π΅Π½ΠΈΠ΅ ΠΊΠΎΠΌΠΏΠ»Π°Π½Π°ΡΠ½ΠΎΡΡΠΈ Π²Π΅ΠΊΡΠΎΡΠΎΠ²:
( ο (,, β ΠΊΠΎΠΌΠΏΠ»Π°Π½Π°ΡΠ½Ρ).
3. ΠΠΏΡΠ΅Π΄Π΅Π»Π΅Π½ΠΈΠ΅ ΠΎΠ±ΡΠ΅ΠΌΠ° ΠΏΠ°ΡΠ°Π»Π»Π΅Π»Π΅ΠΏΠΈΠΏΠ΅Π΄Π° ΠΈ ΡΡΠ΅ΡΠ³ΠΎΠ»ΡΠ½ΠΎΠΉ ΠΏΠΈΡΠ°ΠΌΠΈΠ΄Ρ (ΡΠ΅ΡΡΠ°ΡΠ΄ΡΠ°):
, .
ΠΡΠΈΠΌΠ΅Ρ. ΠΠΎΠΌΠΏΠ»Π°Π½Π°ΡΠ½Ρ Π»ΠΈ Π²Π΅ΠΊΡΠΎΡΡ ,ΠΈ, Π΅ΡΠ»ΠΈ .
Π Π΅ΡΠ΅Π½ΠΈΠ΅. ΠΡΡΠΈΡΠ»ΠΈΠΌ ΡΠΌΠ΅ΡΠ°Π½Π½ΠΎΠ΅ ΠΏΡΠΎΠΈΠ·Π²Π΅Π΄Π΅Π½ΠΈΠ΅ Π²Π΅ΠΊΡΠΎΡΠΎΠ²:
Π²Π΅ΠΊΡΠΎΡΡ ,ΠΈΠ½Π΅ ΠΊΠΎΠΌΠΏΠ»Π°Π½Π°ΡΠ½Ρ.
ΠΡΠΈΠΌΠ΅Ρ. ΠΠΎΠΊΠ°Π·Π°ΡΡ, ΡΡΠΎ Π²Π΅ΠΊΡΠΎΡΡ ,ΠΈΠΊΠΎΠΌΠΏΠ»Π°Π½Π°ΡΠ½Ρ.
Π Π΅ΡΠ΅Π½ΠΈΠ΅. Π Π°ΡΡΠΌΠΎΡΡΠΈΠΌ ΠΌΠ°ΡΡΠΈΡΡ, ΡΠΎΡΡΠ°Π²Π»Π΅Π½Π½ΡΡ ΠΈΠ· ΠΊΠΎΠΎΡΠ΄ΠΈΠ½Π°Ρ Π²Π΅ΠΊΡΠΎΡΠΎΠ² ,ΠΈ
,
Ρ. ΠΊ. ΠΎΠΏΡΠ΅Π΄Π΅Π»ΠΈΡΠ΅Π»Ρ ΠΌΠ°ΡΡΠΈΡΡ ΡΠ°Π²Π΅Π½ Π½ΡΠ»Ρ,
ΡΠΎ Π²Π΅ΠΊΡΠΎΡΡ Π»ΠΈΠ½Π΅ΠΉΠ½ΠΎ Π·Π°Π²ΠΈΡΠΈΠΌΡ, ΡΠ»Π΅Π΄ΠΎΠ²Π°ΡΠ΅Π»ΡΠ½ΠΎ
ΠΎΠ½ΠΈ ΠΊΠΎΠΌΠΏΠ»Π°Π½Π°ΡΠ½Ρ.
ΠΡΠΈΠΌΠ΅Ρ. ΠΡΡΠΈΡΠ»ΠΈΡΡ ΠΎΠ±ΡΠ΅ΠΌ ΡΠ΅ΡΡΠ°ΡΠ΄ΡΠ° Ρ Π²Π΅ΡΡΠΈΠ½Π°ΠΌΠΈ Π² ΡΠΎΡΠΊΠ°Ρ ΠΈ Π΅Π³ΠΎ Π²ΡΡΠΎΡΡ, ΠΎΠΏΡΡΠ΅Π½Π½ΡΡ ΠΈΠ· Π²Π΅ΡΡΠΈΠ½ΡΠ½Π° Π³ΡΠ°Π½Ρ, Π΅ΡΠ»ΠΈ
Π Π΅ΡΠ΅Π½ΠΈΠ΅. ΠΠ°ΠΉΠ΄Π΅ΠΌ ΠΊΠΎΠΎΡΠ΄ΠΈΠ½Π°ΡΡ Π²Π΅ΠΊΡΠΎΡΠΎΠ²:
, ,.
ΠΡΡΠΈΡΠ»ΠΈΠΌ ΠΎΠ±ΡΠ΅ΠΌ:
.
ΠΠΎΡΠΊΠΎΠ»ΡΠΊΡ ΠΎΠ±ΡΠ΅ΠΌ ΡΠ΅ΡΡΠ°ΡΠ΄ΡΠ° , ΡΠΎ Π²ΡΡΠΎΡΠ°.
ΠΡΡΠΈΡΠ»ΠΈΠΌ ΠΏΠ»ΠΎΡΠ°Π΄Ρ ΠΎΡΠ½ΠΎΠ²Π°Π½ΠΈΡ ΡΠ΅ΡΡΠ°ΡΠ΄ΡΠ°
.
ΠΡΠ°ΠΊ, Π²ΡΡΠΎΡΠ° .
ΠΠ½ΠΎΠ³ΠΎΠΌΠ΅ΡΠ½ΠΎΠ΅ ΠΈΡΡΠΈΡΠ»Π΅Π½ΠΈΠ΅. ΠΠΎΡΠ΅ΠΌΡ Π²Π΅Π»ΠΈΡΠΈΠ½Π° ΠΏΠ΅ΡΠ΅ΠΊΡΠ΅ΡΡΠ½ΠΎΠ³ΠΎ ΠΏΡΠΎΠΈΠ·Π²Π΅Π΄Π΅Π½ΠΈΡ a ΠΈ b Π΄Π°Π΅Ρ ΠΏΠ»ΠΎΡΠ°Π΄Ρ ΠΏΠ°ΡΠ°Π»Π»Π΅Π»ΠΎΠ³ΡΠ°ΠΌΠΌΠ°, Π½Π°ΡΡΠ½ΡΡΠΎΠ³ΠΎ Π½Π° a ΠΈ b?
ΡΠΏΡΠΎΡΠΈΠ»
ΠΠ·ΠΌΠ΅Π½Π΅Π½ΠΎ 11 ΠΌΠ΅ΡΡΡΠ΅Π² Π½Π°Π·Π°Π΄
ΠΡΠΎΡΠΌΠΎΡΡΠ΅Π½ΠΎ 24ΠΊ ΡΠ°Π·
$\begingroup$
Π― ΠΏΡΡΠ°Π»ΡΡ Π½Π°ΠΉΡΠΈ ΡΡΠΎ, Π½ΠΎ ΠΌΠ½ΠΎΠ³ΠΈΠ΅ Π²Π΅Π±-ΡΠ°ΠΉΡΡ ΠΏΡΠΎΡΡΠΎ ΡΠΊΠ°Π·ΡΠ²Π°ΡΡ ΡΡΠΎ Π±Π΅Π· Π΄ΠΎΠΊΠ°Π·Π°ΡΠ΅Π»ΡΡΡΠ² ΠΈ Π±Π΅Π· ΠΈΠ½ΡΡΠΈΡΠΈΠΈ. Π― Π½Π°Π΄Π΅ΡΡΡ ΠΈΠ·ΡΡΠΈΡΡ Π΅Π³ΠΎ Π½Π΅ΠΌΠ½ΠΎΠ³ΠΎ Π»ΡΡΡΠ΅, ΡΡΠΎΠ±Ρ Π½Π΅ Π·Π°Π±ΡΡΡ, ΠΊΠ°ΠΊ Π²ΡΡΠΈΡΠ»ΡΡΡ ΡΠΊΠΎΠ±ΠΈΠ°Π½ ΠΏΡΠΈ ΡΠ°Π±ΠΎΡΠ΅ Ρ ΠΏΠΎΠ²Π΅ΡΡ
Π½ΠΎΡΡΠ½ΡΠΌΠΈ ΠΈΠ½ΡΠ΅Π³ΡΠ°Π»Π°ΠΌΠΈ, Π³Π΄Π΅ ΡΠ΅ΠΎΡΠ΅ΠΌΠ° ΠΎ Π΄ΠΈΠ²Π΅ΡΠ³Π΅Π½ΡΠΈΠΈ Π½Π΅ΠΏΡΠΈΠΌΠ΅Π½ΠΈΠΌΠ°.
ΠΡΠ»ΠΈ Ρ Π²Π°Ρ Π΅ΡΡΡ Ρ ΠΎΡΠΎΡΠΈΠΉ ΠΎΠ½Π»Π°ΠΉΠ½-ΡΠΏΡΠ°Π²ΠΎΡΠ½ΠΈΠΊ, ΠΏΠΎΠΆΠ°Π»ΡΠΉΡΡΠ°, ΠΏΡΠ΅Π΄ΠΎΡΡΠ°Π²ΡΡΠ΅ Π΅Π³ΠΎ π
Π‘ΠΏΠ°ΡΠΈΠ±ΠΎ,
- ΠΌΠ½ΠΎΠ³ΠΎΠΌΠ΅ΡΠ½ΠΎΠ΅ ΠΈΡΡΠΈΡΠ»Π΅Π½ΠΈΠ΅
- ΠΏΠ»ΠΎΡΠ°Π΄Ρ
- ΠΏΠ΅ΡΠ΅ΠΊΡΠ΅ΡΡΠ½ΠΎΠ΅ ΠΏΡΠΎΠΈΠ·Π²Π΅Π΄Π΅Π½ΠΈΠ΅
- ΠΏΠΎΠ²Π΅ΡΡ Π½ΠΎΡΡΠ½ΡΠ΅ ΠΈΠ½ΡΠ΅Π³ΡΠ°Π»Ρ
$\endgroup$
1
$\begingroup$
ΠΡΠ±Π΅ΡΠΈΡΠ΅ ΠΊΠΎΠΎΡΠ΄ΠΈΠ½Π°ΡΡ ΡΠ°ΠΊ, ΡΡΠΎΠ±Ρ Π΄Π²Π° Π²Π΅ΠΊΡΠΎΡΠ° $\vec a, \vec b$ Π»Π΅ΠΆΠ°Π»ΠΈ Π² ΠΏΠ»ΠΎΡΠΊΠΎΡΡΠΈ $xy$, Π° $\vec a$ Π²Π΄ΠΎΠ»Ρ ΠΎΡΠΈ $x$. (ΠΠ±ΡΠ°ΡΠΈΡΠ΅ Π²Π½ΠΈΠΌΠ°Π½ΠΈΠ΅, ΡΡΠΎ ΠΏΠΎΠΊΠ° Π²Ρ Π²ΡΠ±ΡΠ°Π»ΠΈ Π΅Π΄ΠΈΠ½ΠΈΡΡ Π΄Π»ΠΈΠ½Ρ , ΡΠΎΡΠ½ΠΎΠ΅ Π½Π°ΠΏΡΠ°Π²Π»Π΅Π½ΠΈΠ΅, ΠΊΠΎΡΠΎΡΠΎΠ΅ Π²Ρ Π²ΡΠ±Π΅ΡΠ΅ΡΠ΅ Π΄Π»Ρ ΠΎΡΠ΅ΠΉ ΠΊΠΎΠΎΡΠ΄ΠΈΠ½Π°Ρ, Π½ΠΈΡΠ΅Π³ΠΎ Π½Π΅ ΠΌΠ΅Π½ΡΠ΅Ρ. ΠΠ΅ΠΊΡΠΎΡΡ ΠΈ ΠΈΡ
Π²Π΅ΠΊΡΠΎΡΠ½ΠΎΠ΅ ΠΏΡΠΎΠΈΠ·Π²Π΅Π΄Π΅Π½ΠΈΠ΅ ΠΆΠΈΠ²ΡΡ Π² ΠΏΡΠΎΡΡΡΠ°Π½ΡΡΠ²Π΅ Π±Π΅Π· ΠΊΠΎΠΎΡΠ΄ΠΈΠ½Π°Ρ, ΠΏΡΠΎΡΡΠΎ ΠΏΠ»Π°Π²Π°Ρ. ΠΡ ΠΏΡΠΎΡΡΠΎ Π²Π²ΠΎΠ΄ΠΈΠΌ ΠΊΠΎΠΎΡΠ΄ΠΈΠ½Π°ΡΡ, ΡΡΠΎΠ±Ρ ΡΠΏΡΠΎΡΡΠΈΡΡ ΠΊΠΎΠ½ΠΊΡΠ΅ΡΠ½ΡΠ΅ Π²ΡΡΠΈΡΠ»Π΅Π½ΠΈΡ. 2} = |a_1b_2|$, ΠΎΡΠ΅Π²ΠΈΠ΄Π½ΠΎ. ΠΠΎ Ρ ΠΏΠ°ΡΠ°Π»Π»Π΅Π»ΠΎΠ³ΡΠ°ΠΌΠΌΠ° Π΅ΡΡΡ ΠΎΡΠ½ΠΎΠ²Π°Π½ΠΈΠ΅ $|a_1|$ ΠΈ Π²ΡΡΠΎΡΠ° $|b_2|$, Π° ΡΡΠΎ Π·Π½Π°ΡΠΈΡ, ΡΡΠΎ ΠΏΠ»ΠΎΡΠ°Π΄Ρ ΠΏΠ°ΡΠ°Π»Π»Π΅Π»ΠΎΠ³ΡΠ°ΠΌΠΌΠ° Π·Π°Π΄Π°Π΅ΡΡΡ ΡΠΎΡΠ½ΠΎ ΡΠ°ΠΊΠΈΠΌ ΠΆΠ΅ Π²ΡΡΠ°ΠΆΠ΅Π½ΠΈΠ΅ΠΌ.
$\endgroup$
9
$\begingroup$
ΠΠ΅ΡΠΎΡΡΠ½ΠΎ, ΠΏΠΎΡΠΎΠΌΡ ΡΡΠΎ ΠΎΡΠ²Π΅Ρ ΠΏΡΠΎΡΡ Ρ ΡΠΎΡΠΊΠΈ Π·ΡΠ΅Π½ΠΈΡ ΠΊΠ»Π°ΡΡΠΈΡΠ΅ΡΠΊΠΎΠΉ 2D-Π³Π΅ΠΎΠΌΠ΅ΡΡΠΈΠΈ.
$||\vec u\times \vec v||=||\vec u||.||\vec v||.\sin(\vec u,\vec v)$
ΠΠΎ ΠΏΠ»ΠΎΡΠ°Π΄Ρ ΠΏΠ°ΡΠ°Π»Π»Π΅Π»ΠΎΠ³ΡΠ°ΠΌΠΌ, ΠΎΠΏΡΠ΅Π΄Π΅Π»ΡΠ΅ΠΌΡΠΉ $\vec u$ ΠΈ $\vec v$, ΠΏΡΠ΅Π΄ΡΡΠ°Π²Π»ΡΠ΅Ρ ΡΠΎΠ±ΠΎΠΉ ΠΏΡΠΎΠΈΠ·Π²Π΅Π΄Π΅Π½ΠΈΠ΅ ΠΎΡΠ½ΠΎΠ²Π°Π½ΠΈΡ Π½Π° Π²ΡΡΠΎΡΡ. ΠΡΠ»ΠΈ Π²Π·ΡΡΡ $\vec u$ Π·Π° ΠΎΡΠ½ΠΎΠ²Ρ, Π²ΡΡΠΎΡΠ° Π±ΡΠ΄Π΅Ρ $h=||\vec v||.\sin(\vec u,\vec v)$, ΠΎΡΡΡΠ΄Π° ΠΈ ΡΠ΅Π·ΡΠ»ΡΡΠ°Ρβ¦
$\endgroup$
1
$\begingroup$
$|ΠΓΠ|=|Π| Γ|B|\sin \theta $, Π³Π΄Π΅ $\theta$ β ΡΠ³ΠΎΠ» ΠΌΠ΅ΠΆΠ΄Ρ A ΠΈ B. ΠΡΠΏΠΎΠ»ΡΠ·ΡΡ ΠΏΡΠΎΡΡΡΡ ΡΡΠΈΠ³ΠΎΠ½ΠΎΠΌΠ΅ΡΡΠΈΡ, ΠΏΡΡΡΡ Π²ΡΡΠΎΡΠΎΠΉ ΠΏΠ°ΡΠ°Π»Π»Π΅Π»ΠΎΠ³ΡΠ°ΠΌΠΌΠ° Π±ΡΠ΄Π΅Ρ Π±ΡΠΊΠ²Π° X
ΠΠ»ΠΎΡΠ°Π΄Ρ ΠΏΠ°ΡΠ°Π»Π»Π΅Π»ΠΎΠ³ΡΠ°ΠΌΠΌΠ° ΠΎΠΏΡΠ΅Π΄Π΅Π»ΡΠ΅ΡΡΡ ΠΎΡΠ½ΠΎΠ²Π°Π½ΠΈΠ΅ΠΌ Γ Π²ΡΡΠΎΡΠ°
$\sin\theta=X/|A|$ ΠΡΠΎ ΠΎΠ·Π½Π°ΡΠ°Π΅Ρ, ΡΡΠΎ $X= |A| \sin\theta $
Π‘Π»Π΅Π΄ΠΎΠ²Π°ΡΠ΅Π»ΡΠ½ΠΎ, ΠΏΠ»ΠΎΡΠ°Π΄Ρ Π·Π°Π΄Π°Π΅ΡΡΡ ΠΎΡΠ½ΠΎΠ²Π°Π½ΠΈΠ΅ΠΌ $|B| Γ |Π| \sin\ΡΠ΅ΡΠ° $ ΠΠΎ ΡΡΡΠΈ, ΡΡΠΎ Π²Π΅Π»ΠΈΡΠΈΠ½Π° ΠΏΠ΅ΡΠ΅ΠΊΡΠ΅ΡΡΠ½ΠΎΠ³ΠΎ ΠΏΡΠΎΠΈΠ·Π²Π΅Π΄Π΅Π½ΠΈΡ A ΠΈ B
$\endgroup$
1
$\begingroup$
ΠΠΏΡΠ΅Π΄Π΅Π»ΠΈΠΌ $A \times_1 B, A \times_2 B$ ΠΊΠ°ΠΊ Π²Π΅ΠΊΡΠΎΡΡ, ΠΏΠ΅ΡΠΏΠ΅Π½Π΄ΠΈΠΊΡΠ»ΡΡΠ½ΡΠ΅ $A, B$, Ρ ΠΎΡΠΈΠ΅Π½ΡΠ°ΡΠΈΠ΅ΠΉ, Π²ΡΠ±ΡΠ°Π½Π½ΠΎΠΉ ΠΏΠΎ ΠΏΡΠ°Π²ΠΈΠ»Ρ ΠΏΡΠ°Π²ΠΎΠΉ ΡΡΠΊΠΈ, ΠΈ Π²Π΅Π»ΠΈΡΠΈΠ½ΠΎΠΉ $|A||B|sin \theta$ ΡΠΎΠΎΡΠ²Π΅ΡΡΡΠ²Π΅Π½Π½ΠΎ. Π°Π±ΡΠΎΠ»ΡΡΠ½ΠΎΠ΅ Π·Π½Π°ΡΠ΅Π½ΠΈΠ΅
$$
\begin{vmatrix}
\bf{i} & \bf{j} & \bf{k} \\
Π°_1 ΠΈ Π°_2 ΠΈ Π°_3 \\
Π±_1 ΠΈ Π±_2 ΠΈ Π±_3
\end{vmatrix}
$$
ΠΡΠΎΡΡΠΎΠ΅ Π³Π΅ΠΎΠΌΠ΅ΡΡΠΈΡΠ΅ΡΠΊΠΎΠ΅ Π΄ΠΎΠΊΠ°Π·Π°ΡΠ΅Π»ΡΡΡΠ²ΠΎ ΠΏΠΎΠΊΠ°Π·ΡΠ²Π°Π΅Ρ, ΡΡΠΎ $A \times_1 B$ Π»ΠΈΠ½Π΅ΠΉΠ½ΠΎ Π·Π°Π²ΠΈΡΠΈΡ ΠΎΡ $A, B$. Π Π΄Π΅ΡΠ΅ΡΠΌΠΈΠ½Π°Π½ΡΠ½ΡΠ΅ ΡΠ²ΠΎΠΉΡΡΠ²Π° ΠΏΠΎΠΊΠ°Π·ΡΠ²Π°ΡΡ, ΡΡΠΎ $A \times_2 B$ Π»ΠΈΠ½Π΅ΠΉΠ½ΠΎ Π·Π°Π²ΠΈΡΠΈΡ ΡΠ°ΠΊΠΆΠ΅ Π² $A, B$. ΠΡΡΠ°Π΅ΡΡΡ Π·Π°ΠΌΠ΅ΡΠΈΡΡ, ΡΡΠΎ $\times_1, \times_2$ ΡΠΎΠ²ΠΏΠ°Π΄Π°ΡΡ Π½Π° Π±Π°Π·ΠΈΡΠ½ΡΡ
Π²Π΅ΠΊΡΠΎΡΠ°Ρ
$\bf{i}, \bf{j}, \bf{k}$, Π° Π·Π½Π°ΡΠΈΡ, ΠΊΠ°ΠΊ ΠΈ ΠΎΠΆΠΈΠ΄Π°Π»ΠΎΡΡ, $\times_1 = \times_2$ .
$\endgroup$
Π’Π²ΠΎΠΉ ΠΎΡΠ²Π΅Ρ
ΠΠ°ΡΠ΅Π³ΠΈΡΡΡΠΈΡΡΠΉΡΠ΅ΡΡ ΠΈΠ»ΠΈ Π²ΠΎΠΉΠ΄ΠΈΡΠ΅ Π² ΡΠΈΡΡΠ΅ΠΌΡ
ΠΠ°ΡΠ΅Π³ΠΈΡΡΡΠΈΡΡΠΉΡΠ΅ΡΡ Ρ ΠΏΠΎΠΌΠΎΡΡΡ Google
ΠΠ°ΡΠ΅Π³ΠΈΡΡΡΠΈΡΠΎΠ²Π°ΡΡΡΡ ΡΠ΅ΡΠ΅Π· Facebook
ΠΠ°ΡΠ΅Π³ΠΈΡΡΡΠΈΡΡΠΉΡΠ΅ΡΡ, ΠΈΡΠΏΠΎΠ»ΡΠ·ΡΡ ΡΠ»Π΅ΠΊΡΡΠΎΠ½Π½ΡΡ ΠΏΠΎΡΡΡ ΠΈ ΠΏΠ°ΡΠΎΠ»Ρ
ΠΠΏΡΠ±Π»ΠΈΠΊΠΎΠ²Π°ΡΡ ΠΊΠ°ΠΊ Π³ΠΎΡΡΡ
ΠΠ»Π΅ΠΊΡΡΠΎΠ½Π½Π°Ρ ΠΏΠΎΡΡΠ°
Π’ΡΠ΅Π±ΡΠ΅ΡΡΡ, Π½ΠΎ Π½ΠΈΠΊΠΎΠ³Π΄Π° Π½Π΅ ΠΎΡΠΎΠ±ΡΠ°ΠΆΠ°Π΅ΡΡΡ
ΠΠΏΡΠ±Π»ΠΈΠΊΠΎΠ²Π°ΡΡ ΠΊΠ°ΠΊ Π³ΠΎΡΡΡ
ΠΠ»Π΅ΠΊΡΡΠΎΠ½Π½Π°Ρ ΠΏΠΎΡΡΠ°
Π’ΡΠ΅Π±ΡΠ΅ΡΡΡ, Π½ΠΎ Π½Π΅ ΠΎΡΠΎΠ±ΡΠ°ΠΆΠ°Π΅ΡΡΡ
ΠΠ°ΠΆΠΈΠΌΠ°Ρ Β«ΠΠΏΡΠ±Π»ΠΈΠΊΠΎΠ²Π°ΡΡ ΡΠ²ΠΎΠΉ ΠΎΡΠ²Π΅ΡΒ», Π²Ρ ΡΠΎΠ³Π»Π°ΡΠ°Π΅ΡΠ΅ΡΡ Ρ Π½Π°ΡΠΈΠΌΠΈ ΡΡΠ»ΠΎΠ²ΠΈΡΠΌΠΈ ΠΎΠ±ΡΠ»ΡΠΆΠΈΠ²Π°Π½ΠΈΡ, ΠΏΠΎΠ»ΠΈΡΠΈΠΊΠΎΠΉ ΠΊΠΎΠ½ΡΠΈΠ΄Π΅Π½ΡΠΈΠ°Π»ΡΠ½ΠΎΡΡΠΈ ΠΈ ΠΏΠΎΠ»ΠΈΡΠΈΠΊΠΎΠΉ ΠΈΡΠΏΠΎΠ»ΡΠ·ΠΎΠ²Π°Π½ΠΈΡ ΡΠ°ΠΉΠ»ΠΎΠ² cookie
.
Π²Π½Π΅ΡΠ½ΡΡ Π°Π»Π³Π΅Π±ΡΠ° β ΠΠ½ΡΡΠΈΡΠΈΡ Π·Π° ΠΏΠ΅ΡΠ΅ΠΊΡΠ΅ΡΡΠ½ΡΠΌ ΠΏΡΠΎΠΈΠ·Π²Π΅Π΄Π΅Π½ΠΈΠ΅ΠΌ ΠΈ ΠΏΠ»ΠΎΡΠ°Π΄ΡΡ ΠΏΠ°ΡΠ°Π»Π»Π΅Π»ΠΎΠ³ΡΠ°ΠΌΠΌΠ°
ΡΠΏΡΠΎΡΠΈΠ»
ΠΠ·ΠΌΠ΅Π½Π΅Π½ΠΎ 4 ΠΌΠ΅ΡΡΡΠ° Π½Π°Π·Π°Π΄
ΠΡΠΎΡΠΌΠΎΡΡΠ΅Π½ΠΎ 2ΠΊ ΡΠ°Π·
$\begingroup$
ΠΠ΅ΡΠ΅ΠΊΡΠ΅ΡΡΠ½ΠΎΠ΅ ΠΏΡΠΎΠΈΠ·Π²Π΅Π΄Π΅Π½ΠΈΠ΅ Π² 2D ΠΎΠΏΡΠ΅Π΄Π΅Π»ΡΠ΅ΡΡΡ ΡΠ»Π΅Π΄ΡΡΡΠΈΠΌ ΠΎΠ±ΡΠ°Π·ΠΎΠΌ: $|(x_1, y_1) \times (x_2, y_2)| = x_1 y_2 β x_2 y_1.$
Π― ΠΏΡΠ΅ΠΊΡΠ°ΡΠ½ΠΎ ΠΏΠΎΠ½ΠΈΠΌΠ°Ρ ΠΏΠ΅ΡΠ²ΡΡ ΡΠ°ΡΡΡ ΠΎΠΏΡΠ΅Π΄Π΅Π»Π΅Π½ΠΈΡ: $x_1 y_2$, ΠΊΠΎΡΠΎΡΠ°Ρ ΠΏΡΠ΅Π΄ΡΡΠ°Π²Π»ΡΠ΅Ρ ΡΠΎΠ±ΠΎΠΉ ΠΏΡΠΎΡΡΠΎ ΠΏΠ»ΠΎΡΠ°Π΄Ρ ΠΏΡΡΠΌΠΎΡΠ³ΠΎΠ»ΡΠ½ΠΈΠΊΠ°:
Π― ΠΈΠ·ΠΎ Π²ΡΠ΅Ρ
ΡΠΈΠ» ΠΏΡΡΠ°ΡΡΡ ΠΏΠΎΠ½ΡΡΡ Π²ΡΠΎΡΡΡ ΡΠ°ΡΡΡ: $- x_2 y_1.$
Π― ΡΡΠ²ΡΡΠ²ΡΡ, ΡΡΠΎ Π²ΡΠΎΡΠ°Ρ ΡΠ°ΡΡΡ, Π²Π΅ΡΠΎΡΡΠ½ΠΎ, ΡΠ²ΡΠ·Π°Π½Π° Ρ ΠΏΠΎΠ²ΠΎΡΠΎΡΠ°ΠΌΠΈ Π²Π΅ΠΊΡΠΎΡΠΎΠ² $(x_1, y_1)$ ΠΈ $(x_2, y_2)$, ΠΏΠΎΡΠΎΠΌΡ ΡΡΠΎ ΠΏΡΠΈ ΠΏΠΎΠ²ΠΎΡΠΎΡΠ΅ ΠΈΡΡ
ΠΎΠ΄Π½ΠΎΠ³ΠΎ ΠΏΡΡΠΌΠΎΡΠ³ΠΎΠ»ΡΠ½ΠΈΠΊΠ° ΠΌΡ Π΄ΠΎΠ»ΠΆΠ½Ρ ΡΠΎΡ
ΡΠ°Π½ΠΈΡΡ ΠΏΠ»ΠΎΡΠ°Π΄Ρ ΠΈ $- x_2 y_1$ ΠΊΠ°ΠΊΠΈΠΌ-ΡΠΎ ΠΎΠ±ΡΠ°Π·ΠΎΠΌ ΠΊΠΎΠΌΠΏΠ΅Π½ΡΠΈΡΡΠ΅Ρ ΠΈΠ·Π±ΡΡΠΎΠΊ ΠΏΠ»ΠΎΡΠ°Π΄ΠΈ , ΡΡΠΎ ΠΌΡ ΠΏΠΎΠ»ΡΡΠ°Π΅ΠΌ ΠΈΠ· ΠΏΠ΅ΡΠ²ΠΎΠ³ΠΎ ΡΠ»Π°Π³Π°Π΅ΠΌΠΎΠ³ΠΎ $x_1 y_2$.
Π― ΡΡΠ²ΡΡΠ²ΡΡ, ΡΡΠΎ ΠΌΠΎΠ΅ΠΉ ΠΈΠ½ΡΡΠΈΡΠΈΠΈ Π½Π΅ Ρ Π²Π°ΡΠ°Π΅Ρ ΠΌΠ½ΠΎΠ³ΠΈΡ Π΄Π΅ΡΠ°Π»Π΅ΠΉ, ΠΈ Ρ Π±ΡΠ» Π±Ρ ΠΏΡΠΈΠ·Π½Π°ΡΠ΅Π»Π΅Π½ Π·Π° ΠΎΠ±ΡΡΡΠ½Π΅Π½ΠΈΠ΅ Π²ΡΠΎΡΠΎΠ³ΠΎ ΡΠ»Π΅Π½Π° ΠΈ Π΅Π³ΠΎ ΡΠ²ΡΠ·ΠΈ Ρ Π²ΡΠ°ΡΠ΅Π½ΠΈΡΠΌΠΈ.
ΠΠΎΠΉ Π²ΠΎΠΏΡΠΎΡ ΠΎΡΠ»ΠΈΡΠ°Π΅ΡΡΡ ΠΎΡ Π Π°ΡΡΡΠΆΠ΄Π΅Π½ΠΈΡ ΠΎ ΠΏΠ΅ΡΠ΅ΠΊΡΠ΅ΡΡΠ½ΡΡ ΠΏΡΠΎΠΈΠ·Π²Π΅Π΄Π΅Π½ΠΈΡΡ , ΠΈΡΠΏΠΎΠ»ΡΠ·ΡΠ΅ΠΌΡΡ Π΄Π»Ρ Π½Π°Ρ ΠΎΠΆΠ΄Π΅Π½ΠΈΡ ΠΏΠ»ΠΎΡΠ°Π΄ΠΈ, Ρ ΠΎΡΡ Π½Π°Π·Π²Π°Π½ΠΈΡ ΠΏΠΎΡΡΠΈ ΠΈΠ΄Π΅Π½ΡΠΈΡΠ½Ρ. ΠΡΡΠΎΠ³ΠΎΠ½Π°Π»ΡΠ½ΠΎΡΡΡ ΠΌΠΎΠ΅Π³ΠΎ Π²ΠΎΠΏΡΠΎΡΠ° ΠΊ ΡΡΠΎΠΌΡ ΠΌΠΎΠΆΠ½ΠΎ ΡΠ²ΠΈΠ΄Π΅ΡΡ, ΠΏΡΠΎΡΠΈΡΠ°Π² ΡΡΠΈ ΡΠ°ΡΡΠΈ Π²ΠΎΠΏΡΠΎΡΠ°:
Π£ ΠΌΠ΅Π½Ρ Π½Π΅Ρ ΠΏΡΠΎΠ±Π»Π΅ΠΌ Ρ Π²ΡΡΠΈΡΠ»Π΅Π½ΠΈΠ΅ΠΌ ΠΏΠ»ΠΎΡΠ°Π΄ΠΈ ΠΌΠ΅ΠΆΠ΄Ρ Π΄Π²ΡΠΌΡ Π²Π΅ΠΊΡΠΎΡΠ°ΠΌΠΈ.
ΠΠΎ Ρ ΠΌΠ΅Π½Ρ Π΅ΡΡΡ.
β¦ Π½ΠΎ Π½Π΅ ΠΏΠΎΡΠ΅ΠΌΡ Π²ΠΌΠ΅ΡΡΠΎ ΡΠΊΠ°Π»ΡΡΠ½ΠΎΠ³ΠΎ ΠΏΡΠΎΠΈΠ·Π²Π΅Π΄Π΅Π½ΠΈΡ ΠΈΡΠΏΠΎΠ»ΡΠ·ΡΠ΅ΡΡΡ ΠΏΠ΅ΡΠ΅ΠΊΡΠ΅ΡΡΠ½ΠΎΠ΅ ΠΏΡΠΎΠΈΠ·Π²Π΅Π΄Π΅Π½ΠΈΠ΅.
Π‘ΠΊΠ°Π»ΡΡΠ½ΡΠΉ ΠΏΡΠΎΠ΄ΡΠΊΡ Π²ΡΡ ΠΎΠ΄ΠΈΡ Π·Π° ΡΠ°ΠΌΠΊΠΈ ΠΌΠΎΠ΅Π³ΠΎ Π²ΠΎΠΏΡΠΎΡΠ°.
- ΠΏΠ»ΠΎΡΠ°Π΄Ρ
- Π²Π½Π΅ΡΠ½ΡΡ Π°Π»Π³Π΅Π±ΡΠ°
- ΠΏΠ΅ΡΠ΅ΠΊΡΠ΅ΡΡΠ½ΠΎΠ΅ ΠΏΡΠΎΠΈΠ·Π²Π΅Π΄Π΅Π½ΠΈΠ΅
$\endgroup$
9
$\begingroup$
ΠΡΠ»ΠΈ Π²Ρ ΠΏΡΠ΅Π΄ΠΏΠΎΡΠΈΡΠ°Π΅ΡΠ΅ Π³Π΅ΠΎΠΌΠ΅ΡΡΠΈΡΠ΅ΡΠΊΠΎΠ΅ Π²ΠΈΠ΄Π΅Π½ΠΈΠ΅, Π·Π°ΡΠΈΠΊΡΠΈΡΡΠΉΡΠ΅ΡΡ Π½Π° ΡΠΈΠ³ΡΡΠ΅.
ΠΠ»ΠΎΡΠ°Π΄Ρ ΠΏΠ°ΡΠ°Π»Π»Π΅Π»ΠΎΠ³ΡΠ°ΠΌΠΌΠ° $OMPN$, ΠΎΡΠ΅Π²ΠΈΠ΄Π½ΠΎ, ΡΠ°Π²Π½Π° ΠΏΠ»ΠΎΡΠ°Π΄ΠΈ ΡΠΎΠ·ΠΎΠ²ΠΎΠ³ΠΎ ΠΏΠ°ΡΠ°Π»Π»Π΅Π»ΠΎΠ³ΡΠ°ΠΌΠΌΠ° (ΡΠΎ ΠΆΠ΅ ΠΎΡΠ½ΠΎΠ²Π°Π½ΠΈΠ΅ $OM$ ΠΈ ΡΠ° ΠΆΠ΅ Π²ΡΡΠΎΡΠ°, ΠΏΠΎΡΠΎΠΌΡ ΡΡΠΎ ΠΏΡΠΎΡΠΈΠ²ΠΎΠΏΠΎΠ»ΠΎΠΆΠ½ΡΠ΅ ΡΡΠΎΡΠΎΠ½Ρ ΠΏΠ°ΡΠ°Π»Π»Π΅Π»ΡΠ½Ρ). ΠΠΎ ΠΏΠ»ΠΎΡΠ°Π΄Ρ ΡΠΎΠ·ΠΎΠ²ΠΎΠ³ΠΎ ΠΏΠ°ΡΠ°Π»Π»Π΅Π»ΠΎΠ³ΡΠ°ΠΌΠΌΠ° ΡΠ°ΠΊΠΆΠ΅ ΠΎΠΏΡΠ΅Π΄Π΅Π»ΡΠ΅ΡΡΡ ΠΊΠ°ΠΊ: $ \quad Area=|a_1c|$, ΠΈ ΠΌΡ ΠΌΠΎΠΆΠ΅ΠΌ Π½Π°ΠΉΡΠΈ $c$ ΠΊΠ°ΠΊ ΠΏΠ΅ΡΠ΅ΡΠ΅ΡΠ΅Π½ΠΈΠ΅ Π² Π½Π°ΡΠ°Π»Π΅ ΠΊΠΎΠΎΡΠ΄ΠΈΠ½Π°Ρ ΠΏΡΡΠΌΠΎΠΉ ΠΈΠ· $N,P$.
ΠΠ·ΡΡΠΈΠ² ΡΠ»Π΅ΠΌΠ΅Π½ΡΠ°ΡΠ½ΡΡ Π°Π½Π°Π»ΠΈΡΠΈΡΠ΅ΡΠΊΡΡ Π³Π΅ΠΎΠΌΠ΅ΡΡΠΈΡ, Π²Ρ ΠΎΠ±Π½Π°ΡΡΠΆΠΈΡΠ΅, ΡΡΠΎ $$ c=\frac{a_1b_2-a_2b_1}{a_1} $$
ΠΈ ΡΡΠΎ ΠΎΠ·Π½Π°ΡΠ°Π΅Ρ, ΡΡΠΎ:
$$ ΠΠ»ΠΎΡΠ°Π΄Ρ = |a_1b_2-a_2b_1| $$
, ΡΠΎ Π΅ΡΡΡ Π°Π±ΡΠΎΠ»ΡΡΠ½ΠΎΠ΅ Π·Π½Π°ΡΠ΅Π½ΠΈΠ΅ Π΄Π΅ΡΠΌΠΈΠ½Π°ΡΠ° ΠΌΠ°ΡΡΠΈΡΡ, ΡΡΠΎΠ»Π±ΡΠ°ΠΌΠΈ ΠΊΠΎΡΠΎΡΠΎΠΉ ΡΠ²Π»ΡΡΡΡΡ Π΄Π²Π° Π²Π΅ΠΊΡΠΎΡΠ° $\overline {OM}$ ΠΈ $\overline {ON}$, ΠΈΠ»ΠΈ Π°Π±ΡΠΎΠ»ΡΡΠ½ΠΎΠ΅ Π·Π½Π°ΡΠ΅Π½ΠΈΠ΅ Π²Π΅ΠΊΡΠΎΡΠ½ΠΎΠ³ΠΎ ΠΏΡΠΎΠΈΠ·Π²Π΅Π΄Π΅Π½ΠΈΡ ΡΠ΅Ρ ΠΆΠ΅ Π΄Π²ΡΡ Π²Π΅ΠΊΡΠΎΡΠΎΠ².
$\endgroup$
$\begingroup$
- Π€ΠΎΡΠΌΡΠ»Π° ΠΎΡΠ»ΠΈΡΠ½ΠΎ ΡΠ°Π±ΠΎΡΠ°Π΅Ρ Π΄Π»Ρ ΡΡΠ°Π½Π΄Π°ΡΡΠ½ΡΡ Π΅Π΄ΠΈΠ½ΠΈΡΠ½ΡΡ Π²Π΅ΠΊΡΠΎΡΠΎΠ².
- Π Π°ΡΡΠΈΡΠ΅Π½ΠΈΠ΅ ΠΎΠ΄Π½ΠΎΠ³ΠΎ ΠΈΠ· Π²Π΅ΠΊΡΠΎΡΠΎΠ² Π½Π° ΠΊΠΎΠ½ΡΡΠ°Π½ΡΡ $c$ Π΄ΠΎΠ»ΠΆΠ½ΠΎ ΡΠΌΠ½ΠΎΠΆΠ°ΡΡ (ΠΎΡΠΈΠ΅Π½ΡΠΈΡΠΎΠ²Π°Π½Π½ΡΡ) ΠΏΠ»ΠΎΡΠ°Π΄Ρ Π½Π° $c$ ΠΈ Π΄Π΅ΠΉΡΡΠ²ΠΈΡΠ΅Π»ΡΠ½ΠΎ ΡΠΌΠ½ΠΎΠΆΠ°Π΅Ρ Π²Π΅ΠΊΡΠΎΡΠ½ΠΎΠ΅ ΠΏΡΠΎΠΈΠ·Π²Π΅Π΄Π΅Π½ΠΈΠ΅ Π½Π° $c$
- Π‘Π΄Π²ΠΈΠ³ Π²Π΄ΠΎΠ»Ρ $(x_1,y_1)$, Ρ.
Π΅. Π·Π°ΠΌΠ΅Π½Π° $(x_2,y_2)$ Π½Π° $(x_2+cx_1,y_2+cy_1)$ Π½Π΅ ΠΌΠ΅Π½ΡΠ΅Ρ Π½ΠΈ ΠΏΠ»ΠΎΡΠ°Π΄Ρ, Π½ΠΈ Π²Π΅ΠΊΡΠΎΡΠ½ΠΎΠ΅ ΠΏΡΠΎΠΈΠ·Π²Π΅Π΄Π΅Π½ΠΈΠ΅ (Π΄ΠΎΠΏΠΎΠ»Π½ΠΈΡΠ΅Π»ΡΠ½ΡΠ΅ ΡΠ»Π΅Π½Ρ ΠΎΡΠΌΠ΅Π½ΠΈΡΡ)
ΠΠΎΡΠΊΠΎΠ»ΡΠΊΡ Π»ΡΠ±ΠΎΠΉ ΠΏΠ°ΡΠ°Π»Π»Π΅Π»ΠΎΠ³ΡΠ°ΠΌΠΌ ΠΌΠΎΠΆΠ΅Ρ Π±ΡΡΡ ΠΏΠΎΠ»ΡΡΠ΅Π½ ΠΈΠ· ΡΡΠ°Π½Π΄Π°ΡΡΠ½ΡΡ Π΅Π΄ΠΈΠ½ΠΈΡΠ½ΡΡ Π²Π΅ΠΊΡΠΎΡΠΎΠ² Ρ ΠΏΠΎΠΌΠΎΡΡΡ Π½Π΅ΡΠΊΠΎΠ»ΡΠΊΠΈΡ ΡΠ°Π³ΠΎΠ² ΡΠ΄Π²ΠΈΠ³Π°/ΡΠ°ΡΡΡΠΆΠ΅Π½ΠΈΡ, Π²Π΅ΠΊΡΠΎΡΠ½ΠΎΠ΅ ΠΏΡΠΎΠΈΠ·Π²Π΅Π΄Π΅Π½ΠΈΠ΅ ΡΠΎΠΎΠ±ΡΠ°Π΅Ρ Π½Π°ΠΌ ΠΎΡΠΈΠ΅Π½ΡΠΈΡΠΎΠ²Π°Π½Π½ΡΡ ΠΏΠ»ΠΎΡΠ°Π΄Ρ Π΄Π»Ρ Π²ΡΠ΅Ρ ΠΏΠ°ΡΠ°Π»Π»Π΅Π»ΠΎΠ³ΡΠ°ΠΌΠΌΠΎΠ².
$\endgroup$
$\begingroup$
ΠΠ΅ΡΠ΅ΠΊΡΠ΅ΡΡΠ½ΠΎΠ΅ ΠΏΡΠΎΠΈΠ·Π²Π΅Π΄Π΅Π½ΠΈΠ΅ ΠΌΠΎΠΆΠ½ΠΎ ΡΠ°ΠΊΠΆΠ΅ ΠΎΠΏΡΠ΅Π΄Π΅Π»ΠΈΡΡ ΠΊΠ°ΠΊ $a\times b=|a||b|\sin\theta$, Π³Π΄Π΅ $\theta$ ΠΏΡΠ΅Π΄ΡΡΠ°Π²Π»ΡΠ΅Ρ ΡΠΎΠ±ΠΎΠΉ ΡΠ³ΠΎΠ» ΠΌΠ΅ΠΆΠ΄Ρ $a$ ΠΈ $b$.
ΠΠ³ΠΎ ΠΌΠΎΠΆΠ½ΠΎ ΡΠ°ΡΡΠΌΠ°ΡΡΠΈΠ²Π°ΡΡ ΠΊΠ°ΠΊ Π·Π°Π΄Π°ΡΡΠΈΠΉ Π²Π΅ΠΊΡΠΎΡ, ΠΏΠ΅ΡΠΏΠ΅Π½Π΄ΠΈΠΊΡΠ»ΡΡΠ½ΡΠΉ ΠΊ $a$ ΠΈ $b$, Ρ Π΄Π»ΠΈΠ½ΠΎΠΉ $|a||b|\sin\theta$ ΠΈΠ»ΠΈ ΠΊΠ°ΠΊ ΠΏΠ»ΠΎΡΠ°Π΄Ρ ΠΊΠΎΡΠΎΠ³ΠΎ ΠΏΡΡΠΌΠΎΡΠ³ΠΎΠ»ΡΠ½ΠΈΠΊΠ° (Ρ. Π΅. ΠΏΠ°ΡΠ°Π»Π»Π΅Π»ΠΎΠ³ΡΠ°ΠΌΠΌΠ°) Ρ Π΄Π»ΠΈΠ½Π°ΠΌΠΈ ΡΡΠΎΡΠΎΠ½ $a$ ΠΈ $b$ ΠΈ ΠΊΠΎΡΠΎΠΉ ΡΠ΅ΡΡΠΎΠΉ $\theta$.
$\endgroup$
$\begingroup$
ΠΠΎΡ ΠΏΡΠ΅Π΄ΡΡΠ°Π²Π»Π΅Π½ΠΈΠ΅ ΠΎΠΏΡΠ΅Π΄Π΅Π»ΠΈΡΠ΅Π»Ρ Π²ΡΠ΅Ρ
ΠΏΠΎΠ»ΠΎΠΆΠΈΡΠ΅Π»ΡΠ½ΡΡ
Π²Π΅Π»ΠΈΡΠΈΠ½, ΠΊΠΎΡΠΎΡΠΎΠ΅ ΠΌΠΎΠΆΠ΅Ρ ΠΎΠ±Π΅ΡΠΏΠ΅ΡΠΈΡΡ ΠΏΠΎΠ½ΠΈΠΌΠ°Π½ΠΈΠ΅ ΡΠ²ΡΠ·ΠΈ Ρ ΠΏΠ»ΠΎΡΠ°Π΄ΡΠΌΠΈ Π½Π° ΠΏΠ΅ΡΠ²ΠΎΠΌ ΡΡΠΎΠ²Π½Π΅.
$\endgroup$
Π’Π²ΠΎΠΉ ΠΎΡΠ²Π΅Ρ
ΠΠ°ΡΠ΅Π³ΠΈΡΡΡΠΈΡΡΠΉΡΠ΅ΡΡ ΠΈΠ»ΠΈ Π²ΠΎΠΉΠ΄ΠΈΡΠ΅ Π² ΡΠΈΡΡΠ΅ΠΌΡ
ΠΠ°ΡΠ΅Π³ΠΈΡΡΡΠΈΡΡΠΉΡΠ΅ΡΡ Ρ ΠΏΠΎΠΌΠΎΡΡΡ Google
ΠΠ°ΡΠ΅Π³ΠΈΡΡΡΠΈΡΠΎΠ²Π°ΡΡΡΡ ΡΠ΅ΡΠ΅Π· Facebook
ΠΠ°ΡΠ΅Π³ΠΈΡΡΡΠΈΡΡΠΉΡΠ΅ΡΡ, ΠΈΡΠΏΠΎΠ»ΡΠ·ΡΡ ΡΠ»Π΅ΠΊΡΡΠΎΠ½Π½ΡΡ ΠΏΠΎΡΡΡ ΠΈ ΠΏΠ°ΡΠΎΠ»Ρ
ΠΠΏΡΠ±Π»ΠΈΠΊΠΎΠ²Π°ΡΡ ΠΊΠ°ΠΊ Π³ΠΎΡΡΡ
ΠΠ»Π΅ΠΊΡΡΠΎΠ½Π½Π°Ρ ΠΏΠΎΡΡΠ°
Π’ΡΠ΅Π±ΡΠ΅ΡΡΡ, Π½ΠΎ Π½Π΅ ΠΎΡΠΎΠ±ΡΠ°ΠΆΠ°Π΅ΡΡΡ
ΠΠΏΡΠ±Π»ΠΈΠΊΠΎΠ²Π°ΡΡ ΠΊΠ°ΠΊ Π³ΠΎΡΡΡ
ΠΠ»Π΅ΠΊΡΡΠΎΠ½Π½Π°Ρ ΠΏΠΎΡΡΠ°
Π’ΡΠ΅Π±ΡΠ΅ΡΡΡ, Π½ΠΎ Π½Π΅ ΠΎΡΠΎΠ±ΡΠ°ΠΆΠ°Π΅ΡΡΡ
ΠΠ°ΠΆΠΈΠΌΠ°Ρ Β«ΠΠΏΡΠ±Π»ΠΈΠΊΠΎΠ²Π°ΡΡ ΡΠ²ΠΎΠΉ ΠΎΡΠ²Π΅ΡΒ», Π²Ρ ΡΠΎΠ³Π»Π°ΡΠ°Π΅ΡΠ΅ΡΡ Ρ Π½Π°ΡΠΈΠΌΠΈ ΡΡΠ»ΠΎΠ²ΠΈΡΠΌΠΈ ΠΎΠ±ΡΠ»ΡΠΆΠΈΠ²Π°Π½ΠΈΡ, ΠΏΠΎΠ»ΠΈΡΠΈΠΊΠΎΠΉ ΠΊΠΎΠ½ΡΠΈΠ΄Π΅Π½ΡΠΈΠ°Π»ΡΠ½ΠΎΡΡΠΈ ΠΈ ΠΏΠΎΠ»ΠΈΡΠΈΠΊΠΎΠΉ ΠΈΡΠΏΠΎΠ»ΡΠ·ΠΎΠ²Π°Π½ΠΈΡ ΡΠ°ΠΉΠ»ΠΎΠ² cookie
.
ΠΠ»ΠΎΡΠ°Π΄Ρ ΠΏΠ°ΡΠ°Π»Π»Π΅Π»ΠΎΠ³ΡΠ°ΠΌΠΌΠ°
ΠΠ»ΠΎΡΠ°Π΄Ρ ΠΏΠ°ΡΠ°Π»Π»Π΅Π»ΠΎΠ³ΡΠ°ΠΌΠΌΠ°Π§Π°ΡΡΡ 3:
ΠΠ°ΡΠ°Π»Π»Π΅Π»ΠΎΠ³ΡΠ°ΠΌΠΌΡ ΠΈ ΡΡΠ΅ΡΠ³ΠΎΠ»ΡΠ½ΠΈΠΊΠΈ ΠΠ°Π½Ρ Π΄Π²Π° Π²Π΅ΠΊΡΠΎΡΠ° u ΠΈ v Ρ ΠΎΠ±ΡΠΈΠΌ
Π½Π°ΡΠ°Π»ΡΠ½Π°Ρ ΡΠΎΡΠΊΠ°, ΠΌΠ½ΠΎΠΆΠ΅ΡΡΠ²ΠΎ ΠΊΠΎΠ½Π΅ΡΠ½ΡΡ
ΡΠΎΡΠ΅ΠΊ Π²Π΅ΠΊΡΠΎΡΠΎΠ² s u + t v Π΄Π»Ρ 0 Β£ s , t Β£ 1 ΠΎΠΏΡΠ΅Π΄Π΅Π»ΡΠ΅ΡΡΡ ΠΊΠ°ΠΊ ΠΏΠ°ΡΠ°Π»Π»Π΅Π»ΠΎΠ³ΡΠ°ΠΌΠΌ ΠΎΡ
Π²Π°ΡΡΠ²Π°Π΅Ρ u ΠΈ ΠΏΡΠΎΡΠΈΠ² .

|
|
Π’Π΅ΠΎΡΠ΅ΠΌΠ° 3.3: ΠΡΠ»ΠΈ q ΡΠ³ΠΎΠ», ΠΎΠ±ΡΠ°Π·ΠΎΠ²Π°Π½Π½ΡΠΉ u ΠΈ v , ΡΠΎ
|| u Γ v || = || ΠΈ || || ΠΏΡΠΎΡΠΈΠ² || sin(q)
Π Π°ΡΡΠΌΠΎΡΡΠΈΠΌ Π΄Π²Π° ΠΏΡΠΈΠ»ΠΎΠΆΠ΅Π½ΠΈΡ ΡΠ΅ΠΎΡΠ΅ΠΌΡ 3.3. ΠΠΎ-ΠΏΠ΅ΡΠ²ΡΡ , Π΅ΡΠ»ΠΈ ΠΈ ΠΈ v ΠΏΠ°ΡΠ°Π»Π»Π΅Π»ΡΠ½Ρ, ΡΠΎΠ³Π΄Π° q = 0 ΠΈ u Γ v = 0.
Π’Π°ΠΊΠΈΠΌ ΠΎΠ±ΡΠ°Π·ΠΎΠΌ, ΠΏΠ»ΠΎΡΠ°Π΄Ρ ΠΏΠ°ΡΠ°Π»Π»Π΅Π»ΠΎΠ³ΡΠ°ΠΌΠΌΠ°, ΠΎΠ±ΡΠ°Π·ΠΎΠ²Π°Π½Π½ΠΎΠ³ΠΎ ΠΈ ΠΈ ΠΈ , ΡΠ°Π²Π½Π° || ΠΈ |||| ΠΏΡΠΎΡΠΈΠ² || Π³ΡΠ΅Ρ
(q) . ΠΠ΅ΠΉΡΡΠ²ΠΈΡΠ΅Π»ΡΠ½ΠΎ, ΠΈΠΌΠ΅Π΅ΠΌ ΡΠ»Π΅Π΄ΡΡΡΠ΅Π΅:
ΠΠΎΡΠ»Π΅Π΄Π½ΠΈΠΉ ΡΠ΅Π·ΡΠ»ΡΡΠ°Ρ ΡΠ»Π΅Π΄ΡΠ΅Ρ ΠΈΠ· ΡΠΎΠ³ΠΎ ΡΠ°ΠΊΡΠ°, ΡΡΠΎ ΠΈ β ΠΈ Π΄Π΅Π»ΠΈΡ ΠΏΠΎΠΏΠΎΠ»Π°ΠΌ ΠΏΠ°ΡΠ°Π»Π»Π΅Π»ΠΎΠ³ΡΠ°ΠΌΠΌ, ΠΎΠ±ΡΠ°Π·ΠΎΠ²Π°Π½Π½ΡΠΉ ΠΈ ΠΈ ΠΈ .
ΠΡΠΎΠ²Π΅ΡΡΡΠ΅ ΠΏΡΠ°Π²ΠΈΠ»ΡΠ½ΠΎΡΡΡ ΡΡΠ΅Π½ΠΈΡ: ΠΠ°ΠΊΠΎΠ²Π° ΠΏΠ»ΠΎΡΠ°Π΄Ρ ΠΏΠ°ΡΠ°Π»Π»Π΅Π»ΠΎΠ³ΡΠ°ΠΌΠΌΠ°, Π½Π°ΡΡΠ½ΡΡΠΎΠ³ΠΎ Π½Π° u ΠΈ ΠΏΡΠΎΡΠΈΠ² Π² ΠΏΡΠΈΠΌΠ΅ΡΠ΅ 6?ΠΠ ΠΠΠΠ 5 ΠΠ°ΠΉΠ΄ΠΈΡΠ΅ ΠΏΠ»ΠΎΡΠ°Π΄Ρ ΡΡΠ΅ΡΠ³ΠΎΠ»ΡΠ½ΠΈΠΊΠ° Ρ Π²Π΅ΡΡΠΈΠ½Π°ΠΌΠΈ P 1 (2,2) , P 2 (4,4) ΠΈ P 9 (6,) 3 .
Π Π΅ΡΠ΅Π½ΠΈΠ΅: ΠΠ΅Π³ΠΊΠΎ Π²ΠΈΠ΄Π΅ΡΡ, ΡΡΠΎ u = Π±2,2 Γ± ΠΈ v = Π± 4,-1 Ρ . Π ΠΊΠ°ΡΠ΅ΡΡΠ²Π΅ Π²Π΅ΠΊΡΠΎΡΠΎΠ² Π² R 3 , ΠΌΡ ΠΈΠΌΠ΅Π΅ΠΌ u = Π±2,2,0 Γ± ΠΈ v = Π° 4,-1,0 Ρ . Π’Π°ΠΊΠΈΠΌ ΠΎΠ±ΡΠ°Π·ΠΎΠΌ, ΠΈΡ Π²Π΅ΠΊΡΠΎΡΠ½ΠΎΠ΅ ΠΏΡΠΎΠΈΠ·Π²Π΅Π΄Π΅Π½ΠΈΠ΅ ΡΠ°Π²Π½ΠΎ
Π’Π°ΠΊ ΠΊΠ°ΠΊ ΡΡΠ΅ΡΠ³ΠΎΠ»ΡΠ½ΠΈΠΊ ΠΈΠΌΠ΅Π΅Ρ ΠΏΠΎΠ»ΠΎΠ²ΠΈΠ½Ρ ΠΏΠ»ΠΎΡΠ°Π΄ΠΈ ΠΏΠ°ΡΠ°Π»Π»Π΅Π»ΠΎΠ³ΡΠ°ΠΌΠΌΠ°, ΠΎΠ±ΡΠ°Π·ΠΎΠ²Π°Π½Π½ΠΎΠ³ΠΎ u ΠΈ v , ΠΏΠ»ΠΎΡΠ°Π΄Ρ ΡΡΠ΅ΡΠ³ΠΎΠ»ΡΠ½ΠΈΠΊΠ°
Π° 0,0, 2Β·( -1) -4Β·2 β
0,0,-10 β ΠΠ ΠΠΠΠ 6 ΠΠ°ΠΉΠ΄ΠΈΡΠ΅ ΠΏΠ»ΠΎΡΠ°Π΄Ρ ΡΡΠ΅ΡΠ³ΠΎΠ»ΡΠ½ΠΈΠΊΠ° Ρ Π²Π΅ΡΡΠΈΠ½Π°ΠΌΠΈ P 1 ( 3,0,2) , P 2 ( 4,6,1) , ΠΈ P 3 09046 3 5,4) .
ΠΠ±Π»Π°ΡΡΡ = || u Γ v || =
0 2 +0 2 +(-10) 2 = 5 Π΅Π΄ΠΈΠ½ΠΈΡ 2 Π Π΅ΡΠ΅Π½ΠΈΠ΅: ΠΠ»Ρ ΡΡΠΎΠ³ΠΎ ΡΠ½Π°ΡΠ°Π»Π° ΠΏΠΎΡΡΡΠΎΠΈΠΌ Π²Π΅ΠΊΡΠΎΡΡ u ΠΈ ΠΏΡΠΎΡΠΈΠ² :
Π ΠΊΠ°ΡΠ΅ΡΡΠ²Π΅ Π²Π΅ΠΊΡΠΎΡΠΎΠ² Π² R 3 ΠΌΡ ΡΠ΅ΠΏΠ΅ΡΡ ΠΈΠΌΠ΅Π΅ΠΌ u = Π±2,2,0 Γ± ΠΈ v = Π° 4,-1,0 Ρ .
ΠΈ = Π 1 Π 2
= Π±4-3,6-0,1-2 Ρ = Π° 1,6,-1 Ρ v = Π 1 Π 3
= Π0-3,5-0,4-2 Ρ = Π±-3,5,2 Ρ Π’Π°ΠΊΠΈΠΌ ΠΎΠ±ΡΠ°Π·ΠΎΠΌ, ΠΈΡ Π²Π΅ΠΊΡΠΎΡΠ½ΠΎΠ΅ ΠΏΡΠΎΠΈΠ·Π²Π΅Π΄Π΅Π½ΠΈΠ΅ ΡΠ°Π²Π½ΠΎ
ΠΠΎΡΠΊΠΎΠ»ΡΠΊΡ ΡΡΠ΅ΡΠ³ΠΎΠ»ΡΠ½ΠΈΠΊ ΠΈΠΌΠ΅Π΅Ρ ΠΏΠΎΠ»ΠΎΠ²ΠΈΠ½Ρ ΠΏΠ»ΠΎΡΠ°Π΄ΠΈ ΠΏΠ°ΡΠ°Π»Π»Π΅Π»ΠΎΠ³ΡΠ°ΠΌΠΌΠ°, ΠΎΠ±ΡΠ°Π·ΠΎΠ²Π°Π½Π½ΠΎΠ³ΠΎ 90 233 u 90 234 ΠΈ 90 233 v 90 234, ΠΏΠ»ΠΎΡΠ°Π΄Ρ ΡΡΠ΅ΡΠ³ΠΎΠ»ΡΠ½ΠΈΠΊΠ° ΡΠ°Π²Π½Π°
.
6Β·2-5Β·(-1),(-1)Β·(-3)-2Β·1,1Β·5-(-3)Β·6 β
17,1,23 β
ΠΠ±Π»Π°ΡΡΡ = || u Γ v || =
17 2 +2 2 +23 2 = 14,335 Π΅Π΄ΠΈΠ½ΠΈΡ 2 Π ΠΈΡΡΠ½ΠΎΠΊ Maple/Javaview
ΠΠ΅ΡΠ΅ΠΊΡΠ΅ΡΡΠ½ΠΎΠ΅ ΠΏΡΠΎΠΈΠ·Π²Π΅Π΄Π΅Π½ΠΈΠ΅ Π΄Π²ΡΡ Π²Π΅ΠΊΡΠΎΡΠΎΠ²
ΠΠ΅ΡΠ΅ΠΊΡΠ΅ΡΡΠ½ΠΎΠ΅ ΠΏΡΠΎΠΈΠ·Π²Π΅Π΄Π΅Π½ΠΈΠ΅ Π΄Π²ΡΡ
Π²Π΅ΠΊΡΠΎΡΠΎΠ² β ΡΡΠΎ ΠΌΠ΅ΡΠΎΠ΄ ΡΠΌΠ½ΠΎΠΆΠ΅Π½ΠΈΡ Π΄Π²ΡΡ
Π²Π΅ΠΊΡΠΎΡΠΎΠ². ΠΠ΅ΡΠ΅ΠΊΡΠ΅ΡΡΠ½ΠΎΠ΅ ΠΏΡΠΎΠΈΠ·Π²Π΅Π΄Π΅Π½ΠΈΠ΅ ΠΎΠ±ΠΎΠ·Π½Π°ΡΠ°Π΅ΡΡΡ Π·Π½Π°ΠΊΠΎΠΌ ΡΠΌΠ½ΠΎΠΆΠ΅Π½ΠΈΡ (x) ΠΌΠ΅ΠΆΠ΄Ρ Π΄Π²ΡΠΌΡ Π²Π΅ΠΊΡΠΎΡΠ°ΠΌΠΈ. ΠΡΠΎ Π±ΠΈΠ½Π°ΡΠ½Π°Ρ Π²Π΅ΠΊΡΠΎΡΠ½Π°Ρ ΠΎΠΏΠ΅ΡΠ°ΡΠΈΡ, ΠΎΠΏΡΠ΅Π΄Π΅Π»Π΅Π½Π½Π°Ρ Π² ΡΡΠ΅Ρ
ΠΌΠ΅ΡΠ½ΠΎΠΉ ΡΠΈΡΡΠ΅ΠΌΠ΅. ΠΠ΅ΡΠ΅ΠΊΡΠ΅ΡΡΠ½ΠΎΠ΅ ΠΏΡΠΎΠΈΠ·Π²Π΅Π΄Π΅Π½ΠΈΠ΅ Π΄Π²ΡΡ
Π²Π΅ΠΊΡΠΎΡΠΎΠ² β ΡΡΠΎ ΡΡΠ΅ΡΠΈΠΉ Π²Π΅ΠΊΡΠΎΡ, ΠΏΠ΅ΡΠΏΠ΅Π½Π΄ΠΈΠΊΡΠ»ΡΡΠ½ΡΠΉ Π΄Π²ΡΠΌ ΠΈΡΡ
ΠΎΠ΄Π½ΡΠΌ Π²Π΅ΠΊΡΠΎΡΠ°ΠΌ. ΠΠ³ΠΎ Π²Π΅Π»ΠΈΡΠΈΠ½Π° ΠΎΠΏΡΠ΅Π΄Π΅Π»ΡΠ΅ΡΡΡ ΠΏΠ»ΠΎΡΠ°Π΄ΡΡ ΠΏΠ°ΡΠ°Π»Π»Π΅Π»ΠΎΠ³ΡΠ°ΠΌΠΌΠ° ΠΌΠ΅ΠΆΠ΄Ρ Π½ΠΈΠΌΠΈ, Π° Π΅Π³ΠΎ Π½Π°ΠΏΡΠ°Π²Π»Π΅Π½ΠΈΠ΅ ΠΌΠΎΠΆΠ½ΠΎ ΠΎΠΏΡΠ΅Π΄Π΅Π»ΠΈΡΡ ΠΏΠΎ ΠΏΡΠ°Π²ΠΈΠ»Ρ Π±ΠΎΠ»ΡΡΠΎΠ³ΠΎ ΠΏΠ°Π»ΡΡΠ° ΠΏΡΠ°Π²ΠΎΠΉ ΡΡΠΊΠΈ. ΠΠ΅ΡΠ΅ΠΊΡΠ΅ΡΡΠ½ΠΎΠ΅ ΠΏΡΠΎΠΈΠ·Π²Π΅Π΄Π΅Π½ΠΈΠ΅ Π΄Π²ΡΡ
Π²Π΅ΠΊΡΠΎΡΠΎΠ² ΡΠ°ΠΊΠΆΠ΅ ΠΈΠ·Π²Π΅ΡΡΠ½ΠΎ ΠΊΠ°ΠΊ Π²Π΅ΠΊΡΠΎΡΠ½ΠΎΠ΅ ΠΏΡΠΎΠΈΠ·Π²Π΅Π΄Π΅Π½ΠΈΠ΅, ΠΏΠΎΡΠΊΠΎΠ»ΡΠΊΡ ΡΠ΅Π·ΡΠ»ΡΡΠΈΡΡΡΡΠ°Ρ ΠΏΠ΅ΡΠ΅ΠΊΡΠ΅ΡΡΠ½ΠΎΠ³ΠΎ ΠΏΡΠΎΠΈΠ·Π²Π΅Π΄Π΅Π½ΠΈΡ Π²Π΅ΠΊΡΠΎΡΠΎΠ² ΡΠ²Π»ΡΠ΅ΡΡΡ Π²Π΅ΠΊΡΠΎΡΠ½ΠΎΠΉ Π²Π΅Π»ΠΈΡΠΈΠ½ΠΎΠΉ. ΠΠ΄Π΅ΡΡ ΠΌΡ ΡΠ·Π½Π°Π΅ΠΌ Π±ΠΎΠ»ΡΡΠ΅ ΠΎ Π²Π΅ΠΊΡΠΎΡΠ½ΠΎΠΌ ΠΏΡΠΎΠΈΠ·Π²Π΅Π΄Π΅Π½ΠΈΠΈ Π΄Π²ΡΡ
Π²Π΅ΠΊΡΠΎΡΠΎΠ².
1. | ΠΠ΅ΡΠ΅ΠΊΡΠ΅ΡΡΠ½ΠΎΠ΅ ΠΏΡΠΎΠΈΠ·Π²Π΅Π΄Π΅Π½ΠΈΠ΅ Π΄Π²ΡΡ Π²Π΅ΠΊΡΠΎΡΠΎΠ² |
2. | Π€ΠΎΡΠΌΡΠ»Π° ΠΏΠ΅ΡΠ΅ΠΊΡΠ΅ΡΡΠ½ΠΎΠ³ΠΎ ΠΏΡΠΎΠΈΠ·Π²Π΅Π΄Π΅Π½ΠΈΡ |
3. | ΠΡΠ°Π²ΠΈΠ»ΠΎ ΠΏΡΠ°Π²ΠΎΠΉ ΡΡΠΊΠΈ ΠΏΠ΅ΡΠ΅ΠΊΡΠ΅ΡΡΠ½ΠΎΠ³ΠΎ ΠΏΡΠΎΠΈΠ·Π²Π΅Π΄Π΅Π½ΠΈΡ |
4. | Π‘Π²ΠΎΠΉΡΡΠ²Π° ΠΏΠ΅ΡΠ΅ΠΊΡΠ΅ΡΡΠ½ΠΎΠ³ΠΎ ΠΏΡΠΎΠ΄ΡΠΊΡΠ° |
5.![]() | ΠΡΠΎΠ΄ΡΠΊΡ ΡΡΠΎΠΉΠ½ΠΎΠ³ΠΎ ΠΊΡΠ΅ΡΡΠ° |
6. | ΠΡΠΈΠΌΠ΅Ρ ΠΏΠ΅ΡΠ΅ΠΊΡΠ΅ΡΡΠ½ΠΎΠ³ΠΎ ΠΏΡΠΎΠΈΠ·Π²Π΅Π΄Π΅Π½ΠΈΡ |
7. | Π§Π°ΡΡΠΎ Π·Π°Π΄Π°Π²Π°Π΅ΠΌΡΠ΅ Π²ΠΎΠΏΡΠΎΡΡ ΠΎ Π²Π΅ΠΊΡΠΎΡΠ½ΠΎΠΌ ΠΏΡΠΎΠΈΠ·Π²Π΅Π΄Π΅Π½ΠΈΠΈ Π΄Π²ΡΡ Π²Π΅ΠΊΡΠΎΡΠΎΠ² |
ΠΠ΅ΡΠ΅ΠΊΡΠ΅ΡΡΠ½ΠΎΠ΅ ΠΏΡΠΎΠΈΠ·Π²Π΅Π΄Π΅Π½ΠΈΠ΅ Π΄Π²ΡΡ Π²Π΅ΠΊΡΠΎΡΠΎΠ²
ΠΠ΅ΡΠ΅ΠΊΡΠ΅ΡΡΠ½ΠΎΠ΅ ΠΏΡΠΎΠΈΠ·Π²Π΅Π΄Π΅Π½ΠΈΠ΅ β ΡΡΠΎ ΡΠΎΡΠΌΠ° ΡΠΌΠ½ΠΎΠΆΠ΅Π½ΠΈΡ Π²Π΅ΠΊΡΠΎΡΠΎΠ², Π²ΡΠΏΠΎΠ»Π½ΡΠ΅ΠΌΠ°Ρ ΠΌΠ΅ΠΆΠ΄Ρ Π΄Π²ΡΠΌΡ Π²Π΅ΠΊΡΠΎΡΠ°ΠΌΠΈ ΡΠ°Π·Π½ΠΎΠΉ ΠΏΡΠΈΡΠΎΠ΄Ρ ΠΈΠ»ΠΈ Π²ΠΈΠ΄Π°. ΠΠ΅ΠΊΡΠΎΡ ΠΈΠΌΠ΅Π΅Ρ ΠΊΠ°ΠΊ Π²Π΅Π»ΠΈΡΠΈΠ½Ρ, ΡΠ°ΠΊ ΠΈ Π½Π°ΠΏΡΠ°Π²Π»Π΅Π½ΠΈΠ΅. ΠΡ ΠΌΠΎΠΆΠ΅ΠΌ ΡΠΌΠ½ΠΎΠΆΠ°ΡΡ Π΄Π²Π° ΠΈΠ»ΠΈ Π±ΠΎΠ»Π΅Π΅ Π²Π΅ΠΊΡΠΎΡΠΎΠ² Π½Π° ΠΏΠ΅ΡΠ΅ΠΊΡΠ΅ΡΡΠ½ΠΎΠ΅ ΠΏΡΠΎΠΈΠ·Π²Π΅Π΄Π΅Π½ΠΈΠ΅ ΠΈ ΡΠΊΠ°Π»ΡΡΠ½ΠΎΠ΅ ΠΏΡΠΎΠΈΠ·Π²Π΅Π΄Π΅Π½ΠΈΠ΅. ΠΠΎΠ³Π΄Π° Π΄Π²Π° Π²Π΅ΠΊΡΠΎΡΠ° ΠΏΠ΅ΡΠ΅ΠΌΠ½ΠΎΠΆΠ°ΡΡΡΡ Π΄ΡΡΠ³ Ρ Π΄ΡΡΠ³ΠΎΠΌ ΠΈ ΠΏΡΠΎΠΈΠ·Π²Π΅Π΄Π΅Π½ΠΈΠ΅ Π²Π΅ΠΊΡΠΎΡΠΎΠ² ΡΠ°ΠΊΠΆΠ΅ ΡΠ²Π»ΡΠ΅ΡΡΡ Π²Π΅ΠΊΡΠΎΡΠ½ΠΎΠΉ Π²Π΅Π»ΠΈΡΠΈΠ½ΠΎΠΉ, ΡΠΎ ΡΠ΅Π·ΡΠ»ΡΡΠΈΡΡΡΡΠΈΠΉ Π²Π΅ΠΊΡΠΎΡ Π½Π°Π·ΡΠ²Π°Π΅ΡΡΡ ΠΏΠ΅ΡΠ΅ΠΊΡΠ΅ΡΡΠ½ΡΠΌ ΠΏΡΠΎΠΈΠ·Π²Π΅Π΄Π΅Π½ΠΈΠ΅ΠΌ Π΄Π²ΡΡ Π²Π΅ΠΊΡΠΎΡΠΎΠ² ΠΈΠ»ΠΈ Π²Π΅ΠΊΡΠΎΡΠ½ΡΠΌ ΠΏΡΠΎΠΈΠ·Π²Π΅Π΄Π΅Π½ΠΈΠ΅ΠΌ. Π Π΅Π·ΡΠ»ΡΡΠΈΡΡΡΡΠΈΠΉ Π²Π΅ΠΊΡΠΎΡ ΠΏΠ΅ΡΠΏΠ΅Π½Π΄ΠΈΠΊΡΠ»ΡΡΠ΅Π½ ΠΏΠ»ΠΎΡΠΊΠΎΡΡΠΈ, ΡΠΎΠ΄Π΅ΡΠΆΠ°ΡΠ΅ΠΉ Π΄Π²Π° Π·Π°Π΄Π°Π½Π½ΡΡ Π²Π΅ΠΊΡΠΎΡΠ°.
ΠΠΏΡΠ΅Π΄Π΅Π»Π΅Π½ΠΈΠ΅ Π²Π΅ΠΊΡΠΎΡΠ½ΠΎΠ³ΠΎ ΠΏΡΠΎΠΈΠ·Π²Π΅Π΄Π΅Π½ΠΈΡ
ΠΡΠ»ΠΈ A ΠΈ B β Π΄Π²Π° Π½Π΅Π·Π°Π²ΠΈΡΠΈΠΌΡΡ
Π²Π΅ΠΊΡΠΎΡΠ°, ΡΠΎ ΡΠ΅Π·ΡΠ»ΡΡΠ°Ρ Π²Π΅ΠΊΡΠΎΡΠ½ΠΎΠ³ΠΎ ΠΏΡΠΎΠΈΠ·Π²Π΅Π΄Π΅Π½ΠΈΡ ΡΡΠΈΡ
Π΄Π²ΡΡ
Π²Π΅ΠΊΡΠΎΡΠΎΠ² (Ax B) ΠΏΠ΅ΡΠΏΠ΅Π½Π΄ΠΈΠΊΡΠ»ΡΡΠ΅Π½ ΠΎΠ±ΠΎΠΈΠΌ Π²Π΅ΠΊΡΠΎΡΠ°ΠΌ ΠΈ Π½ΠΎΡΠΌΠ°Π»Π΅Π½ ΠΊ ΠΏΠ»ΠΎΡΠΊΠΎΡΡΠΈ, ΡΠΎΠ΄Π΅ΡΠΆΠ°ΡΠ΅ΠΉ ΠΎΠ±Π° Π²Π΅ΠΊΡΠΎΡΠ°. ΠΠ½ ΠΏΡΠ΅Π΄ΡΡΠ°Π²Π»Π΅Π½:
Π Ρ
Π = |Π| |Π| sin ΞΈ
ΠΡ ΠΌΠΎΠΆΠ΅ΠΌ ΠΏΠΎΠ½ΡΡΡ ΡΡΠΎ Π½Π° ΠΏΡΠΈΠΌΠ΅ΡΠ΅, ΡΡΠΎ Π΅ΡΠ»ΠΈ Ρ Π½Π°Ρ Π΅ΡΡΡ Π΄Π²Π° Π²Π΅ΠΊΡΠΎΡΠ°, Π»Π΅ΠΆΠ°ΡΠΈΡ Π² ΠΏΠ»ΠΎΡΠΊΠΎΡΡΠΈ X-Y, ΡΠΎ ΠΈΡ Π²Π΅ΠΊΡΠΎΡΠ½ΠΎΠ΅ ΠΏΡΠΎΠΈΠ·Π²Π΅Π΄Π΅Π½ΠΈΠ΅ Π΄Π°ΡΡ ΡΠ΅Π·ΡΠ»ΡΡΠΈΡΡΡΡΠΈΠΉ Π²Π΅ΠΊΡΠΎΡ Π² Π½Π°ΠΏΡΠ°Π²Π»Π΅Π½ΠΈΠΈ ΠΎΡΠΈ Z, ΠΊΠΎΡΠΎΡΠ°Ρ ΠΏΠ΅ΡΠΏΠ΅Π½Π΄ΠΈΠΊΡΠ»ΡΡΠ½Π° ΠΏΠ»ΠΎΡΠΊΠΎΡΡΠΈ XY . Π‘ΠΈΠΌΠ²ΠΎΠ» Γ ΠΈΡΠΏΠΎΠ»ΡΠ·ΡΠ΅ΡΡΡ ΠΌΠ΅ΠΆΠ΄Ρ ΠΈΡΡ ΠΎΠ΄Π½ΡΠΌΠΈ Π²Π΅ΠΊΡΠΎΡΠ°ΠΌΠΈ. ΠΠ΅ΠΊΡΠΎΡΠ½ΠΎΠ΅ ΠΏΡΠΎΠΈΠ·Π²Π΅Π΄Π΅Π½ΠΈΠ΅ ΠΈΠ»ΠΈ ΠΏΠ΅ΡΠ΅ΠΊΡΠ΅ΡΡΠ½ΠΎΠ΅ ΠΏΡΠΎΠΈΠ·Π²Π΅Π΄Π΅Π½ΠΈΠ΅ Π΄Π²ΡΡ Π²Π΅ΠΊΡΠΎΡΠΎΠ² ΠΏΠΎΠΊΠ°Π·Π°Π½ΠΎ ΠΊΠ°ΠΊ:
\(\overrightarrow{a} \times \overrightarrow{b} = \overrightarrow{c}\)
ΠΠ΄Π΅
- \(\overrightarrow{a}\) ΠΈ \(\overrightarrow{b}\) Π΄Π²Π° Π²Π΅ΠΊΡΠΎΡΠ°.
- \(\overrightarrow{c}\) β ΡΠ΅Π·ΡΠ»ΡΡΠΈΡΡΡΡΠΈΠΉ Π²Π΅ΠΊΡΠΎΡ.
ΠΠ΅ΡΠ΅ΠΊΡΠ΅ΡΡΠ½ΠΎΠ΅ ΠΏΡΠΎΠΈΠ·Π²Π΅Π΄Π΅Π½ΠΈΠ΅ Π΄Π²ΡΡ Π²Π΅ΠΊΡΠΎΡΠΎΠ² ΠΠ½Π°ΡΠ΅Π½ΠΈΠ΅
ΠΡΠΏΠΎΠ»ΡΠ·ΡΠΉΡΠ΅ ΠΈΠ·ΠΎΠ±ΡΠ°ΠΆΠ΅Π½ΠΈΠ΅, ΠΏΠΎΠΊΠ°Π·Π°Π½Π½ΠΎΠ΅ Π½ΠΈΠΆΠ΅, ΠΈ ΠΎΠ±ΡΠ°ΡΠΈΡΠ΅ Π²Π½ΠΈΠΌΠ°Π½ΠΈΠ΅ Π½Π° ΡΠ³Π»Ρ ΠΌΠ΅ΠΆΠ΄Ρ Π²Π΅ΠΊΡΠΎΡΠ°ΠΌΠΈ \(\overrightarrow{a}\) ΠΈ \(\overrightarrow{c}\) ΠΈ ΡΠ³Π»Ρ ΠΌΠ΅ΠΆΠ΄Ρ Π²Π΅ΠΊΡΠΎΡΠ°ΠΌΠΈ \(\ overrightarrow{b}\) ΠΈ \(\overrightarrow{c}\). 9\circ\).Ρ. Π΅. \(\overrightarrow{b}\) ΠΈ \(\overrightarrow{c}\) ΡΠ²Π»ΡΡΡΡΡ ΠΎΡΡΠΎΠ³ΠΎΠ½Π°Π»ΡΠ½ΡΠΌΠΈ Π²Π΅ΠΊΡΠΎΡΠ°ΠΌΠΈ.
Π€ΠΎΡΠΌΡΠ»Π° ΠΏΠ΅ΡΠ΅ΠΊΡΠ΅ΡΡΠ½ΠΎΠ³ΠΎ ΠΏΡΠΎΠΈΠ·Π²Π΅Π΄Π΅Π½ΠΈΡ
Π€ΠΎΡΠΌΡΠ»Π° ΠΏΠ΅ΡΠ΅ΠΊΡΠ΅ΡΡΠ½ΠΎΠ³ΠΎ ΠΏΡΠΎΠΈΠ·Π²Π΅Π΄Π΅Π½ΠΈΡ ΠΌΠ΅ΠΆΠ΄Ρ Π»ΡΠ±ΡΠΌΠΈ Π΄Π²ΡΠΌΡ Π²Π΅ΠΊΡΠΎΡΠ°ΠΌΠΈ Π΄Π°Π΅Ρ ΠΏΠ»ΠΎΡΠ°Π΄Ρ ΠΌΠ΅ΠΆΠ΄Ρ ΡΡΠΈΠΌΠΈ Π²Π΅ΠΊΡΠΎΡΠ°ΠΌΠΈ. Π€ΠΎΡΠΌΡΠ»Π° Π²Π΅ΠΊΡΠΎΡΠ½ΠΎΠ³ΠΎ ΠΏΡΠΎΠΈΠ·Π²Π΅Π΄Π΅Π½ΠΈΡ Π΄Π°Π΅Ρ Π²Π΅Π»ΠΈΡΠΈΠ½Ρ ΡΠ΅Π·ΡΠ»ΡΡΠΈΡΡΡΡΠ΅Π³ΠΎ Π²Π΅ΠΊΡΠΎΡΠ°, ΠΊΠΎΡΠΎΡΡΠΉ ΠΏΡΠ΅Π΄ΡΡΠ°Π²Π»ΡΠ΅Ρ ΡΠΎΠ±ΠΎΠΉ ΠΏΠ»ΠΎΡΠ°Π΄Ρ ΠΏΠ°ΡΠ°Π»Π»Π΅Π»ΠΎΠ³ΡΠ°ΠΌΠΌΠ°, Π½Π°ΡΡΠ½ΡΡΠΎΠ³ΠΎ Π½Π° Π΄Π²Π° Π²Π΅ΠΊΡΠΎΡΠ°.
Π€ΠΎΡΠΌΡΠ»Π° ΠΏΠ΅ΡΠ΅ΠΊΡΠ΅ΡΡΠ½ΠΎΠ³ΠΎ ΠΏΡΠΎΠΈΠ·Π²Π΅Π΄Π΅Π½ΠΈΡ
Π Π°ΡΡΠΌΠΎΡΡΠΈΠΌ Π΄Π²Π° Π²Π΅ΠΊΡΠΎΡΠ° \(\overrightarrow{a}\)= \(a_1\hat i+a_2 \hat j+a_3 \hat k\) ΠΈ \(\overrightarrow{b}\) = \(b_1 \ΡΠ»ΡΠΏΠ° i+b_2 \ΡΠ»ΡΠΏΠ° j+b_3 \ΡΠ»ΡΠΏΠ° k\). ΠΡΡΡΡ ΞΈ β ΡΠ³ΠΎΠ», ΠΎΠ±ΡΠ°Π·ΠΎΠ²Π°Π½Π½ΡΠΉ ΠΌΠ΅ΠΆΠ΄Ρ \(\overrightarrow{a}\) ΠΈ \(\overrightarrow{b}\), Π° \(\hat n\) β Π΅Π΄ΠΈΠ½ΠΈΡΠ½ΡΠΉ Π²Π΅ΠΊΡΠΎΡ, ΠΏΠ΅ΡΠΏΠ΅Π½Π΄ΠΈΠΊΡΠ»ΡΡΠ½ΡΠΉ ΠΏΠ»ΠΎΡΠΊΠΎΡΡΠΈ, ΡΠΎΠ΄Π΅ΡΠΆΠ°ΡΠ΅ΠΉ ΠΎΠ±Π΅ \(\overrightarrow{a }\) ΠΈ \(\overrightarrow{b}\). ΠΠ΅ΡΠ΅ΠΊΡΠ΅ΡΡΠ½ΠΎΠ΅ ΠΏΡΠΎΠΈΠ·Π²Π΅Π΄Π΅Π½ΠΈΠ΅ Π΄Π²ΡΡ
Π²Π΅ΠΊΡΠΎΡΠΎΠ² ΠΎΠΏΡΠ΅Π΄Π΅Π»ΡΠ΅ΡΡΡ ΡΠΎΡΠΌΡΠ»ΠΎΠΉ:
\(\overrightarrow{a} \times \overrightarrow{b} = |a| |b| \sin(\theta) \hat n\)
ΠΠ΄Π΅
- \(\mid \overrightarrow a \mid\ ) β Π²Π΅Π»ΠΈΡΠΈΠ½Π° Π²Π΅ΠΊΡΠΎΡΠ° a ΠΈΠ»ΠΈ Π΄Π»ΠΈΠ½Π° \(\overrightarrow{a}\),
- \(\mid \overrightarrow b \mid\) β Π²Π΅Π»ΠΈΡΠΈΠ½Π° Π²Π΅ΠΊΡΠΎΡΠ° b ΠΈΠ»ΠΈ Π΄Π»ΠΈΠ½Π° \(\overrightarrow{b}\).
ΠΡΠ΅Π΄ΠΏΠΎΠ»ΠΎΠΆΠΈΠΌ, ΡΡΠΎ \(\overrightarrow{a}\) ΠΈ \(\overrightarrow{b}\) β Π΄Π²Π° Π²Π΅ΠΊΡΠΎΡΠ°, ΡΠ°ΠΊΠΈΠ΅ ΡΡΠΎ \(\overrightarrow{a}\)= \(a_1\hat i+a_2 \ hat j+a_3 \hat k\) ΠΈ \(\overrightarrow{b}\) = \(b_1 \hat i+b_2 \hat j+b_3 \hat k\), ΡΠΎ Ρ ΠΏΠΎΠΌΠΎΡΡΡ ΠΎΠΏΡΠ΅Π΄Π΅Π»ΠΈΡΠ΅Π»Π΅ΠΉ ΠΌΡ ΠΌΠΎΠ³Π»ΠΈ Π±Ρ Π½Π°ΠΉΡΠΈ Π²Π΅ΠΊΡΠΎΡΠ½ΠΎΠ΅ ΠΏΡΠΎΠΈΠ·Π²Π΅Π΄Π΅Π½ΠΈΠ΅ ΠΈ Π·Π°ΠΏΠΈΡΠΈΡΠ΅ ΡΠ΅Π·ΡΠ»ΡΡΠ°Ρ Π² Π²ΠΈΠ΄Π΅ ΡΠΎΡΠΌΡΠ»Ρ ΠΏΠ΅ΡΠ΅ΠΊΡΠ΅ΡΡΠ½ΠΎΠ³ΠΎ ΠΏΡΠΎΠΈΠ·Π²Π΅Π΄Π΅Π½ΠΈΡ, ΠΈΡΠΏΠΎΠ»ΡΠ·ΡΡ ΠΌΠ°ΡΡΠΈΡΠ½ΡΡ Π·Π°ΠΏΠΈΡΡ.
ΠΠ΅ΡΠ΅ΠΊΡΠ΅ΡΡΠ½ΠΎΠ΅ ΠΏΡΠΎΠΈΠ·Π²Π΅Π΄Π΅Π½ΠΈΠ΅ Π΄Π²ΡΡ Π²Π΅ΠΊΡΠΎΡΠΎΠ² ΡΠ°ΠΊΠΆΠ΅ ΠΏΡΠ΅Π΄ΡΡΠ°Π²Π»ΡΠ΅ΡΡΡ Ρ ΠΏΠΎΠΌΠΎΡΡΡ ΡΠΎΡΠΌΡΠ»Ρ ΠΏΠ΅ΡΠ΅ΠΊΡΠ΅ΡΡΠ½ΠΎΠ³ΠΎ ΠΏΡΠΎΠΈΠ·Π²Π΅Π΄Π΅Π½ΠΈΡ ΠΊΠ°ΠΊ:
\(\overrightarrow{a} \times \overrightarrow{b} = \hat i (a_2b_3-a_3b_2) \\- \hat j (a_1b_3-a_3b_1)\\+ \hat k (a_1b_2-a_2b_1)\)
ΠΡΠΈΠΌΠ΅ΡΠ°Π½ΠΈΠ΅: \( \hat i, \hat j, \text{ ΠΈ } \hat k \) β Π΅Π΄ΠΈΠ½ΠΈΡΠ½ΡΠ΅ Π²Π΅ΠΊΡΠΎΡΡ Π² Π½Π°ΠΏΡΠ°Π²Π»Π΅Π½ΠΈΠΈ ΠΎΡΠΈ x, ΠΎΡΠΈ y ΠΈ ΠΎΡΠΈ z ΡΠΎΠΎΡΠ²Π΅ΡΡΡΠ²Π΅Π½Π½ΠΎ.
ΠΡΠ°Π²ΠΈΠ»ΠΎ ΠΏΡΠ°Π²ΠΎΠΉ ΡΡΠΊΠΈ β Π²Π΅ΠΊΡΠΎΡΠ½ΠΎΠ΅ ΠΏΡΠΎΠΈΠ·Π²Π΅Π΄Π΅Π½ΠΈΠ΅ Π΄Π²ΡΡ Π²Π΅ΠΊΡΠΎΡΠΎΠ²
ΠΡ ΠΌΠΎΠΆΠ΅ΠΌ ΡΠ·Π½Π°ΡΡ Π½Π°ΠΏΡΠ°Π²Π»Π΅Π½ΠΈΠ΅ Π²Π΅ΠΊΡΠΎΡΠ°, ΠΊΠΎΡΠΎΡΡΠΉ ΠΏΠΎΠ»ΡΡΠ°Π΅ΡΡΡ ΠΏΡΠΈ ΠΏΠ΅ΡΠ΅ΠΊΡΠ΅ΡΡΠ½ΠΎΠΌ ΠΏΡΠΎΠΈΠ·Π²Π΅Π΄Π΅Π½ΠΈΠΈ Π΄Π²ΡΡ Π²Π΅ΠΊΡΠΎΡΠΎΠ² ΠΏΠΎ ΠΏΡΠ°Π²ΠΈΠ»Ρ ΠΏΡΠ°Π²ΠΎΠΉ ΡΡΠΊΠΈ. ΠΡ ΡΠ»Π΅Π΄ΡΠ΅ΠΌ ΡΠ»Π΅Π΄ΡΡΡΠ΅ΠΉ ΠΏΡΠΎΡΠ΅Π΄ΡΡΠ΅, ΡΡΠΎΠ±Ρ ΡΠ·Π½Π°ΡΡ Π½Π°ΠΏΡΠ°Π²Π»Π΅Π½ΠΈΠ΅ ΡΠ΅Π·ΡΠ»ΡΡΠ°ΡΠ° ΠΏΠ΅ΡΠ΅ΠΊΡΠ΅ΡΡΠ½ΠΎΠ³ΠΎ ΠΏΡΠΎΠΈΠ·Π²Π΅Π΄Π΅Π½ΠΈΡ Π΄Π²ΡΡ Π²Π΅ΠΊΡΠΎΡΠΎΠ²:
- ΠΠ°ΠΏΡΠ°Π²ΡΡΠ΅ ΡΠΊΠ°Π·Π°ΡΠ΅Π»ΡΠ½ΡΠΉ ΠΏΠ°Π»Π΅Ρ Π² Π½Π°ΠΏΡΠ°Π²Π»Π΅Π½ΠΈΠΈ ΠΏΠ΅ΡΠ²ΠΎΠ³ΠΎ Π²Π΅ΠΊΡΠΎΡΠ° (\(\overrightarrow{A}\)).
- Π‘ΠΎΠ²ΠΌΠ΅ΡΡΠΈΡΠ΅ ΡΡΠ΅Π΄Π½ΠΈΠΉ ΠΏΠ°Π»Π΅Ρ Π² Π½Π°ΠΏΡΠ°Π²Π»Π΅Π½ΠΈΠΈ Π²ΡΠΎΡΠΎΠ³ΠΎ Π²Π΅ΠΊΡΠΎΡΠ° \(\overrightarrow{B}\).
- Π’Π΅ΠΏΠ΅ΡΡ Π±ΠΎΠ»ΡΡΠΎΠΉ ΠΏΠ°Π»Π΅Ρ ΡΠΊΠ°Π·ΡΠ²Π°Π΅Ρ Π² Π½Π°ΠΏΡΠ°Π²Π»Π΅Π½ΠΈΠΈ Π²Π΅ΠΊΡΠΎΡΠ½ΠΎΠ³ΠΎ ΠΏΡΠΎΠΈΠ·Π²Π΅Π΄Π΅Π½ΠΈΡ Π΄Π²ΡΡ Π²Π΅ΠΊΡΠΎΡΠΎΠ².
ΠΠΎΡΠΌΠΎΡΡΠΈΡΠ΅ Π½Π° ΠΈΠ·ΠΎΠ±ΡΠ°ΠΆΠ΅Π½ΠΈΠ΅ Π½ΠΈΠΆΠ΅, ΡΡΠΎΠ±Ρ ΠΏΠΎΠ½ΡΡΡ ΡΡΠΎ Π»ΡΡΡΠ΅.
Π‘Π²ΠΎΠΉΡΡΠ²Π° ΠΏΠ΅ΡΠ΅ΠΊΡΠ΅ΡΡΠ½ΠΎΠ³ΠΎ ΠΏΡΠΎΠΈΠ·Π²Π΅Π΄Π΅Π½ΠΈΡ Π΄Π²ΡΡ Π²Π΅ΠΊΡΠΎΡΠΎΠ²
Π‘Π²ΠΎΠΉΡΡΠ²Π° ΠΏΠ΅ΡΠ΅ΠΊΡΠ΅ΡΡΠ½ΠΎΠ³ΠΎ ΠΏΡΠΎΠΈΠ·Π²Π΅Π΄Π΅Π½ΠΈΡ ΠΏΠΎΠΌΠΎΠ³Π°ΡΡ ΡΡΠ½ΠΎ ΠΏΠΎΠ½ΡΡΡ ΡΠΌΠ½ΠΎΠΆΠ΅Π½ΠΈΠ΅ Π²Π΅ΠΊΡΠΎΡΠΎΠ² ΠΈ ΠΏΠΎΠΌΠΎΠ³Π°ΡΡ Π»Π΅Π³ΠΊΠΎ ΡΠ΅ΡΠ°ΡΡ Π²ΡΠ΅ ΠΏΡΠΎΠ±Π»Π΅ΠΌΡ Π²Π΅ΠΊΡΠΎΡΠ½ΡΡ Π²ΡΡΠΈΡΠ»Π΅Π½ΠΈΠΉ. Π‘Π²ΠΎΠΉΡΡΠ²Π° ΠΏΠ΅ΡΠ΅ΠΊΡΠ΅ΡΡΠ½ΠΎΠ³ΠΎ ΠΏΡΠΎΠΈΠ·Π²Π΅Π΄Π΅Π½ΠΈΡ Π΄Π²ΡΡ Π²Π΅ΠΊΡΠΎΡΠΎΠ² ΡΠ»Π΅Π΄ΡΡΡΠΈΠ΅:
- ΠΠ»ΠΈΠ½Π° Π²Π΅ΠΊΡΠΎΡΠ½ΠΎΠ³ΠΎ ΠΏΡΠΎΠΈΠ·Π²Π΅Π΄Π΅Π½ΠΈΡ Π΄Π²ΡΡ
Π²Π΅ΠΊΡΠΎΡΠΎΠ² \(= \overrightarrow{a} \times \overrightarrow{b} = |a| |b| \sin(\theta)\).
- ΠΠ½ΡΠΈΠΊΠΎΠΌΠΌΡΡΠ°ΡΠΈΠ²Π½ΠΎΠ΅ ΡΠ²ΠΎΠΉΡΡΠ²ΠΎ: \(\overrightarrow{a} \times \overrightarrow{b} = β \overrightarrow{b} \times \overrightarrow{a}\)
- Π Π°ΡΠΏΡΠ΅Π΄Π΅Π»ΠΈΡΠ΅Π»ΡΠ½ΠΎΠ΅ ΡΠ²ΠΎΠΉΡΡΠ²ΠΎ: \(\overrightarrow{a} \times (\overrightarrow{b} + \overrightarrow{c}) = (\overrightarrow{a}\times \overrightarrow{b} )+ (\overrightarrow{a}\times \overrightarrow{Ρ})\)
- ΠΠ΅ΡΠ΅ΠΌΠ½ΠΎΠΆΠ΅Π½ΠΈΠ΅ Π½ΡΠ»Π΅Π²ΠΎΠ³ΠΎ Π²Π΅ΠΊΡΠΎΡΠ°: \(\overrightarrow{a}\times \overrightarrow{0} = \overrightarrow{0}\)
- ΠΠ΅ΡΠ΅ΠΌΠ½ΠΎΠΆΠ΅Π½ΠΈΠ΅ Π²Π΅ΠΊΡΠΎΡΠ° Ρ ΡΠ°ΠΌΠΈΠΌ ΡΠΎΠ±ΠΎΠΉ: \(\overrightarrow{a}\times \overrightarrow{a} = \overrightarrow{0}\)
- Π£ΠΌΠ½ΠΎΠΆΠΈΡΡ Π½Π° ΡΠΊΠ°Π»ΡΡΠ½ΡΡ Π²Π΅Π»ΠΈΡΠΈΠ½Ρ: \(\overrightarrow{c}(\overrightarrow{a}\times \overrightarrow{b}) = c\overrightarrow{a}\times \overrightarrow{b} = \overrightarrow{a}\times c\overrightarrow{b}\)
- ΠΠ΅ΡΠ΅ΠΊΡΠ΅ΡΡΠ½ΠΎΠ΅ ΠΏΡΠΎΠΈΠ·Π²Π΅Π΄Π΅Π½ΠΈΠ΅ Π΅Π΄ΠΈΠ½ΠΈΡΠ½ΡΡ
Π²Π΅ΠΊΡΠΎΡΠΎΠ²: \(\overrightarrow{i}\times \overrightarrow{i} =\overrightarrow{j}\times \overrightarrow{j} = \overrightarrow{k}\times \overrightarrow{k} = 0\)
- \(\overrightarrow{i}\times \overrightarrow{j} = \overrightarrow{k}\\ \overrightarrow{j}\times \overrightarrow{k}= \overrightarrow{i}\\\overrightarrow{k}\times \overrightarrow{i} = \overrightarrow{j}\)
- \(\overrightarrow{j}\times \overrightarrow{i} = \overrightarrow{-k}\\ \overrightarrow{k}\times \overrightarrow{j}= \overrightarrow{-i}\\ \overrightarrow{i} \times \overrightarrow{k} = \overrightarrow{-j}\)
Π’ΡΠΎΠΉΠ½ΠΎΠ΅ ΠΏΠ΅ΡΠ΅ΠΊΡΠ΅ΡΡΠ½ΠΎΠ΅ ΠΏΡΠΎΠΈΠ·Π²Π΅Π΄Π΅Π½ΠΈΠ΅
ΠΠ΅ΡΠ΅ΠΊΡΠ΅ΡΡΠ½ΠΎΠ΅ ΠΏΡΠΎΠΈΠ·Π²Π΅Π΄Π΅Π½ΠΈΠ΅ Π²Π΅ΠΊΡΠΎΡΠ° Π½Π° ΠΏΠ΅ΡΠ΅ΠΊΡΠ΅ΡΡΠ½ΠΎΠ΅ ΠΏΡΠΎΠΈΠ·Π²Π΅Π΄Π΅Π½ΠΈΠ΅ Π΄Π²ΡΡ
Π΄ΡΡΠ³ΠΈΡ
Π²Π΅ΠΊΡΠΎΡΠΎΠ² ΠΏΡΠ΅Π΄ΡΡΠ°Π²Π»ΡΠ΅Ρ ΡΠΎΠ±ΠΎΠΉ ΡΡΠΎΠΉΠ½ΠΎΠ΅ ΠΏΠ΅ΡΠ΅ΠΊΡΠ΅ΡΡΠ½ΠΎΠ΅ ΠΏΡΠΎΠΈΠ·Π²Π΅Π΄Π΅Π½ΠΈΠ΅ Π²Π΅ΠΊΡΠΎΡΠΎΠ². Π Π΅Π·ΡΠ»ΡΡΠ°ΡΠΎΠΌ ΡΡΠΎΠΉΠ½ΠΎΠ³ΠΎ ΠΏΠ΅ΡΠ΅ΠΊΡΠ΅ΡΡΠ½ΠΎΠ³ΠΎ ΠΏΡΠΎΠΈΠ·Π²Π΅Π΄Π΅Π½ΠΈΡ ΡΠ²Π»ΡΠ΅ΡΡΡ Π²Π΅ΠΊΡΠΎΡ. Π Π°Π²Π½ΠΎΠ΄Π΅ΠΉΡΡΠ²ΡΡΡΠΈΠΉ Π²Π΅ΠΊΡΠΎΡΠ° ΡΡΠΎΠΉΠ½ΠΎΠ³ΠΎ ΠΏΠ΅ΡΠ΅ΡΠ΅ΡΠ΅Π½ΠΈΡ Π»Π΅ΠΆΠΈΡ Π² ΠΏΠ»ΠΎΡΠΊΠΎΡΡΠΈ Π΄Π°Π½Π½ΡΡ
ΡΡΠ΅Ρ
Π²Π΅ΠΊΡΠΎΡΠΎΠ². ΠΡΠ»ΠΈ a, b ΠΈ c β Π²Π΅ΠΊΡΠΎΡΡ, ΡΠΎ Π²Π΅ΠΊΡΠΎΡΠ½ΠΎΠ΅ ΡΡΠΎΠΉΠ½ΠΎΠ΅ ΠΏΡΠΎΠΈΠ·Π²Π΅Π΄Π΅Π½ΠΈΠ΅ ΡΡΠΈΡ
Π²Π΅ΠΊΡΠΎΡΠΎΠ² Π±ΡΠ΄Π΅Ρ ΠΈΠΌΠ΅ΡΡ Π²ΠΈΠ΄:
\((\overrightarrow{a}\times \overrightarrow{b}) \times \overrightarrow{c} = (\overrightarrow{a}\cdot \overrightarrow{c})\overrightarrow{b} -(\overrightarrow{b}\cdot \overrightarrow{c}) \overrightarrow{a}\)
ΠΠ΅ΡΠ΅ΠΊΡΠ΅ΡΡΠ½ΠΎΠ΅ ΠΏΡΠΎΠΈΠ·Π²Π΅Π΄Π΅Π½ΠΈΠ΅ Π΄Π²ΡΡ Π²Π΅ΠΊΡΠΎΡΠΎΠ² ΠΡΠΈΠΌΠ΅Ρ
ΠΠ΅ΡΠ΅ΠΊΡΠ΅ΡΡΠ½ΠΎΠ΅ ΠΏΡΠΎΠΈΠ·Π²Π΅Π΄Π΅Π½ΠΈΠ΅ ΠΈΠ³ΡΠ°Π΅Ρ ΡΠ΅ΡΠ°ΡΡΡΡ ΡΠΎΠ»Ρ Π² Π½Π΅ΡΠΊΠΎΠ»ΡΠΊΠΈΡ ΠΎΠ±Π»Π°ΡΡΡΡ Π½Π°ΡΠΊΠΈ ΠΈ ΡΠ΅Ρ Π½ΠΈΠΊΠΈ. ΠΠ²Π° ΠΎΡΠ΅Π½Ρ ΠΏΡΠΎΡΡΡΡ ΠΏΡΠΈΠΌΠ΅ΡΠ° ΠΏΠΎΠΊΠ°Π·Π°Π½Ρ Π½ΠΈΠΆΠ΅.
ΠΡΠΈΠΌΠ΅Ρ 1: ΠΡΠΊΡΡΡΠΈΠ΅ ΠΊΡΠ°Π½Π°: ΠΡ ΠΏΡΠΈΠΊΠ»Π°Π΄ΡΠ²Π°Π΅ΠΌ ΡΠ°Π²Π½ΡΠ΅ ΠΈ ΠΏΡΠΎΡΠΈΠ²ΠΎΠΏΠΎΠ»ΠΎΠΆΠ½ΡΠ΅ ΡΠΈΠ»Ρ ΠΊ Π΄Π²ΡΠΌ Π΄ΠΈΠ°ΠΌΠ΅ΡΡΠ°Π»ΡΠ½ΠΎ ΠΏΡΠΎΡΠΈΠ²ΠΎΠΏΠΎΠ»ΠΎΠΆΠ½ΡΠΌ ΠΊΠΎΠ½ΡΠ°ΠΌ ΠΊΡΠ°Π½Π°. Π ΡΡΠΎΠΌ ΡΠ»ΡΡΠ°Π΅ ΠΏΡΠΈΠΌΠ΅Π½ΡΠ΅ΡΡΡ ΠΊΡΡΡΡΡΠΈΠΉ ΠΌΠΎΠΌΠ΅Π½Ρ. Π Π²Π΅ΠΊΡΠΎΡΠ½ΠΎΠΉ ΡΠΎΡΠΌΠ΅ ΠΊΡΡΡΡΡΠΈΠΉ ΠΌΠΎΠΌΠ΅Π½Ρ ΠΏΡΠ΅Π΄ΡΡΠ°Π²Π»ΡΠ΅Ρ ΡΠΎΠ±ΠΎΠΉ Π²Π΅ΠΊΡΠΎΡΠ½ΠΎΠ΅ ΠΏΡΠΎΠΈΠ·Π²Π΅Π΄Π΅Π½ΠΈΠ΅ ΡΠ°Π΄ΠΈΡΡ-Π²Π΅ΠΊΡΠΎΡΠ° (ΠΎΡ ΠΎΡΠΈ Π²ΡΠ°ΡΠ΅Π½ΠΈΡ Π΄ΠΎ ΡΠΎΡΠΊΠΈ ΠΏΡΠΈΠ»ΠΎΠΆΠ΅Π½ΠΈΡ ΡΠΈΠ»Ρ) ΠΈ Π²Π΅ΠΊΡΠΎΡΠ° ΡΠΈΠ»Ρ.
Ρ. Π΅. \(\overrightarrow{T} = \overrightarrow{r} \times \overrightarrow{F}\)
ΠΡΠΈΠΌΠ΅Ρ 2: Π‘ΠΊΡΡΡΠΈΠ²Π°Π½ΠΈΠ΅ Π±ΠΎΠ»ΡΠ° Π³Π°Π΅ΡΠ½ΡΠΌ ΠΊΠ»ΡΡΠΎΠΌ: Π΄Π»ΠΈΠ½Π° Π³Π°Π΅ΡΠ½ΠΎΠ³ΠΎ ΠΊΠ»ΡΡΠ° ΡΠ°Π²Π½Π° ΠΎΠ΄Π½ΠΎΠΌΡ Π²Π΅ΠΊΡΠΎΡΡ. ΠΠ΄Π΅ΡΡ Π½Π°ΠΏΡΠ°Π²Π»Π΅Π½ΠΈΠ΅, Π² ΠΊΠΎΡΠΎΡΠΎΠΌ ΠΌΡ ΠΏΡΠΈΠΊΠ»Π°Π΄ΡΠ²Π°Π΅ΠΌ ΡΡΠΈΠ»ΠΈΠ΅ ΠΊ Π³Π°Π΅ΡΠ½ΠΎΠΌΡ ΠΊΠ»ΡΡΡ (ΡΡΠΎΠ±Ρ Π·Π°ΡΡΠ½ΡΡΡ ΠΈΠ»ΠΈ ΠΎΡΠ»Π°Π±ΠΈΡΡ Π±ΠΎΠ»Ρ), ΡΠ²Π»ΡΠ΅ΡΡΡ Π΄ΡΡΠ³ΠΈΠΌ Π²Π΅ΠΊΡΠΎΡΠΎΠΌ. Π Π΅Π·ΡΠ»ΡΡΠΈΡΡΡΡΠ΅Π΅ Π½Π°ΠΏΡΠ°Π²Π»Π΅Π½ΠΈΠ΅ Π·Π°ΠΊΡΡΡΠΈΠ²Π°Π½ΠΈΡ ΠΏΠ΅ΡΠΏΠ΅Π½Π΄ΠΈΠΊΡΠ»ΡΡΠ½ΠΎ ΠΎΠ±ΠΎΠΈΠΌ Π²Π΅ΠΊΡΠΎΡΠ°ΠΌ.
ΠΠ°ΠΆΠ½ΡΠ΅ ΠΏΡΠΈΠΌΠ΅ΡΠ°Π½ΠΈΡ
- Π ΡΠ΅Π·ΡΠ»ΡΡΠ°ΡΠ΅ ΠΏΠ΅ΡΠ΅ΠΊΡΠ΅ΡΡΠ½ΠΎΠ³ΠΎ ΠΏΡΠΎΠΈΠ·Π²Π΅Π΄Π΅Π½ΠΈΡ Π΄Π²ΡΡ Π²Π΅ΠΊΡΠΎΡΠΎΠ² ΠΏΠΎΠ»ΡΡΠ°Π΅ΡΡΡ Π²Π΅ΠΊΡΠΎΡ, ΠΎΡΡΠΎΠ³ΠΎΠ½Π°Π»ΡΠ½ΡΠΉ Π΄Π²ΡΠΌ Π·Π°Π΄Π°Π½Π½ΡΠΌ Π²Π΅ΠΊΡΠΎΡΠ°ΠΌ.
- ΠΠ°ΠΏΡΠ°Π²Π»Π΅Π½ΠΈΠ΅ Π²Π΅ΠΊΡΠΎΡΠ½ΠΎΠ³ΠΎ ΠΏΡΠΎΠΈΠ·Π²Π΅Π΄Π΅Π½ΠΈΡ Π΄Π²ΡΡ Π²Π΅ΠΊΡΠΎΡΠΎΠ² ΠΎΠΏΡΠ΅Π΄Π΅Π»ΡΠ΅ΡΡΡ ΠΏΡΠ°Π²ΠΈΠ»ΠΎΠΌ Π±ΠΎΠ»ΡΡΠΎΠ³ΠΎ ΠΏΠ°Π»ΡΡΠ° ΠΏΡΠ°Π²ΠΎΠΉ ΡΡΠΊΠΈ, Π° Π²Π΅Π»ΠΈΡΠΈΠ½Π° ΠΎΠΏΡΠ΅Π΄Π΅Π»ΡΠ΅ΡΡΡ ΠΏΠ»ΠΎΡΠ°Π΄ΡΡ ΠΏΠ°ΡΠ°Π»Π»Π΅Π»ΠΎΠ³ΡΠ°ΠΌΠΌΠ°, ΠΎΠ±ΡΠ°Π·ΠΎΠ²Π°Π½Π½ΠΎΠ³ΠΎ ΠΈΡΡ ΠΎΠ΄Π½ΡΠΌΠΈ Π΄Π²ΡΠΌΡ Π²Π΅ΠΊΡΠΎΡΠ°ΠΌΠΈ \(\overrightarrow{a}\) ΠΈ \(\overrightarrow {Π±}\).
- ΠΠ΅ΡΠ΅ΠΊΡΠ΅ΡΡΠ½ΠΎΠ΅ ΠΏΡΠΎΠΈΠ·Π²Π΅Π΄Π΅Π½ΠΈΠ΅ Π΄Π²ΡΡ Π»ΠΈΠ½Π΅ΠΉΠ½ΡΡ Π²Π΅ΠΊΡΠΎΡΠΎΠ² ΠΈΠ»ΠΈ ΠΏΠ°ΡΠ°Π»Π»Π΅Π»ΡΠ½ΡΡ Π²Π΅ΠΊΡΠΎΡΠΎΠ² ΡΠ²Π»ΡΠ΅ΡΡΡ Π½ΡΠ»Π΅Π²ΡΠΌ Π²Π΅ΠΊΡΠΎΡΠΎΠΌ.
β Π’Π°ΠΊΠΆΠ΅ ΠΏΡΠΎΠ²Π΅ΡΡΡΠ΅:
- Π’ΠΈΠΏΡ Π²Π΅ΠΊΡΠΎΡΠΎΠ²
- ΠΠ΅ΠΊΡΠΎΡΠ½ΡΠ΅ ΡΠΎΡΠΌΡΠ»Ρ
- ΠΠΎΠΌΠΏΠΎΠ½Π΅Π½ΡΡ Π²Π΅ΠΊΡΠΎΡΠ°
- ΠΠ°Π»ΡΠΊΡΠ»ΡΡΠΎΡ ΠΏΠ΅ΡΠ΅ΠΊΡΠ΅ΡΡΠ½ΠΎΠ³ΠΎ ΠΏΡΠΎΠΈΠ·Π²Π΅Π΄Π΅Π½ΠΈΡ
- Π‘Π»ΠΎΠΆΠ΅Π½ΠΈΠ΅ Π²Π΅ΠΊΡΠΎΡΠΎΠ²
ΠΡΠΈΠΌΠ΅ΡΡ ΠΏΠ΅ΡΠ΅ΠΊΡΠ΅ΡΡΠ½ΠΎΠ³ΠΎ ΠΏΡΠΎΠΈΠ·Π²Π΅Π΄Π΅Π½ΠΈΡ Π΄Π²ΡΡ Π²Π΅ΠΊΡΠΎΡΠΎΠ²
ΠΡΠΈΠΌΠ΅Ρ 1: ΠΠ²Π° Π²Π΅ΠΊΡΠΎΡΠ° ΠΈΠΌΠ΅ΡΡ ΡΠΊΠ°Π»ΡΡΠ½ΡΡ Π²Π΅Π»ΠΈΡΠΈΠ½Ρ ΠΊΠ°ΠΊ β£aβ£=2β3 ΠΈ β£bβ£ = 4, Π° ΡΠ³ΠΎΠ» ΠΌΠ΅ΠΆΠ΄Ρ Π΄Π²ΡΠΌΡ Π²Π΅ΠΊΡΠΎΡΠ°ΠΌΠΈ ΡΠ°Π²Π΅Π½ 60 β .
Π Π°ΡΡΡΠΈΡΠ°ΡΡ Π²Π΅ΠΊΡΠΎΡΠ½ΠΎΠ΅ ΠΏΡΠΎΠΈΠ·Π²Π΅Π΄Π΅Π½ΠΈΠ΅ Π΄Π²ΡΡ Π²Π΅ΠΊΡΠΎΡΠΎΠ².
Π Π΅ΡΠ΅Π½ΠΈΠ΅:
ΠΡ Π·Π½Π°Π΅ΠΌ, ΡΡΠΎ sin60Β° = β3/2
ΠΠ΅ΡΠ΅ΠΊΡΠ΅ΡΡΠ½ΠΎΠ΅ ΠΏΡΠΎΠΈΠ·Π²Π΅Π΄Π΅Π½ΠΈΠ΅ Π΄Π²ΡΡ Π²Π΅ΠΊΡΠΎΡΠΎΠ² ΠΎΠΏΡΠ΅Π΄Π΅Π»ΡΠ΅ΡΡΡ Π²ΡΡΠ°ΠΆΠ΅Π½ΠΈΠ΅ΠΌ \(\overrightarrow{a} \times \overrightarrow{b} \) = |a ||b|sin(ΞΈ)\( \ΡΠ»ΡΠΏΠ° n \) = 2β3Γ4Γβ3/2 = 12\( \ΡΠ»ΡΠΏΠ° n \)
ΠΡΠ²Π΅Ρ: ΠΏΠ΅ΡΠ΅ΠΊΡΠ΅ΡΡΠ½ΠΎΠ΅ ΠΏΡΠΎΠΈΠ·Π²Π΅Π΄Π΅Π½ΠΈΠ΅ ΡΠ°Π²Π½ΠΎ 12n.
ΠΠΎΠΏΡΠΎΡ 2: ΠΠ°ΠΉΠ΄ΠΈΡΠ΅ Π²Π΅ΠΊΡΠΎΡΠ½ΠΎΠ΅ ΠΏΡΠΎΠΈΠ·Π²Π΅Π΄Π΅Π½ΠΈΠ΅ Π΄Π²ΡΡ Π²Π΅ΠΊΡΠΎΡΠΎΠ² \(\overrightarrow{a}\) = (3,4,5) ΠΈ \(\overrightarrow{b}\) = (7,8,9)
Π Π΅ΡΠ΅Π½ΠΈΠ΅:
ΠΠ΅ΡΠ΅ΠΊΡΠ΅ΡΡΠ½ΠΎΠ΅ ΠΏΡΠΎΠΈΠ·Π²Π΅Π΄Π΅Π½ΠΈΠ΅ Π·Π°Π΄Π°Π΅ΡΡΡ ΠΊΠ°ΠΊ
\(\begin{align}a \times b &=\begin{matrix} \hat i & \hat j & \hat k\\ 3 & 4 & 5\\ 7 & 8 & 9 \end{matrix}\end{align}\)
= [(4Γ9)β(5Γ8)] \( \ΡΠ»ΡΠΏΠ° {i}\) β[(3Γ9)β(5Γ7)]\( \ΡΠ»ΡΠΏΠ° {j} \)+[(3Γ8)β(4 Γ7)] \( \hat {k}\)
= (36β40)\( \hat i\) β (27β35)\( \hat j\) +(24β28) \( \ ΡΠ»ΡΠΏΠ° k\) = β4\( \hat i\) + 8\( \hat j\) β4\( \hat k\)
ΠΡΠ²Π΅Ρ: β΄ \(\overrightarrow{a} \times \overrightarrow{ b} \) = β4\( \hat i\) + 8\( \hat j\) β4\( \hat k\)
ΠΡΠΈΠΌΠ΅Ρ 3: ΠΡΠ»ΠΈ \(\overrightarrow{a}\) = (2, -4, 4) ΠΈ \(\overrightarrow{b}\) = (4, 0,3), Π½Π°ΠΉΠ΄ΠΈΡΠ΅ ΡΠ³ΠΎΠ» ΠΌΠ΅ΠΆΠ΄Ρ Π½ΠΈΠΌΠΈ.
Π Π΅ΡΠ΅Π½ΠΈΠ΅
\(\overrightarrow{a}\) = 2i β 4j + 4k
\(\overrightarrow{b}\) = 4i + 0j +3k
ΠΠ΅Π»ΠΈΡΠΈΠ½Π° \(\overrightarrow{a }\) ΡΠ°Π²Π½ΠΎ
β£aβ£=β(2 2 +4 2 +4 2 ) = β36 = 6
ΠΠ΅Π»ΠΈΡΠΈΠ½Π° \(\overrightarrow{b}\) ΡΠ°Π²Π½Π°
β£900 bβ£=β(4 2 +0 2 +3 2 ) = β25 = 5
Π‘ΠΎΠ³Π»Π°ΡΠ½ΠΎ ΡΠΎΡΠΌΡΠ»Π΅ ΠΏΠ΅ΡΠ΅ΠΊΡΠ΅ΡΡΠ½ΠΎΠ³ΠΎ ΠΏΡΠΎΠΈΠ·Π²Π΅Π΄Π΅Π½ΠΈΡ, ΠΌΡ ΠΈΠΌΠ΅Π΅ΠΌ\(\begin{align}\overrightarrow{a}\times \overrightarrow{b} &= \begin{matrix} \hat i & \hat j & \hat k\\ 2 & -4 & 4\\ 4 & 0 & 3 \end{matrix}\\\\&=[(-4\times3) β (4\times0)]\hat i \\ β &[(3\times 2) β (4\times 4)] \hat j \\\\ + &[(2\times 0) -(-4\times 4)]\hat k \\\\&= -12\hat i + 10 \hat j +16 \hat k \\ \overrightarrow{a}\times \overrightarrow{b} &= (-12, 10, 16) \end{align}\)
ΠΠ»ΠΈΠ½Π° \(\overrightarrow{c}\) ΡΠ°Π²Π½Π°
β£cβ£=β(β(12) 2 +10 2 +16 2 )
=β0(144+ +256)
=β500
=10β5
\(\begin{align}\overrightarrow{a}\times \overrightarrow{b} &= \mid a \mid \mid b \mid \sin\ ΡΠ΅ΡΠ°\\\sin\theta &= \dfrac{\overrightarrow{a}\times \overrightarrow{b}}{\mid a \mid \mid b \mid}\end{align}\)
sinΞΈ = 10β 5/(5Γ6)
sinΞΈ = β5/3
ΞΈ = sin β1 (β5/3)
ΞΈ = sin β1 (0,74)
ΞΈ = 48 β
ΠΡΠ²Π΅Ρ: Π£Π³ΠΎΠ» ΠΌΠ΅ΠΆΠ΄Ρ Π²Π΅ΠΊΡΠΎΡΠ°ΠΌΠΈ ΡΠ°Π²Π΅Π½ 904 904 904.
ΠΏΠ΅ΡΠ΅ΠΉΡΠΈ ΠΊ ΡΠ»Π°ΠΉΠ΄ΡΠΏΠ΅ΡΠ΅ΠΉΡΠΈ ΠΊ ΡΠ»Π°ΠΉΠ΄ΡΠΏΠ΅ΡΠ΅ΠΉΡΠΈ ΠΊ ΡΠ»Π°ΠΉΠ΄Ρ
Π Π°Π·Π±ΠΈΠ²Π°ΠΉΡΠ΅ ΡΠ»ΠΎΠΆΠ½ΡΠ΅ ΠΊΠΎΠ½ΡΠ΅ΠΏΡΠΈΠΈ Ρ ΠΏΠΎΠΌΠΎΡΡΡ ΠΏΡΠΎΡΡΡΡ Π²ΠΈΠ·ΡΠ°Π»ΡΠ½ΡΡ ΡΡΡΠ΅ΠΊΡΠΎΠ².
ΠΠ°ΡΠ΅ΠΌΠ°ΡΠΈΠΊΠ° Π±ΠΎΠ»ΡΡΠ΅ Π½Π΅ Π±ΡΠ΄Π΅Ρ ΡΠ»ΠΎΠΆΠ½ΡΠΌ ΠΏΡΠ΅Π΄ΠΌΠ΅ΡΠΎΠΌ, ΠΎΡΠΎΠ±Π΅Π½Π½ΠΎ ΠΊΠΎΠ³Π΄Π° Π²Ρ ΠΏΠΎΠ½ΠΈΠΌΠ°Π΅ΡΠ΅ ΠΊΠΎΠ½ΡΠ΅ΠΏΡΠΈΠΈ Ρ ΠΏΠΎΠΌΠΎΡΡΡ Π²ΠΈΠ·ΡΠ°Π»ΠΈΠ·Π°ΡΠΈΠΉ.
ΠΠ°Π±ΡΠΎΠ½ΠΈΡΠΎΠ²Π°ΡΡ Π±Π΅ΡΠΏΠ»Π°ΡΠ½ΡΠΉ ΠΏΡΠΎΠ±Π½ΡΠΉ ΡΡΠΎΠΊ
ΠΡΠ°ΠΊΡΠΈΡΠ΅ΡΠΊΠΈΠ΅ Π²ΠΎΠΏΡΠΎΡΡ ΠΏΠΎ ΠΏΠ΅ΡΠ΅ΠΊΡΠ΅ΡΡΠ½ΠΎΠΌΡ ΠΏΡΠΎΠΈΠ·Π²Π΅Π΄Π΅Π½ΠΈΡ Π΄Π²ΡΡ Π²Π΅ΠΊΡΠΎΡΠΎΠ²
ΠΏΠ΅ΡΠ΅ΠΉΡΠΈ ΠΊ ΡΠ»Π°ΠΉΠ΄ΡΠΏΠ΅ΡΠ΅ΠΉΡΠΈ ΠΊ ΡΠ»Π°ΠΉΠ΄Ρ
Π§Π°ΡΡΠΎ Π·Π°Π΄Π°Π²Π°Π΅ΠΌΡΠ΅ Π²ΠΎΠΏΡΠΎΡΡ ΠΎ ΠΏΠ΅ΡΠ΅ΠΊΡΠ΅ΡΡΠ½ΠΎΠΌ ΠΏΡΠΎΠΈΠ·Π²Π΅Π΄Π΅Π½ΠΈΠΈ Π΄Π²ΡΡ Π²Π΅ΠΊΡΠΎΡΠΎΠ²
Π§ΡΠΎ ΡΠ°ΠΊΠΎΠ΅ Π²Π΅ΠΊΡΠΎΡΠ½ΠΎΠ΅ ΠΏΡΠΎΠΈΠ·Π²Π΅Π΄Π΅Π½ΠΈΠ΅ Π΄Π²ΡΡ Π²Π΅ΠΊΡΠΎΡΠΎΠ²?
Π ΡΠ΅Π·ΡΠ»ΡΡΠ°ΡΠ΅ ΠΏΠ΅ΡΠ΅ΠΊΡΠ΅ΡΡΠ½ΠΎΠ³ΠΎ ΠΏΡΠΎΠΈΠ·Π²Π΅Π΄Π΅Π½ΠΈΡ Π΄Π²ΡΡ Π²Π΅ΠΊΡΠΎΡΠΎΠ² ΠΏΡΠΈ ΡΠΌΠ½ΠΎΠΆΠ΅Π½ΠΈΠΈ ΡΡΠ΅ΡΠΈΠΉ Π²Π΅ΠΊΡΠΎΡ ΠΏΠ΅ΡΠΏΠ΅Π½Π΄ΠΈΠΊΡΠ»ΡΡΠ΅Π½ Π΄Π²ΡΠΌ ΠΈΡΡ ΠΎΠ΄Π½ΡΠΌ Π²Π΅ΠΊΡΠΎΡΠ°ΠΌ. ΠΠ΅Π»ΠΈΡΠΈΠ½Π° ΡΠ΅Π·ΡΠ»ΡΡΠΈΡΡΡΡΠ΅Π³ΠΎ Π²Π΅ΠΊΡΠΎΡΠ° ΠΎΠΏΡΠ΅Π΄Π΅Π»ΡΠ΅ΡΡΡ ΠΏΠ»ΠΎΡΠ°Π΄ΡΡ ΠΏΠ°ΡΠ°Π»Π»Π΅Π»ΠΎΠ³ΡΠ°ΠΌΠΌΠ° ΠΌΠ΅ΠΆΠ΄Ρ Π½ΠΈΠΌΠΈ, Π° Π΅Π³ΠΎ Π½Π°ΠΏΡΠ°Π²Π»Π΅Π½ΠΈΠ΅ ΠΌΠΎΠΆΠ½ΠΎ ΠΎΠΏΡΠ΅Π΄Π΅Π»ΠΈΡΡ ΠΏΠΎ ΠΏΡΠ°Π²ΠΈΠ»Ρ ΠΏΡΠ°Π²ΠΎΠΉ ΡΡΠΊΠΈ. a Γ b = c, Π³Π΄Π΅ c β Π²Π΅ΠΊΡΠΎΡΠ½ΠΎΠ΅ ΠΏΡΠΎΠΈΠ·Π²Π΅Π΄Π΅Π½ΠΈΠ΅ Π΄Π²ΡΡ Π²Π΅ΠΊΡΠΎΡΠΎΠ² a ΠΈ b.
ΠΠ°ΠΊΠΎΠ² ΡΠ΅Π·ΡΠ»ΡΡΠ°Ρ Π²Π΅ΠΊΡΠΎΡΠ½ΠΎΠ³ΠΎ ΠΏΠ΅ΡΠ΅ΠΊΡΠ΅ΡΡΠ½ΠΎΠ³ΠΎ ΠΏΡΠΎΠΈΠ·Π²Π΅Π΄Π΅Π½ΠΈΡ?
ΠΠΎΠ³Π΄Π° ΠΌΡ Π½Π°Ρ
ΠΎΠ΄ΠΈΠΌ Π²Π΅ΠΊΡΠΎΡΠ½ΠΎΠ΅ ΠΏΡΠΎΠΈΠ·Π²Π΅Π΄Π΅Π½ΠΈΠ΅ Π΄Π²ΡΡ
Π²Π΅ΠΊΡΠΎΡΠΎΠ², ΠΌΡ ΠΏΠΎΠ»ΡΡΠ°Π΅ΠΌ Π΄ΡΡΠ³ΠΎΠΉ Π²Π΅ΠΊΡΠΎΡ, Π²ΡΡΠΎΠ²Π½Π΅Π½Π½ΡΠΉ ΠΏΠ΅ΡΠΏΠ΅Π½Π΄ΠΈΠΊΡΠ»ΡΡΠ½ΠΎ ΠΏΠ»ΠΎΡΠΊΠΎΡΡΠΈ, ΡΠΎΠ΄Π΅ΡΠΆΠ°ΡΠ΅ΠΉ Π΄Π²Π° Π²Π΅ΠΊΡΠΎΡΠ°. ΠΠ΅Π»ΠΈΡΠΈΠ½Π° ΡΠ΅Π·ΡΠ»ΡΡΠΈΡΡΡΡΠ΅Π³ΠΎ Π²Π΅ΠΊΡΠΎΡΠ° ΡΠ²Π»ΡΠ΅ΡΡΡ ΠΏΡΠΎΠΈΠ·Π²Π΅Π΄Π΅Π½ΠΈΠ΅ΠΌ Π³ΡΠ΅Ρ
Π° ΡΠ³Π»Π° ΠΌΠ΅ΠΆΠ΄Ρ Π²Π΅ΠΊΡΠΎΡΠ°ΠΌΠΈ ΠΈ Π²Π΅Π»ΠΈΡΠΈΠ½ΠΎΠΉ Π΄Π²ΡΡ
Π²Π΅ΠΊΡΠΎΡΠΎΠ². Π° Γ Π± = | Π° | |Π±| Π³ΡΠ΅Ρ
ΞΈ.
Π§ΡΠΎ ΡΠ°ΠΊΠΎΠ΅ ΡΠΊΠ°Π»ΡΡΠ½ΠΎΠ΅ ΠΏΡΠΎΠΈΠ·Π²Π΅Π΄Π΅Π½ΠΈΠ΅ ΠΈ Π²Π΅ΠΊΡΠΎΡΠ½ΠΎΠ΅ ΠΏΡΠΎΠΈΠ·Π²Π΅Π΄Π΅Π½ΠΈΠ΅ Π΄Π²ΡΡ Π²Π΅ΠΊΡΠΎΡΠΎΠ²?
ΠΠ΅ΠΊΡΠΎΡΡ ΠΌΠΎΠΆΠ½ΠΎ ΡΠΌΠ½ΠΎΠΆΠ°ΡΡ Π΄Π²ΡΠΌΡ ΡΠ°Π·Π½ΡΠΌΠΈ ΡΠΏΠΎΡΠΎΠ±Π°ΠΌΠΈ, Ρ. Π΅. ΡΠΊΠ°Π»ΡΡΠ½ΡΠΌ ΠΏΡΠΎΠΈΠ·Π²Π΅Π΄Π΅Π½ΠΈΠ΅ΠΌ ΠΈ ΠΏΠ΅ΡΠ΅ΠΊΡΠ΅ΡΡΠ½ΡΠΌ ΠΏΡΠΎΠΈΠ·Π²Π΅Π΄Π΅Π½ΠΈΠ΅ΠΌ. Π Π΅Π·ΡΠ»ΡΡΠ°ΡΡ ΠΎΠ±ΠΎΠΈΡ
ΡΡΠΈΡ
ΡΠΌΠ½ΠΎΠΆΠ΅Π½ΠΈΠΉ Π²Π΅ΠΊΡΠΎΡΠΎΠ² ΡΠ°Π·Π»ΠΈΡΠ½Ρ. Π ΡΠ΅Π·ΡΠ»ΡΡΠ°ΡΠ΅ ΡΠΊΠ°Π»ΡΡΠ½ΠΎΠ΅ ΠΏΡΠΎΠΈΠ·Π²Π΅Π΄Π΅Π½ΠΈΠ΅ Π΄Π°Π΅Ρ ΡΠΊΠ°Π»ΡΡΠ½ΡΡ Π²Π΅Π»ΠΈΡΠΈΠ½Ρ, ΡΠΎΠ³Π΄Π° ΠΊΠ°ΠΊ Π²Π΅ΠΊΡΠΎΡΠ½ΠΎΠ΅ ΠΏΡΠΎΠΈΠ·Π²Π΅Π΄Π΅Π½ΠΈΠ΅ Π΄Π°Π΅Ρ Π²Π΅ΠΊΡΠΎΡΠ½ΡΡ Π²Π΅Π»ΠΈΡΠΈΠ½Ρ. Π‘ΠΊΠ°Π»ΡΡΠ½ΠΎΠ΅ ΠΏΡΠΎΠΈΠ·Π²Π΅Π΄Π΅Π½ΠΈΠ΅ β ΡΡΠΎ ΡΠΊΠ°Π»ΡΡΠ½ΠΎΠ΅ ΠΏΡΠΎΠΈΠ·Π²Π΅Π΄Π΅Π½ΠΈΠ΅ Π΄Π²ΡΡ
Π²Π΅ΠΊΡΠΎΡΠΎΠ², Π° ΠΏΠ΅ΡΠ΅ΠΊΡΠ΅ΡΡΠ½ΠΎΠ΅ ΠΏΡΠΎΠΈΠ·Π²Π΅Π΄Π΅Π½ΠΈΠ΅ Π΄Π²ΡΡ
Π²Π΅ΠΊΡΠΎΡΠΎΠ² β ΡΡΠΎ Π²Π΅ΠΊΡΠΎΡΠ½ΠΎΠ΅ ΠΏΡΠΎΠΈΠ·Π²Π΅Π΄Π΅Π½ΠΈΠ΅ Π΄Π²ΡΡ
Π²Π΅ΠΊΡΠΎΡΠΎΠ². Π‘ΠΊΠ°Π»ΡΡΠ½ΠΎΠ΅ ΠΏΡΠΎΠΈΠ·Π²Π΅Π΄Π΅Π½ΠΈΠ΅ ΡΠ°ΠΊΠΆΠ΅ ΠΈΠ·Π²Π΅ΡΡΠ½ΠΎ ΠΊΠ°ΠΊ ΡΠΊΠ°Π»ΡΡΠ½ΠΎΠ΅ ΠΏΡΠΎΠΈΠ·Π²Π΅Π΄Π΅Π½ΠΈΠ΅, Π° ΠΏΠ΅ΡΠ΅ΠΊΡΠ΅ΡΡΠ½ΠΎΠ΅ ΠΏΡΠΎΠΈΠ·Π²Π΅Π΄Π΅Π½ΠΈΠ΅ ΡΠ°ΠΊΠΆΠ΅ ΠΈΠ·Π²Π΅ΡΡΠ½ΠΎ ΠΊΠ°ΠΊ Π²Π΅ΠΊΡΠΎΡΠ½ΠΎΠ΅ ΠΏΡΠΎΠΈΠ·Π²Π΅Π΄Π΅Π½ΠΈΠ΅. ΠΠ΅ΠΊΡΠΎΡΠ½ΠΎΠ΅ ΠΏΡΠΎΠΈΠ·Π²Π΅Π΄Π΅Π½ΠΈΠ΅ Π΄Π²ΡΡ
Π²Π΅ΠΊΡΠΎΡΠΎΠ² Π·Π°Π΄Π°Π΅ΡΡΡ ΠΊΠ°ΠΊ: \(\overrightarrow{a} \times \overrightarrow{b} = |a| |b| \sin(\theta) \hat n\) ΠΈ ΡΠΎΡΠΌΡΠ»Π° ΡΠΊΠ°Π»ΡΡΠ½ΠΎΠ³ΠΎ ΠΏΡΠΎΠΈΠ·Π²Π΅Π΄Π΅Π½ΠΈΡ Π΄Π²ΡΡ
Π²Π΅ΠΊΡΠΎΡΠΎΠ² Π·Π°Π΄Π°Π΅ΡΡΡ ΠΊΠ°ΠΊ: \(\overrightarrow{a}. \overrightarrow{b} = |a| |b| \cos(\theta)\)
ΠΠ°ΠΊ Π½Π°ΠΉΡΠΈ Π²Π΅ΠΊΡΠΎΡΠ½ΠΎΠ΅ ΠΏΡΠΎΠΈΠ·Π²Π΅Π΄Π΅Π½ΠΈΠ΅ Π΄Π²ΡΡ Π²Π΅ΠΊΡΠΎΡΠΎΠ²?
ΠΠ΅ΡΠ΅ΠΊΡΠ΅ΡΡΠ½ΠΎΠ΅ ΠΏΡΠΎΠΈΠ·Π²Π΅Π΄Π΅Π½ΠΈΠ΅ Π΄Π²ΡΡ Π²Π΅ΠΊΡΠΎΡΠΎΠ² ΠΎΠΏΡΠ΅Π΄Π΅Π»ΡΠ΅ΡΡΡ ΠΏΠΎ ΡΠΎΡΠΌΡΠ»Π΅: \(\overrightarrow{a} \times \overrightarrow{b} = |a| |b| \sin(\theta) \hat n\)
ΠΠ΄Π΅
- |\(\overrightarrow a\)| Π²Π΅Π»ΠΈΡΠΈΠ½Π° ΠΈΠ»ΠΈ Π΄Π»ΠΈΠ½Π° \(\overrightarrow{a}\),
- |\(\overrightarrow b\)| ΡΠ²Π»ΡΠ΅ΡΡΡ Π²Π΅Π»ΠΈΡΠΈΠ½ΠΎΠΉ ΠΈΠ»ΠΈ Π΄Π»ΠΈΠ½ΠΎΠΉ \(\overrightarrow{b}\)
ΠΠΎΡΠ΅ΠΌΡ ΠΏΠ΅ΡΠ΅ΠΊΡΠ΅ΡΡΠ½ΠΎΠ΅ ΠΏΡΠΎΠΈΠ·Π²Π΅Π΄Π΅Π½ΠΈΠ΅ ΡΠ²Π»ΡΠ΅ΡΡΡ ΡΠΈΠ½ΡΡΠΎΠΈΠ΄Π°Π»ΡΠ½ΡΠΌ?
ΠΠΎΡΠΊΠΎΠ»ΡΠΊΡ ΞΈ β ΡΡΠΎ ΡΠ³ΠΎΠ» ΠΌΠ΅ΠΆΠ΄Ρ Π΄Π²ΡΠΌΡ ΠΈΡΡ ΠΎΠ΄Π½ΡΠΌΠΈ Π²Π΅ΠΊΡΠΎΡΠ°ΠΌΠΈ, ΠΈΡΠΏΠΎΠ»ΡΠ·ΡΠ΅ΡΡΡ sin ΞΈ, ΠΏΠΎΡΠΊΠΎΠ»ΡΠΊΡ ΠΏΠ»ΠΎΡΠ°Π΄Ρ ΠΏΠ°ΡΠ°Π»Π»Π΅Π»ΠΎΠ³ΡΠ°ΠΌΠΌΠ° ΠΏΠΎΠ»ΡΡΠ°Π΅ΡΡΡ ΠΏΡΡΠ΅ΠΌ ΠΏΠ΅ΡΠ΅ΠΊΡΠ΅ΡΡΠ½ΠΎΠ³ΠΎ ΠΏΡΠΎΠΈΠ·Π²Π΅Π΄Π΅Π½ΠΈΡ Π΄Π²ΡΡ Π²Π΅ΠΊΡΠΎΡΠΎΠ².
ΠΡΠ΅Π³Π΄Π° Π»ΠΈ ΠΏΠΎΠ»ΠΎΠΆΠΈΡΠ΅Π»ΡΠ½ΠΎΠ΅ ΠΏΡΠΎΠΈΠ·Π²Π΅Π΄Π΅Π½ΠΈΠ΅ Π΄Π²ΡΡ Π²Π΅ΠΊΡΠΎΡΠΎΠ²?
ΠΠΎΠ³Π΄Π° ΡΠ³ΠΎΠ» ΠΌΠ΅ΠΆΠ΄Ρ Π΄Π²ΡΠΌΡ ΠΈΡΡ ΠΎΠ΄Π½ΡΠΌΠΈ Π²Π΅ΠΊΡΠΎΡΠ°ΠΌΠΈ ΠΈΠ·ΠΌΠ΅Π½ΡΠ΅ΡΡΡ ΠΎΡ 180Β° Π΄ΠΎ 360Β°, Π²Π΅ΠΊΡΠΎΡΠ½ΠΎΠ΅ ΠΏΡΠΎΠΈΠ·Π²Π΅Π΄Π΅Π½ΠΈΠ΅ ΡΡΠ°Π½ΠΎΠ²ΠΈΡΡΡ ΠΎΡΡΠΈΡΠ°ΡΠ΅Π»ΡΠ½ΡΠΌ. ΠΡΠΎ ΡΠ²ΡΠ·Π°Π½ΠΎ Ρ ΡΠ΅ΠΌ, ΡΡΠΎ sin ΞΈ ΠΎΡΡΠΈΡΠ°ΡΠ΅Π»Π΅Π½ Π΄Π»Ρ 180Β°< ΞΈ <360Β°.
Π ΡΠ΅ΠΌ ΡΠ°Π·Π½ΠΈΡΠ° ΠΌΠ΅ΠΆΠ΄Ρ ΡΠΊΠ°Π»ΡΡΠ½ΡΠΌ ΠΏΡΠΎΠΈΠ·Π²Π΅Π΄Π΅Π½ΠΈΠ΅ΠΌ ΠΈ ΠΏΠ΅ΡΠ΅ΠΊΡΠ΅ΡΡΠ½ΡΠΌ ΠΏΡΠΎΠΈΠ·Π²Π΅Π΄Π΅Π½ΠΈΠ΅ΠΌ Π΄Π²ΡΡ Π²Π΅ΠΊΡΠΎΡΠΎΠ²?
ΠΡΠΈ ΡΠΌΠ½ΠΎΠΆΠ΅Π½ΠΈΠΈ Π²Π΅ΠΊΡΠΎΡΠΎΠ² ΡΠΊΠ°Π»ΡΡΠ½ΠΎΠ΅ ΠΏΡΠΎΠΈΠ·Π²Π΅Π΄Π΅Π½ΠΈΠ΅ ΠΈΡΡ
ΠΎΠ΄Π½ΡΡ
Π²Π΅ΠΊΡΠΎΡΠΎΠ² Π΄Π°Π΅Ρ ΡΠΊΠ°Π»ΡΡΠ½ΡΡ Π²Π΅Π»ΠΈΡΠΈΠ½Ρ, ΡΠΎΠ³Π΄Π° ΠΊΠ°ΠΊ ΠΏΠ΅ΡΠ΅ΠΊΡΠ΅ΡΡΠ½ΠΎΠ΅ ΠΏΡΠΎΠΈΠ·Π²Π΅Π΄Π΅Π½ΠΈΠ΅ Π΄Π²ΡΡ
Π²Π΅ΠΊΡΠΎΡΠΎΠ² Π΄Π°Π΅Ρ Π²Π΅ΠΊΡΠΎΡΠ½ΡΡ Π²Π΅Π»ΠΈΡΠΈΠ½Ρ. Π‘ΠΊΠ°Π»ΡΡΠ½ΠΎΠ΅ ΠΏΡΠΎΠΈΠ·Π²Π΅Π΄Π΅Π½ΠΈΠ΅ β ΡΡΠΎ ΠΏΡΠΎΠΈΠ·Π²Π΅Π΄Π΅Π½ΠΈΠ΅ Π²Π΅Π»ΠΈΡΠΈΠ½Ρ Π²Π΅ΠΊΡΠΎΡΠΎΠ² ΠΈ cos ΡΠ³Π»Π° ΠΌΠ΅ΠΆΠ΄Ρ Π½ΠΈΠΌΠΈ. Π° . Π± = |Π°| |Π±| cosΞΈ. ΠΠ΅ΠΊΡΠΎΡΠ½ΠΎΠ΅ ΠΏΡΠΎΠΈΠ·Π²Π΅Π΄Π΅Π½ΠΈΠ΅ β ΡΡΠΎ ΠΏΡΠΎΠΈΠ·Π²Π΅Π΄Π΅Π½ΠΈΠ΅ Π²Π΅Π»ΠΈΡΠΈΠ½Ρ Π²Π΅ΠΊΡΠΎΡΠΎΠ² Π½Π° ΡΠΈΠ½ΡΡ ΡΠ³Π»Π° ΠΌΠ΅ΠΆΠ΄Ρ Π½ΠΈΠΌΠΈ. Π° Γ Π± = | Π° | |Π±| Π³ΡΠ΅Ρ
ΞΈ.
Π§ΡΠΎ ΡΠ°ΠΊΠΎΠ΅ ΡΠΎΡΠΌΡΠ»Π° Π²Π΅ΠΊΡΠΎΡΠ½ΠΎΠ³ΠΎ ΠΏΡΠΎΠΈΠ·Π²Π΅Π΄Π΅Π½ΠΈΡ Π΄Π»Ρ Π΄Π²ΡΡ Π²Π΅ΠΊΡΠΎΡΠΎΠ²?
Π€ΠΎΡΠΌΡΠ»Π° Π²Π΅ΠΊΡΠΎΡΠ½ΠΎΠ³ΠΎ ΠΏΡΠΎΠΈΠ·Π²Π΅Π΄Π΅Π½ΠΈΡ ΠΎΠΏΡΠ΅Π΄Π΅Π»ΡΠ΅Ρ Π²Π΅ΠΊΡΠΎΡΠ½ΠΎΠ΅ ΠΏΡΠΎΠΈΠ·Π²Π΅Π΄Π΅Π½ΠΈΠ΅ Π΄Π»Ρ Π»ΡΠ±ΡΡ Π΄Π²ΡΡ Π·Π°Π΄Π°Π½Π½ΡΡ Π²Π΅ΠΊΡΠΎΡΠΎΠ², Π·Π°Π΄Π°Π²Π°Ρ ΠΏΠ»ΠΎΡΠ°Π΄Ρ ΠΌΠ΅ΠΆΠ΄Ρ ΡΡΠΈΠΌΠΈ Π²Π΅ΠΊΡΠΎΡΠ°ΠΌΠΈ. Π€ΠΎΡΠΌΡΠ»Π° ΠΏΠ΅ΡΠ΅ΠΊΡΠ΅ΡΡΠ½ΠΎΠ³ΠΎ ΠΏΡΠΎΠΈΠ·Π²Π΅Π΄Π΅Π½ΠΈΡ ΠΈΠΌΠ΅Π΅Ρ Π²ΠΈΠ΄ \(\overrightarrow{A} Γ \overrightarrow{B} =|A||B| sinβ‘ΞΈ\), Π³Π΄Π΅ |A| = Π²Π΅Π»ΠΈΡΠΈΠ½Π° Π²Π΅ΠΊΡΠΎΡΠ° A, |B| = Π²Π΅Π»ΠΈΡΠΈΠ½Π° Π²Π΅ΠΊΡΠΎΡΠ° B, Π° ΞΈ = ΡΠ³ΠΎΠ» ΠΌΠ΅ΠΆΠ΄Ρ Π²Π΅ΠΊΡΠΎΡΠ°ΠΌΠΈ A ΠΈ B.
ΠΠ°ΠΊ Π½Π°ΠΉΡΠΈ Π²Π΅Π»ΠΈΡΠΈΠ½Ρ Π²Π΅ΠΊΡΠΎΡΠ½ΠΎΠ³ΠΎ ΠΏΡΠΎΠΈΠ·Π²Π΅Π΄Π΅Π½ΠΈΡ Π΄Π²ΡΡ Π²Π΅ΠΊΡΠΎΡΠΎΠ²?
ΠΠ΅ΡΠ΅ΠΊΡΠ΅ΡΡΠ½ΠΎΠ΅ ΠΏΡΠΎΠΈΠ·Π²Π΅Π΄Π΅Π½ΠΈΠ΅ Π΄Π²ΡΡ Π²Π΅ΠΊΡΠΎΡΠΎΠ² β ΡΡΠΎ Π΅ΡΠ΅ ΠΎΠ΄ΠΈΠ½ Π²Π΅ΠΊΡΠΎΡ, Π²Π΅Π»ΠΈΡΠΈΠ½Π° ΠΊΠΎΡΠΎΡΠΎΠ³ΠΎ ΠΎΠΏΡΠ΅Π΄Π΅Π»ΡΠ΅ΡΡΡ Π²ΡΡΠ°ΠΆΠ΅Π½ΠΈΠ΅ΠΌ \(\overrightarrow{a} \times \overrightarrow{b} = \hat i (a_2b_3-a_3b_2) \\- \hat j (a_1b_3-a_3b_1) \\+ \ΡΠ»ΡΠΏΠ° k (a_1b_2-a_2b_1)\)
Π§ΡΠΎ ΡΠ°ΠΊΠΎΠ΅ ΡΠΎΡΠΌΡΠ»Π° ΠΏΠ΅ΡΠ΅ΠΊΡΠ΅ΡΡΠ½ΠΎΠ³ΠΎ ΠΏΡΠΎΠΈΠ·Π²Π΅Π΄Π΅Π½ΠΈΡ Ρ ΠΈΡΠΏΠΎΠ»ΡΠ·ΠΎΠ²Π°Π½ΠΈΠ΅ΠΌ ΠΌΠ°ΡΡΠΈΡΠ½ΠΎΠΉ Π·Π°ΠΏΠΈΡΠΈ?
ΠΠ»Ρ Π΄Π²ΡΡ
Π·Π°Π΄Π°Π½Π½ΡΡ
Π²Π΅ΠΊΡΠΎΡΠΎΠ² \(\overrightarrow{a}\) ΠΈ \(\overrightarrow{b}\) ΠΌΡ ΠΌΠΎΠΆΠ΅ΠΌ Π½Π°ΠΉΡΠΈ Π²Π΅ΠΊΡΠΎΡΠ½ΠΎΠ΅ ΠΏΡΠΎΠΈΠ·Π²Π΅Π΄Π΅Π½ΠΈΠ΅, ΠΈΡΠΏΠΎΠ»ΡΠ·ΡΡ ΠΎΠΏΡΠ΅Π΄Π΅Π»ΠΈΡΠ΅Π»ΠΈ. ΠΠ°ΠΏΡΠΈΠΌΠ΅Ρ, \(\overrightarrow{a}\)= \(a_1\hat i+a_2 \hat j+a_3 \hat k\) ΠΈ \(\overrightarrow{b}\) = \(b_1 \hat i+b_2 \hat j+b_3 \hat k\), ΡΠΎ ΠΌΡ ΠΌΠΎΠΆΠ΅ΠΌ Π·Π°ΠΏΠΈΡΠ°ΡΡ ΡΠ΅Π·ΡΠ»ΡΡΠ°Ρ ΠΊΠ°ΠΊ \(\overrightarrow{a} \times \overrightarrow{b} = \hat i (a_2b_3-a_3b_2) \\- \hat j (a_1b_3- a_3b_1)\\+ \ΡΠ»ΡΠΏΠ° k (a_1b_2-a_2b_1)\)
ΠΠ°ΠΊ ΠΈΡΠΏΠΎΠ»ΡΠ·ΠΎΠ²Π°ΡΡ ΡΠΎΡΠΌΡΠ»Ρ ΠΏΠ΅ΡΠ΅ΠΊΡΠ΅ΡΡΠ½ΠΎΠ³ΠΎ ΠΏΡΠΎΠΈΠ·Π²Π΅Π΄Π΅Π½ΠΈΡ?
Π Π°ΡΡΠΌΠΎΡΡΠΈΠΌ Π·Π°Π΄Π°Π½Π½ΡΠ΅ Π²Π΅ΠΊΡΠΎΡΡ.
- Π¨Π°Π³ 1: ΠΡΠΎΠ²Π΅ΡΡΡΠ΅ ΠΊΠΎΠΌΠΏΠΎΠ½Π΅Π½ΡΡ Π²Π΅ΠΊΡΠΎΡΠΎΠ² |A| = Π²Π΅Π»ΠΈΡΠΈΠ½Π° Π²Π΅ΠΊΡΠΎΡΠ° A, |B| = ΠΌΠΎΠ΄ΡΠ»Ρ Π²Π΅ΠΊΡΠΎΡΠ° B ΠΈ ΞΈ = ΡΠ³ΠΎΠ» ΠΌΠ΅ΠΆΠ΄Ρ Π²Π΅ΠΊΡΠΎΡΠ°ΠΌΠΈ A ΠΈ B.
- Π¨Π°Π³ 2: ΠΠΎΠ΄ΡΡΠ°Π²ΡΡΠ΅ Π·Π½Π°ΡΠ΅Π½ΠΈΡ Π² ΡΠΎΡΠΌΡΠ»Ρ Π²Π΅ΠΊΡΠΎΡΠ½ΠΎΠ³ΠΎ ΠΏΡΠΎΠΈΠ·Π²Π΅Π΄Π΅Π½ΠΈΡ: \ ((\vec {A Γ B})=|A||B|\text{Sinβ‘}\vec{ΞΈ_n}\)
ΠΠ°ΠΏΡΠΈΠΌΠ΅Ρ, Π΅ΡΠ»ΠΈ \(\vec {A}=a\hat{i} + b\hat{j}+c\hat{k}\) ΠΈ \( \vec{B}=d\hat{i } + e\hat{j}+f\hat{k}\), Π·Π°ΡΠ΅ΠΌ \({\vec{A Γ B}} = \begin{matrix} \hat{i} & \hat{j} & \hat{ k} \\ a & b & c \\ d & e & f \end{matrix}\)
\({\vec{A Γ B}} = \ΡΠ»ΡΠΏΠ°{i}(bf-ce) β \ΡΠ»ΡΠΏΠ°{j}(af-cd) + \ΡΠ»ΡΠΏΠ°{k}(ae-bd)\)
Π§ΡΠΎ ΡΠ°ΠΊΠΎΠ΅ ΠΏΡΠ°Π²ΠΈΠ»ΠΎ Π±ΠΎΠ»ΡΡΠΎΠ³ΠΎ ΠΏΠ°Π»ΡΡΠ° ΠΏΡΠ°Π²ΠΎΠΉ ΡΡΠΊΠΈ Π΄Π»Ρ ΠΏΠ΅ΡΠ΅ΠΊΡΠ΅ΡΡΠ½ΠΎΠ³ΠΎ ΠΏΡΠΎΠΈΠ·Π²Π΅Π΄Π΅Π½ΠΈΡ Π΄Π²ΡΡ Π²Π΅ΠΊΡΠΎΡΠΎΠ²?
ΠΡΠ°Π²ΠΈΠ»ΠΎ ΠΏΡΠ°Π²ΠΎΠΉ ΡΡΠΊΠΈ Π΄Π»Ρ Π²Π΅ΠΊΡΠΎΡΠ½ΠΎΠ³ΠΎ ΠΏΡΠΎΠΈΠ·Π²Π΅Π΄Π΅Π½ΠΈΡ Π΄Π²ΡΡ
Π²Π΅ΠΊΡΠΎΡΠΎΠ² ΠΏΠΎΠΌΠΎΠ³Π°Π΅Ρ ΠΎΠΏΡΠ΅Π΄Π΅Π»ΠΈΡΡ Π½Π°ΠΏΡΠ°Π²Π»Π΅Π½ΠΈΠ΅ ΡΠ΅Π·ΡΠ»ΡΡΠΈΡΡΡΡΠ΅Π³ΠΎ Π²Π΅ΠΊΡΠΎΡΠ°. ΠΡΠ»ΠΈ ΠΌΡ Π½Π°ΠΏΡΠ°Π²ΠΈΠΌ ΠΏΡΠ°Π²ΡΡ ΡΡΠΊΡ Π² Π½Π°ΠΏΡΠ°Π²Π»Π΅Π½ΠΈΠΈ ΠΏΠ΅ΡΠ²ΠΎΠΉ ΡΡΡΠ΅Π»ΠΊΠΈ, Π° ΠΏΠ°Π»ΡΡΡ ΡΠΎΠ³Π½Π΅ΠΌ Π² Π½Π°ΠΏΡΠ°Π²Π»Π΅Π½ΠΈΠΈ Π²ΡΠΎΡΠΎΠΉ, ΡΠΎ Π½Π°Ρ Π±ΠΎΠ»ΡΡΠΎΠΉ ΠΏΠ°Π»Π΅Ρ ΠΎΠΊΠ°ΠΆΠ΅ΡΡΡ Π² Π½Π°ΠΏΡΠ°Π²Π»Π΅Π½ΠΈΠΈ Π²Π΅ΠΊΡΠΎΡΠ½ΠΎΠ³ΠΎ ΠΏΡΠΎΠΈΠ·Π²Π΅Π΄Π΅Π½ΠΈΡ Π΄Π²ΡΡ
Π²Π΅ΠΊΡΠΎΡΠΎΠ². ΠΡΠ°Π²ΠΈΠ»ΠΎ Π±ΠΎΠ»ΡΡΠΎΠ³ΠΎ ΠΏΠ°Π»ΡΡΠ° ΠΏΡΠ°Π²ΠΎΠΉ ΡΡΠΊΠΈ Π΄Π°Π΅Ρ ΡΠΎΡΠΌΡΠ»Ρ Π²Π΅ΠΊΡΠΎΡΠ½ΠΎΠ³ΠΎ ΠΏΡΠΎΠΈΠ·Π²Π΅Π΄Π΅Π½ΠΈΡ Π΄Π»Ρ Π½Π°Ρ
ΠΎΠΆΠ΄Π΅Π½ΠΈΡ Π½Π°ΠΏΡΠ°Π²Π»Π΅Π½ΠΈΡ ΡΠ΅Π·ΡΠ»ΡΡΠΈΡΡΡΡΠ΅Π³ΠΎ Π²Π΅ΠΊΡΠΎΡΠ°.
ΠΠΎΠ½ΠΈΠΌΠ°Π½ΠΈΠ΅ ΠΏΠ΅ΡΠ΅ΠΊΡΠ΅ΡΡΠ½ΠΎΠ³ΠΎ ΠΏΡΠΎΠΈΠ·Π²Π΅Π΄Π΅Π½ΠΈΡ β BetterExplained
ΠΠ·ΡΠ² Π΄Π²Π° Π²Π΅ΠΊΡΠΎΡΠ°, ΠΌΡ ΠΌΠΎΠΆΠ΅ΠΌ Π·Π°ΠΏΠΈΡΠ°ΡΡ ΠΊΠ°ΠΆΠ΄ΡΡ ΠΊΠΎΠΌΠ±ΠΈΠ½Π°ΡΠΈΡ ΠΊΠΎΠΌΠΏΠΎΠ½Π΅Π½ΡΠΎΠ² Π² ΡΠ΅ΡΠΊΡ:
ΠΡΠ° Π·Π°Π²Π΅ΡΡΠ΅Π½Π½Π°Ρ ΡΠ΅ΡΠΊΠ° ΠΏΡΠ΅Π΄ΡΡΠ°Π²Π»ΡΠ΅Ρ ΡΠΎΠ±ΠΎΠΉ Π²Π½Π΅ΡΠ½ΠΈΠΉ ΠΏΡΠΎΠ΄ΡΠΊΡ , ΠΊΠΎΡΠΎΡΡΠΉ ΠΌΠΎΠΆΠ½ΠΎ ΡΠ°Π·Π΄Π΅Π»ΠΈΡΡ Π½Π°:
Π‘ΠΊΠ°Π»ΡΡΠ½ΡΠΉ ΠΏΡΠΎΠ΄ΡΠΊΡ , Π²Π·Π°ΠΈΠΌΠΎΠ΄Π΅ΠΉΡΡΠ²ΠΈΡ ΠΌΠ΅ΠΆΠ΄Ρ ΠΏΠΎΠ΄ΠΎΠ±Π½ΡΠΌΠΈ ΠΈΠ·ΠΌΠ΅ΡΠ΅Π½ΠΈΡΠΌΠΈ (
x*x
,y*y
,z*z
)ΠΠ΅ΡΠ΅ΠΊΡΠ΅ΡΡΠ½ΠΎΠ΅ ΠΏΡΠΎΠΈΠ·Π²Π΅Π΄Π΅Π½ΠΈΠ΅ , Π²Π·Π°ΠΈΠΌΠΎΠ΄Π΅ΠΉΡΡΠ²ΠΈΡ ΠΌΠ΅ΠΆΠ΄Ρ ΡΠ°Π·Π»ΠΈΡΠ½ΡΠΌΠΈ ΠΈΠ·ΠΌΠ΅ΡΠ΅Π½ΠΈΡΠΌΠΈ (
x*y
,y*z
,z*x
ΠΈ Ρ.Π΄.)
Π‘ΠΊΠ°Π»ΡΡΠ½ΠΎΠ΅ ΠΏΡΠΎΠΈΠ·Π²Π΅Π΄Π΅Π½ΠΈΠ΅ ($\vec{a} \cdot \vec{b}$) ΠΈΠ·ΠΌΠ΅ΡΡΠ΅Ρ ΡΡ
ΠΎΠ΄ΡΡΠ²ΠΎ, ΠΏΠΎΡΠΊΠΎΠ»ΡΠΊΡ ΠΎΠ½ΠΎ Π½Π°ΠΊΠ°ΠΏΠ»ΠΈΠ²Π°Π΅Ρ Π²Π·Π°ΠΈΠΌΠΎΠ΄Π΅ΠΉΡΡΠ²ΠΈΡ ΡΠΎΠ»ΡΠΊΠΎ Π² ΡΠΎΠ²ΠΏΠ°Π΄Π°ΡΡΠΈΡ
ΠΈΠ·ΠΌΠ΅ΡΠ΅Π½ΠΈΡΡ
. ΠΡΠΎ ΠΏΡΠΎΡΡΠΎΠΉ ΡΠ°ΡΡΠ΅Ρ Ρ 3 ΠΊΠΎΠΌΠΏΠΎΠ½Π΅Π½ΡΠ°ΠΌΠΈ.
ΠΠ΅ΡΠ΅ΠΊΡΠ΅ΡΡΠ½ΠΎΠ΅ ΠΏΡΠΎΠΈΠ·Π²Π΅Π΄Π΅Π½ΠΈΠ΅ (ΠΎΠ±ΠΎΠ·Π½Π°ΡΠ°Π΅ΠΌΠΎΠ΅ ΠΊΠ°ΠΊ $\vec{a} \times \vec{b}$) Π΄ΠΎΠ»ΠΆΠ½ΠΎ ΠΈΠ·ΠΌΠ΅ΡΡΡΡ ΠΏΠΎΠ»Π΄ΡΠΆΠΈΠ½Ρ Β«ΠΏΠ΅ΡΠ΅ΠΊΡΠ΅ΡΡΠ½ΡΡ Π²Π·Π°ΠΈΠΌΠΎΠ΄Π΅ΠΉΡΡΠ²ΠΈΠΉΒ». ΠΡΡΠΈΡΠ»Π΅Π½ΠΈΠ΅ Π²ΡΠ³Π»ΡΠ΄ΠΈΡ ΡΠ»ΠΎΠΆΠ½ΡΠΌ, Π½ΠΎ ΠΊΠΎΠ½ΡΠ΅ΠΏΡΠΈΡ ΠΏΡΠΎΡΡΠ°: ΡΡΠΌΠΌΠΈΡΡΠΉΡΠ΅ 6 ΠΈΠ½Π΄ΠΈΠ²ΠΈΠ΄ΡΠ°Π»ΡΠ½ΡΡ ΡΠ°Π·Π»ΠΈΡΠΈΠΉ Π΄Π»Ρ ΠΏΠΎΠ»ΡΡΠ΅Π½ΠΈΡ ΠΎΠ±ΡΠ΅ΠΉ ΡΠ°Π·Π½ΠΈΡΡ.
ΠΠΌΠ΅ΡΡΠΎ ΡΠΎΠ³ΠΎ, ΡΡΠΎΠ±Ρ Π΄ΡΠΌΠ°ΡΡ Β«ΠΠΎΠ³Π΄Π° ΠΌΠ½Π΅ Π½ΡΠΆΠ½ΠΎ ΠΏΠ΅ΡΠ΅ΠΊΡΠ΅ΡΡΠ½ΠΎΠ΅ ΠΏΡΠΎΠΈΠ·Π²Π΅Π΄Π΅Π½ΠΈΠ΅?Β» ΠΏΠΎΠ΄ΡΠΌΠ°ΠΉΡΠ΅: Β«ΠΠΎΠ³Π΄Π° ΠΌΠ½Π΅ Π½ΡΠΆΠ½ΠΎ Π²Π·Π°ΠΈΠΌΠΎΠ΄Π΅ΠΉΡΡΠ²ΠΈΠ΅ ΠΌΠ΅ΠΆΠ΄Ρ ΡΠ°Π·Π½ΡΠΌΠΈ ΠΈΠ·ΠΌΠ΅ΡΠ΅Π½ΠΈΡΠΌΠΈ?Β».
ΠΠ±Π»Π°ΡΡΡ, Π½Π°ΠΏΡΠΈΠΌΠ΅Ρ, ΠΎΠ±ΡΠ°Π·ΠΎΠ²Π°Π½Π° Π²Π΅ΠΊΡΠΎΡΠ°ΠΌΠΈ, ΡΠΊΠ°Π·ΡΠ²Π°ΡΡΠΈΠΌΠΈ Π² ΡΠ°Π·Π½ΡΠ΅ ΡΡΠΎΡΠΎΠ½Ρ (ΡΠ΅ΠΌ ΠΎΡΡΠΎΠ³ΠΎΠ½Π°Π»ΡΠ½Π΅Π΅, ΡΠ΅ΠΌ Π»ΡΡΡΠ΅). ΠΠ΅ΠΉΡΡΠ²ΠΈΡΠ΅Π»ΡΠ½ΠΎ, ΠΏΠ΅ΡΠ΅ΠΊΡΠ΅ΡΡΠ½ΠΎΠ΅ ΠΏΡΠΎΠΈΠ·Π²Π΅Π΄Π΅Π½ΠΈΠ΅ ΠΈΠ·ΠΌΠ΅ΡΡΠ΅Ρ ΠΏΠ»ΠΎΡΠ°Π΄Ρ, ΠΎΡ Π²Π°ΡΡΠ²Π°Π΅ΠΌΡΡ Π΄Π²ΡΠΌΡ ΡΡΠ΅Ρ ΠΌΠ΅ΡΠ½ΡΠΌΠΈ Π²Π΅ΠΊΡΠΎΡΠ°ΠΌΠΈ (ΠΈΡΡΠΎΡΠ½ΠΈΠΊ):
(Β«ΠΠ΅ΡΠ΅ΠΊΡΠ΅ΡΡΠ½ΠΎΠ΅ ΠΏΡΠΎΠΈΠ·Π²Π΅Π΄Π΅Π½ΠΈΠ΅Β» ΠΏΡΠ΅Π΄ΠΏΠΎΠ»Π°Π³Π°Π΅Ρ ΡΡΠ΅Ρ ΠΌΠ΅ΡΠ½ΡΠ΅ Π²Π΅ΠΊΡΠΎΡΡ, Π½ΠΎ ΠΊΠΎΠ½ΡΠ΅ΠΏΡΠΈΡ ΡΠ°ΡΠΏΡΠΎΡΡΡΠ°Π½ΡΠ΅ΡΡΡ Π½Π° Π±ΠΎΠ»Π΅Π΅ Π²ΡΡΠΎΠΊΠΈΠ΅ ΠΈΠ·ΠΌΠ΅ΡΠ΅Π½ΠΈΡ.)
ΠΠ»ΡΡΠ΅Π²Π°Ρ ΠΈΠ½ΡΡΠΈΡΠΈΡ ΡΠ΅Π»ΠΊΠ½ΡΠ»Π°? ΠΠ΅ΡΠ΅ΠΉΠ΄Π΅ΠΌ ΠΊ Π΄Π΅ΡΠ°Π»ΡΠΌ.
ΠΠΏΡΠ΅Π΄Π΅Π»Π΅Π½ΠΈΠ΅ ΠΏΠ΅ΡΠ΅ΠΊΡΠ΅ΡΡΠ½ΠΎΠ³ΠΎ ΠΏΡΠΎΠΈΠ·Π²Π΅Π΄Π΅Π½ΠΈΡ
Π‘ΠΊΠ°Π»ΡΡΠ½ΠΎΠ΅ ΠΏΡΠΎΠΈΠ·Π²Π΅Π΄Π΅Π½ΠΈΠ΅ ΠΏΡΠ΅Π΄ΡΡΠ°Π²Π»ΡΠ΅Ρ ΡΡ ΠΎΠ΄ΡΡΠ²ΠΎ ΠΌΠ΅ΠΆΠ΄Ρ Π²Π΅ΠΊΡΠΎΡΠ°ΠΌΠΈ Π² Π²ΠΈΠ΄Π΅ ΠΎΠ΄Π½ΠΎΠ³ΠΎ ΡΠΈΡΠ»Π°:
ΠΠ°ΠΏΡΠΈΠΌΠ΅Ρ, ΠΌΡ ΠΌΠΎΠΆΠ΅ΠΌ ΡΠΊΠ°Π·Π°ΡΡ, ΡΡΠΎ ΡΠ΅Π²Π΅Ρ ΠΈ Π²ΠΎΡΡΠΎΠΊ ΡΡ
ΠΎΠ΄Π½Ρ Π½Π° 0%, ΠΏΠΎΡΠΊΠΎΠ»ΡΠΊΡ $(0, 1) \cdot (1, 0) = 0$. ΠΠ»ΠΈ ΡΡΠΎ Π‘Π΅Π²Π΅Ρ ΠΈ Π‘Π΅Π²Π΅ΡΠΎ-ΠΠΎΡΡΠΎΠΊ ΠΏΠΎΡ
ΠΎΠΆΠΈ Π½Π° 70% ($\cos(45) = 0,707$, ΠΏΠΎΠΌΠ½ΠΈΡΠ΅, ΡΡΠΎ ΡΡΠΈΠ³Π³Π΅ΡΠ½ΡΠ΅ ΡΡΠ½ΠΊΡΠΈΠΈ ΠΏΡΠ΅Π΄ΡΡΠ°Π²Π»ΡΡΡ ΡΠΎΠ±ΠΎΠΉ ΠΏΡΠΎΡΠ΅Π½ΡΡ.) Π‘Ρ
ΠΎΠ΄ΡΡΠ²ΠΎ ΠΏΠΎΠΊΠ°Π·ΡΠ²Π°Π΅Ρ ΠΊΠΎΠ»ΠΈΡΠ΅ΡΡΠ²ΠΎ ΠΎΠ΄Π½ΠΎΠ³ΠΎ Π²Π΅ΠΊΡΠΎΡΠ°, ΠΊΠΎΡΠΎΡΠΎΠ΅ Β«ΠΏΡΠΎΡΠ²Π»ΡΠ΅ΡΡΡΒ» Π² Π΄ΡΡΠ³ΠΎΠΌ.
ΠΠΎΠ»ΠΆΠ½ΠΎ Π»ΠΈ ΠΏΠ΅ΡΠ΅ΠΊΡΠ΅ΡΡΠ½ΠΎΠ΅ ΠΏΡΠΎΠΈΠ·Π²Π΅Π΄Π΅Π½ΠΈΠ΅, ΡΠ°Π·Π½ΠΎΡΡΡ ΠΌΠ΅ΠΆΠ΄Ρ Π²Π΅ΠΊΡΠΎΡΠ°ΠΌΠΈ, ΡΠΎΠΆΠ΅ Π±ΡΡΡ ΠΎΠ΄Π½ΠΈΠΌ ΡΠΈΡΠ»ΠΎΠΌ?
ΠΠΎΠΏΡΠΎΠ±ΡΠ΅ΠΌ. Π‘ΠΈΠ½ΡΡ β ΡΡΠΎ ΡΠ°Π·Π½ΠΈΡΠ° Π² ΠΏΡΠΎΡΠ΅Π½ΡΠ°Ρ , ΠΏΠΎΡΡΠΎΠΌΡ ΠΌΡ ΠΌΠΎΠ³Π»ΠΈ Π±Ρ Π½Π°ΠΏΠΈΡΠ°ΡΡ:
Π ΡΠΎΠΆΠ°Π»Π΅Π½ΠΈΡ, Π½Π°ΠΌ Π½Π΅ Ρ Π²Π°ΡΠ°Π΅Ρ Π½Π΅ΠΊΠΎΡΠΎΡΡΡ Π΄Π΅ΡΠ°Π»Π΅ΠΉ. ΠΠΎΠΏΡΡΡΠΈΠΌ, ΠΌΡ ΡΠΌΠΎΡΡΠΈΠΌ Π²Π½ΠΈΠ· ΠΏΠΎ ΠΎΡΠΈ x: ΠΈ y, ΠΈ z ΡΠΊΠ°Π·ΡΠ²Π°ΡΡ Π½Π° 100% ΠΎΡ Π½Π°Ρ. Π§ΠΈΡΠ»ΠΎ Π²ΡΠΎΠ΄Π΅ Β«100%Β» Π³ΠΎΠ²ΠΎΡΠΈΡ Π½Π°ΠΌ ΠΎ Π±ΠΎΠ»ΡΡΠΎΠΉ ΡΠ°Π·Π½ΠΈΡΠ΅, Π½ΠΎ ΠΌΡ Π½Π΅ Π·Π½Π°Π΅ΠΌ, Π² ΡΠ΅ΠΌ ΠΎΠ½Π° Π·Π°ΠΊΠ»ΡΡΠ°Π΅ΡΡΡ! ΠΠ°ΠΌ Π½ΡΠΆΠ½Π° Π΄ΠΎΠΏΠΎΠ»Π½ΠΈΡΠ΅Π»ΡΠ½Π°Ρ ΠΈΠ½ΡΠΎΡΠΌΠ°ΡΠΈΡ, ΡΡΠΎΠ±Ρ ΡΠΊΠ°Π·Π°ΡΡ Π½Π°ΠΌ, ΡΡΠΎ Β«ΡΠ°Π·Π½ΠΈΡΠ° ΠΌΠ΅ΠΆΠ΄Ρ $\vec{x}$ ΠΈ $\vec{y}$ ΡΠΎΡΡΠ°Π²Π»ΡΠ΅Ρ Π½Π° Β» ΠΈ Β«ΡΠ°Π·Π½ΠΈΡΠ° ΠΌΠ΅ΠΆΠ΄Ρ $\vec{x}$ ΠΈ $\vec{z} $ β ΡΡΠΎ , ΡΡΠΎ Β».
ΠΡΠ°ΠΊ, Π΄Π°Π²Π°ΠΉΡΠ΅ ΠΏΡΠ΅Π΄ΡΡΠ°Π²ΠΈΠΌ ΠΏΠ΅ΡΠ΅ΠΊΡΠ΅ΡΡΠ½ΠΎΠ΅ ΠΏΡΠΎΠΈΠ·Π²Π΅Π΄Π΅Π½ΠΈΠ΅ Π² Π²ΠΈΠ΄Π΅ Π²Π΅ΠΊΡΠΎΡΠ°:
Π Π°Π·ΠΌΠ΅Ρ ΠΏΠ΅ΡΠ΅ΠΊΡΠ΅ΡΡΠ½ΠΎΠ³ΠΎ ΠΏΡΠΎΠΈΠ·Π²Π΅Π΄Π΅Π½ΠΈΡ β ΡΡΠΎ ΡΠΈΡΠ»ΠΎΠ²Π°Ρ Β«Π²Π΅Π»ΠΈΡΠΈΠ½Π° ΡΠ°Π·Π½ΠΈΡΡΒ» (Ρ $\sin(\theta)$ Π² ΠΏΡΠΎΡΠ΅Π½ΡΠ°Ρ ).
Π‘Π°ΠΌΠΎ ΠΏΠΎ ΡΠ΅Π±Π΅ ΡΡΠΎ Π½Π΅ ΠΎΡΠ»ΠΈΡΠ°Π΅Ρ $\vec{x} \times \vec{y}$ ΠΎΡ $\vec{x} \times \vec{z}$.
Π½Π°ΠΏΡΠ°Π²Π»Π΅Π½ΠΈΠ΅ ΠΏΠ΅ΡΠ΅ΠΊΡΠ΅ΡΡΠ½ΠΎΠ³ΠΎ ΠΏΡΠΎΠΈΠ·Π²Π΅Π΄Π΅Π½ΠΈΡ ΠΎΡΠ½ΠΎΠ²Π°Π½ΠΎ Π½Π° ΠΎΠ±ΠΎΠΈΡ Π²Ρ ΠΎΠ΄Π½ΡΡ Π΄Π°Π½Π½ΡΡ : ΡΡΠΎ Π½Π°ΠΏΡΠ°Π²Π»Π΅Π½ΠΈΠ΅ ΠΎΡΡΠΎΠ³ΠΎΠ½Π°Π»ΡΠ½ΠΎ ΠΎΠ±ΠΎΠΈΠΌ (Ρ. Π΅. Π½Π΅ Π² ΠΏΠΎΠ»ΡΠ·Ρ Π½ΠΈ ΡΠΎΠ³ΠΎ, Π½ΠΈ Π΄ΡΡΠ³ΠΎΠ³ΠΎ).
Π’Π΅ΠΏΠ΅ΡΡ $\vec{x} \times \vec{y}$ ΠΈ $\vec{x} \times \vec{z}$ ΠΈΠΌΠ΅ΡΡ ΡΠ°Π·Π½ΡΠ΅ ΡΠ΅Π·ΡΠ»ΡΡΠ°ΡΡ, ΠΊΠ°ΠΆΠ΄ΡΠΉ Ρ Π²Π΅Π»ΠΈΡΠΈΠ½ΠΎΠΉ, ΡΠΊΠ°Π·ΡΠ²Π°ΡΡΠ΅ΠΉ, ΡΡΠΎ ΠΎΠ½ΠΈ Π½Π° Β«100%Β» ΠΎΡΠ»ΠΈΡΠ°ΡΡΡΡ ΠΎΡ $ \vec{Ρ }$.
(ΠΠΎΠ»ΠΆΠ½ΠΎ Π»ΠΈ ΡΠΊΠ°Π»ΡΡΠ½ΠΎΠ΅ ΠΏΡΠΎΠΈΠ·Π²Π΅Π΄Π΅Π½ΠΈΠ΅ ΡΠ°ΠΊΠΆΠ΅ Π±ΡΡΡ Π²Π΅ΠΊΡΠΎΡΠ½ΡΠΌ ΡΠ΅Π·ΡΠ»ΡΡΠ°ΡΠΎΠΌ? ΠΡ, ΠΌΡ ΠΎΡΡΠ»Π΅ΠΆΠΈΠ²Π°Π΅ΠΌ ΡΡ ΠΎΠ΄ΡΡΠ²ΠΎ ΠΌΠ΅ΠΆΠ΄Ρ $\vec{a}$ ΠΈ $\vec{b}$. Π‘Ρ ΠΎΠ΄ΡΡΠ²ΠΎ ΠΈΠ·ΠΌΠ΅ΡΡΠ΅Ρ ΠΏΠ΅ΡΠ΅ΠΊΡΡΡΠΈΠ΅ ΠΌΠ΅ΠΆΠ΄Ρ Π½Π°ΠΏΡΠ°Π²Π»Π΅Π½ΠΈΡΠΌΠΈ ΠΈΡΡ ΠΎΠ΄Π½ΠΎΠ³ΠΎ Π²Π΅ΠΊΡΠΎΡΠ°, ΠΊΠΎΡΠΎΡΠΎΠ΅ Ρ Π½Π°Ρ ΡΠΆΠ΅ Π΅ΡΡΡ.)
ΠΠ΅ΠΎΠΌΠ΅ΡΡΠΈΡΠ΅ΡΠΊΠ°Ρ ΠΈΠ½ΡΠ΅ΡΠΏΡΠ΅ΡΠ°ΡΠΈΡ
ΠΠ²Π° Π²Π΅ΠΊΡΠΎΡΠ° ΠΎΠΏΡΠ΅Π΄Π΅Π»ΡΡΡ ΠΏΠ»ΠΎΡΠΊΠΎΡΡΡ, ΠΈ Π²Π΅ΠΊΡΠΎΡΠ½ΠΎΠ΅ ΠΏΡΠΎΠΈΠ·Π²Π΅Π΄Π΅Π½ΠΈΠ΅ ΡΠΊΠ°Π·ΡΠ²Π°Π΅Ρ Π² Π½Π°ΠΏΡΠ°Π²Π»Π΅Π½ΠΈΠΈ, ΠΎΡΠ»ΠΈΡΠ½ΠΎΠΌ ΠΎΡ ΠΎΠ±ΠΎΠΈΡ (ΠΈΡΡΠΎΡΠ½ΠΈΠΊ):
ΠΠΎΡ ΠΏΡΠΎΠ±Π»Π΅ΠΌΠ°: Π΅ΡΡΡ Π΄Π²Π° ΠΏΠ΅ΡΠΏΠ΅Π½Π΄ΠΈΠΊΡΠ»ΡΡΠ½ΡΡ Π½Π°ΠΏΡΠ°Π²Π»Π΅Π½ΠΈΡ. ΠΠΎ ΡΠΎΠ³Π»Π°ΡΠ΅Π½ΠΈΡ ΠΌΡ ΠΏΡΠ΅Π΄ΠΏΠΎΠ»Π°Π³Π°Π΅ΠΌ Β«ΠΏΡΠ°Π²ΠΎΡΡΠΎΡΠΎΠ½Π½ΡΡ ΡΠΈΡΡΠ΅ΠΌΡΒ» ββ(ΠΈΡΡΠΎΡΠ½ΠΈΠΊ):
ΠΡΠ»ΠΈ Π²Ρ Π΄Π΅ΡΠΆΠΈΡΠ΅ ΠΏΠ΅ΡΠ²ΡΠ΅ Π΄Π²Π° ΠΏΠ°Π»ΡΡΠ°, ΠΊΠ°ΠΊ ΠΏΠΎΠΊΠ°Π·Π°Π½ΠΎ Π½Π° Π΄ΠΈΠ°Π³ΡΠ°ΠΌΠΌΠ΅, Π²Π°Ρ Π±ΠΎΠ»ΡΡΠΎΠΉ ΠΏΠ°Π»Π΅Ρ Π±ΡΠ΄Π΅Ρ ΡΠΊΠ°Π·ΡΠ²Π°ΡΡ Π² Π½Π°ΠΏΡΠ°Π²Π»Π΅Π½ΠΈΠΈ Π²Π΅ΠΊΡΠΎΡΠ½ΠΎΠ³ΠΎ ΠΏΡΠΎΠΈΠ·Π²Π΅Π΄Π΅Π½ΠΈΡ. Π― ΠΏΡΠΎΠ²Π΅ΡΡΡ ΠΏΡΠ°Π²ΠΈΠ»ΡΠ½ΠΎΡΡΡ ΠΎΡΠΈΠ΅Π½ΡΠ°ΡΠΈΠΈ, ΠΏΡΠΎΠ²ΠΎΠ΄Ρ ΠΏΠ°Π»ΡΡΠ΅ΠΌ ΠΎΡ $\vec{a}$ Π΄ΠΎ $\vec{b}$. ΠΠΎΠ³Π΄Π° Π½Π°ΠΏΡΠ°Π²Π»Π΅Π½ΠΈΠ΅ Π²ΡΡΡΠ½Π΅Π½ΠΎ, Π²Π΅Π»ΠΈΡΠΈΠ½Π° Π²Π΅ΠΊΡΠΎΡΠ½ΠΎΠ³ΠΎ ΠΏΡΠΎΠΈΠ·Π²Π΅Π΄Π΅Π½ΠΈΡ ΡΠ°Π²Π½Π° $|a| |Π±| \sin(\theta)$, ΠΊΠΎΡΠΎΡΡΠΉ ΠΏΡΠΎΠΏΠΎΡΡΠΈΠΎΠ½Π°Π»Π΅Π½ Π²Π΅Π»ΠΈΡΠΈΠ½Π΅ ΠΊΠ°ΠΆΠ΄ΠΎΠ³ΠΎ Π²Π΅ΠΊΡΠΎΡΠ° ΠΈ Β«ΠΏΡΠΎΡΠ΅Π½ΡΡ ΡΠ°Π·Π½ΠΈΡΡΒ» (ΡΠΈΠ½ΡΡΡ).
ΠΠ΅ΡΠ΅ΠΊΡΠ΅ΡΡΠ½ΠΎΠ΅ ΠΏΡΠΎΠΈΠ·Π²Π΅Π΄Π΅Π½ΠΈΠ΅ Π΄Π»Ρ ΠΎΡΡΠΎΠ³ΠΎΠ½Π°Π»ΡΠ½ΡΡ Π²Π΅ΠΊΡΠΎΡΠΎΠ²
Π§ΡΠΎΠ±Ρ Π·Π°ΠΏΠΎΠΌΠ½ΠΈΡΡ ΠΏΡΠ°Π²ΠΈΠ»ΠΎ ΠΏΡΠ°Π²ΠΎΠΉ ΡΡΠΊΠΈ, Π½Π°ΠΏΠΈΡΠΈΡΠ΅ xyz
Π·Π°ΠΊΠ°ΠΆΠΈΡΠ΅ Π΄Π²Π°ΠΆΠ΄Ρ: xyzxyz
. ΠΠ°ΡΠ΅ΠΌ Π½Π°ΠΉΠ΄ΠΈΡΠ΅ Π½ΡΠΆΠ½ΡΠΉ ΡΠ°Π±Π»ΠΎΠ½:
-
xy => z
(x
ΠΊΡΠ΅ΡΡy
ΡΠ°Π²Π½ΠΎz
) -
yz => x
(y
ΠΏΠ΅ΡΠ΅ΡΠ΅ΡΡz
ΡΠ°Π²Π½ΠΎx
; ΠΌΡ Π·Π°ΡΠΈΠΊΠ»ΠΈΠ»ΠΈΡΡ:y
Π΄ΠΎz
Π΄ΠΎx
) -
Π·Ρ => Ρ
ΠΡΠ°ΠΊ, xy
ΠΈ yx
ΠΈΠΌΠ΅ΡΡ ΠΏΡΠΎΡΠΈΠ²ΠΎΠΏΠΎΠ»ΠΎΠΆΠ½ΡΠ΅ Π·Π½Π°ΠΊΠΈ, ΠΏΠΎΡΠΎΠΌΡ ΡΡΠΎ ΠΎΠ½ΠΈ ΠΈΠ΄ΡΡ Π²ΠΏΠ΅ΡΠ΅Π΄ ΠΈ Π½Π°Π·Π°Π΄ Π² Π½Π°ΡΠ΅ΠΌ 9ΠΠ°ΡΡΡΠΎΠΉΠΊΠ° 1241 xyzxyz .
Π’Π°ΠΊΠΈΠΌ ΠΎΠ±ΡΠ°Π·ΠΎΠΌ, Π±Π΅Π· ΡΠΎΡΠΌΡΠ»Ρ Π²Ρ ΡΠΌΠΎΠΆΠ΅ΡΠ΅ Π²ΡΡΠΈΡΠ»ΠΈΡΡ:
ΠΠΏΡΡΡ ΠΆΠ΅, ΡΡΠΎ ΠΏΠΎΡΠΎΠΌΡ, ΡΡΠΎ x
ΠΏΠ΅ΡΠ΅ΡΠ΅ΡΠ΅Π½ΠΈΠ΅ y
ΡΠ²Π»ΡΠ΅ΡΡΡ ΠΏΠΎΠ»ΠΎΠΆΠΈΡΠ΅Π»ΡΠ½ΡΠΌ z
Π² ΠΏΡΠ°Π²ΠΎΡΡΠΎΡΠΎΠ½Π½Π΅ΠΉ ΡΠΈΡΡΠ΅ΠΌΠ΅ ΠΊΠΎΠΎΡΠ΄ΠΈΠ½Π°Ρ. Π― ΠΈΡΠΏΠΎΠ»ΡΠ·ΠΎΠ²Π°Π» Π΅Π΄ΠΈΠ½ΠΈΡΠ½ΡΠ΅ Π²Π΅ΠΊΡΠΎΡΡ, Π½ΠΎ ΠΌΡ ΠΌΠΎΠ³Π»ΠΈ ΠΌΠ°ΡΡΡΠ°Π±ΠΈΡΠΎΠ²Π°ΡΡ ΡΠ΅ΡΠΌΠΈΠ½Ρ:
ΠΡΡΠΈΡΠ»Π΅Π½ΠΈΠ΅ ΠΏΠ΅ΡΠ΅ΠΊΡΠ΅ΡΡΠ½ΠΎΠ³ΠΎ ΠΏΡΠΎΠΈΠ·Π²Π΅Π΄Π΅Π½ΠΈΡ
ΠΡΠ΄Π΅Π»ΡΠ½ΡΠΉ Π²Π΅ΠΊΡΠΎΡ ΠΌΠΎΠΆΠ½ΠΎ ΡΠ°Π·Π»ΠΎΠΆΠΈΡΡ Π½Π° 3 ΠΎΡΡΠΎΠ³ΠΎΠ½Π°Π»ΡΠ½ΡΠ΅ ΡΠ°ΡΡΠΈ:
ΠΊΠΎΠΌΠΏΠΎΠ½Π΅Π½ΡΡ (Π½Π°ΠΏΡΠΈΠΌΠ΅Ρ, $a_x \times b_y$) Π³ΠΎΠ»ΠΎΡΡΡΡ Π·Π° ΡΠΎ, ΠΊΡΠ΄Π° Π΄ΠΎΠ»ΠΆΠ΅Π½ ΡΠΊΠ°Π·ΡΠ²Π°ΡΡ ΠΎΡΡΠΎΠ³ΠΎΠ½Π°Π»ΡΠ½ΡΠΉ Π²Π΅ΠΊΡΠΎΡ. 6 ΠΊΠΎΠΌΠΏΠΎΠ½Π΅Π½ΡΠΎΠ², 6 Π³ΠΎΠ»ΠΎΡΠΎΠ², ΠΈ ΠΈΡ ΡΡΠΌΠΌΠ° ΡΠ²Π»ΡΠ΅ΡΡΡ Π²Π΅ΠΊΡΠΎΡΠ½ΡΠΌ ΠΏΡΠΎΠΈΠ·Π²Π΅Π΄Π΅Π½ΠΈΠ΅ΠΌ. (ΠΠΎΡ ΠΎΠΆΠ΅ Π½Π° Π³ΡΠ°Π΄ΠΈΠ΅Π½Ρ, Π³Π΄Π΅ ΠΊΠ°ΠΆΠ΄Π°Ρ ΠΎΡΡ Π³ΠΎΠ»ΠΎΡΡΠ΅Ρ Π·Π° Π½Π°ΠΏΡΠ°Π²Π»Π΅Π½ΠΈΠ΅ Π½Π°ΠΈΠ±ΠΎΠ»ΡΡΠ΅Π³ΠΎ ΡΠ²Π΅Π»ΠΈΡΠ΅Π½ΠΈΡ.)
-
xy => z
ΠΈyx => -z
(ΠΏΡΠ΅Π΄ΠΏΠΎΠ»ΠΎΠΆΠΈΠΌ, ΡΡΠΎ $\vec{a}$ ΡΡΠΎΠΈΡ ΠΏΠ΅ΡΠ²ΡΠΌ, ΠΏΠΎΡΡΠΎΠΌΡxy
ΠΎΠ·Π½Π°ΡΠ°Π΅Ρ $a_x b_y$) -
yz => x
ΠΈzy => -x
-
zx => y
ΠΈxz => -y
xy
ΠΈ yx
ΡΡΠ°ΠΆΠ°ΡΡΡΡ Π² Π½Π°ΠΏΡΠ°Π²Π»Π΅Π½ΠΈΠΈ z
. ΠΡΠ»ΠΈ ΡΡΠΈ ΡΠ»Π΅Π½Ρ ΡΠ°Π²Π½Ρ, Π½Π°ΠΏΡΠΈΠΌΠ΅Ρ, Π² $(2, 1, 0) \times (2, 1, 1)$, ΠΊΠΎΠΌΠΏΠΎΠ½Π΅Π½Ρ ΠΏΠ΅ΡΠ΅ΠΊΡΠ΅ΡΡΠ½ΠΎΠ³ΠΎ ΠΏΡΠΎΠΈΠ·Π²Π΅Π΄Π΅Π½ΠΈΡ Π² z 9 ΠΎΡΡΡΡΡΡΠ²ΡΠ΅Ρ.1242 Π½Π°ΠΏΡΠ°Π²Π»Π΅Π½ΠΈΡ (2 β 2 = 0).
ΠΠΊΠΎΠ½ΡΠ°ΡΠ΅Π»ΡΠ½Π°Ρ ΠΊΠΎΠΌΠ±ΠΈΠ½Π°ΡΠΈΡ:
, Π³Π΄Π΅ $\vec{n}$ β Π΅Π΄ΠΈΠ½ΠΈΡΠ½ΡΠΉ Π²Π΅ΠΊΡΠΎΡ, Π½ΠΎΡΠΌΠ°Π»ΡΠ½ΡΠΉ ΠΊ $\vec{a}$ ΠΈ $\vec{b}$.
ΠΡΡΡΡ ΡΡΠΎ Π²Π°Ρ Π½Π΅ ΠΏΡΠ³Π°Π΅Ρ:
ΠΡΠ΅Π³ΠΎ 6 ΡΠ΅ΡΠΌΠΈΠ½ΠΎΠ², 3 ΠΏΠΎΠ»ΠΎΠΆΠΈΡΠ΅Π»ΡΠ½ΡΡ ΠΈ 3 ΠΎΡΡΠΈΡΠ°ΡΠ΅Π»ΡΠ½ΡΡ
ΠΠ²Π° ΠΈΠ·ΠΌΠ΅ΡΠ΅Π½ΠΈΡ Π³ΠΎΠ»ΠΎΡΡΡΡ Π·Π° ΡΡΠ΅ΡΡΠ΅ (ΠΏΠΎΡΡΠΎΠΌΡ ΡΠ΅ΡΠΌ
z
Π΄ΠΎΠ»ΠΆΠ΅Π½ ΠΈΠΌΠ΅ΡΡ ΡΠΎΠ»ΡΠΊΠΎ ΠΊΠΎΠΌΠΏΠΎΠ½Π΅Π½ΡΡy
ΠΈx
)ΠΠΎΠ»ΠΎΠΆΠΈΡΠ΅Π»ΡΠ½ΡΠΉ/ΠΎΡΡΠΈΡΠ°ΡΠ΅Π»ΡΠ½ΡΠΉ ΠΏΠΎΡΡΠ΄ΠΎΠΊ ΠΎΡΠ½ΠΎΠ²Π°Π½ Π½Π° ΡΠ°Π±Π»ΠΎΠ½Π΅
xyzxyz
ΠΡΠ»ΠΈ Ρ
ΠΎΡΠΈΡΠ΅, Π΅ΡΡΡ Π°Π»Π³Π΅Π±ΡΠ°ΠΈΡΠ΅ΡΠΊΠΎΠ΅ Π΄ΠΎΠΊΠ°Π·Π°ΡΠ΅Π»ΡΡΡΠ²ΠΎ ΡΠΎΠ³ΠΎ, ΡΡΠΎ ΡΠΎΡΠΌΡΠ»Π° ΠΎΠ΄Π½ΠΎΠ²ΡΠ΅ΠΌΠ΅Π½Π½ΠΎ ΠΎΡΡΠΎΠ³ΠΎΠ½Π°Π»ΡΠ½Π° ΠΈ ΠΈΠΌΠ΅Π΅Ρ ΡΠ°Π·ΠΌΠ΅Ρ $|a| |Π±| \sin(\theta)$, Π½ΠΎ ΠΌΠ½Π΅ Π½ΡΠ°Π²ΠΈΡΡΡ ΠΈΠ½ΡΡΠΈΡΠΈΠ²Π½ΠΎΠ΅ Β«ΠΏΡΠΎΠΏΠΎΡΡΠΈΠΎΠ½Π°Π»ΡΠ½ΠΎΠ΅ Π³ΠΎΠ»ΠΎΡΠΎΠ²Π°Π½ΠΈΠ΅Β».
ΠΡΠΈΠΌΠ΅Ρ Π²ΡΠ΅ΠΌΠ΅Π½ΠΈ
ΠΠΏΡΡΡ ΠΆΠ΅, ΠΌΡ Π΄ΠΎΠ»ΠΆΠ½Ρ ΠΏΡΠΎΠΈΠ·Π²Π΅ΡΡΠΈ Π² ΡΠΌΠ΅ ΠΏΡΠΎΡΡΡΠ΅ ΠΏΠ΅ΡΠ΅ΠΊΡΠ΅ΡΡΠ½ΡΠ΅ ΠΏΡΠΎΠΈΠ·Π²Π΅Π΄Π΅Π½ΠΈΡ:
ΠΠΎΡΠ΅ΠΌΡ? ΠΡ ΠΏΠ΅ΡΠ΅ΡΠ΅ΠΊΠ»ΠΈ ΠΎΡΠΈ
x
ΠΈ y
, ΠΏΠΎΠ»ΡΡΠΈΠ² z
(ΠΈΠ»ΠΈ $\vec{i} \times \vec{j} = \vec{k}$, ΠΈΡΠΏΠΎΠ»ΡΠ·ΡΡ ΡΡΠΈ Π΅Π΄ΠΈΠ½ΠΈΡΠ½ΡΠ΅ Π²Π΅ΠΊΡΠΎΡΡ). ΠΠ΅ΡΠ΅Ρ
ΠΎΠ΄ Π² Π΄ΡΡΠ³ΡΡ ΡΡΠΎΡΠΎΠ½Ρ Π΄Π°Π΅Ρ $-\vec{k}$.
ΠΠΎΡ ΠΊΠ°ΠΊ Ρ ΡΠ°ΡΡΠΌΠ°ΡΡΠΈΠ²Π°Ρ Π±ΠΎΠ»Π΅Π΅ ΡΠ»ΠΎΠΆΠ½ΡΠ΅ ΠΏΡΠΈΠΌΠ΅ΡΡ:
ΠΠΎΠ·ΡΠΌΠ΅ΠΌ ΠΏΠΎΡΠ»Π΅Π΄Π½ΠΈΠΉ ΡΠ»Π΅Π½, z-ΠΊΠΎΠΌΠΏΠΎΠ½Π΅Π½ΡΡ.
ΠΡΠΎ (1)(5) ΠΌΠΈΠ½ΡΡ (4)(2), ΠΈΠ»ΠΈ 5 β 8 = -3. Π‘Π½Π°ΡΠ°Π»Π° Ρ ΡΠ΄Π΅Π»Π°Π»
z
, ΠΏΠΎΡΠΎΠΌΡ ΡΡΠΎ ΠΎΠ½ ΠΈΡΠΏΠΎΠ»ΡΠ·ΡΠ΅Ρx
ΠΈy
, ΠΏΠ΅ΡΠ²ΡΠ΅ Π΄Π²Π° ΡΠ΅ΡΠΌΠΈΠ½Π°. ΠΠΎΠΏΡΠΎΠ±ΡΠΉΡΠ΅ ΡΠ²ΠΈΠ΄Π΅ΡΡ (1)(5) ΠΊΠ°ΠΊ Β«Π²ΠΏΠ΅ΡΠ΅Π΄Β», ΠΊΠΎΠ³Π΄Π° Π²Ρ ΡΠΊΠ°Π½ΠΈΡΡΠ΅ΡΠ΅ ΠΎΡ ΠΏΠ΅ΡΠ²ΠΎΠ³ΠΎ Π²Π΅ΠΊΡΠΎΡΠ° ΠΊΠΎ Π²ΡΠΎΡΠΎΠΌΡ, ΠΈ (4)(2) ΠΊΠ°ΠΊ Π½Π°Π·Π°Π΄, ΠΊΠΎΠ³Π΄Π° Π²Ρ Π΄Π²ΠΈΠ³Π°Π΅ΡΠ΅ΡΡ ΠΎΡ Π²ΡΠΎΡΠΎΠ³ΠΎ Π²Π΅ΠΊΡΠΎΡΠ° ΠΊ ΠΏΠ΅ΡΠ²ΠΎΠΌΡ.Π’Π΅ΠΏΠ΅ΡΡ ΠΊΠΎΠΌΠΏΠΎΠ½Π΅Π½Ρ
ΠΈ
: (3)(4) β (6)(1) = 12 β 6 = 6Π’Π΅ΠΏΠ΅ΡΡ ΠΊΠΎΠΌΠΏΠΎΠ½Π΅Π½Ρ
x
: (2)(6) β (5)(3) = 12 β 15 = -3
ΠΡΠΎΠ³ΠΎ $(-3, 6, -3)$, ΡΡΠΎ ΠΌΡ ΠΌΠΎΠΆΠ΅ΠΌ ΠΏΡΠΎΠ²Π΅ΡΠΈΡΡ Ρ ΠΏΠΎΠΌΠΎΡΡΡ Wolfram Alpha.
ΠΠΊΡΠ°ΡΡΠ΅:
ΠΠ΅ΡΠ΅ΠΊΡΠ΅ΡΡΠ½ΠΎΠ΅ ΠΏΡΠΎΠΈΠ·Π²Π΅Π΄Π΅Π½ΠΈΠ΅ ΠΎΡΡΠ»Π΅ΠΆΠΈΠ²Π°Π΅Ρ Π²ΡΠ΅ Β«Π²Π·Π°ΠΈΠΌΠΎΠ΄Π΅ΠΉΡΡΠ²ΠΈΡΒ» ΠΌΠ΅ΠΆΠ΄Ρ ΠΈΠ·ΠΌΠ΅ΡΠ΅Π½ΠΈΡΠΌΠΈ
ΠΡΡΡ 6 Π²Π·Π°ΠΈΠΌΠΎΠ΄Π΅ΠΉΡΡΠ²ΠΈΠΉ (ΠΏΠΎ 2 Π² ΠΊΠ°ΠΆΠ΄ΠΎΠΌ ΠΈΠ·ΠΌΠ΅ΡΠ΅Π½ΠΈΠΈ) ΡΠΎ Π·Π½Π°ΠΊΠ°ΠΌΠΈ, ΠΎΡΠ½ΠΎΠ²Π°Π½Π½ΡΠΌΠΈ Π½Π°
xyzxyz
ΠΏΠΎΡΡΠ΄ΠΊΠ΅
ΠΡΠΈΠ»ΠΎΠΆΠ΅Π½ΠΈΠ΅
Π‘Π²ΡΠ·Ρ Ρ ΠΎΠΏΡΠ΅Π΄Π΅Π»ΠΈΡΠ΅Π»Π΅ΠΌ
ΠΡ ΠΌΠΎΠΆΠ΅ΡΠ΅ Π²ΡΡΠΈΡΠ»ΠΈΡΡ Π²Π΅ΠΊΡΠΎΡΠ½ΠΎΠ΅ ΠΏΡΠΎΠΈΠ·Π²Π΅Π΄Π΅Π½ΠΈΠ΅, ΠΈΡΠΏΠΎΠ»ΡΠ·ΡΡ ΠΎΠΏΡΠ΅Π΄Π΅Π»ΠΈΡΠ΅Π»Ρ ΡΡΠΎΠΉ ΠΌΠ°ΡΡΠΈΡΡ:
ΠΠ΄Π΅ΡΡ Π΅ΡΡΡ ΡΠ΅ΡΠΊΠ°Ρ ΡΠ²ΡΠ·Ρ, ΠΏΠΎΡΠΊΠΎΠ»ΡΠΊΡ ΠΎΠΏΡΠ΅Π΄Π΅Π»ΠΈΡΠ΅Π»Ρ (Β«ΠΏΠ»ΠΎΡΠ°Π΄Ρ/ΠΎΠ±ΡΠ΅ΠΌ ΡΠΎ Π·Π½Π°ΠΊΠΎΠΌΒ») ΠΎΡΡΠ»Π΅ΠΆΠΈΠ²Π°Π΅Ρ Π²ΠΊΠ»Π°Π΄ ΠΎΡΡΠΎΠ³ΠΎΠ½Π°Π»ΡΠ½ΡΡ
ΠΊΠΎΠΌΠΏΠΎΠ½Π΅Π½ΡΠΎΠ².
Π‘ΡΡΠ΅ΡΡΠ²ΡΡΡ ΡΠ΅ΠΎΡΠ΅ΡΠΈΡΠ΅ΡΠΊΠΈΠ΅ ΠΏΡΠΈΡΠΈΠ½Ρ, ΠΏΠΎ ΠΊΠΎΡΠΎΡΡΠΌ Π²Π΅ΠΊΡΠΎΡΠ½ΠΎΠ΅ ΠΏΡΠΎΠΈΠ·Π²Π΅Π΄Π΅Π½ΠΈΠ΅ (ΠΊΠ°ΠΊ ΠΎΡΡΠΎΠ³ΠΎΠ½Π°Π»ΡΠ½ΡΠΉ Π²Π΅ΠΊΡΠΎΡ) Π΄ΠΎΡΡΡΠΏΠ½ΠΎ ΡΠΎΠ»ΡΠΊΠΎ Π² 0, 1, 3 ΠΈΠ»ΠΈ 7 ΠΈΠ·ΠΌΠ΅ΡΠ΅Π½ΠΈΡΡ
. ΠΠ΄Π½Π°ΠΊΠΎ ΠΏΠ΅ΡΠ΅ΠΊΡΠ΅ΡΡΠ½ΠΎΠ΅ ΠΏΡΠΎΠΈΠ·Π²Π΅Π΄Π΅Π½ΠΈΠ΅ ΠΊΠ°ΠΊ ΠΎΡΠ΄Π΅Π»ΡΠ½ΠΎΠ΅ ΡΠΈΡΠ»ΠΎ ΠΏΠΎ ΡΡΡΠ΅ΡΡΠ²Ρ ΡΠ²Π»ΡΠ΅ΡΡΡ ΠΎΠΏΡΠ΅Π΄Π΅Π»ΠΈΡΠ΅Π»Π΅ΠΌ (ΠΏΠ»ΠΎΡΠ°Π΄Ρ ΡΠΎ Π·Π½Π°ΠΊΠΎΠΌ, ΠΎΠ±ΡΠ΅ΠΌ ΠΈΠ»ΠΈ Π³ΠΈΠΏΠ΅ΡΠΎΠ±ΡΠ΅ΠΌ ΠΊΠ°ΠΊ ΡΠΊΠ°Π»ΡΡ).
Π‘ΠΎΠ΅Π΄ΠΈΠ½Π΅Π½ΠΈΠ΅ Ρ Π·Π°Π²ΠΈΡΠΊΠΎΠΌ
ΠΠ·Π³ΠΈΠ± ΠΈΠ·ΠΌΠ΅ΡΡΠ΅Ρ ΡΠΊΡΡΡΠΈΠ²Π°ΡΡΡΡ ΡΠΈΠ»Ρ, ΠΊΠΎΡΠΎΡΡΡ Π²Π΅ΠΊΡΠΎΡΠ½ΠΎΠ΅ ΠΏΠΎΠ»Π΅ ΠΏΡΠΈΠΊΠ»Π°Π΄ΡΠ²Π°Π΅Ρ ΠΊ ΡΠΎΡΠΊΠ΅, ΠΈ ΠΈΠ·ΠΌΠ΅ΡΡΠ΅ΡΡΡ Π²Π΅ΠΊΡΠΎΡΠΎΠΌ, ΠΏΠ΅ΡΠΏΠ΅Π½Π΄ΠΈΠΊΡΠ»ΡΡΠ½ΡΠΌ ΠΏΠΎΠ²Π΅ΡΡ
Π½ΠΎΡΡΠΈ. ΠΡΡΠΊΠΈΠΉ ΡΠ°Π·, ΠΊΠΎΠ³Π΄Π° Π²Ρ ΡΠ»ΡΡΠΈΡΠ΅ Β«ΠΏΠ΅ΡΠΏΠ΅Π½Π΄ΠΈΠΊΡΠ»ΡΡΠ½ΡΠΉ Π²Π΅ΠΊΡΠΎΡΒ», Π½Π°ΡΠΈΠ½Π°ΠΉΡΠ΅ Π΄ΡΠΌΠ°ΡΡ ΠΎ Β«ΠΏΠ΅ΡΠ΅ΠΊΡΠ΅ΡΡΠ½ΠΎΠΌ ΠΏΡΠΎΠΈΠ·Π²Π΅Π΄Π΅Π½ΠΈΠΈΒ».
ΠΠΎΠ·ΡΠΌΠ΅ΠΌ Β«Π΄Π΅ΡΠ΅ΡΠΌΠΈΠ½Π°Π½ΡΒ» ΡΡΠΎΠΉ ΠΌΠ°ΡΡΠΈΡΡ:
ΠΠΌΠ΅ΡΡΠΎ ΡΠΌΠ½ΠΎΠΆΠ΅Π½ΠΈΡ Π²Π·Π°ΠΈΠΌΠΎΠ΄Π΅ΠΉΡΡΠ²ΠΈΠ΅ Π±Π΅ΡΠ΅Ρ ΡΠ°ΡΡΠ½ΡΡ ΠΏΡΠΎΠΈΠ·Π²ΠΎΠ΄Π½ΡΡ. ΠΠ°ΠΊ ΠΈ ΠΏΡΠ΅ΠΆΠ΄Π΅, $\vec{i}$-ΠΊΠΎΠΌΠΏΠΎΠ½Π΅Π½ΡΠ° curl ΠΎΡΠ½ΠΎΠ²Π°Π½Π° Π½Π° Π²Π΅ΠΊΡΠΎΡΠ°Ρ
ΠΈ ΠΏΡΠΎΠΈΠ·Π²ΠΎΠ΄Π½ΡΡ
Π² Π½Π°ΠΏΡΠ°Π²Π»Π΅Π½ΠΈΡΡ
$\vec{j}$ ΠΈ $\vec{k}$.
Π‘Π²ΡΠ·Ρ Ρ ΡΠ΅ΠΎΡΠ΅ΠΌΠΎΠΉ ΠΠΈΡΠ°Π³ΠΎΡΠ°
ΠΠ΅ΡΠ΅ΠΊΡΠ΅ΡΡΠ½ΠΎΠ΅ ΠΏΡΠΎΠΈΠ·Π²Π΅Π΄Π΅Π½ΠΈΠ΅ ΠΈ ΡΠΊΠ°Π»ΡΡΠ½ΠΎΠ΅ ΠΏΡΠΎΠΈΠ·Π²Π΅Π΄Π΅Π½ΠΈΠ΅ ΠΏΠΎΠ΄ΠΎΠ±Π½Ρ ΠΎΡΡΠΎΠ³ΠΎΠ½Π°Π»ΡΠ½ΡΠΌ ΡΡΠΎΡΠΎΠ½Π°ΠΌ ΡΡΠ΅ΡΠ³ΠΎΠ»ΡΠ½ΠΈΠΊΠ°:
ΠΠ»Ρ Π΅Π΄ΠΈΠ½ΠΈΡΠ½ΡΡ
Π²Π΅ΠΊΡΠΎΡΠΎΠ², Π³Π΄Π΅ $|a| = |Π±| = 1$, ΠΈΠΌΠ΅Π΅ΠΌ:
Π― Π½Π΅ΠΌΠ½ΠΎΠ³ΠΎ ΡΡ
ΠΈΡΡΠΈΠ»Π° Π² Π΄ΠΈΠ°Π³ΡΠ°ΠΌΠΌΠ΅ ΡΠ΅ΡΠΊΠΈ, ΡΠ°ΠΊ ΠΊΠ°ΠΊ ΠΌΡ Π΄ΠΎΠ»ΠΆΠ½Ρ ΠΎΡΡΠ»Π΅ΠΆΠΈΠ²Π°ΡΡ ΠΊΠ²Π°Π΄ΡΠ°ΡΡ Π²Π΅Π»ΠΈΡΠΈΠ½ (ΠΊΠ°ΠΊ ΡΡΠΎ Π΄Π΅Π»Π°Π΅ΡΡΡ Π² ΡΠ΅ΠΎΡΠ΅ΠΌΠ΅ ΠΠΈΡΠ°Π³ΠΎΡΠ°).
Advanced Math
ΠΠ΅ΡΠ΅ΠΊΡΠ΅ΡΡΠ½ΠΎΠ΅ ΠΏΡΠΎΠΈΠ·Π²Π΅Π΄Π΅Π½ΠΈΠ΅ ΠΈ Π΄ΡΡΠ·ΡΡ ΡΠ°ΡΡΠΈΡΡΡΡΡΡ Π² Π°Π»Π³Π΅Π±ΡΠ΅ ΠΠ»ΠΈΡΡΠΎΡΠ΄Π° ΠΈ Π³Π΅ΠΎΠΌΠ΅ΡΡΠΈΡΠ΅ΡΠΊΠΎΠΉ Π°Π»Π³Π΅Π±ΡΠ΅. Π― Π²ΡΠ΅ Π΅ΡΠ΅ ΠΈΠ·ΡΡΠ°Ρ ΡΡΠΎ.
ΠΠ΅ΡΠ΅ΠΊΡΠ΅ΡΡΠ½ΠΎΠ΅ ΠΏΡΠΎΠΈΠ·Π²Π΅Π΄Π΅Π½ΠΈΠ΅ ΠΏΠ΅ΡΠ΅ΠΊΡΠ΅ΡΡΠ½ΡΡ
ΠΏΡΠΎΠΈΠ·Π²Π΅Π΄Π΅Π½ΠΈΠΉ
ΠΠ½ΠΎΠ³Π΄Π° Ρ Π²Π°Ρ Π±ΡΠ΄Π΅Ρ ΡΠ°ΠΊΠΎΠΉ ΡΡΠ΅Π½Π°ΡΠΈΠΉ:
ΠΠΎ-ΠΏΠ΅ΡΠ²ΡΡ
, ΠΏΠ΅ΡΠ΅ΠΊΡΠ΅ΡΡΠ½ΠΎΠ΅ ΠΏΡΠΎΠΈΠ·Π²Π΅Π΄Π΅Π½ΠΈΠ΅ Π½Π΅ Π°ΡΡΠΎΡΠΈΠ°ΡΠΈΠ²Π½ΠΎ: ΠΏΠΎΡΡΠ΄ΠΎΠΊ ΠΈΠΌΠ΅Π΅Ρ Π·Π½Π°ΡΠ΅Π½ΠΈΠ΅.
ΠΠ°Π»Π΅Π΅, Π²ΡΠΏΠΎΠΌΠ½ΠΈΠΌ, ΡΡΠΎ Π΄Π΅Π»Π°Π΅Ρ Π²Π΅ΠΊΡΠΎΡΠ½ΠΎΠ΅ ΠΏΡΠΎΠΈΠ·Π²Π΅Π΄Π΅Π½ΠΈΠ΅: Π½Π°Ρ
ΠΎΠ΄ΠΈΡ ΠΎΡΡΠΎΠ³ΠΎΠ½Π°Π»ΡΠ½ΡΠ΅ Π²Π΅ΠΊΡΠΎΡΡ. ΠΡΠ»ΠΈ Π»ΡΠ±ΡΠ΅ Π΄Π²Π΅ ΠΊΠΎΠΌΠΏΠΎΠ½Π΅Π½ΡΡ ΠΏΠ°ΡΠ°Π»Π»Π΅Π»ΡΠ½Ρ ($\vec{a}$ ΠΏΠ°ΡΠ°Π»Π»Π΅Π»ΡΠ½Ρ $\vec{b}$), ΡΠΎ Π½Π΅Ρ ΡΠ°Π·ΠΌΠ΅ΡΠ½ΠΎΡΡΠ΅ΠΉ, ΡΡΠ°Π»ΠΊΠΈΠ²Π°ΡΡΠΈΡ
Π΄ΡΡΠ³ Π΄ΡΡΠ³Π°, ΠΈ Π²Π΅ΠΊΡΠΎΡΠ½ΠΎΠ΅ ΠΏΡΠΎΠΈΠ·Π²Π΅Π΄Π΅Π½ΠΈΠ΅ ΡΠ°Π²Π½ΠΎ Π½ΡΠ»Ρ (ΡΡΠΎ ΠΏΡΠΈΠ²ΠΎΠ΄ΠΈΡ ΠΊ $0 \times \vec {Ρ}$).
ΠΠΎ $\vec{a}$ ΠΈ $\vec{c}$ ΠΌΠΎΠ³ΡΡ Π±ΡΡΡ ΠΏΠ°ΡΠ°Π»Π»Π΅Π»ΡΠ½Ρ, ΠΏΠΎΡΠΊΠΎΠ»ΡΠΊΡ ΠΎΠ½ΠΈ Π½ΠΈΠΊΠΎΠ³Π΄Π° Π½Π΅ ΡΡΠ°ΡΡΠ²ΡΡΡ Π½Π°ΠΏΡΡΠΌΡΡ Π² ΠΏΠ΅ΡΠ΅ΠΊΡΠ΅ΡΡΠ½ΠΎΠΌ ΠΏΡΠΎΠΈΠ·Π²Π΅Π΄Π΅Π½ΠΈΠΈ, Π½Π°ΠΏΡΠΈΠΌΠ΅Ρ:
ΠΠ³ΠΎ! ΠΠ°ΠΊ ΠΌΡ Π²Π΅ΡΠ½ΡΠ»ΠΈΡΡ ΠΊ $\vec{j}$? ΠΡ Π·Π°ΠΏΡΠΎΡΠΈΠ»ΠΈ Π½Π°ΠΏΡΠ°Π²Π»Π΅Π½ΠΈΠ΅, ΠΏΠ΅ΡΠΏΠ΅Π½Π΄ΠΈΠΊΡΠ»ΡΡΠ½ΠΎΠ΅ ΠΊΠ°ΠΊ $\vec{i}$, ΡΠ°ΠΊ ΠΈ $\vec{j}$, ΠΈ ΡΠ½ΠΎΠ²Π° ΡΠ΄Π΅Π»Π°Π»ΠΈ ΡΡΠΎ Π½Π°ΠΏΡΠ°Π²Π»Π΅Π½ΠΈΠ΅ ΠΏΠ΅ΡΠΏΠ΅Π½Π΄ΠΈΠΊΡΠ»ΡΡΠ½ΡΠΌ ΠΊ $\vec{i}$.
ΠΡΡΡ Β«Π΄Π²ΠΎΠΉΠ½ΡΠΌ ΠΏΠ΅ΡΠΏΠ΅Π½Π΄ΠΈΠΊΡΠ»ΡΡΠΎΠΌΒ» ΠΎΠ·Π½Π°ΡΠ°Π΅Ρ, ΡΡΠΎ Π²Ρ Π²Π΅ΡΠ½ΡΠ»ΠΈΡΡ Π½Π° ΠΈΡΡ
ΠΎΠ΄Π½ΡΡ ΠΎΡΡ.
Π‘ΠΊΠ°Π»ΡΡΠ½ΠΎΠ΅ ΠΏΡΠΎΠΈΠ·Π²Π΅Π΄Π΅Π½ΠΈΠ΅ ΠΏΠ΅ΡΠ΅ΠΊΡΠ΅ΡΡΠ½ΡΡ
ΠΏΡΠΎΠΈΠ·Π²Π΅Π΄Π΅Π½ΠΈΠΉ
Π’Π΅ΠΏΠ΅ΡΡ, Π΅ΡΠ»ΠΈ ΠΌΡ Π²ΠΎΠ·ΡΠΌΠ΅ΠΌ
, ΡΡΠΎ ΠΏΡΠΎΠΈΠ·ΠΎΠΉΠ΄Π΅Ρ? ΠΡ Π²ΡΠ½ΡΠΆΠ΄Π΅Π½Ρ ΡΠ½Π°ΡΠ°Π»Π° Π²ΡΠΏΠΎΠ»Π½ΠΈΡΡ $\vec{a} \times \vec{b}$, ΠΏΠΎΡΠΎΠΌΡ ΡΡΠΎ $\vec{b} \cdot \vec{c}$ Π²ΠΎΠ·Π²ΡΠ°ΡΠ°Π΅Ρ ΡΠΊΠ°Π»ΡΡ (ΠΎΠ΄Π½ΠΎ ΡΠΈΡΠ»ΠΎ), ΠΊΠΎΡΠΎΡΡΠΉ Π½Π΅Π»ΡΠ·Ρ ΠΈΡΠΏΠΎΠ»ΡΠ·ΠΎΠ²Π°ΡΡ Π² ΠΏΠ΅ΡΠ΅ΠΊΡΠ΅ΡΡΠ½ΠΎΠΌ ΠΏΡΠΎΠΈΠ·Π²Π΅Π΄Π΅Π½ΠΈΠΈ.
Π§ΡΠΎ ΠΏΡΠΎΠΈΠ·ΠΎΠΉΠ΄Π΅Ρ, Π΅ΡΠ»ΠΈ $\vec{a}$ ΠΈ $\vec{c}$ ΠΏΠ°ΡΠ°Π»Π»Π΅Π»ΡΠ½Ρ? ΠΡΠ°ΠΊ, $\vec{a} \times \vec{b}$ ΠΏΠ΅ΡΠΏΠ΅Π½Π΄ΠΈΠΊΡΠ»ΡΡΠ½ΠΎ $\vec{a}$, ΡΡΠΎ ΠΎΠ·Π½Π°ΡΠ°Π΅Ρ, ΡΡΠΎ ΠΎΠ½ΠΎ ΠΏΠ΅ΡΠΏΠ΅Π½Π΄ΠΈΠΊΡΠ»ΡΡΠ½ΠΎ $\vec{c}$, ΠΏΠΎΡΡΠΎΠΌΡ ΡΠΊΠ°Π»ΡΡΠ½ΠΎΠ΅ ΠΏΡΠΎΠΈΠ·Π²Π΅Π΄Π΅Π½ΠΈΠ΅ Ρ $\vec{c }$ Π±ΡΠ΄Π΅Ρ ΡΠ°Π²Π½ΠΎ Π½ΡΠ»Ρ.
Π― Π½ΠΈΠΊΠΎΠ³Π΄Π° Π½Π΅ Π·Π°ΠΏΠΎΠΌΠΈΠ½Π°Π» ΡΡΠΈ ΠΏΡΠ°Π²ΠΈΠ»Π°, Ρ Π΄ΠΎΠ»ΠΆΠ΅Π½ ΠΏΡΠΎΠ΄ΡΠΌΡΠ²Π°ΡΡ Π²Π·Π°ΠΈΠΌΠΎΠ΄Π΅ΠΉΡΡΠ²ΠΈΠ΅.
ΠΡΡΠ³ΠΈΠ΅ ΡΠΈΡΡΠ΅ΠΌΡ ΠΊΠΎΠΎΡΠ΄ΠΈΠ½Π°Ρ
ΠΠ³ΡΠΎΠ²ΠΎΠΉ Π΄Π²ΠΈΠΆΠΎΠΊ Unity ΠΏΡΠ΅Π΄Π½Π°Π·Π½Π°ΡΠ΅Π½ Π΄Π»Ρ Π»Π΅Π²ΡΠ΅ΠΉ, OpenGL (ΠΈ Π±ΠΎΠ»ΡΡΠΈΠ½ΡΡΠ²ΠΎ ΠΌΠ°ΡΠ΅ΠΌΠ°ΡΠΈΡΠ΅ΡΠΊΠΈΡ
ΠΈ ΡΠΈΠ·ΠΈΡΠ΅ΡΠΊΠΈΡ
ΠΈΠ½ΡΡΡΡΠΌΠ΅Π½ΡΠΎΠ²) β Π΄Π»Ρ ΠΏΡΠ°Π²ΡΠ΅ΠΉ. ΠΠΎΡΠ΅ΠΌΡ?
Π ΠΊΠΎΠΌΠΏΡΡΡΠ΅ΡΠ½ΠΎΠΉ ΠΈΠ³ΡΠ΅
x
ΠΈΠ΄Π΅Ρ ΠΏΠΎ Π³ΠΎΡΠΈΠ·ΠΎΠ½ΡΠ°Π»ΠΈ, y
ΠΏΠΎ Π²Π΅ΡΡΠΈΠΊΠ°Π»ΠΈ, Π° z
ΡΡ
ΠΎΠ΄ΠΈΡ Β«Π² ΡΠΊΡΠ°Π½Β». Π ΡΠ΅Π·ΡΠ»ΡΡΠ°ΡΠ΅ ΠΏΠΎΠ»ΡΡΠ°Π΅ΡΡΡ Π»Π΅Π²ΠΎΡΡΠΎΡΠΎΠ½Π½ΡΡ ΡΠΈΡΡΠ΅ΠΌΠ°. (ΠΠΎΠΏΡΠΎΠ±ΡΠΉΡΠ΅: ΠΈΡΠΏΠΎΠ»ΡΠ·ΡΡ ΠΏΡΠ°Π²ΡΡ ΡΡΠΊΡ, Π²Ρ ΠΌΠΎΠΆΠ΅ΡΠ΅ ΡΠ²ΠΈΠ΄Π΅ΡΡ
x
ΠΊΡΠ΅ΡΡΠ° ΠΈ
Π΄ΠΎΠ»ΠΆΠ½Ρ ΡΠΊΠ°Π·ΡΠ²Π°ΡΡ Π·Π° ΠΏΡΠ΅Π΄Π΅Π»Ρ ΡΠΊΡΠ°Π½Π°).
ΠΡΠΈΠΌΠ΅Π½Π΅Π½ΠΈΠ΅ Π²Π΅ΠΊΡΠΎΡΠ½ΠΎΠ³ΠΎ ΠΏΡΠΎΠΈΠ·Π²Π΅Π΄Π΅Π½ΠΈΡ
ΠΠ°ΠΉΠ΄ΠΈΡΠ΅ Π½Π°ΠΏΡΠ°Π²Π»Π΅Π½ΠΈΠ΅, ΠΏΠ΅ΡΠΏΠ΅Π½Π΄ΠΈΠΊΡΠ»ΡΡΠ½ΠΎΠ΅ Π΄Π²ΡΠΌ Π·Π°Π΄Π°Π½Π½ΡΠΌ Π²Π΅ΠΊΡΠΎΡΠ°ΠΌ.
ΠΠ°ΠΉΠ΄ΠΈΡΠ΅ ΠΎΠ±Π»Π°ΡΡΡ ΡΠΎ Π·Π½Π°ΠΊΠΎΠΌ, Π½Π°ΡΡΠ½ΡΡΡΡ Π½Π° Π΄Π²Π° Π²Π΅ΠΊΡΠΎΡΠ°.
ΠΠΏΡΠ΅Π΄Π΅Π»ΠΈΡΡ, ΡΠ²Π»ΡΡΡΡΡ Π»ΠΈ Π΄Π²Π° Π²Π΅ΠΊΡΠΎΡΠ° ΠΎΡΡΠΎΠ³ΠΎΠ½Π°Π»ΡΠ½ΡΠΌΠΈ (Ρ ΠΎΡΡ ΠΏΡΠΎΠ²Π΅ΡΠΊΠ° ΡΠΊΠ°Π»ΡΡΠ½ΠΎΠ³ΠΎ ΠΏΡΠΎΠΈΠ·Π²Π΅Π΄Π΅Π½ΠΈΡ 0, Π²Π΅ΡΠΎΡΡΠ½ΠΎ, Π²ΡΠΏΠΎΠ»Π½ΡΠ΅ΡΡΡ Π±ΡΡΡΡΠ΅Π΅).
Β«Π£ΠΌΠ½ΠΎΠΆΠ΅Π½ΠΈΠ΅Β» Π΄Π²ΡΡ Π²Π΅ΠΊΡΠΎΡΠΎΠ², ΠΊΠΎΠ³Π΄Π° Π²ΠΊΠ»Π°Π΄ Π²Π½ΠΎΡΡΡ ΡΠΎΠ»ΡΠΊΠΎ ΠΏΠ΅ΡΠΏΠ΅Π½Π΄ΠΈΠΊΡΠ»ΡΡΠ½ΡΠ΅ ΠΏΠ΅ΡΠ΅ΠΊΡΠ΅ΡΡΠ½ΡΠ΅ ΡΠ»Π΅Π½Ρ (Π½Π°ΠΏΡΠΈΠΌΠ΅Ρ, ΠΎΠΏΡΠ΅Π΄Π΅Π»Π΅Π½ΠΈΠ΅ ΠΊΡΡΡΡΡΠ΅Π³ΠΎ ΠΌΠΎΠΌΠ΅Π½ΡΠ°).
Π‘ ΠΊΠ²Π°ΡΠ΅ΡΠ½ΠΈΠΎΠ½Π°ΠΌΠΈ (ΡΠ΅ΡΡΡΠ΅ ΠΊΠΎΠΌΠΏΠ»Π΅ΠΊΡΠ½ΡΡ ΡΠΈΡΠ»Π°) Π²Π΅ΠΊΡΠΎΡΠ½ΠΎΠ΅ ΠΏΡΠΎΠΈΠ·Π²Π΅Π΄Π΅Π½ΠΈΠ΅ Π²ΡΠΏΠΎΠ»Π½ΡΠ΅Ρ ΡΠ°Π±ΠΎΡΡ ΠΏΠΎ Π²ΡΠ°ΡΠ΅Π½ΠΈΡ ΠΎΠ΄Π½ΠΎΠ³ΠΎ Π²Π΅ΠΊΡΠΎΡΠ° Π²ΠΎΠΊΡΡΠ³ Π΄ΡΡΠ³ΠΎΠ³ΠΎ (Π΅ΡΠ΅ ΠΎΠ΄Π½Π° ΡΡΠ°ΡΡΡ Π² ΡΠ°Π·ΡΠ°Π±ΠΎΡΠΊΠ΅!).
Π‘ΡΠ°ΡΡΠ»ΠΈΠ²Π°Ρ ΠΌΠ°ΡΠ΅ΠΌΠ°ΡΠΈΠΊΠ°.
ΠΡΡΠ³ΠΈΠ΅ ΠΏΡΠ±Π»ΠΈΠΊΠ°ΡΠΈΠΈ ΠΈΠ· ΡΡΠΎΠΉ ΡΠ΅ΡΠΈΠΈ
ΠΠ΅ΠΊΡΠΎΡΠ½ΠΎΠ΅ ΠΈΡΡΠΈΡΠ»Π΅Π½ΠΈΠ΅: ΠΏΠΎΠ½ΠΈΠΌΠ°Π½ΠΈΠ΅ ΡΠΊΠ°Π»ΡΡΠ½ΠΎΠ³ΠΎ ΠΏΡΠΎΠΈΠ·Π²Π΅Π΄Π΅Π½ΠΈΡ
ΠΠ΅ΠΊΡΠΎΡΠ½ΠΎΠ΅ ΠΈΡΡΠΈΡΠ»Π΅Π½ΠΈΠ΅: ΠΏΠΎΠ½ΠΈΠΌΠ°Π½ΠΈΠ΅ Π²Π΅ΠΊΡΠΎΡΠ½ΠΎΠ³ΠΎ ΠΏΡΠΎΠΈΠ·Π²Π΅Π΄Π΅Π½ΠΈΡ
ΠΠ΅ΠΊΡΠΎΡΠ½ΠΎΠ΅ ΠΈΡΡΠΈΡΠ»Π΅Π½ΠΈΠ΅: ΠΏΠΎΠ½ΠΈΠΌΠ°Π½ΠΈΠ΅ ΠΏΠΎΡΠΎΠΊΠ°
ΠΠ΅ΠΊΡΠΎΡΠ½ΠΎΠ΅ ΠΈΡΡΠΈΡΠ»Π΅Π½ΠΈΠ΅: ΠΏΠΎΠ½ΠΈΠΌΠ°Π½ΠΈΠ΅ Π΄ΠΈΠ²Π΅ΡΠ³Π΅Π½ΡΠΈΠΈ
ΠΠ΅ΠΊΡΠΎΡΠ½ΠΎΠ΅ ΠΈΡΡΠΈΡΠ»Π΅Π½ΠΈΠ΅: ΠΏΠΎΠ½ΠΈΠΌΠ°Π½ΠΈΠ΅ ΡΠΈΡΠΊΡΠ»ΡΡΠΈΠΈ ΠΈ Π·Π°Π²ΠΈΡΠΊΠ°
ΠΠ΅ΠΊΡΠΎΡΠ½ΠΎΠ΅ ΠΈΡΡΠΈΡΠ»Π΅Π½ΠΈΠ΅: ΠΏΠΎΠ½ΠΈΠΌΠ°Π½ΠΈΠ΅ Π³ΡΠ°Π΄ΠΈΠ΅Π½ΡΠ°
ΠΠΎΠ½ΠΈΠΌΠ°Π½ΠΈΠ΅ ΠΏΠΈΡΠ°Π³ΠΎΡΠ΅ΠΉΡΠΊΠΎΠ³ΠΎ ΡΠ°ΡΡΡΠΎΡΠ½ΠΈΡ ΠΈ Π³ΡΠ°Π΄ΠΈΠ΅Π½ΡΠ°
ΠΠ°ΠΊ Π²ΡΡΠΈΡΠ»ΠΈΡΡ Π²Π΅ΠΊΡΠΎΡΠ½ΠΎΠ΅ ΠΏΡΠΎΠΈΠ·Π²Π΅Π΄Π΅Π½ΠΈΠ΅ ΠΈ ΠΏΠ»ΠΎΡΠ°Π΄Ρ ΠΏΡΡΠΌΠΎΡΠ³ΠΎΠ»ΡΠ½ΠΈΠΊΠ° Π·Π° 3 ΠΏΡΠΎΡΡΡΡ
ΡΠ°Π³Π°
15 ΠΈΡΠ½Ρ 2021 517 ΠΏΡΠΎΡΡΠ΅Π½ΠΈΠΉ
ΠΠ»ΠΎΡΠ°Π΄Ρ ΠΏΡΡΠΌΠΎΡΠ³ΠΎΠ»ΡΠ½ΠΈΠΊΠ° β ΡΡΠΎ ΠΊΠΎΠ»ΠΈΡΠ΅ΡΡΠ²ΠΎ ΠΏΠ»ΠΎΡΠΊΠΈΡ
ΠΎΠ±Π»Π°ΡΡΠ΅ΠΉ, ΠΎΠ³ΡΠ°Π½ΠΈΡΠ΅Π½Π½ΡΡ
ΠΎΠΏΡΠ΅Π΄Π΅Π»Π΅Π½Π½ΠΎΠΉ ΡΠΎΡΠΌΠΎΠΉ.
ΠΠ»ΠΎΡΠ°Π΄Ρ ΠΏΡΡΠΌΠΎΡΠ³ΠΎΠ»ΡΠ½ΠΈΠΊΠ° β ΡΡΠΎ ΠΏΠ»ΠΎΡΠ°Π΄Ρ ΡΠΎΡΠΌΡ ΠΈ ΠΏΠ΅ΡΠΈΠΌΠ΅ΡΡΠ° ΠΏΡΡΠΌΠΎΡΠ³ΠΎΠ»ΡΠ½ΠΈΠΊΠ°. ΠΡΡΠΈΡΠ»Π΅Π½ΠΈΠ΅ ΠΏΠ»ΠΎΡΠ°Π΄ΠΈ Π² 3 ΠΏΡΠΎΡΡΡΡ
ΡΠ°Π³Π° β ΡΡΠΎ ΠΏΡΠΎΡΡΠ°Ρ ΠΏΡΠΎΡΠ΅Π΄ΡΡΠ°, ΠΊΠ°ΠΊ ΠΊΡΡΠΎΠΊ ΠΏΠΈΡΠΎΠ³Π°. Π‘ΡΡΠ΄Π΅Π½ΡΡ ΠΎΠ±ΡΡΠ½ΠΎ ΡΡΠΈΡΠ°ΡΡ, ΡΡΠΎ ΠΈΡ
ΠΎΡΠ΅Π½Ρ ΡΠ»ΠΎΠΆΠ½ΠΎ ΠΏΠΎΠ½ΡΡΡ, Π½ΠΎ ΡΡΠΎ Π½Π΅ΡΠ»ΠΎΠΆΠ½ΠΎ, ΠΏΠΎΡΠΊΠΎΠ»ΡΠΊΡ Ρ ΡΡΠΈΠΌ Π²Ρ Π²ΠΈΠ΄ΠΈΡΠ΅, ΡΡΠΎ Π²ΡΡΠΈΡΠ»Π΅Π½ΠΈΠ΅ ΠΏΠ»ΠΎΡΠ°Π΄ΠΈ ΠΈ Π²Π΅ΠΊΡΠΎΡΠ½ΠΎΠ³ΠΎ ΠΏΡΠΎΠΈΠ·Π²Π΅Π΄Π΅Π½ΠΈΡ Π΄Π²ΡΡ
Π²Π΅ΠΊΡΠΎΡΠΎΠ² ΠΌΠΎΠΆΠ½ΠΎ Π²ΡΠΏΠΎΠ»Π½ΠΈΡΡ Π²ΡΠ΅Π³ΠΎ Π·Π° ΡΡΠΈ ΠΏΡΠΎΡΡΡΡ
ΡΠ°Π³Π°. ΠΠ»Ρ Π²ΡΡΠΈΡΠ»Π΅Π½ΠΈΡ ΠΏΠ»ΠΎΡΠ°Π΄ΠΈ, Ρ ΡΠΎΡΠΊΠΈ Π·ΡΠ΅Π½ΠΈΡ ΡΠΎΡΠΌΡ, ΠΏΡΡΠΌΠΎΡΠ³ΠΎΠ»ΡΠ½ΠΈΠΊ ΡΡΠΈΡΠ°Π΅ΡΡΡ ΡΠ°ΠΌΡΠΌ ΠΏΡΠΎΡΡΡΠΌ.
@james-johnson
ΠΠΆΠ΅ΠΉΠΌΡ ΠΠΆΠΎΠ½ΡΠΎΠ½ Π― ΠΈΡΡΠ»Π΅Π΄ΠΎΠ²Π°ΡΠ΅Π»Ρ ΠΈ Π°Π²ΡΠΎΡ ΡΠ΅Ρ
Π½ΠΈΡΠ΅ΡΠΊΠΈΡ
ΠΌΠ°ΡΠ΅ΡΠΈΠ°Π»ΠΎΠ². ΠΡΠ΅ΠΏΠΎΠ΄Π°Π²Π°ΡΠ΅Π»Ρ ΠΌΠ°ΡΠ΅ΠΌΠ°ΡΠΈΠΊΠΈ Ρ 2007 Π³ΠΎΠ΄Π°. ΠΡΠ±Π»Ρ ΠΏΡΡΠ΅ΡΠ΅ΡΡΠ²ΠΎΠ²Π°ΡΡ, ΠΡΠ±ΠΎΠ²Ρ
Π @james-johnson
ΠΡΠΎΡΡ-ΠΏΡΠΎΠΈΠ·Π²Π΅Π΄Π΅Π½ΠΈΠ΅
ΠΠΎΠΌΠΈΠΌΠΎ ΡΠΊΠ°Π»ΡΡΠ½ΡΡ
ΠΏΡΠΎΠΈΠ·Π²Π΅Π΄Π΅Π½ΠΈΠΉ, ΡΡΡΠ΅ΡΡΠ²ΡΠ΅Ρ Π΄ΡΡΠ³ΠΎΠΉ ΠΏΠΎΠ΄Ρ
ΠΎΠ΄ ΠΊ ΡΠΌΠ½ΠΎΠΆΠ΅Π½ΠΈΡ Π΄Π²ΡΡ
Π²Π΅ΠΊΡΠΎΡΠΎΠ². Π₯ΠΎΡΡ ΡΠΊΠ°Π»ΡΡΠ½ΠΎΠ΅ ΠΏΡΠΎΠΈΠ·Π²Π΅Π΄Π΅Π½ΠΈΠ΅ Π΄Π²ΡΡ
Π²Π΅ΠΊΡΠΎΡΠΎΠ² Π΄Π°Π΅Ρ ΡΠΊΠ°Π»ΡΡ, ΡΠ΅Π·ΡΠ»ΡΡΠΈΡΡΡΡΠΈΠΉ Π²Π΅ΠΊΡΠΎΡ, Π½Π°ΠΏΡΠΎΡΠΈΠ², ΡΠ²Π»ΡΠ΅ΡΡΡ Π²Π΅ΠΊΡΠΎΡΠΎΠΌ Π²Π΅ΠΊΡΠΎΡΠ½ΡΡ
ΠΏΡΠΎΠΈΠ·Π²Π΅Π΄Π΅Π½ΠΈΠΉ. Π’Π°ΠΊΠΈΠΌ ΠΎΠ±ΡΠ°Π·ΠΎΠΌ, Π΅Π³ΠΎ ΠΌΠ°ΡΡΡΠ°Π± ΠΈ Π½Π°ΠΏΡΠ°Π²Π»Π΅Π½ΠΈΠ΅ ΡΠΎΠ²ΠΏΠ°Π΄Π°ΡΡ. Π’Π°ΠΊΠΈΠΌ ΠΎΠ±ΡΠ°Π·ΠΎΠΌ, Π²Π΅ΠΊΡΠΎΡΠ½ΠΎΠ΅ ΠΏΡΠΎΠΈΠ·Π²Π΅Π΄Π΅Π½ΠΈΠ΅ Π²Π΅ΠΊΡΠΎΡΠΎΠ² Π²ΡΡΠ°ΠΆΠ°Π΅ΡΡΡ Π² Π²Π΅Π»ΠΈΡΠΈΠ½Π΅ ΠΈ ΡΠ³Π»Π΅ Π΄Π²ΡΡ
Π²Π΅ΠΊΡΠΎΡΠΎΠ² ΠΈ Π·Π°ΠΏΠΈΡΡΠ²Π°Π΅ΡΡΡ ΠΊΠ°ΠΊ
ΠΠ±Π° Π²Π΅ΠΊΡΠΎΡΠ° ΠΏΠ΅ΡΠΏΠ΅Π½Π΄ΠΈΠΊΡΠ»ΡΡΠ½Ρ Π½Π°ΠΏΡΠ°Π²Π»Π΅Π½ΠΈΡ Π²Π΅ΠΊΡΠΎΡΠ½ΠΎΠ³ΠΎ ΠΏΡΠΎΠΈΠ·Π²Π΅Π΄Π΅Π½ΠΈΡ.
ΠΡΠΏΠΎΠ»ΡΠ·ΡΠ΅ΡΡΡ ΠΏΡΠ°Π²ΠΈΠ»ΠΎ ΠΏΡΠ°Π²ΠΎΠΉ ΡΡΠΊΠΈ Π΄Π»Ρ ΠΏΠΎΠ»ΡΡΠ΅Π½ΠΈΡ ΡΠΎΠΎΡΠ²Π΅ΡΡΡΠ²ΡΡΡΠ΅ΠΉ ΠΎΡΠΈΠ΅Π½ΡΠ°ΡΠΈΠΈ ΠΏΠ΅ΡΠ΅ΠΊΡΠ΅ΡΡΠ½ΠΎΠ³ΠΎ ΠΏΡΠΎΠΈΠ·Π²Π΅Π΄Π΅Π½ΠΈΡ. ΠΠ°ΠΏΡΠ°Π²ΡΡΠ΅ ΡΠΊΠ°Π·Π°ΡΠ΅Π»ΡΠ½ΡΠΉ ΠΏΠ°Π»Π΅Ρ (ΠΏΡΠ°Π²ΠΎΠΉ ΡΡΠΊΠΈ) Π² Π½Π°ΠΏΡΠ°Π²Π»Π΅Π½ΠΈΠΈ ΠΏΠ΅ΡΠ²ΠΎΠ³ΠΎ Π²Π΅ΠΊΡΠΎΡΠ°. ΠΠ°ΡΠ΅ΠΌ Π²ΡΡΠΎΠ²Π½ΡΠΉΡΠ΅ ΡΡΠΊΡ ΠΏΠΎ Π²ΡΠΎΡΠΎΠΌΡ Π²Π΅ΠΊΡΠΎΡΡ ΡΠ°ΠΊ, ΡΡΠΎΠ±Ρ Π²Π°Ρ ΡΡΠ΅Π΄Π½ΠΈΠΉ ΠΏΠ°Π»Π΅Ρ ΡΠΊΠ°Π·ΡΠ²Π°Π» Π½Π° Π½Π΅Π³ΠΎ. Π Π°Π·Π²Π΅ΡΠ½ΠΈΡΠ΅ Π±ΠΎΠ»ΡΡΠΎΠΉ ΠΏΠ°Π»Π΅Ρ. ΠΠ°ΠΏΡΠ°Π²Π»Π΅Π½ΠΈΠ΅ Π²Π°ΡΠ΅Π³ΠΎ Π±ΠΎΠ»ΡΡΠΎΠ³ΠΎ ΠΏΠ°Π»ΡΡΠ° ΡΠΊΠ°Π·ΡΠ²Π°Π΅Ρ Π½Π°ΠΏΡΠ°Π²Π»Π΅Π½ΠΈΠ΅ ΠΏΠ΅ΡΠ΅ΠΊΡΠ΅ΡΡΠ½ΠΎΠ³ΠΎ ΠΏΡΠΎΠΈΠ·Π²Π΅Π΄Π΅Π½ΠΈΡ.
ΠΠ»Ρ Π΄Π²ΡΡ
Π²Π΅ΠΊΡΠΎΡΠΎΠ² ΡΠ½Π°ΡΠ°Π»Π° Π½ΡΠΆΠ½ΠΎ Π³Π°ΡΠ°Π½ΡΠΈΡΠΎΠ²Π°ΡΡ, ΡΡΠΎ ΠΎΠ±Π° Π²Π΅ΠΊΡΠΎΡΠ° ΡΠ²Π»ΡΡΡΡΡ ΡΡΠ΅Ρ
ΠΌΠ΅ΡΠ½ΡΠΌΠΈ, ΡΡΠΎΠ±Ρ ΠΏΠΎΠ»ΡΡΠΈΡΡ Π²Π΅ΠΊΡΠΎΡΠ½ΠΎΠ΅ ΠΏΡΠΎΠΈΠ·Π²Π΅Π΄Π΅Π½ΠΈΠ΅. ΠΡΠ»ΠΈ Π΄Π²Π° Π²Π΅ΠΊΡΠΎΡΠ° Π΄Π°Π½Ρ Π² 3-Ρ
ΠΈΠ·ΠΌΠ΅ΡΠ΅Π½ΠΈΡΡ
, ΡΠΎ ΠΏΡΠΎΠΈΠ·Π²Π΅Π΄Π΅Π½ΠΈΠ΅ Π·Π°Π΄Π°Π΅ΡΡΡ ΠΊΠ°ΠΊ:
ΠΡΡΠΈΡΠ»Π΅Π½ΠΈΠ΅ Π²Π΅ΠΊΡΠΎΡΠ½ΠΎΠ³ΠΎ ΠΏΡΠΎΠΈΠ·Π²Π΅Π΄Π΅Π½ΠΈΡ
Π§ΡΠΎΠ±Ρ Π²ΡΡΠΈΡΠ»ΠΈΡΡ Π²Π΅ΠΊΡΠΎΡΠ½ΠΎΠ΅ ΠΏΡΠΎΠΈΠ·Π²Π΅Π΄Π΅Π½ΠΈΠ΅ Π΄Π²ΡΡ
Π²Π΅ΠΊΡΠΎΡΠΎΠ², ΠΈΡΠΏΠΎΠ»ΡΠ·ΡΠΉΡΠ΅ ΡΡΠ°Π½Π΄Π°ΡΡΠ½ΡΠ΅ Π΅Π΄ΠΈΠ½ΠΈΡΠ½ΡΠ΅ Π²Π΅ΠΊΡΠΎΡΡ i, j ΠΈ k Π΄Π»Ρ R3.
ΠΡΠ»ΠΈ u = (u1, u2, u3) = u1i + u2j + u3k,
ΠΈ v = (v1, v2, v3) = v1i + v2j + v3k,
ΡΠΎΠ³Π΄Π° u Γ v Π·Π°Π΄Π°Π΅ΡΡΡ ΠΊΠ°ΠΊ
u Γ v = (u2v3 β u3v2) i + (u3v1 β u1v3) j + (u1v2 β u2v1) k
ΠΠ»ΠΎΡΠ°Π΄Ρ ΠΏΡΡΠΌΠΎΡΠ³ΠΎΠ»ΡΠ½ΠΈΠΊΠ°
Π§Π΅ΡΡΡΠ΅Ρ
ΡΡΠΎΡΠΎΠ½Π½ΠΈΠΉ ΠΌΠ½ΠΎΠ³ΠΎΡΠ³ΠΎΠ»ΡΠ½ΠΈΠΊ, Π² ΠΊΠΎΡΠΎΡΠΎΠΌ ΠΊΠ°ΠΆΠ΄ΡΠΉ ΡΠ³ΠΎΠ» ΠΊΠ°ΠΆΠ΄ΠΎΠΉ Π²Π΅ΡΡΠΈΠ½Ρ ΡΠ°Π²Π΅Π½ 90Β° ΠΈ ΠΈΠΌΠ΅Π΅Ρ ΠΎΠ΄ΠΈΠ½Π°ΠΊΠΎΠ²ΡΡ Π΄Π»ΠΈΠ½Ρ ΠΏΡΠΎΡΠΈΠ²ΠΎΠΏΠΎΠ»ΠΎΠΆΠ½ΡΡ
ΡΡΠΎΡΠΎΠ½, Π½Π°Π·ΡΠ²Π°Π΅ΡΡΡ ΠΏΡΡΠΌΠΎΡΠ³ΠΎΠ»ΡΠ½ΠΈΠΊΠΎΠΌ.
Π’Π°ΠΊΠΈΠΌ ΠΎΠ±ΡΠ°Π·ΠΎΠΌ, Π΄Π²ΡΡ
ΠΌΠ΅ΡΠ½ΡΠΉ ΠΌΠ½ΠΎΠ³ΠΎΡΠ³ΠΎΠ»ΡΠ½ΠΈΠΊ ΠΏΡΠ΅Π΄ΡΡΠ°Π²Π»ΡΠ΅Ρ ΡΠΎΠ±ΠΎΠΉ ΠΏΡΡΠΌΠΎΡΠ³ΠΎΠ»ΡΠ½ΠΈΠΊ Ρ ΡΠ΅ΡΡΡΡΠΌΡ Π³ΡΠ°Π½ΡΠΌΠΈ, ΡΠ΅ΡΡΡΡΠΌΡ Π²Π΅ΡΡΠΈΠ½Π°ΠΌΠΈ ΠΈ ΡΠ΅ΡΡΡΡΠΌΡ ΠΏΡΡΠΌΡΠΌΠΈ ΡΠ³Π»Π°ΠΌΠΈ. Π ΠΏΡΡΠΌΠΎΡΠ³ΠΎΠ»ΡΠ½ΠΈΠΊΠ΅ Π΄Π²Π΅ ΠΏΡΠΎΡΠΈΠ²ΠΎΠΏΠΎΠ»ΠΎΠΆΠ½ΡΠ΅ ΡΡΠΎΡΠΎΠ½Ρ ΠΏΠ°ΡΠ°Π»Π»Π΅Π»ΡΠ½Ρ. ΠΡΡΠΌΠΎΡΠ³ΠΎΠ»ΡΠ½Π°Ρ ΠΎΠ±Π»Π°ΡΡΡ β ΡΡΠΎ ΠΏΡΠΎΡΡΡΠ°Π½ΡΡΠ²ΠΎ Π² ΡΠΎΡΠΌΠ΅. ΠΠ»ΠΎΡΠ°Π΄Ρ ΠΏΡΡΠΌΠΎΡΠ³ΠΎΠ»ΡΠ½ΠΈΠΊΠ° ΡΠ°ΠΊΠΆΠ΅ ΡΠ²Π»ΡΠ΅ΡΡΡ ΠΏΠ΅ΡΠΈΠΌΠ΅ΡΡΠΎΠΌ ΠΏΡΡΠΌΠΎΡΠ³ΠΎΠ»ΡΠ½ΠΈΠΊΠ°. ΠΡΠΎ ΠΎΡΠ»ΠΈΡΠ°Π΅Ρ ΠΏΠ»ΠΎΡΠ°Π΄Ρ ΠΏΡΡΠΌΠΎΡΠ³ΠΎΠ»ΡΠ½ΠΈΠΊΠ° ΠΎΡ ΠΈΠ½ΡΠ΅Π³ΡΠΈΡΠΎΠ²Π°Π½ΠΈΡ, Π² ΠΊΠΎΡΠΎΡΠΎΠΌ ΠΌΡ Π²ΡΡΠΈΡΠ»ΡΠ΅ΠΌ ΠΏΠ»ΠΎΡΠ°Π΄Ρ ΠΏΠΎΠ΄ ΠΊΡΠΈΠ²ΠΎΠΉ.
ΠΠ»Ρ Π²ΡΡΠΈΡΠ»Π΅Π½ΠΈΡ ΠΏΠ»ΠΎΡΠ°Π΄ΠΈ Ρ ΡΠΎΡΠΊΠΈ Π·ΡΠ΅Π½ΠΈΡ ΡΠΎΡΠΌΡ ΠΏΡΡΠΌΠΎΡΠ³ΠΎΠ»ΡΠ½ΠΈΠΊ ΡΡΠΈΡΠ°Π΅ΡΡΡ ΡΠ°ΠΌΡΠΌ ΠΏΡΠΎΡΡΡΠΌ. ΠΠ»ΠΎΡΠ°Π΄Ρ β ΡΡΠΎ ΠΏΡΠΎΡΡΡΠ°Π½ΡΡΠ²ΠΎ, Π·Π°Π½ΠΈΠΌΠ°Π΅ΠΌΠΎΠ΅ ΠΊΠ°ΠΊΠΎΠΉ-Π»ΠΈΠ±ΠΎ ΡΠΎΡΠΌΠΎΠΉ ΠΈΠ»ΠΈ ΠΏΡΠ΅Π΄ΠΌΠ΅ΡΠΎΠΌ. ΠΠ΄Π½Π°ΠΊΠΎ ΠΎΠΏΠΈΡΠ°ΡΠ΅Π»ΡΠ½ΠΎ ΠΌΡ ΠΌΠΎΠΆΠ΅ΠΌ ΡΠΊΠ°Π·Π°ΡΡ, ΡΡΠΎ ΠΏΠ»ΠΎΡΠ°Π΄Ρ β ΡΡΠΎ ΠΊΠΎΠ»ΠΈΡΠ΅ΡΡΠ²ΠΎ ΠΏΠ»ΠΎΡΠΊΠΎΠΉ ΠΎΠ±Π»Π°ΡΡΠΈ, ΠΎΠ³ΡΠ°Π½ΠΈΡΠ΅Π½Π½ΠΎΠΉ ΠΎΠΏΡΠ΅Π΄Π΅Π»Π΅Π½Π½ΠΎΠΉ ΡΠΎΡΠΌΠΎΠΉ. Π§Π°ΡΡΠΎ ΠΈΡΠΏΠΎΠ»ΡΠ·ΡΠ΅ΠΌΡΠ΅ ΡΠΈΠΏΠΈΡΠ½ΡΠ΅ Π΅Π΄ΠΈΠ½ΠΈΡΡ ΠΏΠ»ΠΎΡΠ°Π΄ΠΈ Π²ΠΊΠ»ΡΡΠ°ΡΡ ΡΠΌ2 (ΠΊΠ²Π°Π΄ΡΠ°ΡΠ½ΡΠΉ ΡΠ°Π½ΡΠΈΠΌΠ΅ΡΡ), ΠΌ2 (ΠΌΠ΅ΡΡ Π² ΠΊΠ²Π°Π΄ΡΠ°ΡΠ΅) ΠΈ ΠΊΠΌ2 (ΠΊΠΈΠ»ΠΎΠΌΠ΅ΡΡ Π² ΠΊΠ²Π°Π΄ΡΠ°ΡΠ΅) .
ΠΠ°ΠΊ Π²ΡΡΠΈΡΠ»ΠΈΡΡ ΠΏΠ»ΠΎΡΠ°Π΄Ρ ΠΏΡΡΠΌΠΎΡΠ³ΠΎΠ»ΡΠ½ΠΈΠΊΠ° Π·Π° 3 ΠΏΡΠΎΡΡΡΡ
ΡΠ°Π³Π°
ΠΡΡΠΈΡΠ»Π΅Π½ΠΈΠ΅ ΠΏΠ»ΠΎΡΠ°Π΄ΠΈ ΠΏΡΡΠΌΠΎΡΠ³ΠΎΠ»ΡΠ½ΠΈΠΊΠ° β ΠΏΡΠΎΡΡΠ°Ρ ΠΏΡΠΎΡΠ΅Π΄ΡΡΠ°, ΠΊΠ°ΠΊ ΠΊΡΡΠΎΠΊ ΠΏΠΈΡΠΎΠ³Π°.
ΠΠ°ΠΊ ΠΈΠ·Π²Π΅ΡΡΠ½ΠΎ, ΠΏΠ»ΠΎΡΠ°Π΄Ρ ΠΏΡΡΠΌΠΎΡΠ³ΠΎΠ»ΡΠ½ΠΈΠΊΠ° ΡΠ°Π²Π½Π° ΠΏΡΠΎΠΈΠ·Π²Π΅Π΄Π΅Π½ΠΈΡ Π΅Π³ΠΎ ΡΠΈΡΠΈΠ½Ρ Π½Π° Π΄Π»ΠΈΠ½Ρ. Π’Π°ΠΊΠΈΠΌ ΠΎΠ±ΡΠ°Π·ΠΎΠΌ, Π²Ρ Π½Π°Π²Π΅ΡΠ½ΡΠΊΠ° ΡΠΌΠΎΠΆΠ΅ΡΠ΅ Π²ΡΡΠΈΡΠ»ΠΈΡΡ ΠΏΠ»ΠΎΡΠ°Π΄Ρ ΠΏΡΡΠΌΠΎΡΠ³ΠΎΠ»ΡΠ½ΠΈΠΊΠ° Π·Π° ΡΡΠΈ ΠΏΡΠΎΡΡΡΡ
ΡΠ°Π³Π°.
ΠΡΡΠΌΠΎΡΠ³ΠΎΠ»ΡΠ½ΠΈΠΊ ΡΠΎΡΡΠΎΠΈΡ ΠΈΠ· 4 ΡΡΠΎΡΠΎΠ½, Π½Π΅ ΡΠ°Π²Π½ΡΡ
ΠΏΠΎ Π΄Π»ΠΈΠ½Π΅. ΠΠ΄Π½Π°ΠΊΠΎ ΠΏΡΠΎΡΠΈΠ²ΠΎΠΏΠΎΠ»ΠΎΠΆΠ½ΡΠ΅ ΡΡΠΎΡΠΎΠ½Ρ ΡΠ°Π²Π½Ρ ΠΏΠΎ Π΄Π»ΠΈΠ½Π΅ Π΄ΡΡΠ³ Π΄ΡΡΠ³Ρ. Π ΠΏΡΡΠΌΠΎΡΠ³ΠΎΠ»ΡΠ½ΠΎΠΉ ΡΠΎΡΠΌΠ΅ Π΅Π³ΠΎ Π΄Π»ΠΈΠ½Π½Π°Ρ ΡΡΠΎΡΠΎΠ½Π° Π½Π°Π·ΡΠ²Π°Π΅ΡΡΡ Π΄Π»ΠΈΠ½ΠΎΠΉ, Π° Π±ΠΎΠ»Π΅Π΅ ΠΊΠΎΡΠΎΡΠΊΠ°Ρ ΡΡΠΎΡΠΎΠ½Π° ΠΈΠ·Π²Π΅ΡΡΠ½Π° ΠΊΠ°ΠΊ ΡΠΈΡΠΈΠ½Π°.
Π’Π°ΠΊΠΈΠΌ ΠΎΠ±ΡΠ°Π·ΠΎΠΌ, ΡΡΠΎΠ±Ρ Π²ΡΡΠΈΡΠ»ΠΈΡΡ ΠΏΠ»ΠΎΡΠ°Π΄Ρ ΠΏΡΡΠΌΠΎΡΠ³ΠΎΠ»ΡΠ½ΠΈΠΊΠ°, ΠΌΡ ΠΌΠΎΠΆΠ΅ΠΌ Π·Π°ΠΏΠΈΡΠ°ΡΡ Π΅Π³ΠΎ ΡΠΎΡΠΌΡΠ»Ρ ΠΊΠ°ΠΊ
A = l x w
Π₯ΠΎΡΡ Π΄Π»Ρ Π²ΡΡΠΈΡΠ»Π΅Π½ΠΈΡ Π² 3 ΠΏΡΠΎΡΡΡΡ
ΡΠ°Π³Π° Π²ΡΠ΅, ΡΡΠΎ Π½Π°ΠΌ Π½ΡΠΆΠ½ΠΎ ΡΠ΄Π΅Π»Π°ΡΡ, ΡΡΠΎ:
- ΠΡΠ΅ΠΆΠ΄Π΅ Π²ΡΠ΅Π³ΠΎ, ΠΎΠΏΡΠ΅Π΄Π΅Π»ΠΈΡΠ΅ ΡΠ°Π²Π½ΡΠ΅ ΠΈ ΠΏΡΠΎΡΠΈΠ²ΠΎΠΏΠΎΠ»ΠΎΠΆΠ½ΡΠ΅ ΡΡΠΎΡΠΎΠ½Ρ ΠΏΡΡΠΌΠΎΡΠ³ΠΎΠ»ΡΠ½ΠΈΠΊΠ°. ΠΠ°ΡΠ΅ΠΌ Π²ΠΎΠ·ΡΠΌΠΈΡΠ΅ ΠΎΠ΄Π½ΠΎ Π·Π½Π°ΡΠ΅Π½ΠΈΠ΅ ΡΠΎ ΡΡΠΎΡΠΎΠ½Ρ Π΄Π»ΠΈΠ½Ρ ΠΈ ΠΎΠ΄Π½ΠΎ ΡΠΎ ΡΡΠΎΡΠΎΠ½Ρ ΡΠΈΡΠΈΠ½Ρ.
- ΠΠ°ΠΏΠΈΡΠΈΡΠ΅ ΠΈΡ
Π² ΠΎΠ±Π»Π°ΡΡΠΈ ΡΠΎΡΠΌΡΠ»Ρ ΠΏΡΡΠΌΠΎΡΠ³ΠΎΠ»ΡΠ½ΠΈΠΊΠ°. ΠΠ°ΠΏΡΠΈΠΌΠ΅Ρ, Π΅ΡΠ»ΠΈ Ρ Π½Π°Ρ Π΅ΡΡΡ 6 ΡΠΌ Π΄Π»Ρ Π΄Π»ΠΈΠ½Ρ ΠΈ 4 ΡΠΌ Π΄Π»Ρ ΡΠΈΡΠΈΠ½Ρ ΠΏΡΡΠΌΠΎΡΠ³ΠΎΠ»ΡΠ½ΠΈΠΊΠ°, ΠΌΡ Π·Π°ΠΏΠΈΡΠ΅ΠΌ ΠΈΡ
ΠΊΠ°ΠΊ 6x4 .
- ΠΠ°ΠΊΠΎΠ½Π΅Ρ, ΡΠΌΠ½ΠΎΠΆΡΡΠ΅ ΠΎΠ±Π° Π·Π½Π°ΡΠ΅Π½ΠΈΡ Π΄ΡΡΠ³ Π½Π° Π΄ΡΡΠ³Π°, ΠΈΠ»ΠΈ Π²Ρ ΡΠ°ΠΊΠΆΠ΅ ΠΌΠΎΠΆΠ΅ΡΠ΅ ΠΈΡΠΏΠΎΠ»ΡΠ·ΠΎΠ²Π°ΡΡ Π°ΡΠΈΡΠΌΠ΅ΡΠΈΡΠ΅ΡΠΊΠΈΠ΅ ΡΠ°Π±Π»ΠΈΡΡ Π΄Π»Ρ ΠΏΠΎΠ»ΡΡΠ΅Π½ΠΈΡ ΠΏΡΠΎΠΈΠ·Π²Π΅Π΄Π΅Π½ΠΈΡ ΠΎΠ±ΠΎΠΈΡ
Π·Π½Π°ΡΠ΅Π½ΠΈΠΉ.
